Sunteți pe pagina 1din 160

PROPERTY

I. Classification of property, NCC 414


A. Immovable or real, 415
B. Movable, 416, 417

Lopez v. Orosa

Real estate connotes the land and the building constructed thereon, it is obvious that the inclusion of the building,
separate and distinct from the land, in the enumeration of what may constitute real properties, could mean only one
thing - that a building is by itself an immovable property.

FACTS:

Enrique Lopez was doing business under the name of Lopez-Castelo Sawmill. He was approached by Vicente Orosa,
inviting him to invest in Plaza Theatre, Inc., a corporation, which would set up a theatre in their area. Although
reluctant to invest, Lopez eventually agreed to supply lumber needed for the construction of the said theatre.

Eventually, the theatre was constructed and Lopez has yet to be paid in full. He demanded payment from Orosa,
who promised that he would be paid after the corporation secures a loan from a bank. Unbeknownst to Lopez, the
corporation already acquired a loan and mortgaged the property to the surety. Lopez was still unpaid and
demanded payment from Orosa, who, in turn, assigned all his shares in Plaza Theatre Inc. to the former.
Dissatisfied, Lopez filed a case against Orosa and Plaza Theatre, praying that land and building be sold to satisfy his
claim. The trial court ruled in favour of Lopez and ordered the sale of the building in a public auction, but not the
land on which it stands.

ISSUE:

Whether a material-man's lien for the value of the materials used in the construction of a building attaches to said
structure alone and does not extend to the land on which the building is adhered to.

RULING: YES.

While it is true that generally, real estate connotes the land and the building constructed thereon, it is obvious that
the inclusion of the building, separate and distinct from the land, in the enumeration of what may constitute real
properties, could mean only one thing - that a building is by itself an immovable property. Moreover, and in view of
the absence of any specific provision of law to the contrary, a building is an immovable property, irrespective of
whether or not said structure and the land on which it is adhered to belong 2 to the same owner.

A close examination of the provision of the Civil Code invoked by appellant reveals that the law gives preference to
unregistered refectionary credits only with respect to the real estate upon which the refection or work was made.
This being so, the inevitable conclusion must be that the lien so created attaches merely to the immovable property
for the construction or repair of which the obligation was incurred. Evidently, therefore, the lien in favor of
appellant for the unpaid value of the lumber used in the construction of the building attaches only to said structure
and to no other property of the obligors.

Associated Insurance v. Iya

A building is an immovable property irrespective of whether or not said structure and the land on which it is adhered
to belong to the same owner.

FACTS:

Adriano Valino and Lucia A. Valino were the owners and possessors of a house of strong materials in Caloocan,
Rizal. To enable her to purchase on credit rice from the NARIC, Lucia A. Valino filed a bond subscribed by the
Associated Insurance and Surety Co., Inc., and as counter-guaranty therefor, the spouses Valino executed an
alleged chattel mortgage on the aforementioned house in favor of the surety company, which encumbrance was
duly registered with the Chattel Mortgage Register of Rizal. At the time said undertaking took place, the parcel of
land on which the house is erected was still registered in the name of the Philippine Realty Corporation but was
able to obtained the same after full payment.

Subsequently, the Valinos, to secure a debt, executed a real estate mortgage over the lot and the house in favor of
Isabel Iya, which was duly registered and annotated.

Valino, failed to satisfy her obligation to the NARIC and also failed to reimburse the surety company. The company
foreclosed the chattel mortgage over the house. As a result thereof, a public sale was conducted and the property
was awarded to the surety company. The surety company then learned of the existence of the real estate mortgage
over the lot and the improvements thereon; thus, said surety company instituted a civil case for the exclusion of the
residential house from the real estate mortgage in favor of defendant Iya and the declaration and recognition of
plaintiff's right to ownership over the same in virtue of the award given during the public auction.

Defendant Isabel Iya filed her answer and alleged that in virtue of the real estate mortgage executed by her co-
defendants, she acquired a real right over the lot and the house constructed thereon; that the auction sale allegedly
conducted as a result of the foreclosure of the chattel mortgage on the house was null and void for non-compliance
with the form required by law.

Defendant surety company, insisted on its right over the building, arguing that as the lot on which the house was
constructed did not belong to the spouses at the time the chattel mortgage was executed, the house might be
considered only as a personal property and that the encumbrance thereof and the subsequent foreclosure
proceedings made pursuant to the provisions of the Chattel Mortgage Law were proper and legal.

ISSUE:

Which of these encumbrances should receive preference over the other?

RULING:

The building is subject to the real estate mortgage, in favor of Isabel Iya's. Isabel Iya's right to foreclose not only the
land but also the building erected thereon is hereby recognized.
As this Court, defining the nature or character of a building, has said:
. . . while it is true that generally, real estate connotes the land and the building constructed thereon, it is
obvious that the inclusion of the building, separate and distinct from the land, in the enumeration of what
may constitute real properties (Art. 415, new Civil Code) could only mean one thing — that a building is
by itself an immovable property . . . Moreover, and in view of the absence of any specific provision to the
contrary, a building is an immovable property irrespective of whether or not said structure and the land on
which it is adhered to belong to the same owner. (Lopez vs. Orosa, G.R. Nos. supra, p. 98).

A building certainly cannot be divested of its character of a realty by the fact that the land on which it is
constructed belongs to another. To hold it the other way, the possibility is not remote that it would result in
confusion, for to cloak the building with an uncertain status made dependent on the ownership of the land, would
create a situation where a permanent fixture changes its nature or character as the ownership of the land changes
hands. In the case at bar, as personal properties could only be the subject of a chattel mortgage and as obviously
the structure in question is not one, the execution of the chattel mortgage covering said building is clearly invalid
and a nullity. The registration of the chattel mortgage of a building of strong materials produce no effect as far as
the building is concerned (Leung Yee vs. Strong Machinery Co., 37 Phil., 644).

Tumalad v. Vicencio
41 SCRA 143 (1971)
If a house belonging to a person stands on a rented land belonging to another person, it may be mortgaged as a
personal property as so stipulated in the document of mortgage. If the owner declared his house to be a chattel, the
principle of estoppel will bar him from claiming otherwise.

FACTS:

Alberta Vicencio and Emiliano Simeon (respondents) obtained a loan from the Tumalads (petitioners) in the
amount of P4,800. To secure said loan, they executed on September 1, 1955 a chattel mortgage over their house
made of strong materials which was erected in a lot being rented from Madrigal & Company, Inc. located in Quiapo,
Manila in favour of the petitioners which was registered in the Registry of Deeds. The parties agreed that in default
of any of the amortizations, the entire balance will become due and demandable and that the chattel mortgage
constituted over the subject house will be enforced in accordance with Special Act No. 3135 and authorized the
sheriff of the City of Manila to sell the same after the necessary publication. Respondents defaulted in their
payment which resulted to the extrajudicial foreclosure of the property and was consequently sold at a public
auction where the petitioners were declared as the highest bidders. The petitioners filed an action with the MTC,
praying that respondents be ordered to pay rentals over the subject house up until such time possession is
surrendered to them. In turn, the respondents alleged that the chattel mortgage is null and void for it covers an
immovable property, specifically the subject house.

ISSUE:

Whether a house made of strong materials cannot be covered by a chattel mortgage since it is a real property.

RULING:

No, a house made of strong materials may be covered by a chattel mortgage notwithstanding the pronouncement
in Article 415 of the Civil Code which clearly separated buildings from lands in its enumeration of immovable
properties. The Court allowed certain exceptions in considering buildings, such as a house, to be movable despite
its classification as an immovable, an example of which is when it has been expressly designated to be a movable
property in the agreement of the parties.

Under the Deed of Mortgage Vicencio executed, the house was expressly designated as subject of the Chattel
Mortgage; it also contained a stipulation which states that "the mortgagor….voluntarily cedes, sells and
transfers by way of Chattel Mortgage the property together with its leasehold rights over the lot on which it is
constructed and participation..." Although there is no specific statement referring to the subject house as personal
property, yet by transferring a property by way of chattel mortgage, respondents could only have intended to treat
the subject house as a chattel. Further, the subject house stood on a rented lot to which respondents merely had a
temporary right as lessee, and although the Court held that this cannot in itself determine the status of the
property, it can be combined with other factors to sustain the interpretation that the parties, particularly the
mortgagors, intended to treat the house as personalty. Furthermore, respondents can no longer be allowed to
impugn the validity of the chattel mortgage for the principle of estoppel applies to them in treating the house as a
personalty.

Board of Assessments v. MERALCO


10 SCRA 68, January 31, 1964

FACTS:

Philippine Commission enacted Act No. 484 which authorized the Municipal Board of Manila to grant a franchise to
construct, maintain and operate an electric street railway and electric light, heat and power system in the City of
Manila. Charles M. Swift was awarded the said franchise. Respondent Manila Electric Co. became the transferee and
owner of the franchise.
Meralco's electric power is generated by its hydro-electric plant located at Botocan Falls, Laguna and is transmitted
to the City of Manila by means of electric transmission wires. These electric transmission wires which carry high
voltage current, are fastened to insulators attached on steel towers constructed by respondent.

Petitioner City Assessor of Quezon City declared the aforesaid steel towers for real property tax. After denying
respondent's petition to cancel these declarations, an appeal was taken by respondent to the Board of Assessment
Appeals of Quezon City, which required respondent to pay real property tax on the said steel towers. Respondent
paid the amount under protest, and filed a petition for review in the CTA.

The CTA held that the steel towers are personal properties and are not subject to real property tax.

ISSUE:

Whether or not the steel poles/towers are immovable properties

RULING:

No, they are not immovable properties as defined in Art. 415. The steel towers or supports in question, do not
come within the objects mentioned in par. 1 of Art. 415, because they do not constitute buildings or constructions
adhered to the soil. They are not construction analogous to buildings nor adhering to the soil. They are removable
and merely attached to a square metal frame by means of bolts, which when unscrewed could easily be dismantled
and moved from place to place. They also cannot be included under par 3, as they are not attached to an immovable
in a fixed manner, and they can be separated without breaking the material or causing deterioration upon the
object to which they are attached. Each of these steel towers or supports consists of steel bars or metal strips,
joined together by means of bolts, which can be disassembled by unscrewing the bolts and reassembled by
screwing the same. These steel towers or supports do not also fall under par 5, for they are not machineries,
receptacles, instruments or implements, and even if they were, they are not intended for industry or works on the
land. Petitioner is not engaged in an industry or works in the land in which the steel supports or towers are
constructed.

Berkenkotter v. Cu Unjieng
G.R. No. 41643. July 31, 1935

A mortgage constituted on a sugar central includes not only the land on which it is built but also the buildings,
machinery and accessories installed at the time the mortgage was constituted as well as the buildings, machinery and
accessories belonging to the mortgagor, installed after the constitution thereof.

FACTS:

On 26 April 1926, the Mabalacat Sugar Company obtained from Cu Unjieng e Hijos, a loan secured by a first
mortgage constituted on 2 parcels of land “with all its buildings, improvements, sugar-cane mill, steel railway,
telephone line, apparatus, utensils and whatever forms part or is a necessary complement of said sugar-cane mill,
steel railway, telephone line, now existing or that may in the future exist in said lots.” On 5 October 1926, the
Mabalacat Sugar Company decided to increase the capacity of its sugar central by buying additional machinery and
equipment, so that instead of milling 150 tons daily, it could produce 250. The estimated cost of said additional
machinery and equipment was P100,000. In order to carry out this plan, B. A. Green, president of said corporation,
proposed to the B. H. Berkenkotter, to advance the necessary amount for the purchase of said machinery and
equipment, promising to reimburse him as soon as he could obtain an additional loan from the mortgagees, Cu
Unjieng e Hijos, and that in case B. A. Green should fail to obtain an additional loan from Cu Unjieng e Hijos, said
machinery and equipment would become security therefor, said B. A. Green binding himself not to mortgage nor
encumber them to anybody until Berkenkotter be fully reimbursed for the corporation’s indebtedness to him..
Having agreed to said proposition made in a letter dated 5 October 1926, B. H. Berkenkotter, on 9 October 1926,
delivered the sum of P1,710 to B. A. Green, the total amount supplied by him to B. A. Green having been P25,750.
Furthermore, B. H. Berkenkotter had a credit of P22,000 against said corporation for unpaid salary. With the loan
of P25,750 and said credit of P22,000, the Mabalacat Sugar Co., Inc., purchased the additional machinery and
equipment. On 10 June 1927, B. A. Green applied to Cu Unjieng e Hijos for an additional loan of P75,000 offering as
security the additional machinery and equipment acquired by said B. A. Green and installed in the sugar central
after the execution of the original mortgage deed, on 27 April 1927, together with whatever additional equipment
acquired with said loan. B. A. Green failed to obtain said loan. The CFI Manila dismissed Berkenkotter’s complaint,
with costs. Hence, the appeal. The Supreme Court affirmed the appealed judgment in all its parts, with costs to the
appellant.

ISSUE:

Whether or not the additional machinery is considered an improvement subject to the mortgage executed in favor
of Mabalacat Sugar Co., Inc by Cu Unjieng e Hijos.

HELD:

Machinery has character of real property Article 334, paragraph 5, of the Civil Code gives the character of real
property to machinery, liquid containers, instruments or implements intended by the owner of any building or
land for use in connection with any industry or trade being carried on therein and which are expressly adapted to
meet the requirements of such trade or industry. The installation of a machinery and equipment in a mortgaged
sugar central, in lieu of another of less capacity, for the purpose of carrying out the industrial functions of the latter
and increasing production, constitutes a permanent improvement on said sugar central and subjects said
machinery and equipment to the mortgage constituted thereon (article 1877, Civil Code)

Davao Sawmill v. Castillo


61 Phil. 709 (1935)

Machinery which is movable in its nature only becomes immobilized when placed in a plant by the owner of the
property or plant. Such result would not be accomplished, therefore, by the placing of machinery in a plant by a tenant
or a usufructuary or any person having only a temporary right.

FACTS:
The Davao Saw Mill Co., Inc., is the holder of a lumber concession. It has operated a sawmill in Davao. However, the
land upon which the business was conducted belonged to another person. On the land the sawmill company
erected a building which housed the machinery used by it. Some of the implements thus used were clearly personal
property, the conflict concerning machines which were placed and mounted on foundations of cement.
In the contract of lease between the sawmill company and the owner of the land there appeared the following
provision: XXX That on the expiration of the period agreed upon, all the improvements and buildings introduced
and erected by the party of the second part shall pass to the exclusive ownership of the party of the first part
without any obligation on its part to pay any amount for said improvements and buildings XXX
In another action, wherein the Davao Light was the plaintiff and the Davao, Saw, Mill Co., Inc., was the defendant, a
judgment was rendered in favor of the plaintiff; a writ of execution issued thereon, and the properties now in
question were levied upon as personalty by the sheriff. Indeed the plaintiff, and the defendant herein having
consummated the sale, proceeded to take possession of the machinery and other properties described in the
corresponding certificates of sale executed in its favor by the sheriff of Davao.
ISSUE:

Whether the subject property is a personal property.

RULING: YES.

It must be pointed out that while not conclusive, the characterization of the property as chattels by the appellant is
indicative of intention and impresses upon the property the character determined by the parties.

So far as the subject-matter with which we are dealing — machinery placed in the plant — it is plain, that
machinery which is movable in its nature only becomes immobilized when placed in a plant by the owner of the
property or plant. Such result would not be accomplished, therefore, by the placing of machinery in a plant by a
tenant or a usufructuary or any person having only a temporary right.

The distinction rests, upon the fact that one only having a temporary right to the possession or enjoyment of
property is not presumed by the law to have applied movable property belonging to him so as to deprive him of it
by causing it by an act of immobilization to become the property of another.

Under such conditions the tenant in putting in the machinery was acting but as the agent of the owner in
compliance with the obligations resting upon him, and the immobilization of the machinery which resulted arose
in legal effect from the act of the owner in giving by contract a permanent destination to the machinery.

II. Character of Ownership, NCC 419-425


A. Public domain

Government v. Cabangis
G.R. No. L-28379, March 27, 1929

FACTS:

Lots 36, 39 and 40, which are subject to cadastral proceeding of the City of Manila were formerly a part of a large
parcel of land belonging to the predecessor of the herein claimants and appellees.

From the year 1896 said land began to wear away, due to the action of the waves of Manila Bay, until the year 1901
when the said lots became completely submerged in water in ordinary tides, and remained in such a state. On
1912, the Government undertook the dredging of Vitas Estuary in order to facilitate navigation, depositing all the
sand and silt taken from the bed of the estuary on the low lands which were completely covered with water,
surrounding that belonging to the Philippine Manufacturing Company, thereby slowly and gradually forming the
lots, the subject matter of this proceeding.

Nobody had declared lot 39 for the purposes of taxation, and it was only in the year 1926 that Dr. Pedro Gil, in
behalf of the claimants and appellees, declared lot No. 40 for such purpose.

The claimants-appellees contend that inasmuch as the said lots once formed a part of a large parcel of land
belonging to their predecessors, whom they succeeded, and their immediate predecessor in interest having taken
possession thereof, said lots belong to them.

ISSUE:

To which does the ownership of the reclaimed land belong to?

RULING:

The Government owns the reclaimed land in the sense that it has become property of public dominion, because in
letting it remained submerged, the claimants-appellees may be said to have abandoned the same. Having become
part of the sea or seashore, it became property for public use. When the government took steps to make it land
again, its status as public dominion remained unchanged. As provided by Article 5 of the Law of Waters,

ART. 5. Lands reclaimed from the sea in consequence of works constructed by the State, or by the provinces,
pueblos or private persons, with proper permission, shall become the property of the party constructing such
works, unless otherwise provided by the terms of the grant of authority.

Therefore, the claimants- appellees are not entitled to the land.

MIAA v. City of Pasay


G.R. No. 163072, April 2, 2009
The term "ports x x x constructed by the State" includes airports and seaports. The Airport Lands and Buildings of
MIAA are intended for public use, and at the very least intended for public service. Whether intended for public use or
public service, the Airport Lands and Buildings are properties of public dominion. As properties of public dominion, the
Airport Lands and Buildings are owned by the Republic and thus exempt from real estate tax under Section 234(a) of
the Local Government Code

FACTS:

Petitioner Manila International Airport Authority (MIAA) operates and administers the Ninoy Aquino International
Airport (NAIA) Complex under Executive Order No. 903 (EO 903), otherwise known as the Revised Charter of the
Manila International Airport Authority. EO 903 was issued on 21 July 1983 by then President Ferdinand E. Marcos.
Under Sections 3 and 22 of EO 903, approximately 600 hectares of land, including the runways, the airport tower,
and other airport buildings, were transferred to MIAA. The NAIA Complex is located along the border between
Pasay City and Parañaque City.

On 28 August 2001, MIAA received Final Notices of Real Property Tax Delinquency from the City of Pasay for the
taxable years 1992 to 2001On 24 August 2001, the City of Pasay, through its City Treasurer, issued notices of levy
and warrants of levy for the NAIA Pasay properties. MIAA received the notices and warrants of levy on 28 August
2001. Thereafter, the City Mayor of Pasay threatened to sell at public auction the NAIA Pasay properties if the
delinquent real property taxes remain unpaid.

On 29 October 2001, MIAA filed with the Court of Appeals a petition for prohibition and injunction with prayer for
preliminary injunction or temporary restraining order. The petition sought to enjoin the City of Pasay from
imposing real property taxes on, levying against, and auctioning for public sale the NAIA Pasay properties.

On 30 October 2002, the Court of Appeals dismissed the petition and upheld the power of the City of Pasay to
impose and collect realty taxes on the NAIA Pasay properties. MIAA filed a motion for reconsideration, which the
Court of Appeals denied. Hence, this petition.

ISSUE:

The issue raised in this petition is whether the NAIA Pasay properties of MIAA are exempt from real property tax.

HELD:

YES. In Manila International Airport Authority v. Court of Appeals6 (2006 MIAA case), this Court already resolved
the issue of whether the airport lands and buildings of MIAA are exempt from tax under existing laws. The 2006
MIAA case originated from a petition for prohibition and injunction which MIAA filed with the Court of Appeals,
seeking to restrain the City of Parañaque from imposing real property tax on, levying against, and auctioning for
public sale the airport lands and buildings located in Parañaque City. The only difference between the 2006 MIAA
case and this case is that the 2006 MIAA case involved airport lands and buildings located in Parañaque City while
this case involved airport lands and buildings located in Pasay City. The 2006 MIAA case and this case raised the
same threshold issue: whether the local government can impose real property tax on the airport lands, consisting
mostly of the runways, as well as the airport buildings, of MIAA. In the 2006 MIAA case, this Court held:

The Airport Lands and Buildings of MIAA are properties devoted to public use and thus are properties of public
dominion. Properties of public dominion are owned by the State or the Republic. Article 420 of the Civil Code
provides:

Art. 420. The following things are property of public dominion:

(1) Those intended for public use, such as roads, canals, rivers, torrents, ports and bridges constructed
by the State, banks, shores, roadsteads, and others of similar character;
(2) Those which belong to the State, without being for public use, and are intended for some public
service or for the development of the national wealth.

The term "ports x x x constructed by the State" includes airports and seaports. The Airport Lands and Buildings of
MIAA are intended for public use, and at the very least intended for public service. Whether intended for public use
or public service, the Airport Lands and Buildings are properties of public dominion. As properties of public
dominion, the Airport Lands and Buildings are owned by the Republic and thus exempt from real estate tax under
Section 234(a) of the Local Government Code.

Furthermore, the airport lands and buildings of MIAA are properties of public dominion intended for public use,
and as such are exempt from real property tax under Section 234(a) of the Local Government Code. However,
under the same provision, if MIAA leases its real property to a taxable person, the specific property leased becomes
subject to real property tax. In this case, only those portions of the NAIA Pasay properties which are leased to
taxable persons like private parties are subject to real property tax by the City of Pasay.

MIAA v. CA, et al.


G.R. No. 155650, July 20, 2006

No one can dispute that properties of public dominion mentioned in Article 420 of the Civil Code, like "roads, canals,
rivers, torrents, ports and bridges constructed by the State," are owned by the State. The term "ports" includes
seaports and airports.

FACTS:

Petitioner Manila International Airport Authority (MIAA) operates the Ninoy Aquino International Airport (NAIA)
Complex in Parañaque City under Executive Order No. 903, otherwise known as the Revised Charter of the Manila
International Airport Authority ("MIAA Charter"). Executive Order No. 903 was issued on 21 July 1983 by then
President Ferdinand E. Marcos. Subsequently, Executive Order Nos. 9091 and 2982 amended the MIAA Charter.

As operator of the international airport, MIAA administers the land, improvements and equipment within the NAIA
Complex. The MIAA Charter transferred to MIAA approximately 600 hectares of land,3 including the runways and
buildings ("Airport Lands and Buildings") then under the Bureau of Air Transportation.4 The MIAA Charter further
provides that no portion of the land transferred to MIAA shall be disposed of through sale or any other mode
unless specifically approved by the President of the Philippines.

The Office of the Government Corporate Counsel (OGCC) opined that the Local Government Code of 1991 withdrew
the exemption from real estate tax granted to MIAA under Section 21 of the MIAA Charter. Thus, MIAA negotiated
with respondent City of Parañaque to pay the real estate tax imposed by the City. MIAA then paid some of the real
estate tax already due.

MIAA then received Final Notices of Real Estate Tax Delinquency from the City of Parañaque for the taxable years
1992 to 2001.

The OGCC pointed out that Section 206 of the Local Government Code requires persons exempt from real estate tax
to show proof of exemption. The OGCC opined that Section 21 of the MIAA Charter is the proof that MIAA is exempt
from real estate tax.

ISSUE:

Whether the Airport Lands and Buildings of MIAA are property of public dominion.

HELD:

Yes, the Airport Lands and Buildings of MIAA are property of public dominion and therefore owned by the
State or the Republic of the Philippines.
ARTICLE 419. Property is either of public dominion or of private ownership.
ARTICLE 420. The following things are property of public dominion:
(1) Those intended for public use, such as roads, canals, rivers, torrents, ports and bridges constructed
by the State, banks, shores, roadsteads, and others of similar character;
(2) Those which belong to the State, without being for public use, and are intended for some public service or
for the development of the national wealth. (Emphasis supplied)
ARTICLE 421. All other property of the State, which is not of the character stated in the preceding article, is
patrimonial property.
ARTICLE 422. Property of public dominion, when no longer intended for public use or for public service, shall
form part of the patrimonial property of the State.

No one can dispute that properties of public dominion mentioned in Article 420 of the Civil Code, like "roads,
canals, rivers, torrents, ports and bridges constructed by the State," are owned by the State. The term "ports"
includes seaports and airports. The MIAA Airport Lands and Buildings constitute a "port" constructed by the
State. Under Article 420 of the Civil Code, the MIAA Airport Lands and Buildings are properties of public dominion
and thus owned by the State or the Republic of the Philippines.

The Airport Lands and Buildings are devoted to public use because they are used by the public for international
and domestic travel and transportation. The fact that the MIAA collects terminal fees and other charges from
the public does not remove the character of the Airport Lands and Buildings as properties for public use. The
operation by the government of a tollway does not change the character of the road as one for public use. Someone
must pay for the maintenance of the road, either the public indirectly through the taxes they pay the government,
or only those among the public who actually use the road through the toll fees they pay upon using the road. The
tollway system is even a more efficient and equitable manner of taxing the public for the maintenance of public
roads.

Republic v. Vega
G.R. No. 177790, January 17, 2011

To establish that the land subject of the application is alienable and disposable public land, the general rule remains:
all applications for original registration under the Property Registration Decree must include both (1) a CENRO or
PENRO certification and (2) a certified true copy of the original classification made by the DENR Secretary. As an
exception, however, the courts—in their sound discretion and based solely on the evidence presented on record—may
approve the application, pro hac vice, on the ground of substantial compliance showing that there has been a positive
act of government to show the nature and character of the land and an absence of effective opposition from the
government. This exception shall only apply to applications for registration currently pending before the trial court
prior to this Decision and shall be inapplicable to all future applications.

FACTS:

Respondents Vega sought to register a parcel of land, claiming that they inherited the same from their deceased
mother. Respondent-intervenors Buhay claimed a portion of the lot in question. The Republic, through the Office of
the Solicitor General, opposed the claim. The Republic maintains that the parcel of land is public domain, and that
respondents failed to substantiate that such was alienable. Respondents presented as witness an officer from
CENRO who testified that the land in question is indeed alienable. The RTC ruled in favor of the respondents and
ordered titles to be issued in favor of Vega and Buhay. The Republic appealed the case to the Court of Appeals,
which affirmed the findings of the lower court. The Republic files a Petition for Review on Certiorari. The Republic
claims that respondents were unable to prove that the parcel of land in question is not part of the public domain.
Respondent-intervenor Buhay challenged the petition as it raises a question of fact, which is outside the scope of
Rule 45, a Petition for Review on Certiorari.

ISSUE:

Is the parcel of land in dispute part of public domain?


HELD:

The rule for registration of government land is that there must be open, continuous, exclusive and notorious
possession and occupation of alienable government land. The fact of occupation and that the land is alienable
government land must be proven. Here, the Republic does not question the fact of occupation, but that of the
alienability of the land. They also contended that the testimony of the CENRO officer is insufficient. It has been held
in Jurisprudence that a CENRO certificate is inadequate proof that the land is alienable. There must also be
certification from the Secretary of Natural Resources. However, in light of a recent ruling, the CENRO certification
is held to be substantial compliance to the needed proof. Since respondents sought certification from the CENRO
before, they are in good faith in claiming the land. The proof that they presented may be considered as competent
and sufficient proof. It is to be noted, however, that this ruling applies pro hac vice.

Victoria v. Republic
G.R. No. 179673, June 8, 2011

To prove that the land subject of the application for registration is alienable, an applicant must establish the existence
of a positive act of the government such as a presidential proclamation or an executive order; an administrative
action; investigation reports of Bureau of Lands investigators; and a legislative act or statute.

FACTS:

Petitioner Natividad Sta. Ana Victoria applied for registration of a 1,729-square meter lot in Taguig City, before the
MeTC. The OSG, representing the respondent Republic of the Philippines, opposed the application in the usual
form.

The Conversion/Subdivision Plan Victoria presented in evidence showed that the land is inside the alienable and
disposable area as certified by the Bureau of Forest Development on January 3, 1968. The Republic did not present
any evidence in support of its opposition.

The MeTC rendered a decision granting the application for registration. However, the Republic appealed to the CA
pointing out in its brief that Victoria failed to present evidence that the subject property is alienable and disposable
land of the public domain and that she failed to establish the kind of possession required for registration.

In her brief, Victoria replied that the Conversion/Subdivision Plan she submitted carried a notation that the subject
property is within alienable and disposable area. Further, she attached to her brief a Certification dated November
6, 2006 issued by the Department of Environment and Natural Resources (DENR), verifying the subject property as
within the alienable and disposable land of the public domain.

CA rendered judgment reversing and setting aside the MeTC decision because Victoria failed to prove that the
subject lot is alienable and disposable land of the public domain.

ISSUE:

Whether or not Victoria amply proved that the subject lot is alienable and disposable land of the public domain.

RULING:

Yes. To prove that the land subject of the application for registration is alienable, an applicant must establish the
existence of a positive act of the government such as a presidential proclamation or an executive order; an
administrative action; investigation reports of Bureau of Lands investigators; and a legislative act or statute. The
applicant may secure a certification from the government that the lands applied for are alienable and disposable,
but the certification must show that the DENR Secretary had approved the land classification and released the land
of the pubic domain as alienable and disposable, and that the land subject of the application for registration falls
within the approved area per verification through survey by the PENRO or CENRO. The applicant must also present
a copy of the original classification of the land into alienable and disposable, as declared by the DENR Secretary or
as proclaimed by the President.

In compliance, the OSG submitted a certification from the DENR stating that Senior Forest Management Specialist
Corazon D. Calamno, who signed Victoria’s DENR Certification, is authorized to issue certifications regarding status
of public land as alienable and disposable land. The OSG also submitted a certified true copy of Forestry
Administrative Order 4-1141 dated January 3, 1968, signed by then Secretary of Agriculture and Natural Resources
Arturo R. Tanco, Jr., which declared portions of the public domain covered by Bureau of Forestry Map LC-2623,
approved on January 3, 1968, as alienable and disposable. Since the OSG does not contest the authenticity of the
DENR Certification, it seems too hasty for the CA to altogether disregard the same simply because it was not
formally offered in evidence before the court below. More so when even the OSG failed to present any evidence in
support of its opposition to the application for registration during trial at the MeTC. The attack on Victoria’s proof
to establish the nature of the subject property was made explicit only when the case was at the appeal stage in the
Republic’s appellant’s brief. Only then did Victoria find it necessary to present the DENR Certification, since she
had believed that the notation in the Conversion/Subdivision Plan of the property was sufficient.

Land Bank v. Cacayurin


G.R. No. 191667, April 17, 2013

Town plazas are properties of public dominion, to be devoted to public use and to be made available to the public in
general. They are outside the commerce of man and cannot be disposed of or even leased by the municipality to
private parties.

FACTS:

In Agoo, La Union, the Municipality’s Sangguniang Bayan (SB) passed certain resolutions to implement a multi-
phased plan (Redevelopment Plan) to redevelop the Agoo Public Plaza where the Imelda Garden and Jose Rizal
Monument were situated. There were to 2 phases of the said plan. To finance both phases, SB passed a resolution
for each phase authorizing then Mayor Eufranio Eriguel to obtain a loan from Land Bank and incidental thereto.

In phase 1, the Municipality mortgage a 2,323.75 sq.m. lot situated at the southeastern portion of the Agoo Plaza as
collateral. As additional security, it further authorized the assignment of a portion of its internal revenue allotment
and the monthly income from the proposed project in favor of Land Bank. Terms were confirmed, approved and
ratified thru a resolution. Land Bank extended a ₱4M loan in favor of the Municipality, the proceeds of which were
used to construct 10 kiosks. After completion, these kiosks were rented out. In phase 2, for approving the
construction of a commercial center on Agoo Plaza, Mayor Eriguel was again authorized to obtain a loan from Land
Bank, posting as well the same securities as that of the 1st Loan. Land Bank granted a 2nd loan in favor of the
Municipality the amount of ₱28M.

Unlike phase 1 of the plan, the construction of the commercial center at the Agoo Plaza was objected to by Eduardo
Cacayuran and some residents of the Municipality. These residents claimed that the conversion of the Agoo Plaza
into a commercial center, as funded by the proceeds from the 2 Loans, were "highly irregular, violative of the law,
and detrimental to public interests, and will result to wanton desecration of the said historical and public park."

Cacayuran wrote a letter addressed to the Mayor, Vice-Mayor and the members of the SB, expressing the growing
public clamor against the conversion of the Agoo Plaza into a commercial center. Unable to get any response,
Cacayuran, invoking his right as a taxpayer, filed a Complaint against the said Officers and Land Bank, assailing, the
validity of the Subject Loans on the ground that Agoo Plaza used as collateral is property of public dominion and
therefore, beyond the commerce of man. Land Bank claimed that it is not privy to the Officers’ acts of destroying
the Agoo Plaza and asserted that Cacayuran did not have a cause of action against it since he was not privy to any of
the Subject Loans.

During the pendency of the proceedings, the construction of the commercial center was completed and the said
structure later became known as the Agoo’s People Center. SB passed Municipal Ordinance declaring the area
where the APC stood as patrimonial property of the Municipality.
RTC was in favor of Cacayuran. On appeal, CA affirmed with modification the RTC’s ruling, excluding the Vice
Mayor from any personal liability arising from the Subject Loans.

ISSUE:

Whether the Subject Loans are ultra vires.

RULING:

An act which is outside of the municipality’s jurisdiction is considered as a void ultra vires act, while an act
attended only by an irregularity but remains within the municipality’s power is considered as an ultra vires act
subject to ratification and/or validation. To the former belongs municipal contracts which (a) are entered into
beyond the express, implied or inherent powers of the LGU; and (b) do not comply with the substantive
requirements of law e.g., when expenditure of public funds is to be made, there must be an actual appropriation
and certificate of availability of funds; while to the latter belongs those which (a) are entered into by the improper
department, board, officer of agent; and (b) do not comply with the formal requirements of a written contract e.g.,
the Statute of Frauds.

Applying these principles to the case at bar, it is clear that the Subject Loans belong to the first class of ultra vires
acts deemed as void.

Records disclose that the said loans were executed by the Municipality for the purpose of funding the conversion of
the Agoo Plaza into a commercial center pursuant to the Redevelopment Plan. However, the conversion of the said
plaza is beyond the Municipality’s jurisdiction considering the property’s nature as one for public use and
thereby, forming part of the public dominion. Accordingly, it cannot be the object of appropriation either by
the State or by private persons. Nor can it be the subject of lease or any other contractual undertaking.

In this relation, Art. 1409(1) CC provides that a contract whose purpose is contrary to law, morals, good customs,
public order or public policy is considered void and as such, creates no rights or obligations or any juridical
relations. Given the unlawful purpose behind the Subject Loans which is to fund the commercialization of Agoo
Plaza pursuant to said Plan, they are considered as ultra vires in the primary sense thus, rendering them void and
in effect, non-binding on the Municipality.

The land on which the Agoo Plaza is situated cannot be converted into patrimonial property – as the SB tried
to when it passed a Municipal Ordinance – absent any express grant by the national government. As public
land used for public use, the foregoing lot rightfully belongs to and is subject to the administration and
control of the Republic of the Philippines. Hence, without the said grant, the Municipality has no right to claim
it as patrimonial property.

While the Subject Loans cannot bind the Municipality for being ultra vires, the officers who authorized the passage
of the Subject Resolutions are personally liable.

Dream Village v. BCDA


G.R. No.192896, July 24, 2013

FACTS:

Petitioner Dream Village Neighborhood Association, Inc. (Dream Village) claims to represent more than 2,000
families who have been occupying a 78,466-square meter lot in Western Bicutan, Taguig City since 1985 "in the
concept of owners continuously, exclusively and... notoriously." The lot used to be part of the Hacienda de
Maricaban (Maricaban), owned by Dolores Casal y Ochoa and registered under a Torrens title,Original Certificate of
Title (OCT) No. 291, issued on October 17, 1906 by the Registry of Deeds of Rizal. Maricaban covered several
parcels of land with a total area of over 2,544 hectares spread out over Makati, Pasig, Taguig, Pasay, and
Parañaque.
Following the purchase of Maricaban by the government of the United States of America (USA) early in the
American colonial period, to be converted into the military reservation known as Fort William Mckinley, Transfer
Certificate of Title (TCT) No. 192 was issued in the... name of the USA to cancel OCT No. 291.[10] The US
government later transferred 30 has. of Maricaban to the Manila Railroad Company, for which TCT No. 192 was
cancelled by TCT Nos. 1218 and 1219, the first in the name of the Manila Railroad Company... for 30 has., and the
second in the name of the USA for the rest of the Maricaban property.

On January 29, 1914, TCT No. 1219 was cancelled and replaced by TCT No. 1688, and later that year, on September
15, 1914, TCT No. 1688 was cancelled and replaced by TCT No. 2288, both times in the name of the USA.[12] On
December 6, 1956, the USA formally... ceded Fort William Mckinley to the Republic of the Philippines (Republic),
and on September 11, 1958, TCT No. 2288 was cancelled and replaced by TCT No. 61524, this time in the name of
the Republic.[13] On July 12, 1957, President Carlos P. Garcia issued
Proclamation No. 423 withdrawing from sale or settlement the tracts of land within Fort William Mckinley, now
renamed Fort Bonifacio, and reserving them for military purposes.

On January 7, 1986, President Ferdinand E. Marcos issued Proclamation No. 2476 declaring certain portions of Fort
Bonifacio alienable and disposable[15] in the manner provided under Republic Act (R.A.) Nos. 274 and 730, in
relation to the Public Land Act,[16] thus allowing the sale to the settlers of home lots in Upper Bicutan, Lower
Bicutan, Signal Village, and Western Bicutan.

On October 16, 1987, President Corazon C. Aquino issued Proclamation No. 172 amending Proclamation No. 2476
by limiting to Lots 1 and 2 of the survey Swo-13-000298 the areas in Western Bicutan open for disposition.

On March 13, 1992, R.A. No. 7227 was passed[19] creating the Bases Conversion and Development Authority
(BCDA) to oversee and accelerate the conversion of Clark and Subic military reservations and their extension
camps (John Hay Station, Wallace Air Station, O'Donnell Transmitter Station, San Miguel Naval Communications
Station and Capas Relay Station) to productive civilian uses. Section 8[20] of the said law provides that the capital
of the BCDA will be provided from sales proceeds or transfers of... lots in nine (9) military camps in Metro Manila,
including 723 has. of Fort Bonifacio. The law, thus, expressly authorized the President of the Philippines "to sell the
above lands, in whole or in part, which are hereby declared alienable and disposable pursuant to the... provisions of
existing laws and regulations governing sales of government properties,"[21] specifically to raise capital for the
BCDA. Titles to the camps were transferred to the BCDA for this purpose,[22] and TCT No. 61524 was... cancelled
on January 3, 1995 by TCT Nos. 23888, 23887, 23886, 22460, 23889, 23890, and 23891, now in the name of the
BCDA.

Excepted from disposition by the BCDA are: a) approximately 148.80 has. reserved for the National Capital Region
(NCR) Security Brigade, Philippine Army officers' housing area, and Philippine National Police jails and support
services (presently known as Camp Bagong Diwa); b)... approximately 99.91 has. in Villamor Air Base for the
Presidential Airlift Wing, one squadron of helicopters for the NCR and respective security units; c) twenty one (21)
areas segregated by various presidential proclamations; and d) a proposed 30.15 has. as relocation site for...
families to be affected by the construction of Circumferential Road 5 and Radial Road 4, provided that the
boundaries and technical description of these exempt areas shall be determined by an actual ground survey.

Now charging the BCDA of wrongfully asserting title to Dream Village and unlawfully subjecting its members to
summary demolition, resulting in unrest and tensions among the residents,[25] on November 22, 1999, the latter
filed a letter-complaint with the COSLAP to seek its assistance in the verification survey of the subject 78,466-sq m
property, which they claimed is within Lot 1 of Swo-13-000298 and thus is covered by Proclamation No. 172. They
claim that they have been occupying the area for thirty (30) years "in the concept... of owners continuously,
exclusively and notoriously for several years," and have built their houses of sturdy materials thereon and
introduced paved roads, drainage and recreational and religious facilities. Dream Village, thus, asserts that the lot
is not among those... transferred to the BCDA under R.A. No. 7227, and therefore patent applications by the
occupants should be processed by the Land Management Bureau (LMB).
Respondent BCDA in its Answer[28] dated November 23, 2000 questioned the jurisdiction of the COSLAP to hear
Dream Village's complaint, while asserting its title to the subject property pursuant to R.A. No. 7227. It argued that
under Executive Order (E.O.) No. 561 which created the COSLAP, its task is merely to coordinate the various
government offices and agencies involved in the settlement of land problems or disputes, adding that BCDA does
not fall in the enumeration in Section 3 of E.O. No. 561, it being neither a... pastureland-lease holder, a timber
concessionaire, or a government reservation grantee, but the holder of patrimonial government property which
cannot be the subject of a petition for classification, release or subdivision by the occupants of Dream Village.

COSLAP Ruling

On the basis of the DENR's verification survey report, the COSLAP resolved that Dream Village lies outside of BCDA,
and particularly, outside of Swo-00-0001302, and thus directed the LMB of the DENR to process the applications of
Dream Village's members for sales patent, noting... that in view of the length of time that they "have been openly,
continuously and notoriously occupying the subject property in the concept of an owner, x x x they are qualified to
apply for sales patent on their respective occupied lots pursuant to R.A. Nos. 274 and 730 in... relation to the
provisions of the Public Land Act."

On the question of its jurisdiction over the c... omplaint, the COSLAP cited the likelihood that the summary eviction
by the BCDA of more than 2,000 families in Dream Village could stir up serious social unrest, and maintained that
Section 3(2) of E.O. No. 561 authorizes it to "assume jurisdiction and resolve land problems or disputes which are
critical and explosive in nature considering, for instance, the large number of parties involved, the presence or
emergence of social tension or unrest, or other similar critical situations requiring... immediate action," even as
Section 3(2)(d) of E.O. No. 561 also allows it to take cognizance of "petitions for classification, release and/or
subdivision of lands of the public domain," exactly the ultimate relief sought by Dream Village. Rationalizing that it
was... created precisely to provide a more effective mechanism for the expeditious settlement of land problems "in
general," the COSLAP invoked as its authority the 1990 case of Bañaga v. COSLAP,[33] where this Court said:

It is true that Executive Order No. 561 provides that the COSLAP may take cognizance of cases which are "critical
and explosive in nature considering, for instance, the large number of parties involved, the presence or emergence
of social tension or unrest, or other... similar critical situations requiring immediate action." However, the use of
the word "may" does not mean that the COSLAP's jurisdiction is merely confined to the above mentioned cases.
The provisions of the said Executive Order are clear that the COSLAP was created as a means... of providing a more
effective mechanism for the expeditious settlement of land problems in general, which are frequently the source of
conflicts among settlers, landowners and cultural minorities. Besides, the COSLAP merely took over from the
abolished PACLAP whose functions,... including its jurisdiction, power and authority to act on, decide and resolve
land disputes (Sec. 2, P.D. No. 832) were all assumed by it. The said Executive Order No. 561 containing said
provision, being enacted only on September 21, 1979, cannot affect the exercise of... jurisdiction of the PACLAP
Provincial Committee of Koronadal on September 20, 1978. Neither can it affect the decision of the COSLAP which
merely affirmed said exercise of jurisdiction.

In its Motion for Reconsideration[35] filed on May 20, 2004, the BCDA questioned the validity of the survey results
since it was conducted without its representatives present, at the same time denying that it received a notification
of the DENR verification... survey.[36] It maintained that there is no basis for the COSLAP's finding that the
members of Dream Village were in open, continuous, and adverse possession in the concept of owner, because not
only is the property not among those declared alienable and... disposable, but it is a titled patrimonial property of
the State.

CA Ruling
The CA in its Decision[47] dated September 10, 2009 ruled that the COSLAP has no jurisdiction over the complaint
because the question of whether Dream Village is within the areas declared as available for disposition in
Proclamation No. 172 is beyond its... competence to determine, even as the land in dispute has been under a
private title since 1906, and presently its title is held by a government agency, the BCDA, in contrast to the case of
Bañaga relied upon by Dream Village, where the disputed land was part of the public domain... and the disputants
were applicants for sales patent thereto.
Dream Village's motion for reconsideration was denied in the appellate court's Order[48] of July 13, 2010.

ISSUE:

Whether or not COSLAP had jurisdiction over the controversy.

RULING:

No. Under the law, [E.O. No. 561], the COSLAP has two options in acting on a land dispute or problem lodged before
it, namely, (a) refer the matter to the agency having appropriate jurisdiction for settlement/resolution; or (b)
assume jurisdiction if the matter is one of those... enumerated in paragraph 2(a) to (e) of the law, if such case is
critical and explosive in nature, taking into account the large number of the parties involved, the presence or
emergence of social tension or unrest, or other similar critical situations requiring immediate action.

In resolving whether to assume jurisdiction over a case or to refer the same to the particular agency concerned, the
COSLAP has to consider the nature or classification of the land involved, the parties to the case, the nature of the
questions raised, and the need for immediate... and urgent action thereon to prevent injuries to persons and
damage or destruction to property. The law does not vest jurisdiction on the COSLAP over any land dispute or
problem.

Thus, in Machado, it was held that the COSLAP cannot invoke Section 3(2)(e) of E.O. No. 561 to assume jurisdiction
over "other similar land problems of grave urgency," since the statutory construction principle of ejusdem generis
prescribes that where general words... follow an enumeration of persons or things, by words of a particular and
specific meaning, such general words are not to be construed in their widest extent but are to be held as applying
only to persons or things of the same kind as those specifically mentioned.

Following this rule COSLAP's jurisdiction is limited to disputes involving lands in which the government has a
proprietary or regulatory interest,... or public lands covered with a specific license from the government such as a
pasture... lease agreements, a timber concessions, or a reservation grants,... and where moreover, the dispute is
between occupants/squatters and pasture lease agreement holders or timber concessionaires; between
occupants/squatters and government reservation... grantees; and between occupants/squatters and public land
claimants or applicants.

In Machado, the high court ruled that COSLAP has no jurisdiction in disputes over private lands between private
parties, reiterating the essential rules contained in Section 3 of E.O. No. 561 governing the exercise by COSLAP of
its jurisdiction, to wit:

Under these terms, the COSLAP has two different rules in acting on a land dispute or problem lodged before it, e.g.,
COSLAP can assume jurisdiction only if the matter is one of those enumerated in paragraph 2(a) to (e) of the law.
Otherwise, it should refer... the case to the agency having appropriate jurisdiction for settlement or resolution. In
resolving whether to assume jurisdiction over a case or to refer it to the particular agency concerned, the COSLAP
considers: (a) the nature or classification of the land involved; (b) the... parties to the case; (c) the nature of the
questions raised; and (d) the need for immediate and urgent action thereon to prevent injury to persons and
damage or destruction to property. The terms of the law clearly do not vest on the COSLAP the general power to
assume... jurisdiction over any land dispute or problem. Thus, under EO 561, the instances when the COSLAP may
resolve land disputes are limited only to those involving public lands or those covered by a specific license from
the government, such as pasture lease agreements, timber... concessions, or reservation grants.

The land dispute in Bañaga was between private individuals who were free patent applicants over unregistered
public lands.

In contrast, the present petition involves land titled to... and managed by a government agency which has been
expressly reserved by law for a specific public purpose other than for settlement.
Thus, as we have advised in Longino... the law does not vest jurisdiction on the COSLAP over any land dispute or
problem, but it has to... consider the nature or classification of the land involved, the parties to the case, the nature
of the questions raised, and the need for immediate and urgent action thereon to prevent injuries to persons and
damage or destruction to property.

Republic v. Cortez
G.R. No. 186639. February 5, 2014

Article 422 of the Civil Code states that "[p]roperty of public dominion, when no longer intended for public use or for
public service, shall form part of the patrimonial property of the State." Article 420 (2) makes clear that those
property "which belong to the State, without being for public use, and are intended for some public service or for the
development of the national wealth" are public dominion property. It is only when such alienable and disposable lands
are expressly declared by the State to be no longer intended for public service or for the development of the national
wealth that the period of acquisitive prescription can begin to run. Such declaration shall be in the form of a law duly
enacted by Congress or a Presidential Proclamation in cases where the President is duly authorized by law.

FACTS:

Emmanuel Cortez filed with the RTC an application for judicial confirmation of title/ registration over Lot No.
2697-B of the Pateros Cadastre. Cortez claimed that the entire lot was inherited by his mother from the latter’s
parents and when their mother died, he (Cortez) and his siblings executed an EJ Settlement of Estate. He alleged
that the subject property had been in the possession of his family since time immemorial; that the subject parcel of
land is not part of the reservation of the DENR and is, in fact, classified as alienable and disposable by the Bureau of
Forest Development (BFD). RTC granted the application. The Republic appealed, alleging that no document was
presented that would establish Cortez’ claims. Petitioner further alleged that there was no certification from any
government agency that the subject property had already been declared alienable and disposable. As such, Cortez’
possession of the subject property, no matter how long, cannot confer ownership or possessory rights. CA
dismissed the appeal.

ISSUE:

W/N CA erred in affirming the decision which granted the application

RULING:

YES. Applicants for original registration of title to land must establish compliance with the provisions of Section 14
of P.D. No. 1529, which pertinently provides that:

Sec. 14. Who may apply. The following persons may file in the proper Court of First Instance an application for
registration of title to land, whether personally or through their duly authorized representatives: (1) Those who by
themselves or through their predecessors-in interest have been in open, continuous, exclusive and notorious
possession and occupation of alienable and disposable lands of the public domain under a bona fide claim of
ownership since June 12, 1945, or earlier. (2) Those who have acquired ownership of private lands by prescription
under the provision of existing laws. Xxx

Cortez failed to comply with the legal requirements for the registration of the subject property under Section 14(1)
and (2) of P.D. No. 1529. "Under Section 14(1) [of P.D. No. 1529], applicants for registration of title must
sufficiently establish first, that the subject land forms part of the disposable and alienable lands of the public
domain; second, that the applicant and his predecessors-in-interest have been in open, continuous, exclusive, and
notorious possession and occupation of the same; and third, that it is under a bona fide claim of ownership since
June 12, 1945, or earlier." All requirements were not satisfied in this case. To prove that the subject property forms
part of the alienable and disposable lands of the public domain, Cortez adduced in evidence an Advance Plan
(survey plan by Geodetic Engr. Fernandez, certified by the Lands Management Bureau of DENR) with a notation
stating in effect that the said properties are alienable and disposable. To prove that the land subject of an
application for registration is alienable, an applicant must establish the existence of a positive act of the
government such as a presidential proclamation or an executive order, an administrative action, investigation
reports of Bureau of Lands investigators, and a legislative act or statute. The applicant may also secure a
certification from the Government that the lands applied for are alienable and disposable. Other than his bare claim
that his family possessed the subject property since time immemorial, Cortez failed to present any evidence to
show that he and his predecessors-in-interest indeed possessed the subject property prior to 1946; it is a mere
claim and not factual proof of possession.

Fortuna v. Republic
G.R. No. 173423, March 5, 2014

An applicant for registration of title acquired through a public land grant must present incontrovertible evidence that
the land subject of the application is alienable or disposable by establishing the existence of a positive act of the
government.

FACTS:

Spouses Fortuna filed an application for registration of a land identified as Lot No. 4457. The Spouses claimed that
they have been in quiet, peaceful, adverse and uninterrupted possession of said lot for more than 50 years as
evidenced by the lot’s survey plan, technical description and certificate of assessment. However, respondent
Republic of the Philippines (Republic) opposed the application.

The RTC granted the application of the spouses. On appeal, the Republic argued that the spouses Fortuna did not
present an official proclamation from the government that the lot has been classified as alienable and disposable
agricultural land. It also claimed that the possession of the spouses dates back only to 1948, thus, failing to meet
the June 12, 1945 cut-off period provided under PD 1529 or the Property Registration Decree (PRD). The CA
reversed and set aside the decision of the RTC. Although it found that the spouses Fortuna were able to establish
the alienable and disposable nature of the land, they failed to show that they complied with the length of
possession that the law requires, i.e., since June 12, 1945.

ISSUE:

Whether Spouses Fortuna sufficiently complied with the requisites for the acquisition of title to alienable lands of
the public domain.

RULING:

No, Spouses Fortuna did not.

The Court ruled that it is essential for any applicant for registration of title to land derived through a public grant
to establish foremost the alienable and disposable nature of the land. The Court explained that jurisprudence has
required that an applicant for registration of title acquired through a public land grant must present
incontrovertible evidence that the land subject of the application is alienable or disposable by establishing the
existence of a positive act of the government, such as a presidential proclamation or an executive order; an
administrative action; investigation reports of Bureau of Lands investigators; and a legislative act or a statute.

In this case, the Court ruled that the notation in the survey plan and the certification from the DENR-CENRO, which
were relied upon by the CA in ruling that the spouses were able to establish the alienable and disposable nature of
the land, were not evidence of a positive act from the government reclassifying the lot as alienable and disposable
agricultural land of the public domain. Hence, for failure to present incontrovertible evidence that Lot No. 4457 has
been reclassified as alienable and disposable land of the public domain though a positive act of the Executive
Department, the spouses Fortuna’s claim of title through a public land grant under the PLA should be denied.

Homeowners Association of Talayan Village, Inc. v. J.M. Tuason & Co., Inc., Talayan Holdings, Inc., Quezon
City Mayor and Equitable Banking Corporation
G.R. No. 203883, Nov. 10, 2015
FACTS:

The Quezon City Council passed an Ordinance directing all subdivision owners to turn over to the city government
the open spaces in city subdivisions which were required to be equivalent to 6% of the total land area being
developed. In compliance with said ordinance, J.M. Tuason, through Araneta, executed in favor of the city
government a Deed of Donation and Acceptance over its subdivisions’ open spaces which included, among others,
Block 494. Block 494 became the site of the Talayan Village Barangay Hall at the expense of Homeowners
Association of Talayan Village (HATVI). For failure of J.M. Tuason to pay its realty taxes, however, Block 494 was
scheduled for a tax delinquency sale by the city government. The Barangay Captain and the homeowners of
Talayan Vilage made known their opposition/objection to the impending sale. J.M. Tuason executed a Unilateral
Deed of Absolute Sale transferring Block 494 in favor of respondent Talayan Holdings, Inc. (THI) having caused the
cancellation of the previous TCT in favor of the latter.

Inquiring from the designated broker, Eastcoast Properties & Holdings Corp. (EPHC), Dr. Rosario Agustin received
a letter confirming that the land being sold was Block 494 and that the same was previously purchased in a
delinquency sale by J.M. Tuason. THI eventually obtained a loan from Equitable Banking Corporation (Equitable
Bank), secured by real estate mortgages over the four lots into which Block 494 had been subdivided. HATVI filed
against respondents a complaint for annulment of sale, cancellation of titles and mortgage, acceptance of donation
and damages. Contending that it had no knowledge of the delinquency sale and that its members purchased their
respective home lots on the belief that Block 494 was an open space for use as in fact it was used - as a public park,
HATVI argued that the subject parcel is beyond the commerce of man. RTC dismissed the case.

ISSUE:

Whether or not Block 494 is a public domain which is beyond the commerce of man

RULING: NO.

Considering that P.D. 1216 does not provide for the retroactive application of its provisions, the CA cannot be
faulted for ruling that the applicable law is the Land Registration Act whose lack of requirement for the reservation
of open spaces in subdivisions was filled in by the requirement for the same in the ordinances passed by the
Quezon City government. Having already designated sufficient open spaces for the Sta. Mesa Heights subdivision to
an excess of 48,679.040 square meters, J.M. Tuason was admitted by the parties to have complied with said
ordinances by executing the Deed of Donation over Block 494 in favor of the Quezon City government. The
development of Talayan Village was pursuant to Subdivision Plan PSD-52256 which was approved by the CFI of
Rizal in LRC (GLRO) Rec. No. 7681. Rather than Block 494, said subdivision plan significantly designated Block 503
as the park/open space for said subdivision. That J.M., Tuason donated Block 494 to the Quezon City government in
compliance with the latter's ordinances also did not operate to divest the property of its private character. In
addition to the fact that the donation was not embodied in a public document as provided under Article 749 of the
Civil Code of the Philippines, the record is entirely bereft of showing that said donation was duly accepted in
accordance with Article 745 of the same Code. The purpose of the formal requirement for acceptance of a donation
is to ensure that such acceptance is duly communicated to the donor. Since the donation is considered perfected
only upon the moment the donor is apprised of such acceptance, it has been ruled that lack of such acceptance, as
expressly provided under the law, renders the donation null and void.

B. Patrimonial property of the State

Laurel v. Garcia
187 SCRA 797

Roppongi property is of public dominion and not a patrimonial property. As such, it is outside the commerce man and
it cannot be alienated. The fact that the Roppongi site has not been used for a long time does not automatically
convert it to patrimonial property.
FACTS:

The subject Roppongi property in this case is one of the four properties in Japan acquired by the Philippine
government under the Reparations Agreement entered into with Japan, as part of the indemnification to the
Filipino people for World War II. It was expressly indicated that such property is for the use by the government
sector. It became the site of the Philippine Embassy until the latter was transferred to Nampeidai on July 22, 1976.
Consequently, the Roppongi property has remained undeveloped since that time. Amidst opposition by various
sectors, the Executive branch has been pushing its decision to sell the reparations properties starting with the
Roppongi lot. Hence, the petitioners are seeking to enjoin respondents from proceeding with the bidding for the
sale of the 3179 sqm land at 306 Roppongi, Tokyo, Japan.

ISSUE:

(1) Whether the Roppongi property is of public dominion or patrimonial.


(2) Whether the Roppongi property and others of its kind can be alienated by the Philippine Government.
(3) Whether the Chief Executive, her officers and agents, have the authority and jurisdiction to sell the Roppongi
property.

RULING:

(1) The Roppongi property is of public dominion.

The nature of the Roppongi lot as property for public service is expressly spelled out. It is dictated by the terms
of the Reparations Agreement and the corresponding contract of procurement. It is classified under par. 2 of Art.
420 of the Civil Code as property belonging to the State and intended for some public service. There can be no
doubt that it is of public dominion unless it is convincingly shown that the property has become patrimonial. The
fact that the Roppongi site has not been used for a long time for actual Embassy service does not automatically
convert it to patrimonial property. A property continues to be part of the public domain until there is a
formal declaration on the part of the government to withdraw it from being such. An abandonment of the
intention to use the Roppongi property for public service and to make it patrimonial property under Art.
422 of the Civil Code must be definite. Abandonment cannot be inferred from the non-use alone. A mere
transfer of the Philippine Embassy to Nampeidai in 1976 is not relinquishment of the Roppongi property’s original
purpose.

(2) The Roppongi property cannot by alienated.

As property of public dominion, the Roppongi lot is outside the commerce of man. It cannot be alienated. Its
ownership is a special collective ownership for general use and enjoyment, an application to the satisfaction of
collective needs, and resides in the social group. The purpose is not to serve the State as a juridical person, but the
citizens; it is intended for the common and public welfare and cannot be the object of appropriation.

(3) The Executive branch does not have authority to sell the Roppongi property.

Section 63 (c) of RA No. 6657 (Comprehensive Agrarian Reform Program or CARP Law) which provides as one of
the sources of funds for its implementation, the proceeds of the disposition of the properties of the Government in
foreign countries, did not withdraw the Roppongi property from being classified as one of public dominion when it
mentions Philippine properties abroad. Section 63 (c) refers to properties which are alienable and not to those
reserved for public use or service. RA No. 6657, therefore, does not authorize the Executive Department to sell the
Roppongi property. It merely enumerates possible sources of future funding to augment (as and when needed) the
Agrarian Reform Fund created under EO No. 299. Obviously, any property outside of the commerce of man cannot
be tapped as a source of funds

Republic v. Rizalvo
G.R. No. 172011, March 7, 2011
FACTS:

On December 7, 2000, respondent Teodoro P. Rizalvo, Jr. filed before the MTC of Bauang, La Union an application
for the registration of a parcel of land referred to in Survey Plan Psu-200706,4 located in Bauang, La Union and
containing an area of 8,957 square meters. Respondent alleged that he is the owner in fee simple of the subject
parcel of land, that he obtained title over the land by virtue of a Deed of Transfer, alleging that he bought the
property from his mother and that he is currently in possession of the land. He thus presented a tax declaration
under his name and a Proof of Payment of real property taxes from 1952 up to the time of his filing of the
application. OSG filed an Opposition alleging that neither respondent nor his predecessors-in-interest had been in
open, continuous, exclusive and notorious possession and occupation of the subject property since June 12, 1945
or earlier and that the tax declarations and tax payment receipts did not constitute competent and sufficient
evidence of ownership, and that the subject property was a portion of public domain and hence not subject to
private acquisition.

ISSUE:

WON respondent and his predecessors-in-interest were in open, continuous, adverse, and public possession of the
land in question in the manner and length of time required by law as to entitle respondent to judicial confirmation
of imperfect title.

RULING: NO.

Applicant failed to comply with PD 1529. An applicant for judicial confirmation of imperfect title must prove
compliance with Section 14 of PD No. 1529 or the Property Registration Decree. Among the requirements, one of
which was not satisfied because Rizalvo merely presented a certification and report from the DENR-CENRO dated
July 17, 2001 certifying that the land in question entirely falls within the alienable and disposable zone since
January 21, 1987; that it has not been earmarked for public use; and that it does not encroach any area devoted to
general public use.

Unfortunately, such certification and report is not enough in order to commence the thirty (30)-year prescriptive
period under Section 14 (2). There is no evidence in this case indicating any express declaration by the state that
the subject land is no longer intended for public service or the development of the national wealth. Thus, there
appears no basis for the application of the thirty (30)-year prescriptive period in this case.

However, even if the DENR-CENRO report was enough, Rizalvo would still not be entitled to the registration of the
land. Indeed, even assuming arguendo that the DENR-CENRO certification and report is enough to signify that the
land is no longer intended for public service or the development of the national wealth, respondent is still not
entitled to registration because the land was certified as alienable and disposable in 1987, while the application for
registration was filed on December 7, 2000, a mere thirteen (13) years after and far short of the required thirty
(30) years under existing laws on prescription.

Republic v. Cortez
G.R. No. 186639. February 5, 2014

Article 422 of the Civil Code states that "[p]roperty of public dominion, when no longer intended for public use or for
public service, shall form part of the patrimonial property of the State." Article 420 (2) makes clear that those
property "which belong to the State, without being for public use, and are intended for some public service or for the
development of the national wealth" are public dominion property. It is only when such alienable and disposable lands
are expressly declared by the State to be no longer intended for public service or for the development of the national
wealth that the period of acquisitive prescription can begin to run. Such declaration shall be in the form of a law duly
enacted by Congress or a Presidential Proclamation in cases where the President is duly authorized by law.

FACTS:
Emmanuel Cortez filed with the RTC an application for judicial confirmation of title/ registration over Lot No.
2697-B of the Pateros Cadastre. Cortez claimed that the entire lot was inherited by his mother from the latter’s
parents and when their mother died, he (Cortez) and his siblings executed an EJ Settlement of Estate. He alleged
that the subject property had been in the possession of his family since time immemorial; that the subject parcel of
land is not part of the reservation of the DENR and is, in fact, classified as alienable and disposable by the Bureau of
Forest Development (BFD). RTC granted the application. The Republic appealed, alleging that no document was
presented that would establish Cortez’ claims. Petitioner further alleged that there was no certification from any
government agency that the subject property had already been declared alienable and disposable. As such, Cortez’
possession of the subject property, no matter how long, cannot confer ownership or possessory rights. CA
dismissed the appeal.

ISSUE:

W/N CA erred in affirming the decision which granted the application

RULING:

YES. Applicants for original registration of title to land must establish compliance with the provisions of Section 14
of P.D. No. 1529, which pertinently provides that:

Sec. 14. Who may apply. The following persons may file in the proper Court of First Instance an application for
registration of title to land, whether personally or through their duly authorized representatives: (1) Those who by
themselves or through their predecessors-in interest have been in open, continuous, exclusive and notorious
possession and occupation of alienable and disposable lands of the public domain under a bona fide claim of
ownership since June 12, 1945, or earlier. (2) Those who have acquired ownership of private lands by prescription
under the provision of existing laws. Xxx

Cortez failed to comply with the legal requirements for the registration of the subject property under Section 14(1)
and (2) of P.D. No. 1529. "Under Section 14(1) [of P.D. No. 1529], applicants for registration of title must
sufficiently establish first, that the subject land forms part of the disposable and alienable lands of the public
domain; second, that the applicant and his predecessors-in-interest have been in open, continuous, exclusive, and
notorious possession and occupation of the same; and third, that it is under a bona fide claim of ownership since
June 12, 1945, or earlier." All requirements were not satisfied in this case. To prove that the subject property forms
part of the alienable and disposable lands of the public domain, Cortez adduced in evidence an Advance Plan
(survey plan by Geodetic Engr. Fernandez, certified by the Lands Management Bureau of DENR) with a notation
stating in effect that the said properties are alienable and disposable. To prove that the land subject of an
application for registration is alienable, an applicant must establish the existence of a positive act of the
government such as a presidential proclamation or an executive order, an administrative action, investigation
reports of Bureau of Lands investigators, and a legislative act or statute. The applicant may also secure a
certification from the Government that the lands applied for are alienable and disposable. Other than his bare claim
that his family possessed the subject property since time immemorial, Cortez failed to present any evidence to
show that he and his predecessors-in-interest indeed possessed the subject property prior to 1946; it is a mere
claim and not factual proof of possession.

Republic v. Rovency Realty Development Corporation


G.R. No. 190817, Jan. 10, 2018

An applicant for land registration must exhibit that it and its predecessors-in-interest had been in open, continuous,
exclusive, and notorious possession and occupation of the land under a bona fide claim of ownership since 12 June
1945 or earlier. It has been held that possession is open when it is patent, visible, apparent, notorious, and not
clandestine; it is continuous when uninterrupted, unbroken, and not intermittent or occasional; it is exclusive when
the adverse possessor can show exclusive dominion over the land and an appropriation of it to his own use and benefit;
and notorious when it is so conspicuous, that it is generally known and talked of by the public or the people in the
neighborhood.
FACTS:

On 22 March 2001, RRDC filed before the RTC an Amended Application for Registration covering a parcel of land
identified as Lot No. 3009 (subject land) with 31.8 hectares situated in Barangay Balulang, Cagayan de Oro City.

An opposition to the application was filed by the Heirs of Paulino Avancena. They alleged, that the subject land was
already claimed and owned by the late Atty. Paulino Avancena (Paulino), their father and predecessor-in-interest,
as early as 1926; that Paulino had been in open, continuous, notorious, adverse, and exclusive possession and
occupation of the subject land. Furthermore, the heirs alleged that the subject land exceeds the twelve (12)-hectare
limit for confirmation of imperfect title set by Section 47 of Commonwealth Act (CA.) No. 141, as amended by
Republic Act (R.A.) No. 6940; and that the subject land forms part of the public domain belonging to the Republic
and, thus, not subject to private appropriation.

RTC granted RRDC's application for registration of the land. It explained that Section 3 of Article XII of the 1987
Constitution, the constitutional provision which provided for the 12-hectare limit in the acquisition of land, covers
only agricultural lands of the public domain. It ratiocinated that when the subject land was acquired through
acquisitive prescription by RRDC's predecessors-in-interest, it was converted into a private property and, as such,
it ceased to be part of the public domain. Thus, when RRDC acquired the subject land by purchase, it was no longer
within the ambit of the constitutional limitation.

CA affirmed RTC.

ISSUE:

WON the land applied for registration of title is in excess of what is allowed by law.

RULING:

YES, it is in excess of what is allowed by law.

As can be clearly gleaned from its language, Section 3, Article XII applies only to lands of the public domain. Private
lands are, therefore, outside of the prohibitions and limitations stated therein. Thus, the appellate court correctly
declared that the 12-hectare limitation on the acquisition of lands under Section 3, Article XII of the 1987
Constitution has no application to private lands.

RRDC failed to present a certified true copy of the original classification approved by the DENR Secretary declaring
the subject land alienable and disposable. Furthermore, RRDC also failed to prove that it and its individual
predecessors-in-interest sufficiently complied with the required period and nature of possession.

An applicant for land registration must exhibit that it and its predecessors-in-interest had been in open,
continuous, exclusive, and notorious possession and occupation of the land under a bona fide claim of ownership
since 12 June 1945 or earlier. It has been held that possession is open when it is patent, visible, apparent,
notorious, and not clandestine; it is continuous when uninterrupted, unbroken, and not intermittent or occasional;
it is exclusive when the adverse possessor can show exclusive dominion over the land and an appropriation of it to
his own use and benefit; and notorious when it is so conspicuous, that it is generally known and talked of by the
public or the people in the neighborhood.

C. Private ownership, NCC 421

Tan Toco v. Municipality of Iloilo


G.R. No. 24950, 25 March 1926

Property for public use of the municipality is not within the commerce of man so long as it is used by the public and,
consequently, said property is also inalienable. The movable and immovable property of a municipality, necessary for
governmental purposes, may not be attached and sold for the payment of a judgment against the municipality.
FACTS:

Tan Toco sued the Municipality of Iloilo for the purchase price of two strips of land which the Municipality had
appropriated for widening the street. The Court of First Instance of Iloilo sentenced the Municipality to pay Tan
Toco the amount claimed for such strips of land. On the account of lack of funds, the Municipality failed to pay. Tan
Toco had a writ of execution issued against the property of the Municipality, by virtue of which the auto trucks
used fro street sprinkling, one police patrol automobile, the police stations, the structures used as markets.

The provincial fiscal of Iloilo filed a motion with the court praying that the attachment be dissolved, being null and
void, since the properties are exempt from execution.

ISSUE:

Whether or not the said subject properties of the Municipality are exempt from execution.

RULING:

Yes, they are exempt from execution. The Civil Code divides the property of provinces and (municipalities) into
property for public use and patrimonial property. According to the Article 344 of the Civil Code (now Art. 424),
provincial roads and foot-path, squares, streets, fountains, and public waters, drives and public improvements of
general benefit built at the expense of the said towns or provinces, are property for public use. All other property
possessed by the said towns and provinces is patrimonial and shall be subject to the provision of the Civil Code
except as provided by special laws. The property for public use of the State is not within the commerce of man and,
consequently, is unalienable and not subject to prescription. Likewise, property for public use of the municipality is
not within the commerce of man so long as it is used by the public and, consequently, said property is also
inalienable. The movable and immovable property of a municipality, necessary for governmental purposes, may
not be attached and sold for the payment of a judgment against the municipality. The supreme reason for this rule
is the character of the public use to which such kind of property is devoted. The necessity for government service
justifies that the property of public use of the municipality be exempt from execution. Even the municipal income,
according to jurisprudence and authorities under the United States Supreme Court, is exempt from levy and
execution.

Zamboanga v Zamboanga, G.R. No. L-24440, June 30, 1969

Casimiro Development Corp. v. Mateo


G.R. No. 175485, 24 July 2011

If a person purchases a piece of land on the assurance that the seller’s title thereto is valid, he should not run the risk
of being told later that his acquisition was ineffectual after all, which will not only be unfair to him as the purchaser,
but will also erode public confidence in the system and will force land transactions to be attended by complicated and
not necessarily conclusive investigations and proof of ownership.

FACTS:

A Registered parcel of land in Las Pinas City was originally owned by Isaias Lara, respondents’ maternal
grandfather. Subsequently, when the land was passed on to his heirs, the latter effected the transfer of the full and
exclusive ownership of the land to Felicidad Lara-Mateo. A Deed of sale was then executed in favor of Laura, one of
Felicidad’s children, who applied for land registration wherein an OCT was thereafter issued. Laura used the
subject proprty as collateral to secure a succession of loans, one of which was to use such as security in an
obligation secured by a Real Estate Mortgage with China Bank. The latter failed in her obligation, prompting the
foreclosure of the lot in favor of China Bank. A TCT of was eventually issued in the name of China Bank. Casimiro
Development Corp. thereafter purchased the property from China Bank and in the meanwhile Felicidad died
intestate.
CDC brought action for unlawful detainer in the MeTC against the sons of Felicidad who remained in the property;
the latter claimed that MeTC did not have jurisdiction because the land was classified as agricultural; that the
jurisdiction belonged to the Department of Agrarian Reform Adjudication Board (DARAB); that they had been in
continuous and open possession of the land even before World War II and had presumed themselves entitled to a
government grant of the land; and that CDCs title was invalid, considering that the land had been registered before
its being declared alienable.

MeTC ruled in favor of CDC, RTC against, CA and SC in favor CDC. Upon finality of the decision before the Supreme
Court, Respondent brought action for quieting of title, RTC favored CDC but the CA ruled against CDC contending
that CDC was a not a buyer in good faith.

ISSUE:

Whether or not the land as a private land was properly acquired by CDC.

RULING:

YES. There is no doubt that the land in question, although once a part of the public domain, has already been placed
under the Torrens system of land registration before CDC became the registered owner by purchase from China
Bank; OCT has already been issued to attest to the fact that the person named in the certificate is the owner of the
property therein described, subject to liens and encumbrances as thereon noted or what the law warrants or
reserves. Neither the respondent nor his siblings opposed the transactions causing various transfers, and even
acknowledged the registration of the land under the name of Laura.

CDC was an innocent purchaser for value. Considering that China Bank’s TCT was clean title, that is, it was free
from any lien or encumbrance, CDC had the right to rely, when it purchased the property, solely upon the face of
the certificate of title in the name of China Bank.

Gatchalian v. Flores
G.R. No. 225176, Jan. 19, 2018

In Ejectment proceedings the only issue for the Court's resolution is, who between the parties is entitled to the physical
or material possession of the subject property. Issues as to ownership are not involved, except only for the purpose of
determining the issue of possession.
Absent any evidence of expropriation proceedings, or sale to the municipal government the property remains private
in nature.

FACTS:

The Gatchalians owned a piece of land land in Parañaque, whose TCT was named under their parents. In 2011 they
filed an ejectment suit against the Flores’ with the MeTC based on a survey conducted on their said lots. The
respondents denied the allegations and averred that it was the Gatchalians who usurped the said lot by building a
fence over the said property. More so a part of the said lot is a public road known as "Don Juan Street Cat-
Mendoza" constituted as a right of way for the GAT Mendoza Housing area and therefore owned by the City
Government of Parañaque based on Ordinance No. 88-04, series of 1988 constituting it as "Don Juan St. Gat-
Mendoza”. The MeTC favored the Gatchalians ordering the respondents to vacate the said property. The RTC
reverses the said decision. The CA overturns the RTC decision, and reinstates the MeTC ruling.

ISSUES:

Did the Gatchalians have the right to evict the respondents?


Does the municipal government own the said property?

RULING:
The court ruled yes on the first issue. It is settled that in ejectment proceedings, the only issue for the Court's
resolution is, who between the parties is entitled to the physical or material possession of the subject property.
Issues as to ownership are not involved, except only for the purpose of determining the issue of possession.

In the instant case, petitioner asserts that he is entitled to the possession of the road lot being one of the co-owners
of the same since it is registered under the name of petitioner's parents.

It is well-settled that an "owner of aregistered land does not lose his rights over a property on the ground of laches
as long as the opposing claimant's possession was merely tolerated by the owner."

A torrens title is irrevocable and its validity can only be challenged in a direct proceeding. A torrens title is an
indefeasible and imprescriptible title to a property in favor of the person in whose name the title appears. The
owner is entitled to all the attributes of ownership of the property, including possession. The person who has a
torrens title over a land is entitled to possession thereof. As such, petitioner can file an ejectment case against
herein respondents who encroached upon a portion of petitioner's property.

In the case of Abellana, Sr. v. Court of Appeals, the Court held that "the road lots in a private subdivision are private
property, hence, the local government should first acquire them by donation, purchase or expropriation, if they are
to be utilized as a public road." Otherwise, they remain to be private properties of the owner-developer.

Contrary to the position of petitioners, the use of the subdivision roads by the general public does not strip it of its
private character. The road is not converted into public property by mere tolerance of the subdivision owner of the
public's passage through it. To repeat, "the local government should first acquire them by donation, purchase or
expropriation, if they are to be utilized as a public road."

There is no such thing as an automatic cessation to the government of subdivision road lots, an actual transfer
must first be effected by the subdivision owner.

III. Essential Form


IV. Designation
V. Susceptibility to Substitution
VI. Aptitude for repeated use, NCC 418
VII. Susceptibility to Division
VIII. Existence in Time
IX. Dependence

Arriola v. Arriola
G.R. No. 177703, January 28, 2008

Even if the family home has passed by succession to the co-ownership of the heirs, or has been willed to any one of
them, this fact alone cannot transform the family home into an ordinary property, much less dispel the protection cast
upon it by the law.

FACTS:

Fidel Arriola who married twice died and is survived by his legal heirs: John Nabor Arriola (respondent), his son
with his first wife, and Vilma G. Arriola, his second wife and his other son, Anthony Ronald Arriola (petitioners).

On February 16, 2004, the RTC rendered a decision ordering the partition of the parcel of land left by the decedent
Fidel S. Arriola. However, the parties failed to agree on how to divide the property. The respondent proposed to
sell it through public auction. The petitioners initially agreed but refused to include in the auction, the house
standing on the subject land alleging that it is a family home.
The CA, citing Articles 440, 445 and 446 of the Civil Code held that as the deceased owned the subject land, he also
owned the subject house, which is a mere accessory to the land. Both properties form part of the estate of the
deceased and are held in co-ownership by his heirs, the parties herein.

ISSUE:
Whether the subject property may be part of the partition by public auction

RULING:

No.

Article 159 imposes the proscription against the immediate partition of the family home regardless of its
ownership. This signifies that even if the family home has passed by succession to the co-ownership of the heirs, or
has been willed to any one of them, this fact alone cannot transform the family home into an ordinary property,
much less dispel the protection cast upon it by the law.

Thus, for 10 years from said date or until March 10, 2013, or for a longer period, if there is still a minor beneficiary
residing therein, the family home he constituted cannot be partitioned, much less when no compelling reason
exists for the court to otherwise set aside the restriction and order the partition of the property.

BUNDLE OF RIGHTS

A. Ownership, NCC 427 - 439

Lunod v Meneses, G.R. No. 4223, August 19, 1908

NAPOCOR v. Ibrahim
G.R. No. 168732, June 29, 2007

In the past, the Court has held that if the government takes property without expropriation and devotes the property
to public use, after many years, the property owner may demand payment of just compensation in the event
restoration of possession is neither convenient nor feasible.

FACTS:

Respondent Lucban Ibrahim, in his personal capacity and in behalf of his co-heirs, instituted an action against the
petitioner for the recovery of possession of land and damages before the RTC of Lanao del Sur. They alleged that
they were co-owners of the parcels of land subject of the dispute and that NAPOCOR, through alleged stealth and
without respondent’s knowledge and prior consent, took possession of the sub-terrain area of their lands and
constructed therein underground tunnels. The tunnels were apparently being used by NAPOCOR in siphoning the
water of Lake Lanao and in the operation of NAPOCORs Agus II, III, IV, V, VI, VII projects located in Saguiran, Lanao
del Sur; Nangca and Balo-i in Lanao del Norte; and Ditucalan and Fuentes in Iligan City.

On October 7, 1992, respondents demanded that NAPOCOR pay damages and vacate the sub-terrain portion of
their lands but the same remained unheeded.
Petitioner NAPOCOR, controverted the same by alleging that the complaint states no cause of action because the
respondents were not even in the possession of the subject property. Further it averred that respondents failed to
show proof that they were really owners of the subject lands in question.

Petitioner maintains that sub terrain portion where the underground tunnels were constructed does not belong to
respondents because even conceding the fact that respondents owned the property, their right to subsoil of the
same does not extend beyond what is necessary to enable them to obtain all the utility and convenience that such
property can normally give. In any case, respondents were able to use the subject property even with the existence
of the tunnels, such as the fact of establishment of residence by respondent.
Both RTC and CA ruled that the sub terrain portions of the property similarly belongs to the respondents. Hence it
ordered petitioner the fair market value of the same.

ISSUE:

Whether the respondents were only entitled for the ownership of the sub terrain portions of lot but to the
exclusion of the subsoil portion?

HELD:

NO. The sub terrain portion of the property also belongs to the respondents.

ART. 437 of the New Civil Code provides “The owner of a parcel of land is the owner of its surface and of everything
under it, and he can construct thereon any works or make any plantations and excavations which he may deem
proper, without detriment to servitudes and subject to special laws and ordinances. He cannot complain of the
reasonable requirements of aerial navigation.”

In the past, the Court has held that if the government takes property without expropriation and devotes the
property to public use, after many years, the property owner may demand payment of just compensation in the
event restoration of possession is neither convenient nor feasible. This is in accordance with the principle that
persons shall not be deprived of their property except by competent authority and for public use and always upon
payment of just compensation.

Significantly, though, landowners cannot be deprived of their right over their land until expropriation proceedings
are instituted in court. The court must then see to it that the taking is for public use, that there is payment of just
compensation and that there is due process of law.

Notwithstanding the fact that petitioner only occupies the sub-terrain portion, it is liable to pay not merely an
easement fee but rather the full compensation for land. This is so because in this case, the nature of the easement
practically deprives the owners of its normal beneficial use. Respondents, as the owners of the property thus
expropriated, are entitled to a just compensation which should be neither more nor less, whenever it is possible to
make the assessment, than the money equivalent of said property

Aneco Realty v. Landex


560 SCRA 182 (2008)

Article 430 of the Civil Code gives every owner the right to enclose or fence his land or tenement by means of walls,
ditches, hedges or any other means. The right to fence flows from the right of ownership. As owner of the land, Landex
may fence his property subject only to the limitations and restrictions provided by law. Absent a clear legal and
enforceable right, as here, We will not interfere with the exercise of an essential attribute of ownership.

FACTS:

Fernandez Hermanos Development, Inc. (FHDI) is the original owner of a tract of land in San Francisco Del Monte,
Quezon City. FHDI subdivided the land into thirty-nine (39) lots. It later sold twenty-two (22) lots to petitioner
Aneco and the remaining seventeen (17) lots to respondent Landex. The dispute arose when Landex started the
construction of a concrete wall on one of its lots. To restrain construction of the wall, Aneco filed a complaint for
injunction with the RTC in Quezon City. Aneco later filed two (2) supplemental complaints seeking to demolish the
newly-built wall and to hold Landex liable for two million pesos in damages.

Landex filed its Answer alleging, among others, that Aneco was not deprived access to its lots due to the
construction of the concrete wall. Landex claimed that Aneco has its own entrance to its property along Miller
Street, Resthaven Street, and San Francisco del Monte Street. The Resthaven access, however, was rendered
inaccessible when Aneco constructed a building on said street. Landex also claimed that FHDI sold ordinary lots,
not subdivision lots, to Aneco based on the express stipulation in the deed of sale that FHDI was not interested in
pursuing its own subdivision project.

RTC dismissed the complaint for injunction which was later affirmed by the CA. CA held that Aneco knew that the
subject property ceased to be a road lot when its former owner sold it to Aneco not as subdivision lots and without
the intention of pursuing the subdivision project. Hence, this petition.

ISSUE:

Whether or not Aneco may enjoin Landex from constructing a concrete wall on its own property.

RULING:

NO. We agree with the RTC and the CA that the complaint for injunction against Landex should be dismissed for
lack of merit. What is involved here is an undue interference on the property rights of a landowner to build a
concrete wall on his own property. It is a simple case of a neighbor, petitioner Aneco, seeking to restrain a
landowner, respondent Landex, from fencing his own land.

Article 430 of the Civil Code gives every owner the right to enclose or fence his land or tenement by means of walls,
ditches, hedges or any other means. The right to fence flows from the right of ownership. As owner of the land,
Landex may fence his property subject only to the limitations and restrictions provided by law. Absent a clear legal
and enforceable right, as here, We will not interfere with the exercise of an essential attribute of ownership. Hence,
petition is denied.

Padilla v. Velasco, et al.


GR No. 169956, January 19, 2009

In accion publiciana, the principal issue is possession, and ownership is merely ancillary thereto. Only in cases where
the possession cannot be resolved without resolving the issue of ownership may the trial court delve into the claim of
ownership.

FACTS:

Respondents are the heirs of Dr. Artemio A. Velasco (Artemio), who acquired the subject lot. Respondents entered
the property as trustees by virtue of a deed of sale executed by the Rural Bank of Pagsanjan. Respondents
demanded that petitioners vacate the property, but the latter refused. Thereafter, petitioners performed acts of
dominion over the property. Respondents filed a complaint for accion publiciana, accounting and damages and the
brother of the deceased was presented as a witness. He presented the following: Kasulatan ng Bilihang Tuluyan
executed by spouses Sacluti in favor of Artemio, and declared that he was present during the signing of the
instrument, tax declarations and tax receipts covering Lot No. 2161 which were all in the name of Artemio and a
certification from the LRA. Petitioner’s witness, Trinidad, residing in the area since birth, said that the land
belonged to Nonong Velasco because he used to buy coconuts harvested from the said land and it was Nonong who
caused the gathering of such. A geodetic engineer testified that he conducted a survey of the land based on the
technical description of the property and the map from the Bureau of Lands. The purpose of the survey was to
verify if the area occupied by petitioners was Lot No. 2161, and he confirmed that it was.

ISSUES:

1. Who between the parties has a better right of possession?


2. Whether or not the complaint for accion publiciana has already prescribed.

RULING:

1. The respondents were able to establish lawful possession of Lot No. 2161 when the petitioners occupied
the same, it was the subject of a Decree based on the decision in Cad. Case No. 11, LRC Record No. 208. The Original
Certificate of Title to the land was issued to Sps. Sacluti. The original owners of the land sold the same to Artemio.
From the date of sale, until Artemios death he was in continuous possession of the land. When Artemio died, his
brother acted as administrator of the land with Tomas Vivero as caretaker. In 1987, petitioners occupied the
property by virtue of a deed of sale between the Rural Bank of Pagsanjan and the Solomon spouses. The land
bought by the latter from the Bank is denominated as Lot No. 76-pt and previously owned by Valeriano. However,
it was proved during trial that the land occupied by petitioners was Lot No. 2161 in the name of Artemio, whereas
the land sold by the bank to the petitioners was Lot No. 76-pt. It can readily be deduced that respondents are
legally entitled to the possession of Lot No. 2161. Petitioners put in issue that Lot No. 2161 and Lot 76-pt are the
same, and that the land was owned by Valeriano when it was foreclosed by the bank. This is a collateral attack on
the title over the property, a title to a registered land cannot be collaterally attacked.

2. The case filed by respondents for accion publiciana has not prescribed. The action was filed with the RTC
on October 14, 1991. Petitioners dispossessed respondents of the property in October 1987. At the time of the
filing of the complaint, only four (4) years had elapsed from the time of dispossession. Under Article 555(4) of the
Civil Code of the Philippines, the real right of possession is not lost till after the lapse of ten years. It is settled that
the remedy of accion publiciana prescribes after the lapse of ten years. Thus, the instant case was filed within the
allowable period.

Heirs of Domingo Valientes v. Ramas


G.R. No. 157852, December 15, 2010

Articles 1141, 1134 and 1137 of the Civil Code, however, are general rules on prescription which should give way to
the special statute on registered lands, Presidential Decree No. 1529, otherwise known as the Property Registration
Decree. Under the Torrens System as enshrined in P.D. No. 1529, the decree of registration and the certificate of title
issued become incontrovertible upon the expiration of one year from the date of entry of the decree of registration,
without prejudice to an action for damages against the applicant or any person responsible for the fraud.

FACTS:

Petitioners claim that they are the heirs of deceased Domingo Valientes who was the owner of a parcel of land
Zamboanga del Sur then covered by Original Certificate of Title (OCT) No. P-18,208 of the Register of Deeds of
Zamboanga del Sur. In 1939, Domingo Valientes mortgaged the subject property to secure his loan to the spouses
Belen. In the 1950s, the Valientes family failed to retrieve the subject property from the spouses Belen. Through an
allegedly forged document purporting to be a deed of sale of the subject property between Domingo Valientes and
the spouses Belen, the latter obtained Transfer Certificate of Title (TCT) No. T-5,427 in their name. On February 28,
1970, Maria Valientes Bucoy and Vicente Valientes, legitimate children of the late Domingo Valientes, had their
Affidavit of Adverse Claim duly entered in the Memorandum of Encumbrances at the back of TCT No. T-5,427.
Upon the death of the spouses Belen, their surviving heirs Brigida Sescon Belen and Maria Lina Belen executed an
extra-judicial settlement with partition and sale in favor of private respondent Vilma Valencia-Minor, the present
possessor of the subject property.

ISSUE:

Whether or not the action of reconveyance has already prescribed.

RULING:

Yes. With regard to the issue of prescription, this Court has ruled a number of times before that an action for
reconveyance of a parcel of land based on implied or constructive trust prescribes in ten years, the point of
reference being the date of registration of the deed or the date of the issuance of the certificate of title over the
property. But this rule applies only when the plaintiff is not in possession of the property, since if a person claiming
to be the owner thereof is in actual possession of the property, the right to seek reconveyance, which in effect seeks
to quiet title to the property, does not prescribe.
As discussed above, Civil Case No. 98-021 was filed more than 28 years from the issuance of TCT No. T-5,427. This
period is unreasonably long for a party seeking to enforce its right to file the appropriate case. Thus, petitioners’
claim that they had not slept on their rights is patently unconvincing.

Datu v. Lantud
G.R. No. 162551, July 18, 2011

The Torrens title is conclusive evidence with respect to the ownership of the land described therein, and other matters
which can be litigated and decided in land registration proceedings. Tax declarations and tax receipts cannot prevail
over a certificate of title which is an incontrovertible proof of ownership. An original certificate of title issued by the
Register of Deeds under an administrative proceeding is as indefeasible as a certificate of title issued under judicial
proceedings. However, the Court has ruled that indefeasibility of title does not attach to titles secured by fraud and
misrepresentation.

FACTS:

Respondent alleged in his Complaint that he is the owner of a parcel of residential lot located at Marinaut, Marawi
City, covered by OCT No. P-658. That petitioner, through his daughter and with several armed men, forcibly and
unlawfully entered his property and destroyed the nursery buildings, cabbage seedlings and other improvements.
Barangay Captain and his councilmen prepared and issued a decision in writing stating that Datu is the owner of
the subject parcel of land. Petitioner alleged that OCT No. P-658 was secured in violation of laws and through fraud,
deception and misrepresentation, considering that the subject parcel of land is a residential lot and the title issued
is a free patent. Moreover, respondent and his predecessors-in-interest had never taken actual possession or
occupied the land under litigation. Respondent had been residing on the lot for more than 30 years, applied for a
title thereto and was issued OCT No. P-658. He paid the corresponding real estate taxes for the land. Datu testified
that the land under litigation is only a portion of the 1,800 square meters of land that he inherited from his father.
Trial court rendered a Decision in favor of petitioner. CA rendered a Decision reversing the decision held that the
allegation of fraud in securing OCT No. P-658 on the ground that the property in dispute is a residential lot and not
subject of a free patent cannot be given weight as it was supported only by testimonial evidence that did not show
how (by metes and bounds) and why the property in dispute could not have been the subject of a free patent.

ISSUE:

Whether or not the Court of Appeals erred in sustaining the validity of OCT No. P-658 and confirming respondent
as owner of the property in dispute.

RULING:

NO. The trial court found that "the lot under litigation as clearly described in the complaint is a residential lot and a
free patent title thereto cannot validly be issued." This finding was one of the bases for the trial court's declaration
that the issuance of OCT was tainted with fraud and irregularities and is, therefore, spurious; thus, OCT No. P-658
is null and void. It should be pointed out that the allegation in the Complaint that the land is residential was made
only by respondent, but the true classification of the disputed land as residential was not shown to have been made
by the President, upon recommendation by the Secretary of Environment and Natural Resources, pursuant to
Section 9 of Commonwealth Act No. 141, otherwise known as The Public Land Act.

In an accion reinvindicatoria, the person who claims that he has a better right to the property must first fix the
identity of the land he is claiming by describing the location, area and boundaries thereof. In this case, petitioner
claims that the property in dispute is part of his larger property. However, petitioner failed to identify his larger
property by providing evidence of the metes and bounds thereof, so that the same may be compared with the
technical description contained in the title of respondent, which would have shown whether the disputed property
really formed part of petitioner's larger property.

In fine, the Court of Appeals did not err in confirming that respondent is the owner of the parcel of land covered by
OCT No. P-658.
Corpuz v. Agustin
G.R. No. 183822, Jan. 18, 2012

Where the parties to an ejectment case raise the issue of ownership, the courts may pass upon that issue to determine
who between the parties has the better right to possess the property. However, where the issue of ownership is
inseparably linked to that of possession, adjudication of the ownership issue is not final and binding, but only for the
purpose of resolving the issue of possession.

FACTS:

Ruben Corpuz filed a complaint for ejectment against Spouses Agustin on the allegation that he is the registered
owner of two parcels of land located in Santa Joaquina, Laoag City.

Ruben alleged that the aforesaid parcels of land were originally owned by his father who allowed the Spouses
Agustin to occupy the subject properties. Ruben later acquired the same parcels of land through a Deed of
Quitclaim executed by his father in his favor in March 1971. Despite demand to vacate, the Agustins refused to
leave the premises.

In their Answer, the Spouses Agustin interposed the defense that on June 5, 1971, Francisco Corpuz disposed of
subject property by executing a Deed of Absolute Sale in their favor for a consideration. The Deed of Sale however,
though notarized, was not registered.

ISSUE:

Who between the parties has the right to possession of the disputed properties - petitioner, who is the registered
owner; or respondents, who have a notarized yet unregistered Deed of Absolute Sale over the same properties

RULING:

The respondents have a better right of possession.

In ejectment proceedings, the courts resolve the basic question of who is entitled to physical possession of the
premises, possession referring to possession de facto, and not possession de jure. Where the parties to an
ejectment case raise the issue of ownership, the courts may pass upon that issue to determine who between the
parties has the better right to possess the property. However, where the issue of ownership is inseparably linked to
that of possession, adjudication of the ownership issue is not final and binding, but only for the purpose of
resolving the issue of possession.

Indeed, a title issued under the Torrens system is entitled to all the attributes of property ownership, which
necessarily includes possession. However, we cannot lose sight of the fact that the present petitioner has instituted
an unlawful detainer case against respondents. It is an established fact that for more than three decades, the latter
have been in continuous possession of the subject property, which, as such, is in the concept of ownership and
not by mere tolerance of petitioner's father. Under these circumstances, petitioner cannot simply oust
respondents from possession through the summary procedure of an ejectment proceeding.

In this case, petitioner has not proven that respondents' continued possession of the subject properties was by
mere tolerance of his father, except by a mere allegation thereof. In fact, petitioner has not established when
respondents' possession of the properties became unlawful - a requisite for a valid cause of action in an unlawful
detainer case.

Edralin v. Phil. Veterans Bank


G.R. No. 168523, 2011
The right to possess a property follows the right of ownership; consequently, it would be illogical to hold that a person
having ownership of a parcel of land is barred from seeking possession thereof.

FACTS:

Veterans Bank granted the Edralins)a loan and as security thereof, petitioners executed a Real Estate Mortgage
(REM) in favor of Veterans Bank over a real property registered in the name of Fernando Edralin. The Edralins
failed to pay their obligation to Veterans Bank. Thus, Veterans Bank filed a Petition for Extrajudicial Foreclosure.
Upon the Edralins failure to redeem the property during the one-year period, Veterans Bank acquired absolute
ownership of the subject property. Consequently, Veterans Bank caused the consolidation of ownership of the
subject property in its name.

Despite the foregoing, the Edralins failed to vacate and surrender possession of the subject property to Veterans
Bank. Veterans Bank filed an ex-parte petition for issuance of a writ of possession. The trial court found no merit in
the Veterans Banks application. It explained that, under paragraph (d) of the REM, the Veterans Bank agreed to
take possession of the Edralins property without any judicial intervention. The court held that granting the writ of
possession to the Veterans Bank will violate the contractual agreement of the parties. It also held that, assuming
the contract allowed for the issuance of a writ of possession, Veterans Banks right to seek possession had already
prescribed. Upon denial of a motion for reconsideration, Veterans Bank to file a Petition for Mandamus with Prayer
for Issuance of a Preliminary Mandatory Injunction before the CA, which was granted, thus this petition for review
by the Edralins.

ISSUE:
Is Veterans Bank entitled to a writ of possession?

HELD:

Petitioners argue that Veterans Bank is not entitled to a writ of possession because it failed to properly consolidate
its title over the subject property, and that the deed of sale issued by the bank in its own favor constitutes a pactum
commissorium. They also argue that the period for filing for a petition for the issuance of a writ of possession has
prescribed.

The elements of pactum commissorium, which enable the mortgagee to acquire ownership of the mortgaged
property without the need of any foreclosure proceedings, are: (1) there should be a property mortgaged by way of
security for the payment of the principal obligation, and (2) there should be a stipulation for automatic
appropriation by the creditor of the thing mortgaged in case of non-payment of the principal obligation within the
stipulated period. The second element is missing here. That Veterans Bank went through all the stages of
extrajudicial foreclosure indicates that there was no pactum commissorium.

In addition, the purchasers right to request for the issuance of the writ of possession of the land never prescribes,
as already established by jurisprudence. The right to possess a property merely follows the right of
ownership, and it would be illogical to hold that a person having ownership of a parcel of land is barred
from seeking possession thereof.

Ermitaño v. Paglas
G.R. No. 174436, January 23, 2013

It is settled that during the period of redemption, it cannot be said that the mortgagor is no longer the owner of the
foreclosed property, since the rule up to now is that the right of a purchaser at a foreclosure sale is merely inchoate
until after the period of redemption has expired without the right being exercised.

FACTS:

In November 1999, respondent and petitioner executed a Contract of Lease wherein petitioner leased in favor of
respondent a house and lot. The contract period is one year, with a monthly rental rate of ₱13,500. After the
execution of the lease contract, respondent learned that the subject property was mortgaged in favor of a certain
Charlie Yap (Yap) and that the same was already foreclosed with Yap as the purchaser in an extra-judicial
foreclosure sale. In June 2000, respondent bought the subject property from Yap and a Deed of Sale was executed
as evidence of the contract. However, it was made clear in the said Deed that the property was still subject to
petitioner's right of redemption. Prior to respondent's purchase of the subject property, petitioner filed a suit for
the declaration of nullity of the mortgage in favor of Yap as well as the sheriff's provisional certificate of sale.
Meanwhile, in May 2000, petitioner sent two letters demanding respondent to pay the rentals which are due and to
vacate the leased premises. Respondent ignored both letters. Thus, in August 2001, petitioner filed with the MTCC
a case of unlawful detainer against respondent.

ISSUE:

Whether respondent’s subsequent acquisition of ownership of the disputed property is a sufficient excuse from
refusing to pay the rentals due to petitioner.

RULING:

No. There is no dispute that at the time that respondent purchased Yap's rights over the subject property,
petitioner's right of redemption as a mortgagor has not yet expired. Under Act. No. 3135, the purchaser in a
foreclosure sale has, during the redemption period, only an inchoate right and not the absolute right to the
property with all the accompanying incidents. He only becomes an absolute owner of the property if it is not
redeemed during the redemption period. Act. No. 3135, as amended, allows the purchaser at the foreclosure sale to
take possession of the property only upon the filing of a bond, in an amount equivalent to the use of the property
for a period of 12 months, to indemnify the mortgagor in case it be shown that the sale was made in violation of the
mortgage or without complying with the requirements of the law.

In the instant case, there is neither evidence nor allegation that respondent filed a petition and bond in accordance
with the provisions of Section 7 of Act No. 3135. In addition, respondent defaulted in the payment of her rents.
Thus, absent respondent's filing of such petition and bond prior to the expiration of the period of redemption,
coupled with her failure to pay her rent, she did not have the right to possess the subject property. On the other
hand, petitioner, as mortgagor and owner, was entitled not only to the possession of the disputed house and lot but
also to the rents, earnings and income derived therefrom.

Heirs of Isip v. Quintos


G.R. No. 172008, Aug. 2012

Actual possession of land consists in the manifestation of acts of dominion over it of such a nature as those a party
would naturally exercise over his own property. It is not necessary that the owner of a parcel of land should himself
occupy the property as someone in his name may perform the act. In other words, the owner of real estate has
possession, either when he himself is physically in occupation of the property, or when another person who recognizes
his rights as owner is in such occupancy.

FACTS:

Petitioners filed with MeTC a complaint against forcible entry against respondents alleging that the latter
succeeded in entering the deep well compound and once inside the premises, prevented the petitioners from re-
entering the same through the use of force, intimidation, and threat. Petitioners alleged that since 1986, Isip, Sr.
was in possession of the subject parcel of land and allowed Toyo Keiki to build said deep well compound. They
aver that respondents deceived them to temporarily vacate the premises. Respondents denied the charge.
They asserted that Pontino formerly owned and occupied the disputed lot. That on 1984, he executed a Deed of
Absolute Sale of Rights in favor of Hadji Datu. However, Pontino rescinded the said contract of sale for failure to
pay. Despite the rescission, Hadji Datu sold the lot to Toyo Keiki, unaware of the said rescission. Subsequently
Pontino wrote a letter to Toyo Keiki informing the latter that Hadji Datu never became the owner of the subject lot.
Thereafter, a Deed of Assignment was executed by Pontino with Jedco Corp., thereby relinquishing the right of
possession over the subject premises in favor of De Guzman. MeTC dismissed the action; RTC affirmed MeTC
Decision; CA dismissed petitioners’ petition for review and affirmed RTC.

ISSUE:

Who has the better right of possession over the subject property.

RULING:

Respondents have prior possession de facto. While petitioners allege that Isip, Sr. was in possession of the subject
lot in 1986, evidence on record supports the respondents’ claim that as early as 1984, Pontino not only possessed
and occupied the lot but also had a title over the disputed property. And by virtue of a Deed of Assignment between
Pontino and Jedco Corp., which the latter relinquished in favor of De Guzman, respondents enjoy the right of prior
possession de facto. In addition, the possession of respondents was lawful from the beginning since it was acquired
through lawful means and thus no forcible entry was committed. Moreover, it is clear from the facts that when the
rights over the subject lot was relinquished in favor of De Guzman, Rogelio Sr. was employed in order to help the
respondents run the water distribution system. And although Rogelio Sr. was able to occupy the lot, he was in fact
possessing the same in the name of the respondents. Verily, whatever right to possess petitioners have in this case
cannot be superior to that of the respondents since it was from the latter that their predecessor-in-interest derived
his claim of possession.

Villondo v. Quijano
G.R No. 173606, December 3, 2012

Regardless of the actual condition of the title to the property, the party in peaceable quiet possession shall not be
thrown not by a strong hand, violence, or terror. Neither is the unlawful withholding of property allowed. Courts will
always uphold respect for prior possession.

FACTS:

Valeriana filed a case for forcible entry with preliminary mandatory injunction against Carmen Quijano and her
farm laborers claiming that on August 14, 1999, respondents intruded into her land with the help of three
policemen and other barangay officials. They destroyed the plants therein, harvested the root crops, corn, and
banana, built a hut, fenced off the area, and posted a "NO TRESPASSING" sign.

Valeriana argued that Carmen can never assert ownership over the property because it is a government land. She
claimed that Carmen's parents, Rufo and Constancia Bacalla, were themselves aware that an ownership claim is
worthless. Thus, they ceded their plantations on the subject land to her husband Daniel Villondo (Daniel) for
P2,000.00 as declared in a "Kasabutan".

On the other hand, Carmen interposed that the alleged "Kasabutan" was never brought to her attention by her
parents. In any case, she asserted that such allegation of Valeriana even supports her claim of prior possession.

The respondents also questioned Valeriana's legal personality to sue, contending that "Daniel T. Villondo," the
named tiller in the Certificate of Stewardship No. 146099, is the real party-in-interest and thus should be the
plaintiff in the suit and not Valeriana. They claimed that "Daniel T. Villondo" is actually Valeriana's son Romualdo
Villondo (Romualdo), a construction worker who had never even cultivated the subject land. Respondents refuted
Valeriana's claim that the named tiller in the Certificate refers to her husband "Daniel P. Villondo," who was
awarded by the government a Certificate of Stewardship over another parcel of land in 1983. Because of this, they
asserted that Valeriana is misleading the court by making it appear that she has successional rights from her
husband as steward. To support this, respondents submitted the respective stewardship applications as well as
other documents indicating that Daniel P. Villondo and Daniel T. Villondo are different persons. Notably, Regino's
Affidavit admits that Daniel T. Villondo refers to Romualdo.

The MTCC ruled in favor of Valeriana. The RTC Reversed the said Decision and the CA affirmed the RTC ruling.
ISSUE:

Whether Valeriana is a real party-in-interest in the forcible entry case she filed?

RULING:

YES. Sans the presence of the awardee of the Certificate of Stewardship, the provision clearly allows Valeriana to
institute the action for the recovery of the physical possession of the property against the alleged usurper. She has
a right or interest to protect as she was the one dispossessed and thus, she can file the action for forcible entry. Any
judgment rendered by the courts below in the forcible entry action will bind and definitely affect her claim to
possess the subject property. The fact that Valeriana is not the holder of the Certificate of Stewardship is not in
issue in a forcible entry case. This matter already delves into the character of her possession. We emphasize that in
ejectment suits, it does not even matter if the party's title to the property is questionable.

The MTCC correctly considered Valeriana as a real party-in-interest and correctly delved strictly with the issue of
physical possession. Notably, the CA, other than dismissing the case for lack of cause of action, did not seem to
dispute the MTCC's fascual finding of Valeriana's prior physical possession. Absent any evidence of respondents'
prior physical possession. Valeriana, who has cogently convinced us that she was dispossessed of the land by force,
is entitled to stay on the property until she is lawfully ejected by others who can prove in a separate proceeding
that they have a better right.

Top Management v. Fajardo


June 15, 2011

Quieting of title is a common law remedy for the removal of any cloud, doubt, or uncertainty affecting title to real
property. In an action for quieting of title, the plaintiffs must show not only that there is a cloud or contrary interest
over the subject real property, but that they have a valid title to it.

In Degollacion v. Register of Deeds of Cavite, SC held that if two certificates of title purport to include the same land,
whether wholly or partly, the better approach is to trace the original certificates from which the certificates of title
were derived.

FACTS:

In 1964, Emilio Gregorio (Gregorio) filed an application for registration of title over Lots at Las Pias, Rizal, before
the CFI of Rizal. CFI issued an order declaring as abandoned the reserved oppositions of Jose T. Velasquez and
Pablo Velasquez. Thereafter, the case proceeded to trial. Meanwhile, in 1965, Velasquez also filed an application for
registration of title over six lots situated at Almanza, Las Pias, Rizal, before the same court.

In January 1966, the CFI rendered a decision declaring Gregorio to be the absolute owner of the lots. An order was
issued by said court for the issuance of the decree of registration, since the decision had become final. On March
1966, the same court promulgated a decision adjudicating Lots to Velasquez. Said court ordered the issuance of a
decree of registration in view of the finality of the decision.

In the meantime, the LRA called the attention of the Director of Lands regarding the overlapping of Lots awarded
to Velasquez, with adjudicated to Gregorio, and requested that portions of these lots that are not in conflict be
segregated. Velasquez petitioned the CFI to set aside the award earlier made in favor of Gregorio on the ground of
lack of jurisdiction and to give due course to his application over the said lots.

CFI issued an Order in declaring that the application of Velasquez be given due course insofar as Lots which are
identical to the other lots and the decision in favor of Gregorio respecting the same lots as null and void.
Corresponding certificates were issued in favor of Velasquez. Gregorio appealed to the CA. The CA reversed CFI
Decision.
Meanwhile, on appeal, Gregorio entered into an agreement with Tomas Trinidad (Trinidad) and Luis Fajardo
(Fajardo) entitled Kasunduan na may Pambihirang Kapangyarihan. By virtue of this agreement, Fajardo would
finance the cost of the litigation and in return he would be entitled to one-half of the subject property if the appeal
is successful.

After the CA rendered a favorable ruling, Fajardo and Trinidad filed Civil Case before the RTC of Pasig to enforce
their agreement with Gregorio. RTC rendered in their favor. However, the writ issued remained unsatisfied. Deputy
Sheriff Estrellado executed the Officers Deed of Conveyance, however, the Register of Deeds of Las Pias said that it
could not be pursued because the subject property was already sold to other parties. CFI reinstated that the RD
annotate the said Deed of Conveyance.

Petitioner Top Management Programs Corporation sought the annulment of the CFI orders in reinstating the order
and directing the issuance of new certificates of title in the name of Trinidad and Fajardo, on the ground of extrinsic
fraud. CA rendered its decision dismissing the petition for annulment.

Petitioner filed before the RTC of Makati Quieting of Title With Damages. Petitioner alleged that the issuance of
TCT in the name of Fajardo -- who obtained the same from the court in a case without the knowledge of petitioner
who was not a party therein -- despite the existence of another TCT in its name constitutes a cloud upon the title of
petitioner. Petitioner claimed that it acquired the same property in good faith and for value from the original
owners thereof.

Petitioner further claims that it is a buyer in good faith who had no knowledge of any defect in the title of his
predecessor-in-interest. It paid the purchase price and acquired its title long before it discovered the right to
compensation of private respondent through the Officers Deed of Conveyance.

Private respondent counters that petitioners assertion of the existence of clerical errors in the annotations of the
entries is, at the very least, an admission that said title is indeed defective. Obviously, petitioner may not file a
petition to quiet its title and at the same time seek, in the same proceeding, the corrections of the entries therein.

ISSUE:

WON the Quieting of Title be allowed

RULING:

No. Quieting of title is a common law remedy for the removal of any cloud, doubt, or uncertainty affecting title to
real property. In an action for quieting of title, the plaintiffs must show not only that there is a cloud or contrary
interest over the subject real property, but that they have a valid title to it.
In Degollacion v. Register of Deeds of Cavite, SC held that if two certificates of title purport to include the same land,
whether wholly or partly, the better approach is to trace the original certificates from which the certificates of title
were derived.

The TCT in the name of the heirs of Emilio Gregorio issued on April 29, 1986, on its face showed badges of
irregularity in its issuance. First, there was inconsistency in the technical description stated for the covered
portion. Second, the decree number and date of issuance, as well as OCT number clearly indicate that the original
decree pertained to Velasquez and not Gregorio. Third, the name of the registered owner in the original certificate
is not Velasquez or Gregorio but “Delta Motor Corp.” And fourth, the certificate from which TCT No. 107729 was
supposedly a transfer should have been the OCT (of Gregorio) and not those unfamiliar TCT numbers indicated
therein.

The foregoing errors are not mere typographical as petitioner claims, but serious discrepancies in the registration
process. In fact, it is not far-fetched that the erroneous entries could have been intended to create the impression
that TCT No. 107729 was a separate and distinct title from the previously issued TCT No. S-91911 even if they
pertain to one and the same lot adjudicated to Emilio Gregorio. Such conclusion is reinforced by the unexplained
inaction or failure of the heirs of Gregorio to rectify the alleged errors in their title before selling the property to
petitioner. The heirs of Gregorio knew that their TCT No. S-91911 bore encumbrances in favor of third parties,
notably the notice of pending litigation (Lis Pendens) involving the property covered by said title before the CFI of
Pasig, Metro Manila in Civil Case No. 35305, which Trinidad caused to be annotated thereon. The issuance of a new
certificate with exactly identical entries as that of TCT No. S-91911 (as to its original registration) would mean that
the aforesaid annotations had to be carried over to such new certificate.

Petitioner being a mere transferee at the time the decision of the RTC of Pasig in Civil Case No. 35305 had become
final and executory on December 6, 1988, it is bound by the said judgment which ordered the heirs of Emilio
Gregorio to convey Lots 1, 2, 3 & 4, Psu-204875 in favor of private respondent and Trinidad. As such buyer of one
of the lots to be conveyed to private respondent pursuant to the court’s decree with notice that said properties are
in litigation, petitioner merely stepped into the shoes of its vendors who lost in the case. Such vested right
acquired by the private respondent under the final judgment in his favor may not be defeated by the subsequent
issuance of another certificate of title to the heirs of Gregorio respecting the same parcel of land. For it is well-
settled that being an involuntary transaction, entry of the notice of lis pendens in the primary entry book of the
Register of Deeds is sufficient to constitute registration and such entry is notice to all persons of such claim.

Mananquil v. Moico
G.R. No. 180076, Nov, 21, 2012

FACTS:

Lots 18 and 19 in Dagat-Dagatan, Navotas form part of the land previously expropriated by the National Housing
Authority (NHA) and placed under its Tondo Dagat-Dagatan Foreshore Development Project – where occupants,
applicants or beneficiaries may purchase lots on installment basis. In October 1984, Lot 18 was awarded
to spouses Iluminardo and Prescilla Mananquil under a Conditional Contract to Sell. Lot 19, on the other hand,
was sold to Prescilla in February 1980 by its occupant. When the spouses died, the heirs of Illuminardo filed for
extrajudicial settlement of estates of the spouses over lots 18 and 19. They appropriated to themselves the
properties by leasing it. However, herein respondent Moico bought the properties from one Eulogio who is the
alleged child of Prescilla from her first marriage who in turn evicted the tenants of the herein petitioners. Upon
finding out of the same, the Mananquils filed for an action to quiet the title against herein Moico claiming title over
the said properties.

ISSUE:

Whether or not petitioners have legal title over the subject lots entitling for the relief of quieting of title.

HELD:

No. An action for quieting of title is essentially a common law remedy grounded on equity.

The competent court is tasked to determine the respective rights of the complainant and other claimants, not only
to place things in their proper place, to make the one who has no rights to said immovable respect and not disturb
the other, but also for the benefit of both, so that he who has the right would see every cloud of doubt over the
property dissipated, and he could afterwards without fear introduce the improvements he may desire, to use, and
even to abuse the property as he deems best. But “for an action to quiet title to prosper, two
indispensable requisites must concur, namely:

(1) the plaintiff or complainant has a legal or an equitable title to or interest in the real property subject of the
action; and
(2) the deed, claim, encumbrance, or proceeding claimed to be casting cloud on his title must be shown to be in
fact invalid or inoperative despite its prima facie appearance of validity or legal efficacy.”

Contrary to petitioners’ stand, the issue relating to the grant of rights, title or award by the NHA determines
whether the case for quieting of title may be maintained. If the petitioners are legitimate successors to or
beneficiaries of Iluminardo upon his death – under the certificate of title, award, or grant, or under the special law
or specific terms of the NHA program/project – then they possess the requisite interest to maintain suit; if not,
then Civil Case No. 2741-MN must necessarily be dismissed.

From the evidence adduced below, it appears that the petitioners have failed to show their qualifications or right
to succeed Iluminardo in his rights under the NHA program/project. They failed to present any title, award, grant,
document or certification from the NHA or proper government agency which would show that Iluminardo and
Prescilla have become the registered owners/beneficiaries/ awardees of Lots 18 and 19, or that petitioners are
qualified successors or beneficiaries under the Dagat-Dagatan program/project, taking over Iluminardo’s rights
after his death. They did not call to the witness stand competent witnesses from the NHA who can attest to their
rights as successors to or beneficiaries of Lots 18 and 19. They failed to present proof, at the very least, of the
specific law, provisions, or terms that govern the Tondo Dagat-Dagatan Foreshore Development Project which
would indicate a modicum of interest on their part. For this reason, their rights or interest in the property could
not be established.

Chung v. Mondragon
G.R. No. 179754, November 21, 2012

FACTS:

Petitioners Joaquin G. Chung, Jr., Paz Royeras-Soler, and Mansueto Maceda are descendants of Rafael Mondragon
(Rafael) by his first wife, Eleuteria Calunia (Eleuteria), while respondent Jack Daniel Mondragon (Jack Daniel) is
Rafaels descendant by his second wife, Andrea Baldos (Andrea). OCT No. 22447 is registered in the name of "Heirs
of Andrea Baldos represented by Teofila G. Maceda" and covers 16,177 square meters of land in Macrohon,
Southern Leyte (the land). Chung, et al. claim that from 1921 up to 2000, Rafael appeared as owner of the land in
its tax declaration, and that a free patent was issued in 1987 in the name of Andreas heirs upon application of
Teofila G.Maceda (Teofila), who is petitioners sister. On the other hand, respondents Bourbon, et al. claim that
Andrea is the exclusive owner of the land, having inherited the same from her father and that after Andrea died, his
son Fortunato Mondragon inherited the land; and when the latter died, his son Jack Daniel (herein respondent)
came into possession and enjoyment thereof. Sometime in the year 2000, Jack Daniel sold a 1,500-square meter
portion of the land to his co-respondent Clarinda Regis-Schmitz (Regis-Schmitz). On the claim that Jack Daniel had
no right to sell a portion of the land and that the sale to Regis-Schmitz created a cloud upon their title, Chung, Jr., et
al. filed an action to quiet title. The RTC dismissed the complaint of Chung, Jr., et al. The CA sustained the trial court.

ISSUE:

Whether or not the action to quiet title should prosper?

HELD:

The petition lacks merit. The issues in a case for quieting of title are fairly simple; the plaintiff need to prove only
two things, namely: "(1) the plaintiff or complainant has a legal or an equitable title to or interest in the real
property subject of the action; and (2) that the deed, claim, encumbrance or proceeding claimed to be casting a
cloud on his title must be shown to be in fact invalid or inoperative despite its prima facie appearance of validity or
legal efficacy. It is evident from the title that the land belongs to no other than the heirs of Andrea Baldos, Rafaels
second wife. The land could not have belonged to Rafael, because he is not even named in OCT No. 22447.With
greater reason may it be said that the land could not belong to petitioners, who are Rafaels children by his first
wife Eleuteria. Unless Eleuteria and Andrea were related by blood such fact is not borne out by the record they
could not be heirs to each other. Add to this is the fact that petitioners are not in possession of the land. Petitioners
do not possess legal or equitable title to the land. Petition is DENIED.

Paraguya v. Crucillo
G.R. No. 200265, December 2, 2013
It is an established rule that a Torrens certificate of title is conclusive proof of ownership. Verily, a party may seek its
annulment on the basis of fraud or misrepresentation. However, such action must be seasonably filed, else the same
would be barred.

FACTS:

On December 19, 1990, Paraguya filed before the RTC a Complaint against Sps. Crucillo and the RD for the
annulment of OCT No. P-17729 alleging that Escurel obtained the title through fraud and deceit. Paraguya claimed
that she is the lawful heir to the subject properties left by her grandfather, the late Ildefonso Estabillo. Sps. Crucillo
filed their answer with motion to dismiss, averring that Paraguya’s complaint had already been barred by
prescription.

Paraguya testified as to how she came about owning the subject properties, presenting a titulo posesorio issued in
the name of Estabillo. For their part, Sps. Crucillo presented several witnesses who testified that Escurel had been
in possession of the subject properties in the concept of an owner as early as 1957.

The RTC granted Paraguya’s complaint, ordering the annulment of the subject title However, the CA reversed the
RTC’s ruling and ordered the dismissal of Paraguya’s complaint.

ISSUE:

WON the CA correctly dismissed Paraguya’s complaint for annulment of title.

RULING:

Yes. The Court sustained the CA’s dismissal of the complaint since it was more than eleven (11) years from the
title’s date of entry on August 24, 1979.

A Torrens certificate of title is conclusive proof of ownership. A party may seek its annulment on the basis of fraud.
However the period to contest a decree of registration shall be one (1) year from the date of its entry and that, after
the lapse of the said period, the Torrens certificate of title issued thereon becomes incontrovertible and
indefeasible.

The Court noted that Paraguya’s complaint is likewise in the nature of an action for reconveyance. Despite this,
Paraguya’s complaint remains dismissible on the same ground because the prescriptive period for actions for
reconveyance is ten (10) years reckoned from the date of issuance of the certificate of title, except when the owner
is in possession of the property in which case the action for reconveyance becomes imprescriptible.

The Court pointed out that Paraguya’s complaint should be dismissed altogether since she merely relied on
the titulo posesorio. Based on PD 892, entitled "Discontinuance of the Spanish Mortgage System of Registration and
of the Use of Spanish Titles as Evidence in Land Registration Proceedings," Spanish titles can no longer be used as
evidence of ownership after six (6) months from the effectivity of the law, or starting August 16, 1976.

Okabe v. Saturnino
G.R. No. 196040, August 26, 2014

It is thus settled that the buyer in a foreclosure sale becomes the absolute owner of the property purchased if it is not
redeemed during the period of one year after the registration of the sale. As such, he is entitled to the possession of the
said property and can demand it at any time following the consolidation of ownership in his name and the issuance to
him of a new transfer certificate of title.

FACTS:

The subject of the controversy is an 81 square meter property located in Makati City, which was initially covered
by TCT No. 175741 under the name of the wife of respondent Ernesto A. Saturnino. Sometime in 1994, the couple
obtained a loan with the Philippine National Bank (PNB), which was secured by the subject property. Because of
the couple’s failure to settle their loan obligation with the bank, PNB extrajudicially foreclosed the mortgage.

On August 24, 1999, the Certificate of Sale was inscribed on TCT No. 175741. Considering that the property was
not redeemed by respondent during the redemption period, consolidation of ownership was inscribed on October
13, 2006 and a new TCT was issued in favor of PNB. Without taking possession of the subject property, PNB sold
the land to petitioner Fe H. Okabe on June 17, 2008. TCT No. 225265 was later issued in petitioner’s name on
August 13, 2008.

On November 27, 2008, petitioner filed with the Regional Trial Court (RTC) of Makati City an Ex-Parte Petition for
Issuance of Writ of Possession over the subject property.

The RTC ruled, among other things, that the right of the petitioner to be placed in absolute possession of the
subject property was a consequence of her right of ownership and that petitioner cannot be deprived of said
possession being now the registered owner of the property. Meanwhile, on November 23, 2009, the RTC rendered
a Decision in favor of petitioner, which granted her ex-parte petition and ordered that the corresponding writ of
possession over the subject property be issued in her favor.

The CA reversed RTC’s decision and opined, among other things, that although it may be true that by virtue of the
contract of sale, petitioner obtained the same rights of a purchaser-owner and which rights she derived from
erstwhile mortgagee turned owner PNB, this does not mean that the right to file an ex-parte motion for a writ of
possession under Act 3135 had also been transferred to the petitioner. Such a special right is granted only to
purchasers in a sale made under the provisions of Act 3135.

ISSUE:

Whether or not, in the case at bar, an ex parte petition for the issuance of a writ of possession was the proper
remedy of the petitioner in obtaining possession of the subject property.

RULING:

No. Section 7 of Act No. 3135,28 as amended by Act No. 4118,29 states:

Section 7. In any sale made under the provisions of this Act, the purchaser may petition the Court of First Instance
of the province or place where the property or any part thereof is situated, to give him possession thereof during
the redemption period, furnishing bond in an amount equivalent to the use of the property for a period of twelve
months, to indemnify the debtor in case it be shown that the sale was made without violating the mortgage or
without complying with the requirements of this Act. Such petition shall be made under oath and filed in the form
of an ex parte motion x x x and the court shall, upon approval of the bond, order that a writ of possession issue,
addressed to the sheriff of the province in which the property is situated, who shall execute said order
immediately.

Under the provision cited above, the purchaser or the mortgagee who is also the purchaser in the foreclosure
sale may apply for a writ of possession during the redemption period, upon an ex-parte motion and after
furnishing a bond.

In GC Dalton Industries, Inc. v. Equitable PCI Bank the Court held that the issuance of a writ of possession to a
purchaser in an extrajudicial foreclosure is summary and ministerial in nature as such proceeding is merely an
incident in the transfer of title.

In China Banking Corporation v. Ordinario, the Court held that under Section 7 of Act No. 3135, the purchaser in a
foreclosure sale is entitled to possession of the property.
The Court ruled in Spouses Nicasio Marquez and Anita Marquez v. Spouses Carlito Alindog and Carmen Alindog
that although the Court allowed the purchaser in a foreclosure sale to demand possession of the land during the
redemption period, it still required the posting of a bond under Section 7 of Act No. 3135.

It is thus settled that the buyer in a foreclosure sale becomes the absolute owner of the property purchased if it is
not redeemed during the period of one year after the registration of the sale. As such, he is entitled to the
possession of the said property and can demand it at any time following the consolidation of ownership in his
name and the issuance to him of a new transfer certificate of title.

Jose Yulo Agricultural Corporation v. Sps. Davis


G.R. No. 197709, August 03, 2015

The general rule is that in the case of two certificates of title, purporting to include the same land, the earlier in date
prevails. Successive registrations, where more than one certificate is issued in respect of a particular estate or interest
in land, the person claiming under the prior certificate is entitled to the estate or interest; and that person is deemed
to hold under the prior certificate who is the holder of, or whose claim is derived directly or indirectly from the person
who was the holder of the earliest certificate issued in respect thereof.

FACTS:

A large tract of land, Lot 62-A, was registered in the name of Jose Yulo. It was later subdivided into several smaller
lots. One of the subdivided lots was subdivided further and the resulting lots were either sold or mortgaged by Jose
Yulo. Five of these lots were mortgaged to Nation Bank, which became the owner of the same after foreclosing the
mortgage. The bank sold the lots to Spouses Davis, along with the improvements thereon, namely a house and
fences.

The owners of adjacent lots, spouses Trajera, demanded that the spouses remove the said improvements as it
encroached on their property. The Local Building Office of Negros Occidental also threatened to impose sanctions
on the respondents. Spouses Davis filed a case with the RTC and won. CA affirmed. Both stating that Spouses Davis
were builders in good faith.

ISSUE:

Whether or not the Court of Appeals erred.

RULING: NO.

On the contrary, We are in agreement with the appellate court's pronouncement that respondents' title must be
upheld over that of the petitioner's as it is derived from titles that were issued earlier - in 1971, as opposed to 1979
with respect to petitioner's and the Trajeras' properties.

In this jurisdiction, it is settled that the general rule is that in the case of two certificates of title, purporting to
include the same land, the earlier in date prevails. Successive registrations, where more than one certificate is
issued in respect of a particular estate or interest in land, the person claiming under the prior certificate is entitled
to the estate or interest; and that person is deemed to hold under the prior certificate who is the holder of, or
whose claim is derived directly or indirectly from the person who was the holder of the earliest certificate issued in
respect thereof.

B. Right of Accession, NCC 440 – 474

Bachrach v. Siefert
87 Phil. 117

Pennsylvania rule is more in accord with our statutory laws than the Massachusetts rule. xxx This rule declares that all
earnings of the corporation made prior to the death of the testator stockholder belong to the corpus of the estate, and
that all earnings, when declared as dividends in whatever form, made during the lifetime of the usufructuary or life
tenant.

FACTS:

The deceased E. M. Bachrach, in his will bequeath and devise to Mary McDonald Bachrach all the fruits and usufruct
of the remainder of all his estate after payment of the legacies, bequests, and gifts. The estate of E. M. Bachrach, as
owner of 108,000 shares of stock of the Atok-Big Wedge Mining Co., Inc., received from the latter 54,000 shares
representing 50 per cent stock dividend on the said 108,000 shares. On June 10, 1948, Mary McDonald Bachrach,
as usufructuary or life tenant of the estate, petitioned the lower court to authorize the Peoples Bank and Trust
Company as administrator of the estate of E. M. Bachrach, to her the said 54,000 share of stock dividend by
endorsing and delivering to her the corresponding certificate of stock, claiming that said dividend, although paid
out in the form of stock, is fruit or income and therefore belonged to her as usufructuary or life tenant. Sophie
Siefert and Elisa Elianoff, legal heirs of the deceased, opposed said petition on the ground that the stock dividend in
question was not income but formed part of the capital and therefore belonged not to the usufructuary but to the
remainderman.

While appellants admits that a cash dividend is an income, they contend that a stock dividend is not, but merely
represents an addition to the invested capital. The so-called Massachusetts rule, supports appellants' contention . It
regards cash dividends, however large, as income, and stock dividends, however made, as capital. (Minot vs. Paine,
99 Mass., 101; 96 Am. Dec., 705.) It holds that a stock dividend is not in any true sense any true sense any dividend
at all since it involves no division or severance from the corporate assets of the dividend; that it does not distribute
property but simply dilutes the shares as they existed before; and that it takes nothing from the property of the
corporation, and nothing to the interests of the shareholders.

On the other hand, so called Pennsylvania rule, which prevails in various other jurisdictions in the United States,
supports appellee's contention. This rule declares that all earnings of the corporation made prior to the death of
the testator stockholder belong to the corpus of the estate, and that all earnings, when declared as dividends in
whatever form, made during the lifetime of the usufructuary or life tenant. (Earp's Appeal, 28 Pa., 368.)

ISSUE:

Is a stock dividend fruit or income, which belongs to the usufructuary, or is it capital or part of the corpus of the
estate, which pertains to the remainderman?

RULING:

The usufructuary or life tenant Mary Bachrach is entitled to the shares. Said dividend although paid out in the form
of stock, is fruit or income and therefore belonged to her as usufructuary. Pennsylvania rule is more in accord with
our statutory laws than the Massachusetts rule. Under section 16 of our Corporation Law, no corporation may
make or declare any dividend except from the surplus profits arising from its business. Any dividend, therefore,
whether cash or stock, represents surplus profits. Article 471 of the Civil Code provides that the usufructuary shall
be entitled to receive all the natural, industrial, and civil fruits of the property in usufruct. And articles 474 and 475
provide as follows:

ART. 474. Civil fruits are deemed to accrue day by day, and belong to the usufructuary in proportion to the time
the usufruct may last.
ART. 475. When a usufruct is created on the right to receive an income or periodical revenue, either in money
or fruits, or the interest on bonds or securities payable to bearer, each matured payment shall be considered as
the proceeds or fruits such right.
When it consists of the enjoyment of the benefits arising from an interest in an industrial or commercial
enterprise, the profits of which are not distributed at fixed periods, such profits shall have the same
consideration.lawphil.net
In either case they shall be distributed as civil fruits, and shall be applied in accordance with the rules
prescribed by the next preceding article.

he 108,000 shares of stock are part of the property in usufruct. The 54,000 shares of stock dividend are civil fruits
of the original investment. They represent profits, and the delivery of the certificate of stock covering said dividend
is equivalent to the payment of said profits. Said shares may be sold independently of the original shares, just as
the offspring of a domestic animal may be sold independently of its mother.

Bachrach v. Talisay-Silay
56 Phil. 117 (1931)

The bonus is not based upon the value, importance or any other circumstance of the mortgaged property, but upon the
total value of the debt thereby secured, according to the annual balance, which is distinct from and independent of the
property referred to. As such, it is not the civil fruits as contemplated under the law.

FACTS:

In December 1923, the Talisay-Silay Milling incurred indebtedness from the Philippine National Bank (PNB) which
the former secured using the land of its planters. Talisay-Silay managed to induce its planters, one of whom was
Mariano Ledesma, to mortgage their land in exchange of a compensation embodied in a resolution passed by the
milling company. It undertook to credit the planters-owners of the mortgaged land every year with a sum equal to
2% of the debt secured according to yearly balance, the payment of the bonus being made at once, or in part from
time to time, as soon as the central became free of its obligations to the aforesaid bank, and of those contracted by
virtue of the contract of supervision, and had funds which might be so used, or as soon as it obtained from said
bank authority to make such payment.

Bachrach filed a complaint against Talisay-Silay for the delivery of the amount of P13,850 or promissory notes or
credit for that sum payable on June 30, 1930, as bonus in favour of Mariano Ledesma. PNB filed a third-party claim
alleging that it has preferential right to receive any amount Mariano Ledesma may be entitled to receive from
Talisay-Silay since it forms part of the civil fruits of the land mortgaged by Mariano Ledesma.

ISSUE:

Whether the bonus being claimed is civil fruits.

RULING:

No, the subject bonus bears no immediate, but only a remote accidental relation to the land mentioned, having
been granted as compensation for the risk of having subjected one's land to a lien in favor of the bank, for the
benefit of the entity granting said bonus. Said bonus rose out of Mariano’s risk in mortgaging his land, thus it is not
civil fruits or income from the mortgaged property itself. Hence, the amount of the bonus, according to the
resolution of the central granting it, is not based upon the value, importance or any other circumstance of the
mortgaged property, but upon the total value of the debt thereby secured, according to the annual balance, which is
distinct from and independent of the property referred to. Philippine National Bank has no right to claim and is not
entitled to such bonus for it is not the kind of income which forms part of civil fruits.

Depra v. Dumlao
G.R. No. L-57348 May 16, 1985

The owner of the land the option to acquire the improvements after payment of the proper indemnity, or to oblige the
builder or planter to pay for the land and the sower to pay for the proper rent. It is the owner of the land who is
authorized to exercise the option, because his right is older, and because, by the principle of accession, he is entitled to
the ownership of the accessory thing.

FACTS:
Depra and Dumlao were neighbors. Sometime in 1972, DUMLAO constructed his house on his lot, the kitchen
thereof had encroached on an area of thirty four square meters of DEPRA's property, After the encroachment was
discovered in a relocation survey of DEPRA's lot, his mother, after writing a demand letter asking DUMLAO to
move back from his encroachment, filed an action for Unlawful Detainer against DUMLAO. MTC found that
DUMLAO was a builder in good faith. Neither party appeal so that it would have ordinarily lapsed into finality, but
even then, DEPRA did not accept payment of rentals so that DUMLAO deposited such rentals with the Municipal
Court.

Almost a year later, DEPRA filed a Complaint for Quieting of Title against DUMLAO before the CFI, involving the
very same 34 square meters. DUMLAO, in his Answer, admitted the encroachment but alleged, in the main, that the
present suit is barred by res judicata by virtue of the Decision of the Municipal Court, which had become final and
executory.

ISSUE:

Whether or not the land owner has the right to exercise the option despite the fact that the BPS is also in good faith.

RULING:

Yes. According to Art. 448, DEPRA has the option either to pay for the encroaching part of DUMLAO's kitchen, or to
sell the encroached 34 square meters of his lot to DUMLAO. He cannot refuse to pay for the encroaching part of the
building, and to sell the encroached part of his land, as he had manifested before the Municipal Court. But that
manifestation is not binding because it was made in a void proceeding.

The owner of the building erected in good faith on a land owned by another, is entitled to retain the possession of
the land until he is paid the value of his building, under Article 546. The owner of the land, upon the other hand,
has the option, under Article 448, either to pay for the building or to sell his land to the owner of the building. But
he cannot refuse both to pay for the building and to sell the land and compel the owner of the building to remove it
from the land where it erected. He is entitled to such removal only when, after having chosen to sell his land the
other party fails to pay for the same.

Where the builder, planter or sower has acted in good faith, a conflict of rights arises between the owners, and it
becomes necessary to protect the owner of the improvements without causing injustice to the owner of the land. In
view of the impracticability of creating a state of forced co-ownership, the law has provided a just solution by
giving the owner of the land the option to acquire the improvements after payment of the proper indemnity, or to
oblige the builder or planter to pay for the land and the sower to pay for the proper rent. It is the owner of the land
who is authorized to exercise the option, because his right is older, and because, by the principle of accession, he is
entitled to the ownership of the accessory thing.

Tuatis v. Spouses Escol


G.R. No. 175399 October 27, 2009

Where the builder, planter or sower has acted in good faith, a conflict of rights arises between the owners, and it
becomes necessary to protect the owner of the improvements without causing injustice to the owner of the land. In
view of the impracticability of creating a state of forced co-ownership, the law has provided a just solution by giving
the owner of the land the option to acquire the improvements after payment of the proper indemnity, or to oblige the
builder or planter to pay for the land and the sower the proper rent. He cannot refuse to exercise either option. It is the
owner of the land who is authorized to exercise the option, because his right is older, and because, by the principle of
accession, he is entitled to the ownership of the accessory thing.

FACTS:

Visminda Escol, the seller and Ophelia Tuatis, the buyer entered into a Deed of Sale by Installments, the subject
matter of which is a parcel of land in Sindangan. It provided that upon the failure of the buyer to pay the remaining
balance within the time stipulated, he shall return the land to the seller, and the seller shall return all the amounts
paid by the buyer. Tuatis took possession of the land and constructed a residential building. Tuatis asserted that
she paid Visminda the remaining balance of P3000 in the presence of one Erik Selda and thereafter requested
Visminda to sign the absolute deed of sale. Visminda refused contending that the purchase price has not been fully
paid. The RTC dismissed Tuatis’s complaint and also ruled that Tuatis constructed the building in bad faith for she
had knowledge of the fact that Visminda is still the absolute owner of the land and there was also bad faith on the
part of Visminda since she allowed the construction of the building without opposition on her part. The rights of
the parties must, therefore, be determined as if they both had acted in bad faith. Their rights in such cases are
governed by Article 448 of the Civil Code. The Court of Appeals dismissed the appeal by Tuatis which resulted to
the finality of the appealed decision. Visminda filed a writ of execution. Tuatis then moved that the RTC issue an
order allowing her to buy the subject property and maintained that she has the right to choose between being
indemnified for the value of her building or buying from Visminda the parcel of land. During the pendency of the
motion, the writ of execution was enforced. Tuatis filed with the CA a petition for certiorari, prohibition and
mandamus but the same was denied hence this petition.

ISSUE:

Whether or not Tuatis has the right to choose between being indemnified for the value of her residential building
or buying from Visminda the parcel of land subject of the case pursuant to Article 448 of the civil code.

HELD:

No, under Article 448, the land owner Visminda, has the right to choose and not Tuatis. Taking into consideration
the provisions of the Deed of Sale by installment. Article 448 provides that the owner of the land on which anything
has been built, sown or planted in good faith, shall have the right to appropriate as his own the works, sowing or
planting, after payment of the indemnity provided for in Articles 546 and 548, or to oblige the one who built or
planted to pay the price of the land, and the one who sowed, the proper rent. However, the builder or planter
cannot be obliged to buy the land if its value is considerably more than that of the building or trees. In such case, he
shall pay reasonable rent, if the owner of the land does not choose to appropriate the building or trees after proper
indemnity. The parties shall agree upon the terms of the lease and in case of disagreement, the court shall fix the
terms thereof. According to the provision, the landowner can choose between appropriating the building by paying
the proper indemnity for the same, as provided for in Articles 546 and 548 of the Civil Code; or obliging the builder
to pay the price of the land, unless its value is considerably more than that of the structures, in which case the
builder in good faith shall pay reasonable rent The rule that the choice under Article 448 of the Civil Code belongs
to the owner of the land is in accord with the principle of accession, i.e., that the accessory follows the principal and
not the other way around. Even as the option lies with the landowner, the grant to him, nevertheless, is
preclusive. The landowner cannot refuse to exercise either option and compel instead the owner of the building to
remove it from the land. The raison d’etre for this provision has been enunciated thus: Where the builder, planter
or sower has acted in good faith, a conflict of rights arises between the owners, and it becomes necessary to protect
the owner of the improvements without causing injustice to the owner of the land. In view of the impracticability of
creating a state of forced co-ownership, the law has provided a just solution by giving the owner of the land the
option to acquire the improvements after payment of the proper indemnity, or to oblige the builder or planter to
pay for the land and the sower the proper rent. He cannot refuse to exercise either option. It is the owner of the
land who is authorized to exercise the option, because his right is older, and because, by the principle of accession,
he is entitled to the ownership of the accessory thing.

Vda. De Roxas v. Our Lady’s Foundation, Inc.


G.R. No. 182378, March 2013

To settle the contention, this Court resorts to the provisions of the Civil Code governing encroachment on property.
Under Article 448 pertaining to encroachments in good faith, as well as Article 450 referring to encroachments in bad
faith, the owner of the land encroached upon – petitioner herein – has the option to require respondent builder to pay
the price of the land.
Although these provisions of the Civil Code do not explicitly state the reckoning period for valuing the property,
Ballatan v. Court of Appeals already specifies that in the event that the seller elects to sell the lot, "the price must be
fixed at the prevailing market value at the time of payment.

FACTS:

Salve Dealca Latosa filed before the RTC a Complaint for the recovery of ownership of a portion of her residential
land located at Our Lady’s Village According to her, Atty. Roxas, represented by petitioner herein, encroached on a
quarter of her property by arbitrarily extending his concrete fence beyond the correct limits.

In his Answer, Roxas imputed the blame to Our Lady’s Foundation, Inc. (OLFI). He then filed a Third-Party
Complaint against respondent and claimed that he only occupied the adjoining portion in order to get the
equivalent area of what he had lost when OLFI trimmed his property for the subdivision road. The RTC admitted
the Third-Party Complaint and proceeded to trial on the merits.

Trial Court found that Roxas occupied a total of 112 square meters of Latosa’s lots, and that, in turn, OLFI trimmed
his property by 92 square meters.

RTC issued a Writ of Execution to implement the ruling ordering OLFI to reimburse Roxas for the value of the 92-
square-meter property plus legal interest to be reckoned from the time the amount was paid to the third-party
defendant. The trial court then approved the Sheriff’s Bill, which valued the subject property at ₱2,500 per square
meter or a total of ₱230,000. Adding the legal interest of 12% per annum for 10 years, respondent’s judgment
obligations totaled ₱506,000.

Opposing the valuation of the subject property, OLFI filed a Motion to Quash the Sheriff’s Bill and a Motion for
Inhibition of the RTC judge. It insisted that it should reimburse Roxas only at the rate of ₱40 per square meter, the
same rate that Roxas paid when the latter first purchased the property. Nevertheless, before resolving the Motions
filed by OLFI, the trial court approved an Amended Sheriff’s Bill, which reduced the valuation to ₱1,800 per square
meter.

Although it might be true that the property was originally purchased at ₱40.00 the value of the Philippine Peso has
greatly devaluated since then ₱40.00 may be able to purchase a square meter of land twenty (20) or more years
ago but it could only buy two (2) kilos of rice today. It would be most unfair to the defendants-third party plaintiff
if the third party defendant would only be made to reimburse the purchase price at ₱40.00 per square meter.
Anyway, this Court is in the best position to determine what amount should be reimbursed since it is the one who
rendered the decision which was affirmed in toto by the Appellate Court and this Court is of the opinion and so
holds that that amount should be ₱1,800.00 per square meter.

ISSUE:

Whether the amount to be reimbursed by OLFI is the original purchase price paid 20 or more years ago.

RULING:

To settle the contention, this Court resorts to the provisions of the Civil Code governing encroachment on property.
Under Article 448 pertaining to encroachments in good faith, as well as Article 450 referring to encroachments in
bad faith, the owner of the land encroached upon – petitioner herein – has the option to require respondent builder
to pay the price of the land.

Although these provisions of the Civil Code do not explicitly state the reckoning period for valuing the property,
Ballatan v. Court of Appeals already specifies that in the event that the seller elects to sell the lot, "the price must be
fixed at the prevailing market value at the time of payment."

Ortiz v. Kayanan
G.R. No. L-32974, July 30, 1979
FACTS:

The lot in question was formerly subject of a homestead application by Martin Dolorico II, Ortiz’s ward. Upon
Martin’s death, Ortiz continued cultivation & possession of said property w/o filing any application to acquire title
thereon. Martin’s heirs relinquished their rights over the property in favor of the defendants. The latter filed their
respective sales application, w/c Ortiz opposed. The Secretary of Agriculture dismissed such opposition. On appeal,
the CFI awarded a portion to defendants & the other portion to be subject of auction sale. And that should Ortiz
not be declared a successful bidder thereof, the defendants be ordered to reimburse Ortiz for the improvements he
introduced therein, Ortiz having the right to retain the property until after he has been fully paid therefor

ISSUE:

WON Ortiz is entitled to retain for his own exclusive benefit all the fruits of the property?

HELD:

NO. From the time of the filing of the action in court, possession in good faith ceases & all the fruits that the
possessor may receive from that time must be delivered & paid to the owner or lawful possessor. However, even
after GF ceases, the possessor in fact can still retain the property until he has been fully reimbursed for all the
necessary & useful expenses made by him. This right of retention is accessory to a principal obligation. Considering
that the right of the possessor to receive the fruits terminates when his good faith ceases, it is necessary, in order
that this right to retain may be useful, to concede to the creditor the right to secure reimbursement from the fruits
of the property by utilizing its proceeds for the payment of the interest as well as the principal of the debt while he
remains in possession.

This right of retention is considered not a coercive measure to oblige the debtor to pay, depriving him temporarily
of the enjoyment of the fruits of his property, but as a means of obtaining compensation for the debt. The right of
retention is analogous to a contract of antichresis & it can be considered as a means of extinguishing the
obligation, inasmuch as the right to retain the thing lasts only for the period necessary to enable the creditor to be
reimbursed from the fruits for the necessary & useful expenses.

Narvaez v. Alciso
G.R. No. 165907, July 27, 2009

In a sale with right of repurchase, the applicable provisions are Articles 1606 and 1616 of the Civil Code, not Article
448. Articles 1606 and 1616 state:

Art. 1606. The right referred to in Article 1601, in the absence of an express agreement, shall last four years from the
date of the contract.

Should there be an agreement, the period cannot exceed ten years.

However, the vendor may still exercise the right to repurchase within thirty days from the time final judgment was
rendered in a civil action on the basis that the contract was a true sale with right to repurchase.

Art. 1616. The vendor cannot avail himself of the right of repurchase without returning to the vendee the price of the
sale, and in addition:

(1) The expenses of the contract, and any other legitimate payments made by reason of the sale;

(2) The necessary and useful expenses made on the thing sold.

FACTS:
Larry Ogas owned a parcel of land, and a portion was subjected to a 30-year lease agreement with Esso Standard
Eastern, Inc. Ogas sold the property to his daughter Rose Alciso. Rose later sold the property to Jaime Sansano,
Rose then Repurchased the property then sold it again to Celso Bate. In the deed of sale, it stated that it recognizes
the lease over the property in favor of ESSO, upon sale, the rights over the land as lessor and seller likewise
transfers in full to Celso Bate. The TCT was then cancelled and a new one was issued in the name of Celso Bate.

Celso Bate then sold the property to petitioner here in Narvaez. Rose Alciso demanded a stipulation be made in the
deed of sale allowing her to repurchase the property from the Narvaez. Upon repurchasing the property, Narvaez
and Alciso did not reach an agreement for the price.

Rose Alciso then filed a complaint claiming that the intention of the parties was to enter into a contract of real
estate mortgage and not a contract of sale with right of repurchase.

RTC held that there was a stipulation pour autrui present in the Deed of sale. CA held that the Deed of Sale
contained a stipulation pour autrui

ISSUE:

WON ALCISO could repurchase the property by virtue of the stipulation pour autrui

HELD:

YES. Article 1311, paragraph 2, of the Civil Code states the rule on stipulations pour autrui:

If a contract should contain some stipulation in favor of a third person, he may demand its
fulfillment provided he communicated his acceptance to the obligor before its revocation. A mere
incidental benefit or interest of a person is not sufficient. The contracting parties must have clearly
and deliberately conferred a favor upon a third person.

In Limitless Potentials, Inc. v. Quilala, the Court laid down the requisites of a stipulation pour autrui: (1) there is a
stipulation in favor of a third person; (2) the stipulation is a part, not the whole, of the contract; (3) the contracting
parties clearly and deliberately conferred a favor to the third person the favor is not an incidental benefit; (4) the
favor is unconditional and uncompensated; (5) the third person communicated his or her acceptance of the favor
before its revocation; and (6) the contracting parties do not represent, or are not authorized by, the third party.

All the requisites are present in the instant case: (1) there is a stipulation in favor of Alciso; (2) the stipulation is a
part, not the whole, of the contract; (3) Bate and the Spouses Narvaez clearly and deliberately conferred a favor to
Alciso; (4) the favor is unconditional and uncompensated; (5) Alciso communicated her acceptance of the favor
before its revocation she demanded that a stipulation be included in the 14 August 1981 Deed of Sale of Realty
allowing her to repurchase the property from the Spouses Narvaez, and she informed the Spouses Narvaez that she
wanted to repurchase the property; and (6) Bate and the Spouses Narvaez did not represent, and were not
authorized by, Alciso.

FURTHER, Article 448 is inapplicable in cases involving contracts of sale with right of repurchase it is inapplicable
when the owner of the land is the builder, sower, or planter. In Pecson v. Court of Appeals, the Court held that:

Article 448 does not apply to a case where the owner of the land is the builder, sower, or planter who then later
loses ownership of the land by sale or donation. This Court said so in Coleongco v. Regalado:

Article 361 of the old Civil Code is not applicable in this case, for Regalado
constructed the house on his own land before he sold said land to Coleongco. Article
361 applies only in cases where a person constructs a building on the land of
another in good or in bad faith, as the case may be. It does not apply to a case where
a person constructs a building on his own land, for then there can be no question as
to good or bad faith on the part of the builder.
Elsewise stated, where the true owner himself is the builder of the works on his own land, the issue of good faith or
bad faith is entirely irrelevant.(Emphasis supplied)

Article 448 is inapplicable in the present case because the Spouses Narvaez built the commercial building on the
land that they own. Besides, to compel them to buy the land, which they own, would be absurd. In a sale with right
of repurchase, the applicable provisions are Articles 1606 and 1616 of the Civil Code, not Article 448. Articles 1606
and 1616 state:

Art. 1606. The right referred to in Article 1601, in the absence of an express agreement,
shall last four years from the date of the contract.

Should there be an agreement, the period cannot exceed ten years.

However, the vendor may still exercise the right to repurchase within thirty days from the
time final judgment was rendered in a civil action on the basis that the contract was a true sale with
right to repurchase.

Art. 1616. The vendor cannot avail himself of the right of repurchase without returning to
the vendee the price of the sale, and in addition:

(1) The expenses of the contract, and any other legitimate payments
made by reason of the sale;

(2) The necessary and useful expenses made on the thing sold.

Under Article 1616, Alciso may exercise her right of redemption by paying the Spouses Narvaez (1) the price of the
sale, (2) the expenses of the contract, (3) legitimate payments made by reason of the sale, and (4) the necessary
and useful expenses made on the thing sold. In the present case, the cost of the building constitutes a useful
expense. Useful expenses include improvements which augment the value of the land.

New Regent v Tanjautco, April 16, 2009, GR No. 168800

Fernando v. Acuna
Sept. 14, 2011

Even assuming that Sapang Bayan was a dried-up creek bed, under Article 420, paragraph 1 and Article 502,
paragraph 1 of the Civil Code, rivers and their natural beds are property of public dominion. In the absence of any
provision of law vesting ownership of the dried-up river bed in some other person, it must continue to belong to the
State.

FACTS:

A parcel of land was registered in the names of Spouses Jose Fernando and Lucila Tinio and spouses Antonia
Fernando and Felipe Galvez. When they died inestestate, the property remained undivided. The heirs and
successors-in-interest, herein petitioners (Jose and Zoilo Fernando, Norma Fernando Banares, Rosario Fernando
Tangkencgo, heirs of Tomas Fernando, heirs of Guillermo Fernando, heirs of Iluminada Fernando and heirs of
Germogena Fernando) failed to agree on the division of the subject property. Thus, except for the heirs of
Germogena Fernando, filed a Complaint for partition against the heirs of Germogena Fernando praying that the
subject property be partitioned into 8 equal parts corresponding to the hereditary interest of each group of heirs.

In his Complaint in intervention, 1998, respondent Leon Acuna averred that the portion of the property identified
as Lot 1303 was already adjudicated the petitioners' predecessor-in-interest. He likewise claimed the portion
identified as Lot 1302 was also already adjudicated to other people as well.
TC found that Lot 1302 was already titled in the names of third persons. With respect to Lot 1303 TC found out
that the November 1929 decision was never executed and has already prescribed. TC ordered the reversion of Lot
1303 to the ownership of spouses Jose Fernando and Lucila Tinio and spouses Antonia Fernando and Felipe Galvez
and allowed the partition of Lot 1303 among petitioners as successors-in-interest of said registered owners.
Excluded from the partition, however, were the portions of the property which petitioners admitted had been sold
or transferred to Ruperta Sto. Domingo Villasenor and respondent Acuna.

With respect to Sapang Bayan, TC found that the same had not been alleged in the pleadings nor raised as an issue
during the pre-trial conference. Also, according to the trial court, the parties failed to clearly show whether Sapang
Bayan was previously a dry portion of either Lot 1302 or Lot 1303. Neither was there any proof that Sapang Bayan
was a river that just dried up or that it was an accretion which the adjoining lots gradually received from the effects
of the current of water. It was likewise not established who were the owners of the lots adjoining Sapang Bayan.
The trial court concluded that none of the parties had clearly and sufficiently established their claims over Sapang
Bay.

ISSUE:

Can the petitioners validly claim the ownership of the Sapang Bayan?

HELD:

NO. CA erred in ruling that the principle of accretion is applicable. the Court of Appeals erred in ruling that the
principle of accretion is applicable ("to the owners of lands adjoining the banks of rivers belong the accretion
which they gradually receive from the effects of the current of the waters." Art. 457, CC) The character of the
Sapang Bayan property was not shown to be of the nature that is being referred to in the provision which is an
accretion known as alluvion. In fact the parties could not agree how Sapang Bayan came about. Whether it was a
gradual deposit received from the river current or a dried-up creek bed connected to the main river could not be
ascertained.

Even assuming that Sapang Bayan was a dried-up creek bed, under Article 420, paragraph 1 and Article 502, of the
CC, rivers and their natural beds are property of public dominion. In the absence of any provision of law vesting
ownership of the dried-up river bed in some other person, it must continue to belong to the State.

In Republic v. Court of Appeals Court ruled that lots were portions of the bed of the Meycauayan river and are
therefore classified as property of the public domain under Article 420. They are not open to registration under the
Land Registration act. Furthermore, in Celestial v. Cachopero, the Court similarly ruled that a dried-up creek bed is
property of public dominion: A creek, like the Salunayan Creek, is a recess or arm extending from a river and
participating in the ebb and flow of the sea. As such, under Article 420 the Salunayan Creek, including its natural
bed, is property of the public domain which is not susceptible to private appropriation and and acquisitive
prescription. And, absent any declaration by the government, that a portion of the creek has dried-up does not, by
itself, alter its inalienable character. Therefore, on the basis of the law and jurisprudence on the matter, Sapang
Bayan cannot be adjudged to any of the parties in this case.

Galang v. Reyes
G.R. No. 184746, August 8, 2012

If indeed a property was the former bed of a creek that changed its course and passed through the property of the
claimant, then, pursuant to Article 461, the ownership of the old bed left to dry by the change of course was
automatically acquired by the claimant. Before such a conclusion can be reached, the fact of natural abandonment of
the old course must be shown, that is, it must be proven that the creek indeed changed its course without artificial or
man-made intervention.

FACTS:
Spouses Reyes filed a case for the annulment of Original Certificate of Title (OCT) No. P-928 against spouses Galang
with the RTC. The Reyeses alleged:

That they owned two properties: (1) Ponderosa Heights Subdivision (Ponderosa), and (2) an adjoining property
covered by a TCT with an area of 1,201 sq.m.;
That the properties were separated by the Marigman Creek, which dried up sometime in 1980 when it changed its
course and passed through Ponderosa;
That the Galangs, by employing manipulation and fraud, were able to obtain a certificate of title over the dried up
creek bed from the Department of Environment and Natural Resources (DENR), through its Provincial Office
(PENRO); and
That as the owners of the land where the new course of water passed, they are entitled to the ownership of the
property to compensate them for the loss of the land being occupied by the new creek.

The Galangs denied that the land subject of the complaint was part of a creek and countered that OCT No. P-928
was issued to them after they had complied with the free patent requirements of the DENR, through the PENRO.

The RTC dismissed the complaint for lack of cause of action and for being an erroneous remedy. It added that the
land, having been acquired through a homestead patent, was presumably public land. Therefore, only the State can
institue an action for the annulment of the title covering it. Furthermore, because the Reyeses claimed to have
acquired the property by right of accretion, they should have filed an action for reconveyance, explaining “[t]hat
the remedy of persons whose property had been wrongly or erroneously registered in another’s name is not to set
aside the decree/title, but an action for reconveyance, or if the property has passed into the hands of an innocent
purchaser for value, an action for damages.”

The CA, however, reversed and set aside the RTC Decision.

ISSUE:

Whether or not they were able to prove their cause of action against the Galangs.

RULING:

No. If indeed a property was the former bed of a creek that changed its course and passed through the property of
the claimant, then, pursuant to Article 461, the ownership of the old bed left to dry by the change of course was
automatically acquired by the claimant. Before such a conclusion can be reached, the fact of natural abandonment
of the old course must be shown, that is, it must be proven that the creek indeed changed its course without
artificial or man-made intervention. Thus, the claimant, in this case the Reyeses, must prove three key elements by
clear and convincing evidence. These are: (1) the old course of the creek, (2) the new course of the creek, and (3)
the change of course of the creek from the old location to the new location by natural occurrence.

In this regard, the Reyeses failed to adduce indubitable evidence to prove the old course, its natural abandonment
and the new course. In the face of a Torrens title issued by the government, which is presumed to have been
regularly issued, the evidence of the Reyeses was clearly wanting. Uncorroborated testimonial evidence will not
suffice to convince the Court to order the reconveyance of the property to them. The conflicting claims here are (1)
the title of the Galangs issued by the DENR, through the PENRO, and (2) the claim of the Reyeses, based on
unsubstantiated testimony, that the land in question is the former bed of a dried up creek. As between these two
claims, this Court is inclined to decide in favor of the Galangs who hold a valid and subsisting title to the property
which, in the absence of evidence to the contrary, the Court presumes to have been issued by the PENRO in the
regular performance of its official duty.

Republic v. Arcadio Ivan Santos


G.R. No. 160453, November 12, 2012

By law, accretion - the gradual and imperceptible deposit made through the effects of the current of the water-
belongs to the owner of the land adjacent to the banks of rivers where it forms. The drying up of the river is not
accretion. Hence, the dried-up river bed belongs to the State as property of public dominion, not to the riparian owner,
unless a law vests the ownership in some other person.

FACTS:

Alleging continuous and adverse possession of more than ten years, respondent Arcadio Ivan A. Santos III (Arcadio
Ivan) applied for the registration of Lot 4998-B (the property) in RTC Paranaque City. The property, which had an
area of 1,045 square meters, more or less, was located in Brgy. San Dionisio, Parañaque City, and was bounded in
the Northeast by Lot 4079 belonging to respondent Arcadio C. Santos, Jr. (Arcadio, Jr.), in the Southeast by the
Parañaque River, in the Southwest by an abandoned road, and in the Northwest by Lot 4998-A also owned by
Arcadio Ivan.

Arcadio Ivan later amended his application for land registration to include Arcadio, Jr. as his co-applicant because
of the latter’s co-ownership of the property. He alleged that the property had been formed through accretion and
had been in their joint open, notorious, public, continuous and adverse possession for more than 30 years.

City of Parañaque opposed the application for land registration, stating that it needed the property for its flood
control program; that the property was within the legal easement of 20 meters from the river bank; and that
assuming that the property was not covered by the legal easement, title to the property could not be registered in
favor of the applicants for the reason that the property was an orchard that had dried up and had not resulted from
accretion.

The RTC GRANTED the application for Land Registration of Arcadio Ivan, to which the Republic, through the Office
of the Solicitor General (OSG), appealed. The CA AFFIRMED the RTC’s decision. Hence, this appeal.

ISSUE:

Is the drying up of river bed covered by accretion?

RULING:

No. Accretion is the process whereby the soil is deposited along the banks of rivers. The deposit of soil, to be
considered accretion, must be:
(a) gradual and imperceptible;
(b) made through the effects of the current of the water; and
(c) taking place on land adjacent to the banks of rivers.

Accordingly, respondents should establish the concurrence of the elements of accretion to warrant the grant of
their application for land registration. Respondents did not discharge their burden of proof. As the applicants for
land registration, they carried the burden of proof to establish the merits of their application by a preponderance
of evidence, by which is meant such evidence that is of greater weight, or more convincing than that offered in
opposition to it. They would be held entitled to claim the property as their own and apply for its registration under
the Torrens system only if they established that, indeed, the property was an accretion to their land. They did not
show that the gradual and imperceptible deposition of soil through the effects of the current of the river had
formed Lot 4998-B. Instead, their evidence revealed that the property was the dried-up river bed of the Parañaque
River, leading both the RTC and the CA to themselves hold that Lot 4998-B was "the land which was previously
part of the Parañaque River and became an orchard after it dried up."

RTC and CA GROSSLY ERRED in treating the dried-up river bed as an accretion that became respondents’ property
pursuant to Art. 457 CC. That land was definitely not an accretion. The process of drying up of a river to form dry
land involved the recession of the water level from the river banks, and the dried-up land did not equate to
accretion, which was the gradual and imperceptible deposition of soil on the river banks through the effects of the
current. In accretion, the water level did not recede and was more or less maintained. Hence, respondents as the
riparian owners had no legal right to claim ownership of Lot 4998-B. Considering that the clear and categorical
language of Art. 457 CC has confined the provision only to accretion, we should apply the provision as its clear and
categorical language tells us to.

Greenacres v. Cabral
G.R. No. 175542, June 5, 2013

For an action to quiet title to prosper, two indispensable requisites must concur: (1) the plaintiff or complainant has a
legal or equitable title or interest in the real property subject of the action; and (2) the deed, claim, encumbrance, or
proceeding claimed to be casting a cloud on his title must be shown to be in fact invalid or inoperative despite its
prima facie appearance of validity or legal efficacy.

The petitioner may vindicate its rights in the property through an action for quieting of title, a common law remedy
designed for the removal of any cloud upon, or doubt, or uncertainty affecting title to real property. The action for
quieting of title may be brought whenever there is a cloud on title to real property or any interest in real property by
reason of any instrument, record, claim, encumbrance, or proceeding that is apparently valid or effective, but is, in
truth and in fact, invalid, ineffective, voidable, or unenforceable, and may be prejudicial to said title.

FACTS:

Victoria Cabral was the original owner of a parcel of land in Barangay Pandayan, Meycauayan, Bulacan. Three
Emancipation Patents were issued to the spouses Enrique Moraga and Victoria Soriano . The Spouses Moraga
thereafter caused the cancellation of EP No. 496041 and its conversion to TCT No. 256260 (M). Cabral filed a
complaint before the Provincial Agrarian Reform Adjudicator (PARAD) seeking the cancellation of the
Emancipation Patents issued to the Spouses Moraga on the grounds that these were obtained through fraud and
that the land is not suitable for rice and corn production and has long been classified as residential, commercial,
industrial and nonagricultural land by the Zoning Administrator of the Housing and Land Use Regulatory Board.
PARAD rendered a decision denying the petition. Cabral appealed the decision to the Department of Agrarian
Reform Adjudication Board (DARAB).

While the appeal was pending, the Spouses Moraga subdivided the lot covered by TCT No. 256260 (M) into three
smaller lots, the properties subject of this case. Green Acres purchased five lots from Filcon including the three
subject properties. Except for an already cancelled annotation of a real estate mortgage in favor of Philippine
Commercial International Bank (PCI Bank), the titles were free from any annotations, liens, notices, claims or
encumbrances. The titles of Filcon were cancelled by the Register of Deeds of Meycauayan, Bulacan and new titles
were issued in the name of Green Acres including TCT Nos. T-345660 (M), T-345661 (M) and T-345662 (M)
covering the subject properties. Green Acres then constructed a warehouse building complex on the said lots.

DARAB resolved Cabral’s appeal and rendered judgment ordering the cancellation of the titles issued in the names
of the Spouses Moraga and those of Filcon for having been illegally acquired. When Green Acres learned about the
DARAB decision, it sent a Letter to Filcon, advising the latter that it learned that the properties it bought from
Filcon were the subject of an adverse decision of the DARAB. Fearing that its titles and possession might be
disturbed by the DARAB decision, Green Acres reminded Filcon of its warranties under the deed of sale.

Green Acres filed a Complaint for Quieting of Title, Damages with Application for Preliminary Injunction and Writ
of Preliminary Attachment before the RTC of Malolos, Bulacan against Cabral, the Spouses Moraga, Filcon, the
DARAB and the Registry of Deeds of Meycauayan, Bulacan. Green Acres sought to quiet its title and alleged that it is
a purchaser in good faith and for value, claiming that it had no notice or knowledge of any adverse claim, lien, or
encumbrance on the properties. Cabral denied all the material allegations in the complaint and alleged that Green
Acres never acquired valid title to the subject property, much less, can it claim to be an innocent purchaser for
value. She further averred that a declaratory judgment in a petition to quiet title will effectively subject the DARAB
decision to review. The trial court granted the demurrer filed by the Cabrals and ordered the case dismissed.
Subsequently, Cabral filed with the PARAD a Motion for Issuance of Writ of Execution of the DARAB decision. The
PARAD issued a Resolution denying the Motion for Issuance of Writ of Execution for lack of merit. Cabral filed with
the CA a petition for certiorari under Rule 65 seeking to annul the January 25, 2006 and September 11, 2006
Resolutions, as well as the February 27, 2007 Order of the PARAD.CA denied Cabral’s petition. Both Green Acres
and Cabral are now before this Court seeking the reversal of the CA decisions adverse to them.

Green Acres argues that the DARAB decision is among those enumerated in Article 476 of the Civil Code as a
possible source of a cloud on title to real property. It contends that there can hardly be any doubt that the DARAB
Decision is an "instrument," or if not, a "record" and reflects a "claim" on the properties, while the proceedings
before the DARAB are "proceedings" directed at the real properties now owned by Green Acres which are
"apparently valid or effective" but "unenforceable" against the titles of Green Acres. It also contends that the
appellate court’s reliance on Foster-Gallego v. Spouses Galang is misplaced since nothing in said case supports the
proposition that a decision of a coordinate court cannot be a source of cloud under Article 476 of the Civil Code.
Green Acres submits that Foster-Gallego is not applicable because the ruling there was that an action to quiet title
is not the proper remedy when to remove a cloud on a title, a final and executory decision of the court need to be
reviewed or vacated. In the present case, Green Acres does not seek a review or reversal of the DARAB decision.

Cabral, for her part, insists that the DARAB decision is not among those enumerated in Article 476 which may cast
a cloud on title to real property. As to the applicability of Foster-Gallego, she argues that assuming that the ruling
on the main issue in said case is not directly germane, the pronouncements therein on the nature, function,
purpose and limitations of a case for quieting of title and the power of the courts in such proceedings are
applicable.

ISSUE:

Whether the said DARAB decision in favor of Cabral constitutes a cloud on Green Acres’ title over the subject
properties.

RULING:

Green Acres’ arguments are meritorious.


Article 476 of the Civil Code provides:
Art. 476. Whenever there is a cloud on title to real property or any interest
therein, by reason of any instrument, record, claim, encumbrance or proceeding which is apparently valid or
effective but is in truth and in fact invalid, ineffective, voidable, or unenforceable, and may be prejudicial to said
title, an action may be brought to remove such cloud or to quiet the title. An action may also be brought to prevent
a cloud from being cast upon title to real property or any interest therein.

Quieting of title is a common law remedy for the removal of any cloud upon, doubt, or uncertainty affecting title to
real property. Whenever there is a cloud on title to real property or any interest in real property by reason of any
instrument, record, claim, encumbrance, or proceeding that is apparently valid or effective, but is in truth and in
fact, invalid, ineffective, voidable, or unenforceable, and may be prejudicial to said title, an action may be brought
to remove such cloud or to quiet the title. In such action, the competent court is tasked to determine the respective
rights of the complainant and the other claimants, not only to place things in their proper places, and make the
claimant, who has no rights to said immovable, respect and not disturb the one so entitled, but also for the benefit
of both, so that whoever has the right will see every cloud of doubt over the property dissipated, and he can
thereafter fearlessly introduce any desired improvements, as well as use, and even abuse the property.

For an action to quiet title to prosper, two indispensable requisites must concur: (1) the plaintiff or complainant
has a legal or equitable title or interest in the real property subject of the action; and (2) the deed, claim,
encumbrance, or proceeding claimed to be casting a cloud on his title must be shown to be in fact invalid or
inoperative despite its prima facie appearance of validity or legal efficacy.
There is no dispute as to the first requisite since Green Acres has legal title over the subject properties. The issue
lies in the second requisite.

A cloud on title consists of (1) any instrument, record, claim, encumbrance or proceeding; (2) which is apparently
valid or effective; (3) but is in truth and in fact invalid, ineffective, voidable, or unenforceable; and (4) may be
prejudicial to the title sought to be quieted.

This Court holds that the DARAB decision in favor of Cabral satisfies all four elements of a cloud on title.As Green
Acres correctly points out, the DARAB decision, a final one at that, is both an "instrument" and a "record." Black’s
Law Dictionary defines an instrument as a document or writing which gives formal expression to a legal act or
agreement, for the purpose of creating, securing, modifying or terminating a right. A record, on the other hand, is
defined as a written account of some act, court proceeding, transaction or instrument drawn up under authority of
law, by a proper officer, and designed to remain as a memorial or permanent evidence of the matters to which it
relates. It is likewise a "claim" which is defined as a cause of action or a demand for money or property since Cabral
is asserting her right over the subject lots. More importantly, it is a "proceeding" which is defined as a regular and
orderly progress in form of law including all possible steps in an action from its commencement to the execution of
judgment and may refer not only to a complete remedy but also to a mere procedural step that is part of a larger
action or special proceeding.

Also, the DARAB decision is apparently valid and effective. It is a final decision that has not been reversed, vacated
or nullified. It is likewise apparently effective and may be prejudicial to Green Acres’ titles since it orders the
cancellation of the titles of the Spouses Moraga and Filcon all from which Green Acres derived its titles. However,
as discussed above, it is ineffective and unenforceable against Green Acres because Green Acres was not properly
impleaded in the DARAB proceedings nor was there any notice of lis pendens annotated on the title of Filcon so as
to serve notice to Green Acres that the subject properties were under litigation. As such, Green Acres is an innocent
purchaser for value.

We agree with the CA's suggestion that the petitioner's proper recourse was either an action for quieting of title or
an action for reconveyance of the property. It is timely for the Court to remind that the petitioner will be better off
if it should go to the courts to obtain relief through the proper recourse; otherwise, it would waste its own time and
effort, aside from thereby unduly burdening the dockets of the courts.

The petitioner may vindicate its rights in the property through an action for quieting of title, a common law remedy
designed for the removal of any cloud upon, or doubt, or uncertainty affecting title to real property. The action for
quieting of title may be brought whenever there is a cloud on title to real property or any interest in real property
by reason of any instrument, record, claim, encumbrance, or proceeding that is apparently valid or effective, but is,
in truth and in fact, invalid, ineffective, voidable, or unenforceable, and may be prejudicial to said title. In the action,
the competent court is tasked to determine the respective rights of the plaintiff and the other claimants, not only to
put things in their proper places, and make the claimant, who has no rights to the immovable, respect and not
disturb the one so entitled, but also for the benefit of both, so that whoever has the right will see every cloud of
doubt over the property dissipated, and he can thereafter fearlessly introduce any desired improvements, as well
as use, and even abuse the property.

PNB v. Sps. Marañon


G.R. No. 189316, June 1, 2013

Rent is a civil fruit that belongs to the owner of the property producing it by right of accession. The rightful recipient
of the disputed rent in this case should thus be the owner of the subject lot at the time the rent accrued (Sps.
Maranon).

FACTS:

As security for a loan, Sps. Montealegre mortgaged to PNB a parcel of land located at Cuadra-Smith Streets,
Downtown, Bacolod. They failed to pay the loan and the lot eventually became PNB’s. Sps. Maranon filed a
complaint for annulment of title and reconveyance with damages. They claimed to be the true registered owners of
the lot by virtue of TCT No. T-129577 which was illegally cancelled by TCT No. T-156512 under the name of Emilie
Montealegre who used a falsified Deed of Sale bearing the forged signatures of Spouse Marañon to effect the
transfer of title to the property in her name. PNB averred that it is a mortgagee in good faith and for value and that
its mortgage lien on the property was registered thus valid and binding against the whole world. While the
proceedings were ongoing, Tolete, one of the tenants of the building erected on the lot deposited his rental
payments, amounting to ₱144,000.00, with the Clerk of Court of Bacolod City. RTC ruled in favor Sps. Maranon.
They sought the release of said rental payments.
ISSUE:

W/N Sps. Maranon are entitled to the rental payments

RULING: YES.

Rent is a civil fruit that belongs to the owner of the property producing it by right of accession. The rightful
recipient of the disputed rent in this case should thus be the owner of the subject lot at the time the rent accrued.
Sps. Maranon never lost ownership over the subject lot as the precise consequence of the final and executory
judgment rendered by the RTC whereby the title to the lot was reconveyed to them and the cloud thereon
consisting of Emilie’s fraudulently obtained title was removed. PNB’s mortgagors, Spouses Montealegre, are not the
true owners of the subject lot much less of the building which produced the disputed rent. The foreclosure
proceedings caused by PNB could not have, thus, included the building found on the subject lot and the rent it
yields. PNB’s lien as a mortgagee in good faith pertains to the subject lot alone because the rule that improvements
shall follow the principal in a mortgage under Article 2127 of the Civil Code does not apply under the premises.
Accordingly, since the building was not foreclosed, it remains a property of Spouses Marañon; it is not affected by
non-redemption and is excluded from any consolidation of title made by PNB over the subject lot. Thus, PNB’s
claim for the rent paid by Tolete has no basis.

Heirs of Sarili, et al v. Lagrosa


G.R. No. 193517, Jan. I5 2014

To be deemed a builder in good faith, it is essential that a person asserts title to the land on which he builds, i.e., that
he be a possessor in concept of owner, and that he be unaware that there exists in his title or mode of acquisition any
flaw which invalidates it.

FACTS:

Respondent filed a complaint against Spouses Sarili and the RD of Caloocan City before the RTC alleging that he is
the owner of a parcel of land which was fraudulently conveyed to the spouses. He claimed that he is a resident of
California, USA and during his vacation in the Philippines, he discovered that a new certificate of title to the subject
property was issued by the RD in the name of Victorino married to Isabel Amparo by virtue of a falsified Deed of
Absolute Sale dated February 16, 1978. Thus, he prayed for its annulment and the delivery to him of the possession
of the subject property.

In their answer, the Spouses claimed that they are innocent purchasers for value, having purchased the subject
property from Ramon B. Rodriguez (Ramon), who possessed and presented a SPA to sell/dispose of the same, and,
in such capacity, executed a Deed of Absolute Sale dated November 20, 1992.

The RTC ruling that Ramon’s authority was established, declared the November 20, 1992 deed of sale as “valid,
genuine, lawful and binding” and, as such, had validly conveyed the subject property in favor of Sps. Sarili.
However, on appeal, the same was reversed and set aside by the CA.

ISSUE:

Whether there was a valid conveyance of the property to the spouses.

RULING:

No, there was no valid conveyance.

The Court ruled that a higher degree of prudence is required from one who buys from a person who is not the
registered owner, although the land object of the transaction is registered. In this case, the SPA presented by
Ramos readily indicates flaws in its notarial acknowledgment since the respondent’s community tax certificate
(CTC) number was not indicated thereon. Hence, under the law, the spouses must have conducted an investigation
beyond the subject SPA and into the circumstances of its execution. For failure to comply with such requirement,
the spouses cannot be considered as innocent purchasers for value.

As to the house the spouses had built, the Court finds a need to remand the case. The Court explained that to be be
deemed a builder in good faith, it is essential that a person asserts title to the land on which he builds, i.e., that he
be a possessor in concept of owner, and that he be unaware that there exists in his title or mode of acquisition any
flaw which invalidates it. As for Sps. Sarili, they knew – or at the very least, should have known – from the very
beginning that they were dealing with a person who
possibly had no authority to sell the subject property considering the palpable irregularity in the
subject SPA’s acknowledgment. Yet, relying solely on said document and without any further investigation on
Ramos’s capacity to sell, Spouses Sarili still chose to proceed with its purchase and even built a house thereon.
Based on the foregoing, it cannot be seriously doubted that Spouses Sarili were actually aware of a flaw or defect in
their title or mode of acquisition and have consequently built the house on the subject property in bad faith under
legal contemplation.

Heirs of Pocdo v. Avila


G.R. No. 199146, March 19, 2014

FACTS:

Pocdo Pool, who died in 1942, began his occupation and claim on three lots that were eventually surveyed
in his name. These lots were the subject of a petition to reopen judicial proceedings filed by the Heirs of Pocdo Pool
with the CFI Baguio City. The registration of the lots in the names of the petitioners were granted, but since the
decision was not implemented within the 10 years prescribed period, the Heirs filed their ancestral land claims
with the DENR. The Certificates of Ancestral Lands Claims (CALS) were issued by the DENR for Lots 44 and 45, but
Lot 43 was not approved due to Memorandum Order issued by the DENR Secretary. Polon Pocdo, an heir of Pocdo
Pool, ceded his rights over the three lots to Pacifico Pocdo in exchange for a one hectare lot to be taken from Lot 43.
However, Pacifico entered into a contract with Florencio Pax and Braulio Yaranon revoking the agreement with
Polon. Polon authorized Avila to undertake the segregation of his land from Lot 43 in accord with the amicable
settlement. In exchange, Polon would award to her 2,000 square meters from the 1 hectare lot. After spending
time, money and effort in the execution of the survey, Avila gave the survey results to Polon prompting Polon to
execute a Waiver of Rights.

CENRO of Baguio City issued in favor of Avila a Certificate of Exclusion. The Heirs of Polon Pocdo and his
wife Konon filed an affidavit of cancellation. Suaking cancelled the Certificate of Exclusion. Avila complained to the
Regional Executive Director the unlawful cancellation of her Certificate, RED issued a memorandum setting aside
the revocation. Avila filed an administrative complaint against Suaking, and RED dismissed the letter complaint of
Avila and referred the administrative complaint to the DENR Central Office. RED set aside the order. The Affidavit
of Cancellation was dismissed for lack of jurisdiction and the validity of the Amicable Settlement. RTC dismissed
the case for lack of jurisdiction. The trial court held that the DENR had already declared the disputed property as
public land, which the State, through the DENR, has the sole power to dispose. Thus, the claim of petitioners to
quiet title is not proper since they do not have title over the disputed property.

ISSUE:

Whether or not trial court had jurisdiction to quiet title.

RULING: NO.

Having established that the disputed property is public land, the trial court was therefore correct in
dismissing the complaint to quiet title for lack of jurisdiction. The trial court had no jurisdiction to determine who
among the parties have better right over the disputed property which is admittedly still part of the public domain.
In an action for quieting of title, the complainant is seeking for an adjudication that a claim of title or interest in
property adverse to the claimant is invalid, to free him from the danger of hostile claim, and to remove a cloud
upon or quiet title to land where stale or unenforceable claims or demands exist.
Under Articles 476 and 477 of the Civil Code, the two indispensable requisites in an action to quiet title are:
(1) that the plaintiff has a legal or equitable title to or interest in the real property subject of the action; and (2)
that there is a cloud on his title by reason of any instrument, record, deed, claim, encumbrance or proceeding,
which must be shown to be in fact invalid or inoperative despite its prima facie appearance of validity. In this case,
petitioners, claiming to be owners of the disputed property, allege that respondents are unlawfully claiming the
disputed property by using void documents. However, the records reveal that petitioners do not have legal or
equitable title over the disputed property, which forms part of Lot 43, a public land within the Baguio Townsite
Reservation. It is clear from the facts of the case that petitioners’ predecessors-in-interest, the heirs of Pocdo Pool,
were not even granted a Certificate of Ancestral Land Claim over Lot 43, which remains public land. Thus, the trial
court had no other recourse but to dismiss the case.

Narvasa v. Imbornal
G.R. No. 182908, August 06, 2014

FACTS:

Basilia Imbornal owned a parcel of land situated at Sabangan, Pangasinan which she conveyed to her 3 daughters,
Balbina, Alejandra, and Catalina sometime in 1920. Meanwhile, Catalina’s husband, Ciriaco Abrio (Ciriaco), applied
for and was granted a homestead patent over a 31,367-sq. m. riparian land (Motherland) adjacent to the Cayanga
River in San Fabian, Pangasinan. Ciriaco and his heirs had since occupied the northern portion of the
Motherland, while respondents occupied the southern portion.

The First Accretion, adjoined the southern portion of the Motherland. OCT No. P-318 was issued in the name of
respondent Victoriano married to Esperanza Narvarte, covering the First Accretion. The Second Accretion, abutted
the First Accretion on its southern portion. OCT No. 21481 was issued in the names of all the respondents covering
the Second Accretion.

ISSUE:

Who are the owners of the first and second accretions?

RULING:

Being the owner of the land adjoining the foreshore area, respondent is the riparian or littoral owner who has
preferential right to lease the foreshore area. Accordingly, therefore, alluvial deposits along the banks of a creek or
a river do not form part of the public domain as the alluvial property automatically belongs to the owner of the
estate to which it may have been added. The only restriction provided for by law is that the owner of the adjoining
property must register the same under the Torrens system; otherwise, the alluvial property may be subject to
acquisition through prescription by third persons.

In this case, Francisco, et al. and, now, their heirs, herein petitioners are not the riparian owners of the Motherland
to which the First Accretion had attached, hence, they cannot assert ownership over the First Accretion.
Consequently, as the Second Accretion had merely attached to the First Accretion, they also have no right over the
Second Accretion. Neither were they able to show that they acquired these properties through prescription as it
was not established that they were in possession of any of them.

Therefore, whether through accretion or, independently, through prescription, the discernible conclusion is that
Francisco et al. and/or petitioners' claim of title over the First and Second Accretions had not been substantiated,
and, as a result, said properties cannot be reconveyed in their favor. This is especially so since on the other end of
the fray lie respondents armed with a certificate of title in their names covering the First and Second Accretions
coupled with their possession thereof, both of which give rise to the superior credibility of their own claim.

Spouses Aquino v. Spouses Aguilar


G.R. No. 182754, June 29, 2015
Builder in bad faith has no right of reimbursement.

FACTS:

Spouses Aquino are the owners of a house and lot located at Makati City as evidenced by Transfer of Certificate.
The subject property is being occupied by Teresa Aquino’s sister Josefina Aguilar, her husband and their family.
Spouses Aguilar stayed on the property with the consent and approval of the Spouses Aquino who were residing in
the United States. While respondents were in possession of the property, the house previously constructed therein
was demolished, and a three (3) storey building built in its place. The respondents occupied half of the third floor
of the building for the new 20 years without payment of rental.

Petitioners sent a letter to respondents informing them that an immediate family needed to use the premises and
demanding the surrender of the property. Respondents failed to heed this demand, prompting the petitioners to
file an ejectment case.

Respondent claimed that they contributed to the improvement of the property and the construction of the building,
both in terms of money and management. Petitioners purportedly agreed to let them contribute to the costs of
construction in exchange for the exclusive use of a portion of the building.

ISSUE:

W/N the respondents are entitled to be reimbursed for the improvements introduced on the property.

RULING:

As builders in bad faith, respondents are not entitled to reimbursement of useful expenses. Based on the findings of
fact of the MeTC and the RTC, petitioners had already warned respondents not to build a structure on the property
as early as 1983. We hold that petitioners, as the owners of the land, have the right to appropriate what has been
built on the property, without any obligation to pay indemnity therefor; and that respondents have no right to a
refund of any improvement built therein, pursuant to Articles 449 and 450 of the Civil Code: Art. 449. He who
builds, plants or sows in bad faith on the land of another, loses what is built, planted or sown without right of
indemnity. a builder in bad faith is entitled to recoup the necessary expenses incurred for the preservation of the
land. The CA correctly ruled that respondents in this case are similarly entitled to this reimbursement. However,
being builders in bad faith, they do not have the right of retention over the premises.

Bliss Development Corp v. Diaz


G.R. No. 213233, August 05, 2015

FACTS:

This case involves a property owned by the petitioner Bliss Development Corporation (BDC), which was
later on sold to Spouses Emiliano and Leonila Melgazo (Sps. Melgazo. A several deeds of transfer of right over the
property were made until such time that it was sold to respondent Montano Diaz (Diaz) in the amount of
P600,000.00. Diaz then paid BDC the amortizations due on the property and BDC issued a permit to occupy the
property in favor of Diaz. Thereafter, BDC informed Diaz that respondent Edgar Arreza (Arreza) was claiming that
the heirs of Sps. Melgazo sold to him the rights over the property.

Diaz filed the present complaint for sum of money against BDC before the RTC, arguing that BDC and
Tapay’s representations led him to believe that he had a good title over the property, but due to the court’s ruling
in the interpleader case, he was constrained to transfer the property to Arreza.

ISSUE:

Whether BDC acted in bad faith in dealing with respondent Diaz.


RULING: YES.

When it allowed Diaz to take over the payment of the amortizations over the subject property. It is
undisputed that Bliss knew about Arreza’s claim in 1991. It even received amortization payments from Arreza. Yet,
Bliss acknowledged the transfer to Diaz and received the monthly amortizations paid by Diaz. Also, Bliss is aware
that should Arreza pursue his claim in court, Diaz may be evicted from the property.

However, Diaz is not a purchaser for value and in good faith. A careful review of the records of this case
reveals that Diaz, in fact, failed to diligently inquire into the title of his predecessor before entering into the
contract of sale. As such, he cannot be considered a buyer in good faith. There is no issue that despite the several
transfers of rights from Nacua to Garcia to Reyes to Tapay to Diaz, title over the property remained in BDC’s name.
When Diaz transacted with Tapay, it was also clear that what was being transferred was merely rights to purchase
the property, and not title over the lot itself; if it were, the sale would have been void because Tapay never had
ownership over the subject property. As the buyer in such a transaction, it was incumbent upon Diaz not only to
inquire as to the right of Tapay to transfer his rights, but also to trace the source of that right to purchase the
property. Had he discharged this duty diligently, he would have found out that Nacua’s right was without basis,
because it was founded on a forged deed. For his failure to inquire diligently and trace the source of the right to
purchase the property, Diaz cannot claim to be a purchaser in good faith and for value.

Both parties being in bad faith, BDC is liable to Diaz for the value of the improvements he
introduced on the subject property.
Art. 448: The owner of the land on which anything has been built, sown or planted in good faith, shall have the right to
appropriate as his own the works, sowing or planting, after payment of the indemnity provided for in Articles 546 and
548, or to oblige the one who built or planted to pay the price of the land, and the one who sowed, the proper rent.
However, the builder or planter cannot be obliged to buy the land if its value is considerably more than that of the
building or trees. In such case, he shall pay reasonable rent, if the owner of the land does not choose to appropriate the
building or trees after proper indemnity. The parties shall agree upon the terms of the lease and in case of
disagreement, the court shall fix the terms thereof.

Art. 453: If there was bad faith, not only on the part of the person who built, planted or sowed on the land of another,
but also on the part of the owner of such land, the rights of one and the other shall be the same as though both had
acted in good faith.

It is understood that there is bad faith on the part of the landowner whenever the act was done with his knowledge
and without opposition on his part.

Art. 546: Necessary expenses shall be refunded to every possessor; but only the possessor in good faith may retain the
thing until he has been reimbursed therefor.

Useful expenses shall be refunded only to the possessor in good faith with the same right of retention, the person who
has defeated him in the possession having the option of refunding the amount of the expenses or of paying the increase
in value which the thing may have acquired by reason thereof.

Art. 548: Expenses for pure luxury or mere pleasure shall not be refunded to the possessor in good faith; but he may
remove the ornaments with which he has embellished the principal thing if it suffers no injury thereby, and if his
successor in the possession does not prefer to refund the amount expended.

Homeowners v. J.M. Tuason


G.R. No. 203833, November 10, 2015

Absent a deed of donation or legitimate acquisition thereof by the government, the area claimed to have been reserved
for public use and/or as an open space still pertained to the subdivision developer.

FACTS:
The subject matter of the instant suit is a parcel of land denominated as Block 494 of the Talayan Village. Block 494
was previously registered in the name of J.M.Tuason. In the subdivision plan approved by the Court of First
Instance (CFI) of Rizal, Block 503 was designated as the park/open space for the subdivision. However, it remained
undivided even as the lots surrounding the same were divided into home lots which were sold to interested
buyers.

In 1962, the Quezon City Council passed an ordinance directing all subdivision owners to turn over to the city
government the open spaces in city subdivisions. In compliance with said ordinance, J.M. Tuason executed in favor
of the city government a Deed of Donation over its subdivisions' open spaces which included, among others, Block
494. No record or document exists to show that the donation was, indeed, accepted.

In 1996, for failure of J.M. Tuason to pay its realty taxes, however, Block 494 was scheduled for a tax delinquency
sale by the city government. Despite opposition from the village, the tax delinquency sale pushed through and
Block 494 was sold to J.M. Tuason, the highest bidder.

Later, J.M. Tuason executed a Unilateral Deed of Absolute Sale transferring Block 494 in favor of respondent
Talayan Holdings, Inc. (THI). THI subdivided Block 494 into four lots. THI eventually obtained a loan from
Equitable Banking Corporation, secured by REM over the 4 lots into which Block 494 had been subdivided. HATVI
subsequently filed against J.M. Tuason, THI, Equitable Bank and Mayor Mathay a complaint for annulment of sale,
cancellation of titles and mortgage, acceptance of donation and damages.

ISSUE:

Whether BLOCK 494 remained a private property.

RULING:

YES. Donation was not in a public document. Moreover, it was not accepted by Quezon City; thus, renders the
donation null and void. Absent a deed of donation or legitimate acquisition thereof by the government, the area
claimed to have been reserved for public use and/or as an open space still pertained to the subdivision developer.
That Block 494 was the subject of the tax delinquency sale conducted by the Quezon City government further
serves to confirm the private character of said property. the Block 494 remained in private ownership, HATVI has
neither factual nor legal basis to question the sale thereof by the Quezon City government for tax delinquency. As
highest bidder at the tax delinquency sale, J.M. Tuason was acting well within its rights when it sold the property to
THI which had the right to rely on what appears on the title covering the same.

De Los Santos v. Abejon


G.R. No. 215820, March 20, 2017

Article 453 of the Civil Code provides that where both the landowner and the builder, planter, or sower acted in bad
faith, they shall be treated as if both of them were in good faith.

FACTS:

The instant case arose from a Complaint for Cancellation of Title with collection of sum of money5 filed by
respondents against petitioners before the RTC. The complaint alleged that Erlinda and her late husband Pedro
Delos Santos (Pedro) borrowed the amount of P100,000.00 from the former’s sister, Teresita, as evidenced by a
Promissory Note dated April 8, 1998. As security for the loan, Erlinda and Pedro mortgaged their property
consisting of 43.50 square meters covered by Transfer Certificate of Title (TCT) No. 131753 (subject land) which
mortgage was annotated on the title.

After Pedro died, Erlinda ended up being unable to pay the loan, and as such, agreed to sell the subject land to
Teresita for P150,000.00, or for the amount of the loan plus an additional P50,000.00. On July 8, 1992, they
executed a Deed of Sale and a Release of Mortgage, and eventually, TCT No. 131753 was cancelled and TCT No.
180286 was issued in the name of “Teresita, Abejon[,] married to Alberto S. Abejon.” Thereafter, respondents
constructed a three (3)-storey building worth P2,000,000.00 on the subject land. Despite the foregoing, petitioners
refused to acknowledge the sale, pointing out that since Pedro died in 1989, his signature in the Deed of Sale
executed in 1992 was definitely forged. As such, respondents demanded from petitioners the amounts of
P150,000.00 representing the consideration for the sale of the subject land and P2,000,000.00 representing the
construction cost of the three (3)-storey building, but to no avail. Thus, respondents filed the instant case.

The RTC ruled that respondents should be reimbursed for the amount of the loan, as well as the expenses incurred
for the construction of the three (3)-storey building in view of petitioners’ categorical admission of their
indebtedness to her, as well as the construction of the building from which they derived benefit being the actual
occupants of the property.

ISSUE:

WON the CA correctly held that petitioners should be held liable to respondents in the aggregate amount of
P2,200,000.00, consisting of the loan obligation of P100,000.00, the construction cost of the three (3)-storey
building in the amount of P2,000,000.

RULING:

As correctly argued by petitioners, it is more accurate to apply the rules on accession with respect to immovable
property, specifically with regard to builders, planters, and sowers, as this case involves a situation where the
landowner (petitioners) is different from the owner of the improvement built therein.

Teresita was apprised of Pedro’s death as early as 1990 when she went on a vacation in the Philippines. As such,
she knew all along that the aforesaid Deed of Sale which contained a signature purportedly belonging to Pedro,
who died in 1989, or three (3) years prior to its execution — was void and would not have operated to transfer any
rights over the subject land to her name. Despite such awareness of the defect in their title to the subject land,
respondents still proceeded in constructing a three (3)-storey building thereon. Indubitably, they should be
deemed as builders in bad faith.

On the other hand, petitioners knew of the defect in the execution of the Deed of Sale from the start, but
nonetheless, still acquiesced to the construction of the three (3)-storey building thereon. Hence, they should
likewise be considered as landowners in bad faith.

In this relation, Article 453 of the Civil Code provides that where both the landowner and the builder, planter, or
sower acted in bad faith, they shall be treated as if both of them were in good faith.

Whenever both the landowner and the builder/planter/sower are in good faith (or in bad faith, pursuant to the
aforecited provision), the landowner is given two (2) options under Article 448 of the Civil Code, namely: (a) he
may appropriate the improvements for himself after reimbursing the buyer (the builder in good faith) the
necessary and useful expenses under Articles 546 and 548.

Applying the aforesaid rule in this case, under the first option, petitioner may appropriate for themselves the three
(3)-storey building on the subject land after payment of the indemnity provided for in Articles 546 and 548 of the
Civil Code, as applied in existing jurisprudence. Under this option, respondents would have a right of retention
over the three (3)-storey building as well as the subject land until petitioners complete the reimbursement. Under
the second option, petitioners may sell the subject land to respondents at a price equivalent to the current market
value thereof. However, if the value of the subject land is considerably more than the value of the three (3)-storey
building, respondents cannot be compelled to purchase the subject land. Rather, they can only be obliged to pay
petitioners reasonable rent.

Leviste v. Legaspi Towers


G.R. No. 199353, April 4, 2018
Article 448 will not apply in the case where the parties are bound by legislation or contract, such as the Condominium
Developer and the unit owner, and the owner builds upon the property of the landowner. What will apply will be the
Condominium Act, the Master Deed, and the By-Laws. Therefore, the Condominium Developer may demolish the illegal
structure built by the unit owner (on the latter’s expense) on top of its unit on the rooftop regardless on the good faith
or bad faith of the builder.

FACTS:

Legaspi Towers (Legaspi) owns a building with 7-floors, a unit in the rooftop which has two levels above, called
Concession 2 and Concession 3. Leviste Management System (Lemans) became the owner of the concession 3.
Lemans decided to build another level on top of Concession 3, to be called Concession 4. The building permit was
obtained from the local government of Makati. However, Legaspi notified Lemans that the construction is illegal
since it was not approved in accordance with the By-laws and Master Deed. Legaspi forbade the entry of the
materials to be used for construction.

ISSUE:

Whether or not the rules under Article 448 will apply, and that Legaspi can only choose between to appropriate the
improvement or to compel the builder to buy the property upon which the improvement WAS BUILT.

RULING:

It is an established rule that the unit owner of the condominium unit does not own the airspace above its unit.
Otherwise, all top unit owners can build a structure above it, and another above it, and so on. So, in this case, it is
clear that Lemans built upon condominium’s property.

Next point, the rules on Art. 448 does not apply where there is a contractual relationship between the parties.
Article 448 is intended precisely to prevent forced co-ownership. And in this case involving a condominium
(landowner) and unit owner (builder), the builder is already in a co-ownership with other unit owners or
stockholders of the condominium corporation. What governs is the special law, Condominium Act, instead of the
General Law, the New Civil Code.

In accordance therefore with the Master Deed, the By-laws of Legaspi Towers, and the Condominium Act, Legaspi
is correct that it has the right to demolish Concession 4 at the expense of LEMANS. Indeed, the application of 448 to
the present case is highly iniquitous, in that an owner, also found to be in good faith, will be forced to either
appropriate the illegal structure (and impliedly be burdened with the cost of its demolition) or to allow the
continuance of such an illegal structure that violates the law and the Master Deed, and threatens the structural
integrity of the condominium building upon the payment of rent. The Court cannot countenance such an unjust
result from an erroneous application of the law and jurisprudence.

Wherefore, Lemans is ordered to remove Concession 4 in its own expense.

C. Quieting of Title. NCC 476-481

Oo v. Lim
G.R. No. 154270, March 9, 2010

Whenever there is a cloud on title to real property or any interest in real property by reason of any instrument, record,
claim, encumbrance, or proceeding that is apparently valid or effective, but is, in truth and in fact, invalid, ineffective,
voidable, or unenforceable, and may be prejudicial to said title, an action may be brought to remove such cloud or to
quiet the title.

FACTS:
Lim filed in the RTC a petition for the reconstitution of the owners duplicate copy of OCT for Lot No. 943 of the
Balamban Cadastre in Cebu City, alleging that said OCT had been lost during World War II by his
mother, Luisa; that Lot No. 943 had been sold in 1937 to Luisa by Spouses Oo; and that Antonio Oo, the only
legitimate heir of Spouses Oo, had executed in 1961 in favor of Luisa a notarized document denominated
as confirmation of sale.

Zosimo Oo and petitioner Teofisto Oo (Oos) opposed Lim’s petition, contending that they had the certificate of title
in their possession as the successors-in-interest of Spouses Oo.

Lim converted the petition into a complaint for quieting of title, averring that he and his predecessor-in-interest
had been in actual possession of the property since 1937, cultivating and developing it, enjoying its fruits, and
paying the taxes corresponding to it. He prayed that the Oos be ordered to surrender the reconstituted owners
duplicate copy of the OCT and that said OCT be cancelled and a new certificate of title be issued in the name of
Luisa.

ISSUE:

Whether or not Lim’s action for quieting of title constituted a collateral attack?

RULING:

No. Action for cancellation of title is not an attack on the title. An action or proceeding is deemed an attack on a title
when its objective is to nullify the title, thereby challenging the judgment pursuant to which the title was decreed.
The attack is direct when the objective is to annul or set aside such judgment, or enjoin its enforcement. On the
other hand, the attack is indirect or collateral when, in an action to obtain a different relief, an attack on the
judgment is nevertheless made as an incident thereof.

Whenever there is a cloud on title to real property or any interest in real property by reason of any instrument,
record, claim, encumbrance, or proceeding that is apparently valid or effective, but is, in truth and in fact, invalid,
ineffective, voidable, or unenforceable, and may be prejudicial to said title, an action may be brought to remove
such cloud or to quiet the title. In such action, the competent court is tasked to determine the respective rights of
the complainant and the other claimants, not only to place things in their proper places, and to make the claimant,
who has no rights to said immovable, respect and not disturb the one so entitled, but also for the benefit of both, so
that whoever has the right will see every cloud of doubt over the property dissipated, and he can thereafter
fearlessly introduce the improvements he may desire, as well as use, and even abuse the property as he deems fit.

The averments readily show that the action was neither a direct nor a collateral attack on OCT No. RO-9969-(O-
20449), for Lim was asserting only that the existing title registered in the name of the petitioners predecessors had
become inoperative due to the conveyance in favor of Lims mother, and resultantly should be cancelled. Lim did
not thereby assail the validity of OCT No. RO-9969-(O-20449),or challenge the judgment by which the title of the
lot involved had been decreed. In other words, the action sought the removal of a cloud from Lims title, and the
confirmation of Lim’s ownership over the disputed property as the successor-in-interest of Luisa.

Nagiat v. Quiazon
G.R. No. 201248, March 11, 2015

FACTS:

Plaintiffs are the heirs of the late Epifanio Makam and Severina Bautista who acquired a house and lot on 20 April
1894 situated in Magalang, Pampanga, consisting of Five Hundred Seventy Seven (577) square meters more or less,
by virtue of a Deed of Sale. From 1894 and up to the present, plaintiffs and through their predecessors-in-interest
have been in open, continuous, adverse and notorious possession for more than a hundred years of the piece of
property mentioned above, constructed their houses thereon and dutifully and faithfully paid the real estate taxes
on the said property; Subsequently, sometime in June 2005, plaintiffs received various demand letters from
defendants demanding plaintiffs to vacate the premises, claiming ownership of the subject property and upon
inquiry from the Office of the Register of Deeds of San Fernando, Pampanga, they were able to confirm that their
property had been titled in the name of herein defendants under TCT No. 213777-R. Hence, the petitioners filed a
complaint for Annulment and Quieting of Title before the RTC praying that the title be cancelled and a new title be
issued in their favor. Respondents denied the allegations in the complaint and proffered affirmative defenses with
counterclaims. They contended as an affirmative defense that the petitioners "have no valid, legal and sufficient
cause of action" against them, because their deed of sale was spurious and could not prevail over a Land
Registration Decree. They also contended that the action was barred by prescription and that petitioners were
guilty of laches in asserting their interest over the subject lot. The Trial court and the CA dismissed the complaint
of the petitioner. The lower courts took into account external factors in the dismissal of the complaint on the
ground of lack of cause of action, such as the doubtful deed of sale that did not properly identify the land. They
assert that since petitioners were given reasonable opportunity to present evidence to prove their cause of action,
they are now estopped from invoking the rule that only allegations in the complaint should be considered.

As regards the allegation of prescription, the petitioners countered that an action to quiet title did not prescribe if
the plaintiffs were in possession of the property in question. They also argued that the Court’s jurisdiction should
not depend on factors beyond what they have stated in their complaint.

ISSUE:

Whether or not the petitioner’s properly alleged an action for Quieting of title

HELD:

YES. In order that an action for quieting of title may prosper, two requisites must concur: (1) the plaintiff or
complainant has a legal or equitable title or interest in the real property subject of the action; and (2) the deed,
claim, encumbrance, or proceeding claimed to be casting cloud on his title must be shown to be in fact invalid or
inoperative despite its prima facie appearance of validity or legal efficacy. t is readily apparent from the complaint
that petitioners alleged that (1) they had an interest over the subject property by virtue of a Deed of Sale, dated
April 20, 1894; and that (2) the title of respondents under TCT No. 213777-R was invalid, ineffective, voidable or
unenforceable. Hypothetically admitting these allegations as true, as is required in determining whether a
complaint fails to state a cause of action, petitioners may be granted their claim. Clearly, the complaint sufficiently
stated a cause of action. In resolving whether or not the complaint stated a cause of action, the trial court should
have limited itself to examining the sufficiency of the allegations in the complaint. It was proscribed from inquiring
into the truth of the allegations in the complaint or the authenticity of any of the documents referred or attached to
the complaint, as these were deemed hypothetically admitted by the respondents

De Guzman v. Tabangao Realty


G.R. No. 154262, February 11, 2015

In Ejectment proceedings the only issue for the Court's resolution is, who between the parties is entitled to the physical
or material possession of the subject property. Issues as to ownership are not involved, except only for the purpose of
determining the issue of possession.

Absent any evidence of expropriation proceedings, or sale to the municipal government the property remains private
in nature.

FACTS:

In 1980 Serafin and Josefino de Guzman purchased on credit oil and lubricating products from Shell Petroleum
Corp. (FSPC_, but they eventually failed to pay for their credit purchases.

FSPC filed a complaint for sum of money against Serafin and Josefino before the RTC of Manila. After trial, decision
was rendered finding Serafin and Josefino liable to pay their outstanding obligation to FSPC and the judgment
became final and executory.
In 1983 FSPC levied upon a parcel of land covered by TCT # 3531 in the name of Spouses De guzman.
At the public auction, the sheriff awarded the certificate of sale to Tabangao Realty as the highest bidder and the
same was annotated on TCT # 3531 on April 13, 1988.

The Spouses De Guzman did not redeem the subject property within one year from registration of the Sheriff’s
Certificate of Sale.

On October 19, 2001, upon the deaths of Spouses De Guzman, their heirs filed a complaint for quieting of title on
the ground that the Sheriff’s Certificate of Sale has lost its effectivity as it had been terminated and extinguished by
prescription, laches and estoppel, since more than 13 years having elapsed from its registration on April 13, 1988
without the buyer taking any step to consolidate its ownership and/or take possession of the property.

ISSUE:

Would an action for quieting of title in order to recover levied land based on the fact that the new owners failed to
utilize the same prosper?

RULING:

The court ruled no. For an action to quiet title to prosper, two indispensable requisites must concur: (1) the
plaintiff or complainant has a legal or equitable title or interest in the real property subject of the action; and (2)
the deed, claim, encumbrances, or proceeding claimed to be casting a cloud on his title must be shown to be in fact
or inoperative despite its prima facie appearance of validity or legal efficacy.

Respondent Tabangao was already substituted to and acquired all the rights, title, interest, and claims of the
Spouses De Guzman to the subject property on April 13, 1989, when the one year redemption period expired. Upon
the deaths of Amelia de Guzman on January 1, 1997 and her husband Serafin de Guzman on April 23, 2001, they
had no more rights, title, interest and claim to the subject property on by succession to complainants as their heirs.

The execution of the final deed of sale and/or conveyance to Tabangao is a mere formality and confirmation of the
title already vested in respondent under Rule 39, Section 33 of the Rules of Court. There is nothing in the rules
requiring the institution of a separate action for execution of such a deed. Therefore, no prescription period for any
action has begun to run.

Caldito v. Obado
G.R. No. 181596, Jan. 30, 2017

In an action to quiet to title, plaintiff must first prove that he has legal or an equitable title to or interest in the real
property subject of the action

FACTS:

The record showed that as early as 1921, the lot was declared for taxation purposes in the name of Felipe Obado
(Felipe). After Felipe's death, Paterno, whom Felipe treated like his own son, subsequently occupied the lot and
continued to pay the realty taxes of the same.

In 1995, an affidavit of ownership was executed stating therein that the lot in question was co-owned by Felipe and
his siblings. On the next day, one of the siblings sold the subject parcel of land to the petitioners for the sum of
P70,000.000 evidenced by a Deed of Absolute Sale. In 2002, the petitioners attempted to build a house on the
subject parcel of land but the respondents prevented them from completing the same. Petitioners then filed an
action to quiet ownership against respondents.

ISSUE:

Whether petitioners were able to prove ownership


RULING:

No.

The petitioners' claim of legal title over the subject parcel of land by virtue of the Deed of Sale and Affidavit of
Ownership issued by Antonio cannot stand because they failed to prove the title of their immediate predecessors-
in-interest.

Although tax declarations or realty tax payment of property are not conclusive evidence of ownership, as in the
instant case, they are good indicia of possession in the concept of owner, for no one in his right mind would be
paying taxes for a property that is not in his actual or constructive possession. Indeed, the respondents'
presentation of the tax declarations and tax receipts which all are of ancient era indicates possession in the concept
of an owner by the respondents and their predecessors-in-interests. It is worthy to note that the respondents'
father Paterno to whom they inherited the entire Lot No. 1633 paid the taxes due under his name from 1961 to
1989; and subsequently, the respondents paid the taxes due after the death of Paterno in 2003.

D. Co-Ownership, NCC 484 – 494


NOTE: Condominium Law, RA 4762 as amended by RA 7899. EXCLUDED

Pardell v Bartolome, G.R. No. L-4656, November 18, 1912

Dailisan v. CA
560 SCRA 351

Ownership of the thing sold is acquired only from the time of delivery thereof, either actual or constructive. Article
1498 of the Civil Code provides that when the sale is made through a public instrument, the execution thereof shall
be equivalent to the delivery of the thing which is the object of the contract, if from the deed the contrary does not
appear or cannot be inferred.

FACTS:

Petitioner and Federico initially agreed to the sale of the 1/2 parcel of land which caused petitioner to make partial
payments to Federico. However, further payments by petitioner was stop by Federico because according to him
some portions thereof were subject of a mortgage. When the mortgage was released, petitioner demanded the
execution of the deed of absolute sale. Both parties agreed that Federico will mortgage it first to petitioner and
upon payment thereof, the deed of absolute sale will be given in favor of the petitioner. After 3 months, the loan
was paid. After which, in accordance to their agreement, a deed of absolute sale was executed in favour of
petitioner.

Petitioner asked for the partition of the lot and caused a resurvey to expedite the partition.However, Federico still
refused to effect the partition and even sent a notice of eviction against petitioner.

According to Federico, petitioner is the husband of his niece and that when the couples house was demolished
during martial law, he allowed them out of pity to occupy one fourth () of his lot. While averring that the property
had been the subject of real estate mortgages in favor of other banks, he admitted that he executed in favor of
petitioner a deed of real estate mortgage as security for a P10,000.00 loan. He was able to pay the said loan which
resulted in the cancellation of the mortgage, he added.

However, Federico denied having voluntarily executed the deed of absolute sale, and instead alleged that when he
was seriously ill.

Petitioner then filed a complaint before the RTC alleging that he purchased the one-fourth of the land of Federico.
RTC ruled that respondents failed to disprove the validity of the deed of absolute sale and ruled in favour of the
petitioner and thus, ordered the partition of the same.

On appeal, CA ruled that the action by petitioner for partition was already barred by prescription.

Hence this petition.

Petitioner posits that the CA erred and insists that his action for specific performance has not prescribed because
upon the execution of the deed of sale, ownership of the subject property has passed to him, the buyer, and an
action for specific performance is only incidental to his claim of ownership.

ISSUE:

Whether Petitioner was already an owner of the one-fourth parcel of land in question so as to make him entitled
for the action for specific performance?

HELD:

YES.

Ownership of the thing sold is acquired only from the time of delivery thereof, either actual or constructive. Article
1498 of the Civil Code provides that when the sale is made through a public instrument, the execution thereof shall
be equivalent to the delivery of the thing which is the object of the contract, if from the deed the contrary does not
appear or cannot be inferred.

The Court notes that Federico had already delivered the portion he sold to petitioner, subject of course to the
execution of a technical survey, when he executed the deed of absolute sale, which is a public instrument.

In view of the delivery in law, coupled with petitioners actual occupation of the portion where his house stands, all
that is needed is its segregation from the rest of the property.

Cruz v. Catapang
G.R. No. 164110, Feb. 12, 2008

Consent of only one co-owner will not warrant the dismissal of the complaint for forcible entry filed against the
builder… A co-owner cannot give valid consent to another to build a house on the co-owned property, which is an act
tantamount to devoting the property to his or her exclusive use.

FACTS:

Petitioner Leonor B. Cruz, Luz Cruz and Norma Maligaya are the co-owners of a parcel of land in Taal, Batangas.
With the consent of Norma Maligaya, one of the co-owners, respondent Teofila M. Catapang built a house on a lot
adjacent to the parcel of land sometime in 1992. The house intruded, however, on a portion of the co-owned
property. In the first week of September 1995, petitioner Leonor B. Cruz visited the property and was surprised to
see a part of respondent’s house intruding unto a portion of the co-owned property. She then made several
demands upon respondent to demolish the intruding structure and to vacate the portion encroaching on their
property. The respondent, however, refused and disregarded her demands.

Petitioner then filed a complaint for forcible entry against respondent before the MCTC Batangas. MCTC decided in
favor of petitioner, ruling that consent of only one of the co-owners is not sufficient to justify defendant’s
construction of the house and possession of the portion of the lot in question. RTC Batangas affirmed the MCTC’s
ruling. After her motion for reconsideration was denied by the RTC, respondent filed a petition for review with the
Court of Appeals, which reversed the RTC’s decision. CA held that there is no cause of action for forcible entry in
this case because respondent’s entry into the property, considering the consent given by co-owner Norma
Maligaya, cannot be characterized as one made through strategy or stealth which gives rise to a cause of action for
forcible entry. Hence, this petition.

ISSUE:

Whether the consent given by a co-owner of a parcel of land to a person to construct a house on the co-owned
property warrants the dismissal of a forcible entry case filed by another co-owner against that person.

RULING:

NO. The Court held that a co-owner cannot devote common property to his or her exclusive use to the prejudice of
the co-ownership. In our view, a co-owner cannot give valid consent to another to build a house on the co-owned
property, which is an act tantamount to devoting the property to his or her exclusive use.

Article 486 states each co-owner may use the thing owned in common provided he does so in accordance with the
purpose for which it is intended and in such a way as not to injure the interest of the co-ownership or prevent the
other co-owners from using it according to their rights. Giving consent to a third person to construct a house on the
co-owned property will injure the interest of the co-ownership and prevent other co-owners from using the
property in accordance with their rights.

Under Article 491, none of the co-owners shall, without the consent of the others, make alterations in the thing
owned in common. It necessarily follows that none of the co-owners can, without the consent of the other co-
owners, validly consent to the making of an alteration by another person, such as respondent, in the thing owned in
common. Alterations include any act of strict dominion or ownership and any encumbrance or disposition has
been held implicitly to be an act of alteration. The construction of a house on the co-owned property is an act of
dominion. Therefore, it is an alteration falling under Article 491 of the Civil Code. There being no consent from all
co-owners, respondent had no right to construct her house on the co-owned property.

Consent of only one co-owner will not warrant the dismissal of the complaint for forcible entry filed against the
builder. The consent given by Norma Maligaya in the absence of the consent of petitioner and Luz Cruz did not vest
upon respondent any right to enter into the co-owned property. Her entry into the property still falls under the
classification "through strategy or stealth."

Plasabas v. CA
GR No. 168139, March 31, 2009

A co-owner may file suit without joining all co-owners as co-plaintiffs because the suit is deemed to be instituted for
the benefit of all. But if the judgment is adverse, the same cannot prejudice the rights of the unimpleaded co-owners.

FACTS:

The property subject of the case was a parcel of coconut land Leyte, declared under a Tax Declaration in the name
of petitioner. Petitioners prayed that judgment be rendered confirming their rights and legal title to the subject
property and ordering the defendants to vacate the occupied portion and to pay damages. Respondents interposed
that the subject land was inherited by all the parties from their common ancestor Francisco Plasabas. Petitioner
Nieves, contrary to her allegations in the complaint, was not the sole and absolute owner of the land. The land
passed on from her ascendants and then to her and her siblings. For failure to implead indispensable parties, the
case was dismissed.

ISSUE:

Whether or not the dismissal was proper

RULING:
It is not. The complaint need not implead the co-owners as parties. Article 487 of the Civil Code provides that any
one of the co-owners may bring an action for ejectment. The article covers all kinds of actions for the recovery of
possession, including an accion publiciana and a reivindicatory action. A co-owner may file suit without necessarily
joining all the other co-owners as co-plaintiffs because the suit is deemed to be instituted for the benefit of all. Any
judgment of the court in favor of the plaintiff will benefit the other co-owners, but if the judgment is adverse, the
same cannot prejudice the rights of the unimpleaded co-owners. There is no need to determine whether
petitioners’ complaint is one for ejectment or for recovery of title. Article 487 applies to both actions.

The allegation of petitioners in their complaint that they are the sole owners of the property in litigation is
immaterial, considering that they acknowledged during the trial that the property is co-owned by Nieves and her
siblings, and that petitioners have been authorized by the co-owners to pursue the case on the latter’s behalf.

The rule is settled that the non-joinder of indispensable parties is not a ground for the dismissal of an action. The
remedy is to implead the non-party claimed to be indispensable. Parties may be added by order of the court on
motion of the party or on its own initiative at any stage of the action and/or at such times as are just. If petitioner
refuses to implead an indispensable party despite the order of the court, the latter may dismiss the
complaint/petition for the plaintiff’s/petitioner's failure to comply therewith.

Republic v. Heirs
549 SCRA 58

Each co-owner shall have the full ownership of his part and of the fruits and benefits pertaining thereto, and he may
therefore alienate, assign or mortgage it, and even substitute another person in its enjoyment, except when personal
rights are involved. But the effect of the alienation of the mortgage, with respect to the co-owners, shall be limited to
the portion which may be allotted to him in the division upon the termination of the co-ownership.

FACTS:

Republic of the Philippines, represented by the MCIAA (hereafter petitioner), in its Answer with Counterclaim,
maintained that from the time the lots were sold to its predecessor-in-interest CAA, it has been in open,
continuous, exclusive, and notorious possession thereof; through acquisitive prescription, it had acquired valid
title to the lots since it was a purchaser in good faith and for value; and assuming arguendo that it did not have just
title, it had, by possession for over 30 years. In light of its finding that the heirs of Tito Dignos did not give notice of
the sale to respondents, the trial court held that the period for legal redemption had not yet lapsed; and the
redemption price should be ¼ of the purchase price paid by the CAA for the two lots.

ISSUE:

Whether or not the petitioner acquires the entire property by prescription.

RULING:

No. As early as 1923, this Court has ruled that even if a co-owner sells the whole property as his, the sale will affect
only his own share but not those of the other co-owners who did not consent to the sale. This is because under the
aforementioned codal provision, the sale or other disposition affects only his undivided share and the transferee
gets only what would correspond to his grantor in the partition of the thing owned in common. From the foregoing,
it may be deduced that since a co-owner is entitled to sell his undivided share, a sale of the entire property by one
co-owner without the consent of the other co-owners is not null and void. However, only the rights of the co-
owner-seller are transferred, thereby making the buyer a co-owner of the property. Petitioner’s predecessor-in-
interest CAA thus acquired only the rights pertaining to the sellers-heirs.

Monteroso v. CA
533 SCRA 66 (2008)
As a matter of law, acquisitive prescription does not apply nor set in against compulsory heirs insofar as their pro-
indiviso share or legitime is concerned, unless said heirs repudiate their share.

FACTS:

Don Fabian B. Monteroso, Sr., married twice and sired eight children, four from each union. Don Fabian married
Soledad Doldol. Out of this marriage were born Soledad, Reygula, Benjamin, and Tirso. Soledad Doldol passed
away. A little over a year later, Don Fabian contracted a 2nd marriage with Sofia Pendejito. After the death of his
first wife, but during the early part of his second marriage, Don Fabian filed before the CFI of Agusan an intestate
proceeding for the estate of his deceased first wife docketed as SP No. 309, apparently to obviate any dispute over
the inheritance of his children from his first marriage. CFI approved it as Project of Partition.

The partition in SP No. 309 covered Parcels F-1 to F-5, and adjudicated to Don Fabian the whole of Parcels F-1, F-2,
and F-3, and one-half of Parcel F-5, while the intestate estate of Soledad Doldol comprised the whole of Parcel F-4
and one-half of Parcel F-5. The children of Don Fabian from his first marriage married accordingly: The eldest,
Soledad to Atty. Cagampang, Sr.; Reygula to Jose Bayan; Benjamin to Nakila; and Tirso to Taña. Benjamin died
leaving behind 4 children with wife Nakila, namely: Ruby, Marlene, Adelita, and Henrieto. A year and a half later,
Don Fabian also passed away. The "F" designation signified that the properties were acquired during the 1st
marriage, to distinguish them from those acquired during the 2nd which are designated as "S" properties.

The children of the late Benjamin filed with the RTC a Complaint for Recovery of Property with Damages against
their uncle, Tirso docketed as Civil Case No. 1292. The complaint involved a portion of Parcel F-4, where their
uncle, Tirso, was entrusted with the 1/4 portion of Parcel F-4 as part of the share from the estate of Soledad
allotted to their father per SP No. 309. However, their uncle refused to surrender and deliver the same when they
demanded such delivery upon their reaching the majority age. Tirso countered that the portion pertaining to
Benjamin was never entrusted to him; it was in the possession of their sister, Soledad Cagampang, who was not
entitled to any share in Parcel F-4, having previously opted to exchange her share in said property for another
parcel of land, i.e., Parcel F-7, then being occupied by her. Soledad Cagampang, citing Article 494 of the Civil Code
and Art. 1965 of the Spanish Civil Code, aver that the right to ask partition is proper only where co-ownership is
recognized.

They also suggest that no co-ownership obtains in this case considering that no less than Tirso avers in his
complaint in Civil Case No. 1332 that from the time of Don Fabian's death in 1948, the lots in question have been in
the exclusive, adverse, and public possession of the Cagampang spouses. Cagampang submit that partition is not
proper, ergo unavailing, but an action for reconveyance which is subject to the rules on extinctive prescription.
Cagampang further assert that there being no co-ownership over the properties sold by Don Fabian to Soledad
Cagampang, Tirso's cause of action, under the Code of Civil Procedure (Act No. 190) in relation to Art. 1116 of the
Civil Code, had already prescribed, either in 1949, i.e., 10 years after the subject properties were registered in
Soledad Cagampang's name, or in 1958, i.e., 10 years after the cause of action accrued in 1948 (death of Don
Fabian). Petitioners contend that the evidence adduced clearly demonstrates that Soledad Cagampang acquired
ownership of the subject properties by virtue of the deeds of sale executed in 1939 by Don Fabian. After the sale,
she registered them under her name and then took exclusive, adverse, and public possession over them. Thus, they
submit that the prescriptive period applicable to the instant case under Act No. 190 had long expired, adding that
the CA erred in finding that Soledad Cagampang repudiated the co-ownership only in 1961 when she and the other
heirs ignored the demand of Tirso for partition. Petitioners alleged that the exclusion of Tirso from the enjoyment
of the fruits of the subject properties since after the death of Don Fabian in 1948 is consistent with Soledad
Cagampang's claim of exclusive ownership and dominion.

ISSUES:

1. Whether the CA committed reversible error in concluding that, "By invoking the benefits of prescription in
their favor, the Cagampang spouses are deemed to have admitted the existence of a co-ownership."
2. Whether the CA committed reversible error in upholding partition as the proper remedy of private respondent
Tirso Monteroso to recover the properties sold by Fabian Monteroso, Sr. to Soledad D. Monteroso de
Cagampang when co-ownership is not pleaded as theory in the Complaint.

3. Whether the CA committed reversible error in holding that the cause of action of private respondent Tirso
Monteroso is not barred by extinctive prescription and laches.

RULING:

1. NO. By invoking the benefits of prescription in their favor, the Cagampang spouses are deemed to have
admitted the existence of a co-ownership among the heirs of Fabian Monteroso, Sr. over the properties
forming the decedent's estate. The fact that Tirso and the other compulsory heirs of Don Fabian were excluded
from the possession of their legitime and the enjoyment of the fruits thereof does not per se argue against the
existence of a co-ownership.

2. NO. Contrary to petitioners' stance, reconveyance is not the proper remedy available to Tirso. Be it
remembered in this regard that Tirso is not asserting total ownership rights over the subject properties, but
only insofar as his legitime from the intestate estate of his father, Don Fabian, is concerned. Tirso being a co-
owner of the subject undivided properties whose rights thereto, as a compulsory heir, accrued at the moment
of death of Don Fabian, vis-à-vis the defense of acquisitive prescription foisted by the Cagampang spouses.
This Court is convinced that Tirso Monteroso's principal cause of action is unmistakably one for partition
which by its very nature is imprescriptible and cannot be barred by laches x x x. The only exception to the rule
on the imprescriptibility of an action for partition is provided in a case where the co-ownership of the
properties sought to be partitioned had been properly repudiated by a co-owner at which instance the remedy
available to the aggrieved heirs lies not in action for partition but for reconveyance which is subject to the
rules on extinctive prescription.

3. NO. Before partition and eventual distribution of Don Fabian's intestate estate, a regime of co-ownership
among the compulsory heirs existed over the undivided estate of Don Fabian. Being a co-owner of that
intestate estate, Tirso's right over a share thereof is imprescriptible. As a matter of law, acquisitive
prescription does not apply nor set in against compulsory heirs insofar as their pro-indiviso share or legitime
is concerned, unless said heirs repudiate their share. The repudiation which must be clear and open as to
amount to an express disavowal of the co-ownership relation happened not when the deeds of absolute sale
were executed in 1939, as these could not have amounted to a clear notice to the other heirs, but in 1961 when
the Cagampang spouses refused upon written demand by Tirso for the partition and distribution of the
intestate estate of Don Fabian. Since then, Tirso was deemed apprised of the repudiation by the Cagampang
spouses.

Quimpo v. Abad
545 SCRA 174 (2008)

Any co-owner may demand at any time the partition of the common property unless a co-owner has repudiated the co-
ownership. This action for partition does not prescribe and is not subject to laches.

FACTS:

Eustaquia Perfecto-Abad (Eustaquia), the owner of four parcels of land in Goa, Camarines Sur, died intestate in
1948 leaving these parcels of land to her grandchild and great grandchildren, namely, Joaquin Quimpo (father of
the petitioners) and respondents Consuelo, Ireneo, Danilo, Marites, Anita and Helen, all surnamed Abad.

In 1966, Joaquin and respondents undertook an oral partition of parcel III and parcel IV. Half of the properties was
given to Joaquin and the other half to the respondents. In 1989, Danilo, Marites, Anita and Helen, who were minors
at the time of the partition, wanted to take possession of the portions allotted to them, but Joaquin prevented them
from occupying the same. Joaquin also refused to heed respondents’ demand for partition of parcels I and II,
prompting respondents to file a complaint for judicial partition and/or recovery of possession with accounting and
damages.

In his defense, Joaquin asserted absolute ownership over parcels III and IV, claiming that he purchased these lands
from Eustaquia in 1946, evidenced by deeds of sale executed on August 23, 1946 and December 2, 1946.

The RTC ruled in favor of respondents. It declared void the purported deeds of sale after finding that at the time of
the execution of these deeds, Joaquin was not gainfully employed and had no known source of income, which
shows that the deeds of sale state a false and fictitious consideration. CA affirmed.

ISSUE:

Whether co-ownership exists among the petitioners and respondents over the subject parcels of land

RULING:

Yes. First, SC took notice of the fact that at the time of the execution of the Deed, Joaquin was not gainfully
employed and had no financial capacity to buy the subject properties.

Second, for forty-three (43) years, Consuelo and Ireneo occupied their portions of the Jose property and
significantly, Joaquin never disturbed their possession. They also installed tenants and Joaquin did not prevent
them from doing so, nor did he assert his ownership over the same. These unerringly point to the fact that there
was indeed an oral partition of parcels III and IV. On general principle, independent and in spite of the statute of
frauds, courts of equity have enforced oral partition when it has been completely or partly performed.

Lastly, Joaquin admitted during pre-trial the relationship of the respondents with Eustaquia. Consuelo was the
grandchild of Eustaquia, while respondents Danilo, Helen, Marites, Anita and also Joaquin Quimpo were
Eustaquia’s great grandchildren. As such, respondents can rightfully ask for the confirmation of the oral partition
over parcels III and IV, and the partition of parcels I and II. Jurisprudence is replete with rulings that any co-owner
may demand at any time the partition of the common property unless a co-owner has repudiated the co-
ownership. This action for partition does not prescribe and is not subject to laches.

Lacbayan v. Samoy
G.R. No. 165427, March 21, 2011

FACTS:

Betty Lacbayan (petitioner) and Bayani S. Samoy (respondent) had an illicit relationship. During their relationship,
they, together with three more incorporators, were able to establish a manpower services company. The company
acquired five parcels of land which were registered in petitioner and respondent’s names, ostensibly as husband
and wife. When their relationship turned sour, they decided to divide the said properties and terminate their
business partnership by executing a Partition Agreement.

Initially, respondent agreed to petitioner’s proposal that the properties in Malvar St. and Don Enrique Heights be
assigned to the latter, while the ownership over the three other properties will go to respondent. However, when
Lacbayan wanted additional demands to be included in the partition agreement, Samoy refused. Feeling aggrieved,
petitioner filed a complaint for judicial partition of the said properties. Petitioner’s contention: She claimed that
they started to live together as husband and wife in 1979 without the benefit of marriage and worked together as
business partners, acquiring real properties amounting to P15, 500,000.00. Respondent’s contention: He
purchased the properties using his own personal funds.

RTC and CA ruled in favor or respondent.

ISSUES:
1. WON an action for partition precludes a settlement on the issue of ownership.
2. Would a resolution on the issue of ownership subject the Torrens title issued over the disputed realties to a
collateral attack?

HELD:

1. No. While it is true that the complaint involved here is one for partition, the same is premised on the existence or
non-existence of co-ownership between the parties. Until and unless this issue of co-ownership is definitely and
finally resolved, it would be premature to effect a partition of the disputed properties. More importantly, the
complaint will not even lie if the claimant, or petitioner in this case, does not even have any rightful interest over
the subject properties.

A careful perusal of the contents of the so-called Partition Agreement indicates that the document involves matters
which necessitate prior settlement of questions of law, basic of which is a determination as to whether the parties
have the right to freely divide among themselves the subject properties.

2. No. There is no dispute that a Torrens certificate of title cannot be collaterally attacked, but that rule is not
material to the case at bar. What cannot be collaterally attacked is the certificate of title and not the title itself. The
certificate referred to is that document issued by the Register of Deeds known as the TCT. In contrast, the title
referred to by law means ownership which is, more often than not, represented by that document.

Moreover, placing a parcel of land under the mantle of the Torrens system does not mean that ownership thereof
can no longer be disputed. Mere issuance of the certificate of title in the name of any person does not foreclose the
possibility that the real property may be under co-ownership with persons not named in the certificate, or that the
registrant may only be a trustee, or that other parties may have acquired interest over the property subsequent to
the issuance of the certificate of title. Needless to say, registration does not vest ownership over a property, but
may be the best evidence thereof.

Tecson v. Fausto
G.R. No. 180683, June 1, 2011

FACTS:

Atty. Fausto and his sister Waldetrudes acquired Lot 2189 in co-ownership with each other. Not long after, they
decided to partition the lot. For this purpose, they adopted a subdivision plan which divided Lot 2189 into two lots,
i.e., Lot 2189-A with an area of 507 sqm. and Lot 2189-B with an area of 508 sqm.

Atty. Fausto died. He was survived by respondents, who are his wife and children.

Waldetrudes on the other hand entered into a Contract to Sell with Aurora Tecson. Waldetrudes undertook to sell
her "ideal share" in Lot 2189 to Aurora upon full payment of the purchase price. A second subdivision plan was
adopted and which drastically altered the division of Lot 2189 under the first plan. Waldetrudes’ Lot 2189-A was
now Lot 2189-B with an increased area of 964 sqm. while Atty. Fausto’s Lot 2189-B was now Lot 2189-A with a
decreased area of 51 sqm.

Thereafter, Waldetrudes sold Lot 2189-B, with an area of 964 sqm. to Aurora. Aurora instead conveyed Lot 2189-B
to her brother Atty. Tecson.

Seven years after, the respondents filed a Complaint for the Declaration of Nullity of Documents, Titles,
Reconveyance and Damages against Waldetrudes and the Tecsons. The respondents seek the recovery of 457 sqm.
of land, which they believe was unlawfully taken from the lawful share of Atty. Fausto in Lot 2189.

The Tecsons for their part claim that Atty. Fausto and Waldetrudes were not actually co-owners in equal share of
Lot 2189 or were not even co-owners at all. According to them, Lot 2189 was originally the conjugal property of
Waldetrudes and her late husband. it was only Waldetrudes who initially declared Lot 2189 for taxation purposes
per Tax Declaration No. 6521.

They also claim that Waldetrudes consented to such a registration only because Atty. Fausto had already
constructed his house on a portion of Lot 2189. The registered co-ownership between Waldetrudes and Atty.
Fausto is, therefore, based merely on the siblings’ actual occupancy of Lot 2189.

ISSUE:

Whether Waldetrudes and Atty. Fausto are co-owners of the property in equal share.

RULING:

YES. First. The mother title of Lot 2189, OCT No. 734, states in no unclear terms that Waldetrudes and Atty. Fausto
were co-owners of the subject lot. The inscription in the original title for Lot 2189 carries more than sufficient
weight to prove the existence of a co-ownership between Waldetrudes and Atty. Fausto.

Second. Other than the bare assertion of the petitioners, there is absolutely no proof on record that Waldetrudes
was the sole beneficial owner of Lot 2189. Tax Declaration No. 6521 simply cannot prevail over OCT No. 734 as
conclusive evidence of the true ownership of Lot 2189.

Third. During the cadastral proceeding involving Lot 2189, Waldetrudes herself stated that Atty. Fausto was a co-
owner of the subject lot.

Fourth. There was likewise no evidence behind the petitioners’ allegation that the registered co-ownership
between Waldetrudes and Atty. Fausto was based on their actual occupancy of Lot 2189. On the contrary, OCT No.
734 categorically states that Waldetrudes and Atty. Fausto are co-owners "in undivided share" of Lot 2189. The
conspicuous silence of OCT No. 734 as to the definite extent of the respective shares of Atty. Fausto and
Waldetrudes in Lot 2189 gives rise to a presumption that they are in equal measure. Article 485 of the Civil Code
provides:

Article 485. x x x.
The portions belonging to the co-owners in the co-ownership shall be presumed equal, unless the contrary
is proved.

Fifth. The equality in terms of share in Lot 2189, was affirmed by Waldetrudes when she testified in open court.

Angeles v. Pascual
G.R. No. 157150, Sept. 21, 2011

The owner of the land has the option to appropriate the building or obliging the builder in good faith to pay the value
of the land. However, the builder cannot be obliged to buy the land if its value is considerably more than that of the
building or trees. In such case, he shall pay reasonable rent, if the owner of the land does not choose to appropriate the
building or trees after proper indemnity. Preference is given to the owner of the land to make such choice.

FACTS:

Pascual (owner of Lot 4) and Angeles (owner of Lot 5) were neighbors. Each of them built a house on his respective
lot, believing that his respective lot was properly delineated. However, when Metrobank bought Lot 3, it was
discovered that Pascual’s house encroached on said lot. Thus, Metrobank successfully ejected Pascual. Pascual, on
the other hand, caused the relocation survey of his own and discovered that Angeles’ house also encroached on his
lot. He demanded rentals for the use of the encroached area, or the removal of Angeles house. Angeles refused the
demand. Accordingly, Pascual sued Angeles for recovery of possession and damages in the RTC. RTC rendered
judgment in favor of Pascual; CA affirmed the RTC, modifying the latter’s ruling and applied Art. 448 of the Civil
Code (which defined the rights of a builder, sower and planter in good faith).
ISSUE:

Whether or not CA’s application of Article 448 of the Civil Code was correct and proper

RULING:

YES. As found and declared by the RTC and CA, Angeles is a builder in good faith which can be drawn from the fact
that he insists that he built his house entirely on his own lot. Good faith consists in the belief of the builder that the
land he is building on is his and in his ignorance of a defect or flaw in his title. Since it is proven that Angeles house
straddled the lot of Pascual, and that Angeles had built his house in good faith, Article 448 of the Civil Code is
unquestionably applicable. Consequently, the land being the principal and the building the accessory, preference is
given to Pascual as the owner of the land to make the choice as between appropriating the building or obliging
Angeles as the builder to pay the value of the land. Contrary to the insistence of Angeles, therefore, no
inconsistency exists between the finding of good faith in his favor and the grant of the reliefs set forth in Article
448 of the Civil Code.

Ampil v. Manahan
11 October 2012

Well established is the rule that ownership over the land cannot be acquired by mere occupation. While it is true that
tax declarations are not conclusive evidence of ownership, they, nevertheless, constitute at least proof that the holder
has a claim of title over the property. It strengthens one's bona fide claim of acquisition of ownership.

FACTS:

On February 14, 2003, Exequiel G. Ampil (Exequiel), as representative of the heirs of the late Albina G. Ampil
(Albina), filed a complaint for ejectment against spouses Perfecto Manahan (Perfecto) and Virginia Manahan,
Teresita Manahan, Almario Manahan, Irene Manahan and all persons claiming rights under them. In the complaint,
it was alleged that Albina was the owner of two (2) adjoining residential lots, situated in Sto. Niño, Paombong,
Bulacan, as evidenced by Tax Declaration No. 020-17-013-0007-00001-L;10 and Lot 74211 with an area of three
hundred eighty-two (382) square meters, as evidenced by Tax Declaration No. 020-17-013-0007-00002-L. They
asserted that during her lifetime, Albina allowed Perfecto and his family to occupy a portion of the said properties
on the condition that they would vacate the same should the need to use it arise.

After the death of Albina in 1986, her heirs, represented by Exequiel, requested Perfecto and family to vacate the
property in question but the latter refused.

The MTC rendered judgment in favor of the petitioners and the RTC affirmed. The CA however, reversed the said
Decision.

ISSUE:

Whether petitioners have the better right to the physical possession of the disputed property?

RULING:

YES. A perusal of the records shows that respondents’ occupation of the lot in question was by mere tolerance. To
prove ownership over the property, the petitioners presented the tax declarations covering the properties and a
certification issued by the Municipality of Paombong, Bulacan, showing that their mother, Albina, had been paying
the corresponding real property taxes thereon. Petitioners also submitted a survey plan, dated August 5, 1968,
prepared by Geodetic Engineer Roberto H. Dimailig, in support of Albina’s application for land registration over the
disputed lots. In fact, on December 14, 2006, the Registry of Deeds of Bulacan issued Katibayan ng Orihinal na
Titulo Blg. P-13627, conferring title over Lot 742 in the names of the heirs of Albina.
On the other hand, respondents could not present proof that they and their predecessors-in-interest had openly
and continuously possessed the subject land since time immemorial. Granting that respondents or their
predecessors-in-interests had been in possession in the concept of an owner since time immemorial, none of them
declared the disputed lots for taxation purposes and, thus, never paid taxes thereon. Respondents' allegation that
they were in peaceful, continuous and adverse possession of the lots in question, unsupported by any evidence, is
not substantial to establish their interest over the property.

Well established is the rule that ownership over the land cannot be acquired by mere occupation. While it is true
that tax declarations are not conclusive evidence of ownership, they, nevertheless, constitute at least proof that the
holder has a claim of title over the property. It strengthens one's bona fide claim of acquisition of ownership.

Figuracion v. Gerilla
February 13, 2013

Co-heirs or co-owners cannot acquire by acquisitive prescription the share of the other co-heirs or co-owners absent a
clear repudiation of the co ownership. The act of repudiation, as a mode of terminating co-ownership, is subject to
certain conditions, to wit: (1) a co-owner repudiates the co-ownership; (2) such an act of repudiation is clearly made
known to the other co-owners; (3) the evidence thereon is clear and conclusive; and (4) he has been in possession
through open, continuous, exclusive, and notorious possession of the property for the period required by law.

FACTS:

Leandro Figuracion died intestate in May 1958. Subject of the dispute are two parcels of land both situated in
Urdaneta, Pangasinan, which were acquired by Leandro during his lifetime. These properties were: (1) Lot No.
2299 and (2) Lot No. 705. Also involved in the controversy is Lot No. 707 originally owned by Eulalio Adviento.

Eulalio begot Agripina Adviento with his first wife Marcela Estioko, whom Eulalio survived. When he remarried,
Eulalio had another daughter, herein petitioner Carolina, with his second wife, Faustina Escabesa.

On November 28, 1961, Agripina executed a Deed of Quitclaim over the eastern half of Lot No. 707 in favor of her
niece, herein respondent Emilia.

On December 11, 1962, petitioner Carolina executed an Affidavit of Self-Adjudication adjudicating unto herself the
entire Lot No. 707 as the sole and exclusive heir of her deceased parents, Eulalio and Faustina. Carolina also
executed a Deed of Absolute Sale over Lot No. 707 in favor of petitioners Hilaria and Felipa, who in turn
immediately caused the cancellation of OCT No. 15867 and the issuance of TCT No. 42244 in their names.

In 1971, Emilia and her family went to the United States and returned to the Philippines only in 1981. Upon her
return and relying on the Deed of Quitclaim, she built a house on the eastern half of Lot No. 707.

The legal debacle of the Figuracions started in 1994 when Hilaria and her agents threatened to demolish the house
of Emilia who, in retaliation, was prompted to seek the partition of Lot No. 707 as well as Lot Nos. 2299 and 705.

RTC rendered its Decision that a partition of Lot Nos. 2299 and 705 will be premature since their ownership is yet
to be transmitted from Leandro to his heirs; Carolina transferred only her one-half (½) share to Felipa and Hilaria
and any conveyance of the other half pertaining to Agripina was void. RTC refused to adjudicate the ownership of
the lot’s eastern half portion in favor of respondent Emilia since a settlement of the estate of Eulalio is yet to be
undertaken.

On appeal, the CA, ruled that the RTC erred in refusing to partition Lot No. 707. The CA agreed with the RTC that a
partition of Lot Nos. 2299 and 705 is indeed premature considering that there is a pending legal controversy with
respect to Lot No. 705 and Lot No. 2299. The CA explained that there is no necessity for placing Lot No. 707 under
judicial administration since Carolina had long sold her ½ pro indiviso share to Felipa and Hilaria.
Respondent Emilia appealed the CA’s decision to the SC. The SC denied the appeal, concurring with the CA’s ruling
that a partition of Lot Nos. 2299 and 705 would be inappropriate considering that: (1) the ownership of Lot No.
705 is still in dispute; and (2) there are still unresolved issues as to the expenses chargeable to the estate of
Leandro.

The petitioners argue that respondent Emilia has no valid basis for her claim of ownership because the Deed of
Quitclaim executed in her favor by Agripina was in fact a deed of donation that contained no acceptance
Respondent Emilia contends that the Deed of Quitclaim should be considered an onerous donation that requires no
acceptance as it is governed by the rules on contracts and not by the formalities for a simple donation.

ISSUES:

1. WON the respondent can compel the partition of Lot 707 – YES.
2. WON the respondent’s right to demand for partition is not barred by laches – YES

RULING:

1. The respondent can compel the partition of Lot No. 707. The first stage in an action for partition is the
settlement of the issue of ownership.

Mere issuance of a certificate of title in the name of any person does not foreclose the possibility that the real
property may be under coownership with persons not named in the certificate, or that the registrant may only be a
trustee, or that other parties may have acquired interest over the property subsequent to the issuance of the
certificate of title. Stated differently, placing a parcel of land under the mantle of the Torrens system does not mean
that ownership thereof can no longer be disputed. The certificate cannot always be considered as conclusive
evidence of ownership. In this case, co-ownership of Lot No. 707 was precisely what respondent Emilia was able to
successfully establish, as correctly found by the RTC and affirmed by the CA.

2. The respondent’s right to demand for partition is not barred by acquisitive prescription or laches.

Records do not reflect conclusive evidence showing the manner of occupation and possession exercised by Hilaria
and Felipa over the lot from the time it was registered in their names. The only evidence of possession extant in the
records dates back only to 1985 when Hilaria and Felipa declared the lot in their names for taxation purposes.50
Prescription can only produce all its effects when acts of ownership, or in this case, possession, do not evince any
doubt as to the ouster of the rights of the other co-owners. Hence, prescription among co-owners cannot take place
when acts of ownership exercised are vague or uncertain.

Anent laches, he Court finds it unavailing in this case in view of the proximity of the period when the co-ownership
was expressly repudiated and when the herein complaint was filed.

On the Partition of Lot No. 707: Under the Old Civil Code which was then in force at the time of Eulalio and
Marcela’s marriage, Lot No. 707 was their conjugal property. When Marcela died, one-half of the lot was
automatically reserved to Eulalio, the surviving spouse, as his share in the conjugal partnership. Marcela’s rights to
the other half, in turn, were transmitted to her legitimate child, Agripina and surviving spouse Eulalio. When he
remarried, Eulalio’s one half portion of the lot representing his share in the conjugal partnership and his
usufructuary right over the other half were brought into his second marriage with Faustina. When Eulalio died on
July 20, 1930, ¼ portion of the lot was reserved for Faustina as her share in the conjugal partnership. The
remaining ¼ were transmitted equally to the widow Faustina and Eulalio’s children, Carolina and Agripina.

Upon the death of Faustina, the shares in Lot No. 707 were in turn inherited by Carolina. Consequently, Agripina is
entitled to 5/8 portion of Lot No. 707 while the remaining 3/8 pertains to Carolina. Thus, when Carolina sold Lot
No. 707 to Hilaria and Felipa, the sale affected only 3/8 portion of the subject lot. Since the Deed of Quitclaim,
bequeathed only the ½ eastern portion of Lot No. 707 in favor of Emilia instead of Agripina’s entire 5/8 share
thereof, the remaining 1/8 portion shall be inherited by Agripina’s nearest collateral relative, who, records show, is
her sister Carolina.
In sum, the CA committed no reversible error in holding that the respondent is entitled to have Lot No. 707
partitioned. The CA judgment must, however, be modified to conform to the above-discussed apportionment of the
lot among Carolina, Hilaria, Felipa and Emilia.

Ining v. Vega
G.R. No. 174727, August 12, 2013

FACTS:

Leon Roldan (Leon), married to Rafaela Menez (Rafaela), is the owner of a 3,120-square meter parcel of land
(subject property) in Kalibo, Aklan. Leon and Rafaela died without issue. Leon was survived by his siblings Romana
Roldan (Romana) and Gregoria Roldan Ining (Gregoria), who are now both deceased.

Romana was survived by her daughter Anunciacion Vega and grandson, herein respondent Leonardo R. Vega
(Leonardo) (also both deceased). Leonardo in turn is survived by his wife Lourdes and children Restonilo I. Vega,
Crispulo M. Vega, Milbuena Vega-Restituto and Lenard Vega, the substituted respondents. Gregoria, on the other
hand, was survived by her six children. In short, herein petitioners, except for Ramon Tresvalles (Tresvalles) and
Roberto Tajonera (Tajonera), are Gregoria’s grandchildren or spouses thereof (Gregoria’s heirs). Tresvalles and
Tajonera are transferees of the said property.

In 1997, acting on the claim that one-half of subject property belonged to him as Romana’s surviving heir,
Leonardo filed with the Regional Trial Court (RTC) of Kalibo, Aklan Civil Case No. 5275 for partition, recovery of
ownership and possession, with damages, against Gregoria’s heirs. In their Answer with counterclaim, Gregoria’s
heirs (through son Antipolo) claimed that Leonardo had no cause of action against them; that they have become
the sole owners of the subject property through Lucimo Sr. who acquired the same in good faith by sale from Juan
Enriquez (Enriquez), who in turn acquired the same from Leon, and Leonardo was aware of this fact.

ISSUE:

Whether the CA committed grave abuse of discretion in reversing the decision of the trial court on the ground that
Lucimo Francisco repudiated the co-ownership only on February 9, 1979.

HELD:

No, the Court of Appeals is correct. The finding that Leon did not sell the property to Lucimo Sr. had long been
settled and had become final for failure of petitioners to appeal. Thus, the property remained part of Leon’s estate.
Leon died without issue; his heirs are his siblings Romana and Gregoria. Gregoria’s and Romana’s heirs are co-
owners of the subject property. No prescription shall run in favor of one of the co-heirs against the others so long
as he expressly or impliedly recognizes the co-ownership. For prescription to set in, the repudiation must be done
by a co-owner. The CA held that prescription began to run against Leonardo only in 1979 – or even in 1980 – when
it has been made sufficiently clear to him that Lucimo Sr. has renounced the co-ownership and has claimed sole
ownership over the property. The CA thus concluded that the filing of Civil Case No. 5275 in 1997, or just under 20
years counted from 1979, is clearly within the period prescribed under Article 1141.

Lucimo Sr. is not a co-owner of the property. Indeed, he is not an heir of Gregoria; he is merely Antipolo’s son-in-
law, being married to Antipolo’s daughter Teodora.

One who is merely related by affinity to the decedent does not inherit from the latter and cannot become a co-
owner of the decedent’s property. Consequently, he cannot effect a repudiation of the co-ownership of the estate
that was formed among the decedent’s heirs.

Recio v. Altamirano
G.R. No. 182349, July 24, 2013
FACTS:

Nena Recio, mother of Reman Recio leased from the Altamiranos a parcel of land with improvements. The
Altamiranos inherited the subject land from their deceased parents, the spouses Aguedo Altamirano and Maria
Vaduvia. The sale of the land to Nena Recio did not materialize. The Altamiranos consolidated the two parcels of
land covered by the TCT and subdivided into 3 parcels of lands. Reman and his family remained in the peaceful
possession of Lot 3. He renewed Nena’s option to buy the subject property. They conducted negotiations with
Alejandro who introduced himself as representing the other heirs. After which, the Altamiranos through Alejandro
entered into an oral contract of sale with the petitioner and made partial payments which Alejandro
received. Then, the petitioner offered to pay the remaining balance, but Alejandro kept on avoiding the petitioner.
Recio filed a case and while its pending, it was discovered that the property was sold to respondents Spouses
Lajarca.The RTC ruled that the Absolute Sale between Altamiranos and the Lajarcas was Null and Void, but the
Court of Appeals modified that the sale between Alejandro and Recio is valid only with respect to the aliquot share
of Alejandro. CA held that Alejandro’s sale of Not. No. 3 did not bind his co-owners because a sale of real property
by one purporting to be an agent of the owner without any written authority from the latter is null and void. An
SPA from co-owners pursuant to Art 1878 of the NCC is necessary.

ISSUE:

Can the contract of sale between Alejandro (representing the share of his co-owners) and Recio be HELD VALID
PURSUANT TO APPARENT AUTHORITY OF AN AGENT BASED ON ESTOPPEL?

HELD:

No. Woodchild Holdings, Inc. vs. Roxas Electric and Construction Company, Inc.stressed that apparent authority
based on estoppel can rise from the principal who knowingly permit the agent with indicia of authority that would
lead a reasonable prudent person to believe that he actually has such authority. Apparent authority of an agent
arises only from acts or conducts on the part of the principal and such act or conduct of the principal must have
been known and relied upon in good faith and as a result of the exercise of a reasonable prudence by a third person
as claimant and such must have produced a change of position to its detriment. In this case, there was no evidence
on record of specific acts which the Altamiranos made before the sale to the petitioner, indicating that they fully
knew of the representation of Alejandro. All that the petitioner relied upon were acts that happened after the sale
to him. Absent the consent of Alejandro’s co-owners, the Court held that the sale between the other Altamarinos
and the petitioner was null and void.

Teodoro v Espino, et al.


G.R. No. 189248, Feb. 5, 2014

Neither party can exclude the other from possession.

FACTS:

The subject property is a portion within Lot No. 2476 registered in the name of Genaro, long deceased ascendant of
all the parties.

Genaro had five children: Santiago; Maria, from whom respondents descended and trace their claim of ownership
and right of possession; Petra, Mariano, Teodoro Teodoro’s father; and Ana. Genaro and his children are all
deceased. Respondents’ respective parents are first cousins of Teodoro Teodoro. All parties are collateral relatives
of Petra Teodoro.

Of all Genaro’s children, only Petra occupied the subject property, living at the ancestral house. After Petra’s death,
her purported will, a holographic will, was probated before RTC, Malolos, Bulacan, which Decision on the will’s
extrinsic validity has become final and executory. In the will, Petra, asserting ownership, devised the subject
property to Teodoro Teodoro, who later demolished the ancestral house. Later, respondents, erected a fence on the
surrounding portion, barricaded its frontage, effectively dispossessing Teodoro Teodoro of the property. After
Teodoro Teodoro’s demand for respondents to vacate the subject property went unheeded, he filed the complaint
for forcible entry against respondents

ISSUE:

WON the act of respondents amounted to Teodoro Teodoro’s unlawful dispossession through the forcible entry of
respondents.

HELD:

YES. The Court agreed with the decision of the RTC which ruled in favor of Teodoro and ordered the ejectment of
respondents from the subject property.

Exclusive ownership of Lot No. 2476 or a portion thereof is not in this case required of Teodoro Teodoro for him to
be entitled to possession. Co-ownership, the finding of both the MTC at first instance and by the RTC on appeal, is
sufficient. The pertinent provisions of the Civil Code are Article 484 and 1078.
The whole of Lot No. 2476 including the portion now litigated is, owing to the fact that it has remained registered
in the name of Genaro who is the common ancestor of both parties herein, co-owned property. All, or both Teodoro
Teodoro and respondents are entitled to exercise the right of possession as co-owners.

Neither party can exclude the other from possession. Although the property remains unpartitioned, the
respondents in fact possess specific areas. Teodoro Teodoro can likewise point to a specific area, which is that
which was possessed by Petra. Teodoro Teodoro cannot be dispossessed of such area, not only by virtue of Petra's
bequeathal in his favor but also because of his own right of possession that comes from his co-ownership of the
property.

Arambulo v. Nolasco
G.R. No. 189420, March 26, 2014

Co-owners such as respondents have over their part, the right of full and absolute ownership. Such right is the same as
that of individual owners which is not diminished by the fact that the entire property is co-owned with others. That
part which ideally belongs to them, or their mental portion, may be disposed of as they please, independent of the
decision of their co-owners. So we rule in this case. The respondents cannot be ordered to sell their portion of the co-
owned properties.

FACTS:

Petitioners, together with their siblings and their mother co-owned a 233sq.m. Land in Tondo, Manila. When their
mother died, she was succeeded by her husband, Genero Nolasco and their children.

On January 8, 1999, petitioners filed a petition for relief alleging that all co-owners, except for Nolasco, have
authorized to sell their respective shares to the properties, saying that in the Civil Code, if one or more co-owners
withhold their consent to the alterations in the thing owned in common, the courts may afford adequate relief.

Nolasco responded that they did not know about the intention to sell, because they were not called to participate in
the negotiations regarding the sale of the property.

RTC ruled in favor of the petitioners and ordered respondents to give their consent to the sale. CA reversed the
decision, holding that the petitioners cannot compel respondents to give consent. CA ruled that the respondents
had the full ownership of their undivided interest in the subject properties, thus, they cannot be compelled to... sell
their undivided shares in the properties.

ISSUE:

WON respondents as co-owners can be compelled by the court to give their consent to the sale.
RULING:

There is co-ownership whenever, as in this case, the ownership of an undivided thing, belongs to different persons.
Article 493 of the Code defines the ownership of the co-owner, clearly establishing that each co-owner shall have
full ownership of his part and of its fruits and benefits. Pertinent to this case, Article 493 dictates that each one of
the parties herein as co-owners with full ownership of their parts can sell their fully owned part. The sale by the
petitioners of their parts shall not affect the full ownership by the respondents of the part that belongs to them.
Their part which petitioners will sell shall be that which may be apportioned to them in the division upon the
termination of the co-ownership. With the full ownership of the respondents remaining unaffected by petitioners’
sale of their parts, the nature of the property, as co-owned, likewise stays. In lieu of the petitioners, their vendees
shall be co-owners with the respondents. The text of Article 493 says so.

The ultimate authorities in civil law, recognized as such by the Court, agree that co-owners such as respondents
have over their part, the right of full and absolute ownership. Such right is the same as that of individual owners
which is not diminished by the fact that the entire property is co-owned with others. That part which ideally
belongs to them, or their mental portion, may be disposed of as they please, independent of the decision of their co-
owners. So we rule in this case. The respondents cannot be ordered to sell their portion of the co-owned
properties. In the language of Rodriguez v. Court of First Instance of Rizal, 88 Phil. 417, (1951), “each party is the
sole judge of what is good for him.”

Petitioners who project themselves as prejudiced co-owners may bring a suit for partition, which is one of the
modes of extinguishing co-ownership. Article 494 of the Civil Code provides that no co-owner shall be obliged to
remain in the co-ownership, and that each co-owner may demand at any time partition of the thing owned in
common insofar as his share is concerned. Corollary to this rule, Article 498 of the Civil Code states that whenever
the thing is essentially indivisible and the co-owners cannot agree that it be allotted to one of them who shall
indemnify the others, it shall be sold and its proceeds accordingly distributed. This is resorted to (a) when the right
to partition the property is invoked by any of the co-owners but because of the nature of the property, it cannot be
subdivided or its subdivision would prejudice the interests of the co-owners, and (b) the co-owners are not in
agreement as to who among them shall be allotted or assigned the entire property upon proper reimbursement of
the co-owners. This is the result obviously aimed at by petitioners at the outset. As already shown, this cannot be
done while the co-ownership exists.

Extraordinary Development Corporation v. BICO


G.R. No. 191090, October 13, 2014

The mere fact that a co-owner purportedly transferred a definite portion of the co-owned lot by metes and bounds to
another, however, does not per se render the sale a nullity.

FACTS:

Apolonio Ballestero and Maria Membre were husband and wife. They had 2 children, Juan and Irenea. When the
spouses died, the parcel of land owned by Apolonio passed to them. Later on, both children, Juan and Irenea, died,
making their heirs co-owners of the said property.

Without consent of the heirs of Irenea, the heirs of Juan sold the subject land to Extraordinary Development
Corporation (EDC). The heirs of Irenea wrote to EDC, stating the land was co-owned and that the land was sold to it
without their consent as co-owners. EDC said it will investigate, but nevertheless caused the registration of the
property in its name. The heirs of Irenea filed an Action for Reconveyance with Damages against EDC. The RTC
ruled in favour of the heirs of Irenea, finding that EDC was not a buyer in good faith. The CA affirmed, however, it
said that the sale was valid and that EDC should be reimbursed by the heirs of Juan of half the purchase price.

ISSUE:
Whether or not the Court of Appeals committed grave error in ruling that the Respondents are entitled to ½ of the
Subject Property despite their utter failure to present evidence which can prove their claim thereto

RULING: NO.

Having established respondents’ co-ownership rights over the subject property, we find no error in the appellate
court’s ruling sustaining the validity of the Deed of Absolute Sale but only with respect to the rights of the heirs of
Juan over one-half of the property. Article 493 of the Civil Code recognizes the absolute right of a co-owner to
freely dispose of his pro indiviso share as well as the fruits and other benefits arising from that share,
independently of the other co-owners.
Art. 493. Each co-owner shall have the full ownership of his part of the fruits and benefits pertaining thereto, and
he may therefore alienate, assign or mortgage it, and even substitute another person in its enjoyment, except when
personal rights are involved. But the effect of the alienation or the mortgage, with respect to the co-owners, shall
be limited to the portion which may be allotted to him in the division upon the termination of the co-ownership.

The mere fact that a co-owner purportedly transferred a definite portion of the co-owned lot by metes and bounds
to another, however, does not per se render the sale a nullity. This much is evident under Article 493 of the Civil
Code and pertinent jurisprudence on the matter. The fact that the agreement in question purported to sell a
concrete portion of the hacienda does not render the sale void, for it is a well-established principle that the binding
force of a contract must be recognized as far as it is legally possible to do so. “Quando res non valet ut ago, valeat
quantum valere potest.” (When a thing is of no force as I do it, it shall have as much force as it can have.)

Apique v. Apique Fahnenstich


G.R. No. 205705, Nov. 2, 2015

A joint account is one that is held jointly by two or more natural persons, or by two or more juridical persons or
entities. Under such setup, the depositors are joint owners or co-owners of the said account, and their share in the
deposits shall be presumed equal, unless the contrary is proved.

FACTS:

Dominador and Evangeline are siblings. When Evangeline left for Germany to work she executed a General and
Special Powers of Attorney constituting Dominador as her attorney-in-fact to purchase real property for her, and to
manage or supervise her business affairs in the Philippines. She opened a joint savings account with Dominador at
the Equitable PCI Bank (EPCIB). Dominador withdrew the amount of P980,000.00 from the subject account and,
thereafter, deposited the money to his own savings account with the same bank. Evangeline learned of such
withdrawal and discovered that Dominador had deposited the amount withdrawn to his own account with the
same bank and that he had withdrawn various amounts from the said account.

Evangeline demanded the return of the amount withdrawn from the joint account, but to no avail. Hence, she filed
a complaint. In her complaint, Evangeline claimed to be the sole owner of the money deposited in the subject
account, and that Dominador has no authority to withdraw the same. In his answer, Dominador asserted, among
others, that he was authorized to withdraw funds from the subject account to answer for the expenses of
Evangeline's projects, considering: (a) that it was a joint account, and (b) the general and special powers of
attorney executed by Evangeline in his favor.

ISSUE:

Whether or not Dominador may withdrew from their joint savings account with EPCIB without the prior consent of
Evangeline

RULING:

Yes. While, Dominador is a co-owner of the subject account as far as the bank is concerned and may, thus, validly
deposit and/or withdraw funds without the consent of his co-depositor, Evangeline as between him and
Evangeline, his authority to withdraw, as well as the amount to be withdrawn, is circumscribed by the
purpose for which the subject account was opened.

A joint account is one that is held jointly by two or more natural persons, or by two or more juridical persons or
entities. Under such setup, the depositors are joint owners or co-owners of the said account, and their share in the
deposits shall be presumed equal, unless the contrary is proved, pursuant to Article 485 of the Civil Code, which
provides:
Art. 485. The share of the co-owners, in the benefits as well as in the charges, shall be proportional to their
respective interests. Any stipulation in a contract to the contrary shall be void.

The portions belonging to the co-owners in the co-ownership shall be presumed equal, unless the
contrary is proved. (Emphasis supplied)

The common banking practice is that regardless of who puts the money into the account, each of the named
account holder has an undivided right to the entire balance, and any of them may deposit and/or withdraw,
partially or wholly, the funds without the need or consent of the other, during their lifetime. Nevertheless, as
between the account holders, their right against each other may depend on what they have agreed upon, and the
purpose for which the account was opened and how it will be operated.

In this case, there is no dispute that the account opened by Evangeline and Dominador was a joint "OR" account. It
is also admitted that: (a) the account was opened for a specific purpose, i.e., to facilitate the transfer of needed
funds for Evangeline's business projects; and (b) Dominador may withdraw funds therefrom "if" there is a need to
meet Evangeline's financial obligations arising from said projects. Hence, while Dominador is a co-owner of the
subject account as far as the bank is concerned and may, thus, validly deposit and/or withdraw funds without the
consent of his co-depositor, Evangeline as between him and Evangeline, his authority to withdraw, as well as the
amount to be withdrawn, is circumscribed by the purpose for which the subject account was opened.

Under the foregoing circumstances, Dominador's right to obtain funds from the subject account was, thus,
conditioned on the necessity of funds for Evangeline's projects. Admittedly, at the time he withdrew the amount of
P980,000.00 from the subject account, there was no project being undertaken for Evangeline.

Tabasondra v. Constantino
G.R. No.196403, Dec. 7, 2016

A co-owner has the right to alienate his pro-indiviso share in the co-owned property even without the consent of the
other co-owners.

FACTS:

Cornelio, Valentina, and Valeriana, all surnamed Tabasondra, were siblings and the registered owners of 3 parcels
of land with a total area of 100, 352 sqm. Valentina and Valeriana both sold their interests and shares to the parcels
of land for a consideration of P10,000 each to Tarcila Constantino and Sebastian Tabasondra (respondents),
Cornelio’s children from his first marriage. Cornelio, Valentina, and Valeriana all died intestate and without
partitioning the 3 parcels of land. Only Cornelio left heirs, and these heirs possessed and occupied the property.

In 2002, Cornelio’s children from his second marriage (petitioners) filed an action for partition, contending that
respondents did not give them their share to the fruits of the property owned in common. The RTC rendered a
decision, partitioning the entire 100,352 sqm. to all of the co-heirs. Respondents appealed the RTC decision to the
CA, which affirmed the RTC decision but modified the area subject of the partition to only 33,450.66 sqm. which
represents Cornelio’s 1/3 share in the total area of the 3 parcels of land.

ISSUE:

Whether the CA correctly limited the partition to only 33,450.66 sqm.


RULING:

Yes, the CA correctly limited the partition to Cornelio’s share only. Cornelio, Valentina, and Valeriana are co-
owners of the 3 parcels of land with a total area of 100,352 sqm. and as co-owners, are entitled to 1/3 of the total
area, which is equivalent to 33,450.66 sqm. Respondents sufficiently established that Valentina and Valeriana sold
their respective shares to the land owned in common in their favor and even presented the deed of absolute sale to
prove the fact of the sale.

The Court upheld the right of Valentina and Valeriana to alienate their pro indiviso shares to Sebastian and Tarcila
even without the knowledge or consent of their co-owner Cornelio because the alienation covered the disposition
of only their respective interests in the common property. According to Article 493 of the Civil Code, each co-owner
"shall have the full ownership of his part and of the fruits and benefits pertaining thereto, and he may therefore
alienate, assign or mortgage it, and even substitute another person in its enjoyment, except when personal rights
are involved," but "the effect of the alienation or the mortgage, with respect to the co-owners, shall be limited to
the portion which may be allotted to him in the division upon the termination of the co-ownership." The
petitioners as Cornelio’s heirs could not validly assail the alienation by Valentina and Valeriana of their shares in
favor of the respondents. As a consequence of the valid sale between respondents and Valeriana and Valentina,
Sebastian and Tarcila became co-owners of 2/3 of the property, with each of them having one-third pro
indiviso share in the three lots, while the remaining one-third was co-owned by the heirs of Cornelio from his first
and second marriage.

Bautista v. Bautista
G.R. No. 202088, March 8, 2017

Although a certificate of title is the best proof of ownership of a piece of land, the mere issuance of the same in the
name of any person does not foreclose the possibility that the real property may be under co-ownership with persons
not named in the certificate or that the registrant may only be a trustee or that other parties may have acquired
interest subsequent to the issuance of the certificate of title. The principle that a trustee who puts a certificate of
registration in his name cannot repudiate the trust by relying on the registration is one of the well-known limitations
upon a title.

FACTS:

The Bautista siblings established a lending business through a common fund from the proceeds of the sale of a
parcel of coconut land they inherited from their mother. Amelia V. Mendoza (Amelia) obtained a loan from
Florencia (one of the siblings), and secured the same with a real estate mortgage situated at Barangay Sta. Monica,
San Pablo City (Sta. Monica property). They later extended the mortgage through a Kasulatan ng Pagdaragdag ng
Sanla, for an additional loan.

Amelia and Florencia renewed the mortgage for and cancelled the previous loan. Subsequently, Amelia and
Florencia executed another Kasulatan ng Pagdaragdag ng Sanla. Florencia, thereafter, received the owner's
duplicate copy of the TCT, which she, in turn, entrusted to Carmelita when she went overseas.

Later, Amelia allegedly sold the subject property to Margarito through a Kasulatan ng Bilihang Tuluyan and,
likewise, cancelled the loan. On the same date, Florencia filed a Petition for the Issuance of a Second Owner's
Duplicate of the TCT. She alleged that she was the mortgagee of the subject property, and that she could not locate,
despite diligent search, the owner's duplicate title in her possession, which she misplaced. Florencia also executed
a SPA in favor of Margarito to represent her in the proceedings.

RTC granted the petition and a TCT was later issued in the name of Margarito. Failing to settle their differences,
petitioners subsequently instituted a Complaint for Partition. Petitioners averred that Margarito and the others
refused to heed their oral and written demands for the partition of the properties they co-owned, which included
the Sta. Monica property. Since no settlement was reached as regards the Sta. Monica property, petitioners
presented copies of their bank transactions with Far East Bank to support their claim of co-ownership over the
same.
ISSUE:

Whether or not the Sta. Monica property is co-owned by both parties.

RULING:

Yes, the Sta. Monica property is co-owned. The object of partition is to enable those who own property as joint
tenants, or coparceners, or tenants in common to put an end to the joint tenancy so as to vest in each a sole estate
in specific property or an allotment in the lands or tenements. Hence, unless and until the issue of co-ownership is
definitively resolved, it would be premature to effect a partition of an estate.

Consequently, the first stage of an action for judicial partition and/or accounting is concerned with the
determination of whether or not a coownership in fact exists and a partition is proper. In the case at bar,
petitioners aver that although the Sta. Monica property was registered solely in Margarito's name, they are co-
owners of the property because it was acquired through the siblings' lending business. Although a certificate of
title is the best proof of ownership of a piece of land, the mere issuance of the same in the name of any person does
not foreclose the possibility that the real property may be under co-ownership with persons not named in the
certificate or that the registrant may only be a trustee or that other parties may have acquired interest subsequent
to the issuance of the certificate of title. The principle that a trustee who puts a certificate of registration in his
name cannot repudiate the trust by relying on the registration is one of the well-known limitations upon a title.

There is an implied trust when a property is sold and the legal estate is granted to one party but the price is paid by
another for the purpose of having the beneficial interest of the property. This is sometimes referred to as
a purchase money resulting trust, the elements of which are: (a) an actual payment of money, property or services,
or an equivalent, constituting valuable consideration; and (b) such consideration must be furnished by the alleged
beneficiary of a resulting trust. A trust, which derives its strength from the confidence one reposes on another
especially between families, does not lose that character simply because of what appears in a legal document. From
the foregoing, this Court finds that an implied resulting trust existed among the parties.

Uy v. Estate of Vipa Fernandez


G.R No. 200612 April 5, 2017

FACTS:

Vipa Fernandez Lahaylahay is the registered owner of a parcel of land situated in Jaro, Iloilo City. Vipa and her
husband Levi Lahaylahay have two children, Grace Joy and Jill Frances. In 1990, a contract of lease was executed
between Vipa and Rafael Uy over the subject property and the improvements thereon to which Rafael bound
himself to pay the amount of P 3,000/mo with provision for a 10% every year thereafter. On 1995, Vipa died
leaving no will or testament whatsoever, Grace Joy became the de facto administrator of the estate of Vipa. In 1998,
Rafael stopped paying the monthly rents. Consequently, the estate of Vipa filed a complaint for unlawful detainer
with MTCC against Rafael. Accordingly, at the time of the filing of the complaint, unpaid rents amounted to
P271,150.00.MTCC rendered a decision ordering Rafael to vacate the premises and to pay the amount of unpaid
rents with 12% interest per annum.On appeal, RTC reversed the decision of MTCC and dismiss the complaint for
unlawful detainer. According the RTC, Grace was the plaintiff not the estate and it had failed to the bring the
dispute to the barangay conciliation; that the property is part of conjugal property and after Vipa’s death the
conjugal partnership was terminated. Levi sold his property to Rafael, thus making him co-owner of the property.
Estate filed a petition for review to CA and reinstated the decision of MTCC.

ISSUE:

Whether or not the CA erred in reversing the RTC’s decision.

RULING:
The petition is partly granted. Complainants by and against corporations, partnerships, or other juridical entities
may not filed with, received or acted upon by the barangay for conciliation. The Estate of Vipa, which is the
complainant below, is a juridical entity that has a personality, separate and distinct from that of Grace Joy. Thus,
there is no necessity to bring the dispute to the barangay conciliation prior to filing of the complaint for unlawful
detainer with MTCC. On the issue of ownership of Rafael, Levi had the right to freely dispose of his undivided
interest. Thus the sale by Levi of his one-half undivided share in the subject property was not necessarily void, for
his right as a co-owner thereof was effectively transferred, making the buyer Rafael, a co-owner of the subject
property. Accordingly, Rafael could no-longer be directed to vacate the subject property since he is already a co-
owner thereof. Nevertheless, Rafael is still bound to pay unpaid rentals from 1998 to 2003.

E. Possession

1. Possession in the concept of a holder, NCC 525, 554, 561, 552, 1128, 526

Aranda v. Republic
G.R. No. 172331, August 24, 2011

While, as a rule, tax declarations or realty tax payments of property are not conclusive evidence of ownership,
nevertheless they are good indicia of possession in the concept of owner, for no one in his right mind would be paying
taxes for a property that is not in his actual or constructive possession – they constitute at least proof that the holder
has a claim of title over the property.

FACTS:

Subject of a petition for original registration before the RTC is a parcel of land situated in Batangas with. The
petition was originally filed by ICTSI Warehousing, Inc. (ICTSI-WI. OSG filed its opposition on grounds that the land
applied for is part of the public domain and the applicant has not acquired a registrable title.

ICTSI-WI sought leave of court to amend the application due to a technicality, the sale between the vendor and
applicant corporation cannot push through and consequently the tax declaration is still in the name of vendor
Ramon Aranda and the land cannot be transferred and declared in the name of ICTSI-WI.
The trial court admitted the Amended Application. Aranda prayed that the liberal provisions of Section 48 of
Commonwealth Act No. 141, as amended, having been in continuous possession of the subject land in the concept
of owner, publicly, openly and adversely for more than thirty (30) years prior to the filing of the application.

In support of the application, petitioner’s sister testified that in 1965 her father Anatalio Aranda donated the
subject land to his brother (petitioner), as evidenced by documents "Pagpapatunay ng Pagkakaloob ng Lupa" which
she and her siblings executed on June 7, 2000. Another witness, Luis Olan, testified that his father Lucio Olan had
open, peaceful, continuous and adverse possession of the land in the concept of owner until his father sold the land
in 1946 to Anatalio Aranda.

ISSUE:

Whether the applicant has acquired a registrable title to the subject land.

RULING: NO.

The Property Registration Decree provides for original registration of land in an ordinary registration proceeding.
Under Section 14(1) thereof, a petition may be granted upon compliance with the following requisites: (a) that the
property in question is alienable and disposable land of the public domain; (b) that the applicants by themselves or
through their predecessors-in-interest have been in open, continuous, exclusive and notorious possession and
occupation; and (c) that such possession is under a bona fide claim of ownership since June 12, 1945 or earlier.

To prove that the land subject of an application for registration is alienable, an applicant must establish the
existence of a positive act of the government such as a presidential proclamation or an executive order; an
administrative action; investigation reports of Bureau of Lands investigators; and a legislative act or a statute. The
applicant may also secure a certification from the Government that the lands applied for are alienable and
disposable.

The status of the land applied for as alienable and disposable was not clearly established. We also agree with the
CA that petitioner’s evidence failed to show that he possessed the property in the manner and for the duration
required by law.

Petitioner presented tax declarations and the deeds of confirmation of the 1946 sale from the original owner to
Anatalio Aranda and the 1965 donation made by the latter in favor of petitioner. But as found by the CA, the history
of the land shows that it was declared for taxation purposes for the first time only in 1981. While, as a rule, tax
declarations or realty tax payments of property are not conclusive evidence of ownership, nevertheless they are
good indicia of possession in the concept of owner, for no one in his right mind would be paying taxes for a
property that is not in his actual or constructive possession – they constitute at least proof that the holder has a
claim of title over the property.

Petitioner likewise failed to prove the alleged possession of his predecessors-in-interest.

2. Concept of Owner, NCC 540

Sps. Decaleng v. Bishop


G.R. No. 171209, June 27, 2012

FACTS:

On February 18, 1992, PEC-EDNP filed before the Regional Trial Court (RTC) of Bontoc, Mountain Province, Branch
36, a Complaint for Accion Reinvindicatoria and Accion Publiciana against Ambrosio Decaleng and Fabian Lopez
(Lopez), docketed as Civil Case No. 797.

PEC-EDNP alleged that it is the owner of two parcels of land in the Municipality of Sagada, located in areas
commonly known as Ken-geka and Ken-gedeng.

According to PEC-EDNP, the Ken-geka property is covered by Certificate of Title No. 15 of the Register of Deeds of
Mountain Province, issued on February 18, 1915, in the name of The Domestic and Foreign Missionary Society of
the Protestant Episcopal Church in the United States (U.S. Episcopal Church). According to Certificate of Title No. 1,
the U.S. Episcopal Church acquired the Ken-geka property by virtue of a sales patent issued by the Governor-
General of the Philippine Islands also on February 18, 1915, in accordance with Section 122 of Act No. 496,6
otherwise known as the Land Registration Act.

PEC-EDNP asserted that the U.S. Episcopal Church donated the Ken-geka property, among other real properties, to
the PEC by virtue of a Deed of Donation8 executed on April 24, 1974. Around the second quarter of 1989, Ambrosio
Decaleng entered and cultivated a portion of about 1,635 square meters of the Ken-geka property despite the
protestations of PEC-EDNP representatives.

The Ken-gedeng property is described in the complaint as:


A certain parcel of land situated at sitio Poblacion, Sagada, Mt. Province, bounded on the North by Tomas Muting &
Kapiz Bacolong; South by Mission Compound, East by Bartolome Gambican; and on the West by Nicolas Imperial
and Lizardo Adriano with an area of 20[,]692 sq. meters more or less and declared for taxation purposes under Tax
Declaration No. 6306 in the name of the Domestic and Foreign Missionary Society of the Protestant Church of the
United States of America.

PEC-EDNP averred that it and its predecessors-in-interest occupied the Ken-gedeng property openly, adversely,
continuously, and notoriously in en concepto de dueño since the American Missionaries arrived in the Mountain
Province in 1901. PEC-EDNP and its predecessors-in-interest have introduced valuable improvements on the Ken-
gedeng property through the years. The Ken-gedeng property was surveyed on August 22, 1947 and said survey
was approved by the Director of Lands on June 15, 1948. During the first quarter of 1987, Ambrosio Decaleng
illegally and forcibly entered two portions of the Ken-gedeng property, one measuring 1,650 square meters
(Portion 1) and the other 419.50 square meters (Portion 2). Ambrosio Decaleng, despite the vehement objections
and conciliatory attitude of PEC-EDNP, cut several matured pine trees within the aforementioned portions of the
Ken-gedeng property, removed the fence and two monuments found therein, and cultivated and planted the same
with plants of economic value. Ambrosio Decaleng made matters worse by selling Portion 2 of the Ken-gedeng
property to Fabian Lopez. Lopez went ahead and purchased Portion 2 despite the warning of PEC-EDNP.

PEC-EDNP contended that Ambrosio Decaleng and Lopez refused to vacate the portions of Ken-geka and Ken-
gedeng properties that they are occupying. Ambrosio Decaleng and Lopez claimed to be the owners of said
portions, but PEC-EDNP maintained that such claim is illegal and baseless in fact and in law. PEC-EDNP likewise
challenged the sale of Portion 2 of Ken-gedeng by Ambrosio Decaleng to Lopez for being unlawful and void.

ISSUE:

Whether or not Decaleng established its ownership and possession over the lots in dispute.

HELD: NO.

The applicant failed to show that he has title to the lot that may be confirmed under the Land Registration Act. He
failed to show that he or any of his predecessors in interest had acquired the lot from the Government, either by
purchase or by grant, under the laws, orders and decrees promulgated by the Spanish Government in the
Philippines, or by possessory information under the Mortgage Law (section 19, Act 496). All lands that were not
acquired from the Government, either by purchase or by grant, belong to the public domain. An exception to the
rule would be any land that should have been in the possession of an occupant and of his predecessors in interest
since time immemorial, for such possession would justify the presumption that the land had never been part of the
public domain or that it had been a private property even before the Spanish conquest. (Carino vs. Insular
Government, 212 U.S., 449; 53 Law. ed., 594.) The applicant does not come under the exception, for the earliest
possession of the lot by his first predecessor in interest began in 1880. (Emphases supplied.)

Neither can the spouses Decaleng claim imperfect title to the properties in Ken-geka and Ken-gedeng for such can
only be acquired by possession of lands of the public domain for the period required by law. Because the spouses
Decaleng failed to provide and prove the necessary details on how and when their predecessors-in-interest came
to possess the disputed properties, there is no way for the Court to determine whether or not said properties were
still part of the public domain when occupied by the spouses Decaleng’s predecessors-in-interest. As the Court
previously found herein, the Ken-geka property was already covered by a Certificate of Title issued in the name of
the U.S. Episcopal Church (the predecessor-in-interest of PEC-EDNP) on February 18, 1915 and the Ken-gedeng
property had been in the possession under claim of title by the U.S. Episcopal Church ever since its arrival in the
Mountain Province in 1901.

Abobon v. Abobon
G.R. No. 155830, August 12, 2012

It is beyond question under the law that the owner has not only the right to enjoy and dispose of a thing without other
limitations than those established by law, but also the right of action against the holder and possessor of the thing in
order to recover it. He may exclude any person from the enjoyment and disposal of the thing, and, for this purpose, he
may use such force as may be reasonably necessary to repel or prevent an actual or threatened unlawful physical
invasion or usurpation of his property.

FACTS:

The controversy involves the rightful possession of a parcel of registered land. The respondents, who were the
registered owners, sued the petitioner, their first cousin, to recover 1the possession of the land in question, stating
that they had only allowed the petitioner to use the land out of pure benevolence, but the petitioner asserted that
the land belonged to him as owner by right of succession from his parents.
Respondents were the plaintiffs in this action for recovery of possession and damages brought against petitioner
Numeriano Abobon (Numeriano) in the MCTC of Labrador-Sual in Pangasinan. They averred that they were the
registered owners of that parcel of unirrigated riceland situated in Poblacion, Labrador, Pangasinan, and that they
had allowed Numeriano, their first cousin, the free use of the land out of benevolence; and that they now
immediately needed the parcel of land for their own use and had accordingly demanded that Numeriano should
vacate and return it to them but he had refused.

In his answer, Numeriano admitted being the first cousin of the respondents and the existence of TCT covering the
land in question, and having received the demand for him to vacate. He alleged, however, that he did not vacate
because he was the owner of the land in question. He asserted that if the land in question related to the unirrigated
riceland with an area of 3,000 square meters that he was presently tilling and covered by tax declaration no. 2 in
the name of his father, Rafael Abobon (Rafael), then the respondents did not have a valid cause of action against
him because he had inherited that portion from his parents; that he and his predecessors-in-interest had also
continuously, publicly and adversely and in the concept of owner possessed the parcel of land for more than 59
years and that assuming that the respondents were the true owners of the land, they were already estopped by
laches from recovering the portion of 3,000 square meters from him.

On August 23, 2000, after due proceedings, the MCTC ruled in favor of the respondents, finding that the
respondents’ parents had purchased the property sometime in 1941 from Emilio with the conformity of Emilio’s
other children, including Rafael. The MCTC further found that the 3,000 square-meter land Numeriano referred to
as donated to his parents was not the same as the land in question due to their boundaries being entirely different.
The MCTC held that the respondents were not guilty of laches because of their numerous acts and transactions
from 1941 until 1996 involving the land in question, specifically: (a) the sale of the land to Juan Mamaril and its
repurchase by Leodogario; (b) the registration of title and ownership; (c) the extrajudicial partition of the property
by the heirs of Leodegario; (d) Numeriano’s free use of the land from 1989 onwards upon being allowed to do so by
the respondents; (e) the verbal demands from the respondents since 1993 for Numeriano to vacate the land; and
(f) the commencement of the action to recover possession against Numeriano. It considered such acts and
transactions as negating any notion of the respondents’ abandonment of their right to assert ownership.2
Numeriano appealed to the Regional Trial Court in Lingayen City, Pangasinan (RTC), which, on April 16, 2001,
upheld the decision of the MCTC. Still dissatisfied, Numeriano appealed via petition for review to the Court of
Appeals, however, the CA rejected Numeriano’s submissions and affirmed the RTC, holding that the respondents
were in possession of a certificate of title that enjoyed the conclusive presumption of validity, by virtue of which
they were entitled to possess the land in question; that the parcel of land that he owned was different from the land
in question; and that his impugning the validity of the respondents’ TCT partook of the nature of an impermissible
collateral attack against the TCT, considering that the validity of a Torrens title could be challenged only directly
through an action instituted for that purpose.

ISSUE:

WON Petitioner is the true owner of the land he is claiming.

RULING:

NO. Agreeing with both lower courts, the CA declared: "(i)n fine, what these decisions are saying is that petitioner
may have evidence that he owns a parcel of land but, based on the evidence he had presented, the said parcel of
land is different from the one he is presently occupying."

Accordingly, we rule for the respondents on the issue of the preferential right to the possession of the land in
question. Their having preferential right conformed to the age-old rule that whoever held a Torrens title in his
name is entitled to the possession of the land covered by the title. Indeed, possession, which is the holding of a
thing or the enjoyment of a right, was but an attribute of their registered ownership.

It is beyond question under the law that the owner has not only the right to enjoy and dispose of a thing without
other limitations than those established by law, but also the right of action against the holder and possessor of the
thing in order to recover it. He may exclude any person from the enjoyment and disposal of the thing, and, for this
purpose, he may use such force as may be reasonably necessary to repel or prevent an actual or threatened
unlawful physical invasion or usurpation of his property.

The Supreme Court must undo the awards of moral and exemplary damages and attorney’s fees as well.

Locsin v. Hizon
G.R. No. 204369, September 17, 2014

An innocent purchaser for value is one who buys the property of another without notice that some other person has a
right to or interest in it, and who pays a full and fair price at the time of the purchase or before receiving any notice of
another person’s claim. As such, a defective title–– or one the procurement of which is tainted with fraud and
misrepresentation––may be the source of a completely legal and valid title, provided that the buyer is an innocent
third person who, in good faith, relied on the correctness of the certificate of title, or an innocent purchaser for value.

Complementing this is the mirror doctrine which echoes the doctrinal rule that every person dealing with registered
land may safely rely on the correctness of the certificate of title issued therefor and is in no way obliged to go beyond
the certificate to determine the condition of the property. The recognized exceptions to this rule are stated as follows:

[A] person dealing with registeredland has a right to rely on the Torrens certificate of title and to dispense with the
need of inquiring further except when the party has actual knowledge of facts and circumstances that would impel a
reasonably cautious man to make such inquiry or when the purchaser has knowledge of a defect or the lack of title in
his vendor or of sufficient facts to induce a reasonably prudent man to inquire into the status of the title of the
property in litigation. The presence of anything which excites or arouses suspicion should then prompt the vendee to
look beyond the certificate and investigate the title of the vendor appearing on the face of said certificate. One who
falls within the exception can neither be denominated an innocent purchaser for value nor a purchaser in good faith
and, hence, does not merit the protection of the law.

FACTS:

Petitioner (Locsin) was the registered owner of a lot located in Quezon City. In 1992, she filed an ejectment case
against Aceron to recover possession over the land in issue. Eventually, the two entered into a compromise
agreement. Locsin later went to the United States without knowing whether Aceron has complied with his part of
the bargain under the compromise agreement. In spite of her absence, however, she continued to pay the real
property taxes on the subject lot. In 1994, after discovering that her copy of TCT was missing, Locsin filed a
petition for administrative reconstruction in order to secure a new one, TCT No. RT-97467. Sometime in early
2002, she then requested her counsel to check the status of the subject lot. It was then that they discovered the
following:

1. One Marylou Bolos (Bolos) had TCT No. RT-97467 cancelled and then secured a new one in her favor by
registering a Deed of Absolute Sale dated November 3, 1979 allegedly executed by Locsin with the Registry of
Deeds;
2. Bolos later sold the subject lot to Bernardo Hizon (Bernardo) for PhP 1.5 million, but it was titled under
Carlos Hizon’s (Carlos’) name on August 12, 1999. Carlos is Bernardo’s son;
3. Bernardo, claiming to be the owner of the property, filed a Motion for Issuance of Writ of Execution for the
enforcement of the court-approved compromise agreement in Civil Case No. 38-6633;
4. The property was already occupied and was, in fact, up for sale.

Locsin, through counsel, sent Carlos a letter requesting the return of the property since her signature in the
purported deed of sale in favor of Bolos was a forgery. In a letter-reply dated May 20, 2002, Carlos denied Locsin’s
request, claiming that he was unaware of any defect or flaw in Bolos’ title and he is, thus, an innocent purchaser for
value and good faith. Bernardo met with Locsin’s counsel and discussed the possibility of a compromise. He ended
the meeting with a promise to come up with a win-win situation for his son and Locsin, a promise which turned out
to be deceitful, for, on July 15, 2002, Locsin learned that Carlos had already sold the property for PhP 1.5 million to
his sister and her husband, herein respondents Lourdes and Jose Manuel Guevara (spouses Guevara), respectively,
who, as early as May 24, 2002, had a new certificate of title, TCT No. N-237083, issued in their names. The spouses
Guevara then immediately mortgaged the said property to secure a PhP 2.5 million loan/credit facility with Damar
Credit Corporation (DCC).

Hence, Locsin filed an action for reconveyance, annulment ofTCT No. N-237083, the cancellation of the mortgage
lien annotated thereon, and damages, against Bolos, Bernardo, Carlos, the Sps. Guevara, DCC, and the Register of
Deeds, Quezon City.

ISSUES:

1) Whether Carlos is an innocent purchaser for value. –No.


2) Whether the spouses Guevara are not innocent purchasers for value.-No.

HELD:

1) No. He is not an innocent purchaser for value. Having knowledge of the foregoing facts, Bernardo and Carlos, to
our mind, should have been impelled to investigate the reason behind the arrangement. They should have been
pressed to inquire into the status of the title of the property in litigation in order to protect Carlos’ interest. It
should have struck them as odd that it was Locsin, not Bolos, who sought the recovery of possession by
commencing an ejectment case against Aceron, and even entered into a compromiseagreement with the latter
years afterthe purported sale in Bolos’ favor. Instead, Bernardo and Carlos took inconsistent positions when they
argued for the validity of the transfer of the property in favor of Bolos, but in the same breath prayed for the
enforcement of the compromise agreement entered into by Locsin.

At this point it is well to emphasize that entering into a compromise agreement is an act of strict dominion. If Bolos
already acquired ownership of the property as early as 1979, it should have been her who entered into a
compromise agreement with Aceron in 1993, not her predecessor-in-interest, Locsin, who, theoretically, had
already divested herself of ownership thereof.

As regards the transfer of the property from Carlos to the spouses Guevara, We find the existence of the sale highly
suspicious. For one, there is a dearth of evidence to support the respondent spouses’ position that the sale was a
bona fide transaction. Evenif we repeatedly sift through the evidence on record, still we cannot findany document,
contract, or deed evidencing the sale in favor of the spouses Guevara. The same goes for the purported payment of
the purchase price of the property in the amount of PhP 1.5 million in favor of Carlos.

a. Presumption of just title, NCC 541

Casimiro Development Corp. v. Mateo


G.R. No. 175485, 24 July 2011

One who deals with property registered under the Torrens system need not go beyond the certificate of title, but only
has to rely on the certificate of title. He is charged with notice only of such burdens and claims as are annotated on the
title.

FACTS:

A Registered parcel of land in Las Pinas City was originally owned by Isaias Lara, respondents’ maternal
grandfather. Subsequently, when the land was passed on to his heirs, the latter effected the transfer of the full and
exclusive ownership of the land to Felicidad Lara-Mateo. A Deed of sale was then executed in favor of Laura, one of
Felicidad’s children, who applied for land registration wherein an OCT was thereafter issued. Laura used the
subject proprty as collateral to secure a succession of loans, one of which was to use such as security in an
obligation secured by a Real Estate Mortgage with China Bank.
The latter failed in her obligation, prompting the foreclosure of the lot in favor of China Bank. A TCT of was
eventually issued in the name of China Bank. Casimiro Development Corp. thereafter purchased the property from
China Bank and in the meanwhile Felicidad died intestate.

CDC brought action for unlawful detainer in the MeTC against the sons of Felicidad who remained in the property;
the latter claimed that MeTC did not have jurisdiction because the land was classified as agricultural; that the
jurisdiction belonged to the Department of Agrarian Reform Adjudication Board (DARAB); that they had been in
continuous and open possession of the land even before World War II and had presumed themselves entitled to a
government grant of the land; and that CDCs title was invalid, considering that the land had been registered before
its being declared alienable.

MeTC ruled in favor of CDC, RTC against, CA and SC in favor CDC. Upon finality of the decision before the Supreme
Court, Respondent brought action for quieting of title, RTC favored CDC but the CA ruled against CDC contending
that CDC was a not a buyer in good faith.

ISSUE:

Whether or not the continuous and open possession of land of the sons of Felicidad entitled them to the
subsequent government grant of the land.

RULING:

NO. One who deals with property registered under the Torrens system need not go beyond the certificate of title,
but only has to rely on the certificate of title. He is charged with notice only of such burdens and claims as are
annotated on the title. China Bank’s TCT’s was a clean title, that is, it was free from any lien or encumbrance, CDC
had the right to rely, when it purchased the property, solely upon the face of the certificate of title in the
name of China Bank. The respondent’s siblings’ possession did not translate to an adverse claim of ownership.
They even characterized their possession only as that of mere agricultural tenants. Under no law was possession
grounded on tenancy a status that might create a defect or inflict a law in the title of the owner. CDC having paid
the full and fair price of the land, was an innocent purchaser for value. The TCT in the name of CDC was declared
valid and subsisting.

b. Good Faith, NCC 544, 545, 546, 547, 548, 1132, 1134, 552

Cabigas v. Limbaco
G.R. No. 175291, July 27, 2011

A purchaser in good faith is one who buys the property of another without notice that some other person has a right to
or interest in such property, and pays a full and fair price for the same at the time of such purchase or before he has
notice of the claim of another person. It is a well-settled rule that a purchaser cannot close his eyes to facts which
should put a reasonable man upon his guard, and then claim that he acted in good faith under the belief that there
was no defect in the title of the vendor. His mere refusal to believe that such defect exists, or his willful closing of his
eyes to the possibility of the existence of a defect in his vendor’s title, will not make him an innocent purchaser for
value, if it afterwards develops that the title was in fact defective, and it appears that he had such notice of the defect
as would have led to its discovery had he acted with that measure of precaution which may reasonably be required of
a prudent man in a like situation.

FACTS:

Petitioners purchased 2 lots from Cobarde in 1980 who in turn had purchased these lots from Ouano in
1948. Nothwithstanding the sale, the 2 lots remained registered in the name of Ouano. Ouano was able to sell these
same lots to the National Airports Corporation (NAC) in 1952 for its airport expansion project. NAC promptly had
the titles for these properties registered in its name. When the airport expansion project fell through, respondent
and the legal heirs of Ouano succeeded in reclaiming title to the 2 lots through an action for reconveyance. The
titles over these lots were thereafter registered in their names then they subdivided the lots and sold them to
various buyers who registered the titles over their respective lots in their names.

ISSUE:

Who has a better right to the subject lots.

HELD:

Cabigas spouses are not buyers in good faith. A purchaser in good faith is one who buys the property of another
without notice that some other person has a right to or interest in such property, and pays a full and fair price for
the same at the time of such purchase or before he has notice of the claim of another person. It is a well-settled rule
that a purchaser cannot close his eyes to the fact which should put a reasonable man upon his guard, and then
claim that he acted in good faith under the belief that there was no defect in the title of the vendor. At the time of
the sale to the Cabigas spouses, the land was registered not in Cobarde’s name but in Ouano’s name. They relied
completely onCobarde’s representation that he owned the properties in question and did not even bother to
perform the most perfunctory of investigations by checking the properties’ titles with the Registry of Deeds. Their
failure to exercise the plain common sense expected of real estate buyers bound them to the consequences of their
own inaction.

The overriding consideration to determine ownership of an immovable property is the good or bad faith of the
buyer, specifically who first registered the ale with the Registry of Deeds in good faith. Since the Cabigas spouses
have no legally recognizable interest in the lots in question, it follows that the petitioners, who are subrogated to
the rights of the former by virtue of succession, also have no legally recognizable rights to the properties that could
be enforced by law.

Heirs of Trazona, et al. v. Cañada, et al.


G.R. No. 175874, December 11, 2013

Persons who occupy land by virtue of tolerance of the owners are not possessors in good faith.

FACTS:

Petitioners owned an untitled parcel of land referred to as Lot No. 5053-H. The property, located in Cebu, is
covered by Tax Declaration No. 07764. The land was purchased from the government in 1940. Since then, Cipriano
had taken possession of the land, cultivated it and diligently paid taxes thereon. In 1949, Dionisio bought the
adjacent parcel of land. It was later found that he had encroached on a small portion of Lot No. 5053-H. He was
then summoned by Cipriano for a confrontation before the barangay captain in 1952. Dionisio offered to buy the
encroached portion, but Cipriano refused the offer. In 1956, the latter gave Dionisio permission to temporarily
build a house on said portion, where it still stands. No action for ejectment was filed against Dionisio during the
lifetime of Cipriano.

The petitioners went to the Office of the Municipal Assessor to secure a copy of Tax Declaration No. 07764, but to
their surprise, they were informed that said Tax Declaration had been cancelled and, in lieu thereof, Tax
Declaration No. 23959 was issued in the name of Dionisio. Apparently, respondents had caused the issuance of Tax
Declaration No. 23959 by submitting a Deed of Absolute Sale dated 27 June 1956 supposedly executed by Cipriano
in favor of Dionisio.

The RTC annulled the assailed deed and ordered the cancellation of Tax Declaration No. 23959, as well as the
reinstatement of Tax Declaration No. 07764. Respondents were also ordered to demolish their residential house on
Lot No. 5053-H. The CA, however, issued a decision reversing that of the RTC.

ISSUE:

Whether or not the CA erred in reversing the RTC’s decision.


RULING:

Yes. The actual possession of Lot No. 5053-H by petitioners has been properly ruled on by the RTC. Much has been
made by the CA of the fact that respondents’ house was standing on the property. However, petitioners have
explained that the house was erected only after Cipriano permitted it.

Dionisio was then well aware that this temporary arrangement may be terminated at any time. Respondents
cannot now refuse to vacate the property or eventually demand reimbursement of necessary and useful expenses
under Articles 448 and 546 of the New Civil Code, because the provisions apply only to a possessor in good faith,
i.e., one who builds on land with the belief that he is the owner thereof. Persons who occupy land by virtue of
tolerance of the owners are not possessors in good faith. Thus, the directive of the RTC for respondents to demolish
their residential house on Lot No. 5053-H was also proper.

Vilbar v. Opinion
G.R. No. 176043, Jan. 15, 2014

Registration is the operative act which gives validity to the transfer or creates a lien upon the land. A certificate of title
serves as evidence of an indefeasible and incontrovertible title to the property in favor of the person whose name
appears therein. Since the spouses Vilbar did not cause the transfer of the certificate title in their name, or at the very
least, annotate or register such sale in the original title in the name of Dulos Realty, have no indefeasible and
incontrovertible title over Lot 20 to support their claim.

FACTS:

Spouses Vilbar claimed they and Dulos Realty and Development Corporation (Dulos Realty), entered into a
Contract to Sell involving a 108-square meter lot designated as Lot 20-B located in Airmen’s Village, Las Piñas City
P19,440.00. Said lot which is also covered and embraced by the same certificate of title is the subject of another
Contract to Sell between Elena Guingon (Elena) and Dulos Realty. Spouses Vilbar took possession of the Lot in the
concept of owners and exercised acts of ownership thereon with the permission of Dulos Realty after making some
advance payment. Upon full payment of the purchase price for Lot, Dulos Realty executed a duly notarized Deed of
Absolute Sale in favor of spouses Vilbar and their co-purchaser Elena. Dulos Realty also surrendered and delivered
the owner’s duplicate copy of Lot to the buyers and new owners of the property.

However, spouses Vilbar and Elena were not able to register and transfer the title in their names because
Dulos Realty allegedly failed to have the lot formally subdivided despite its commitment to do so. In contrast,
Opinion claimed that he legally acquired Lots through extra-judicial foreclosure of mortgage constituted over the
said properties by Otilio Gorospe, Sr. and Otilio "Lito" Gorospe, Jr. (Gorospes) in his favor. Opinion alleged that on
the Gorospes borrowed P440,000.00 and, to secure the loan, executed a Deed of Real Estate Mortgage over the
subject lots. The Gorospes defaulted, prompting Opinion to file a Petition for Extra-Judicial Foreclosure of Real
Estate Mortgage. Subsequently, the subject properties were sold at a public auction where Opinion emerged as the
highest bidder. A Certificate of Sale was issued in his favor and subsequently annotated on the TCTs of the
properties. The Gorospes failed to redeem the properties within the reglementary period resulting in the eventual
cancellation of their titles.

ISSUE:

W/N Vilbar is the owner of the said lots in controversy

RULING: NO.

Torrens system; certificate of title; a certificate of title serves as evidence of an indefeasible and incontrovertible
title to the property in favor of the person whose name appears therein. “a certificate of title serves as evidence of
an indefeasible and incontrovertible title to the property in favor of the person whose name appears therein.”
Having no certificate of title issued in their names, spouses Vilbar have no indefeasible and incontrovertible title
over Lots to support their claim. Further, it is an established rule that “registration is the operative act which gives
validity to the transfer or creates a lien upon the land.” “Any buyer or mortgagee of realty covered by a Torrens
certificate of title is charged with notice only of such burdens and claims as are annotated on the title.” Failing to
annotate the deed for the eventual transfer of title over Lots in their names, the spouses Vilbar cannot claim a
greater right over Opinion, who acquired the property with clean title in good faith and registered the same in his
name by going through the legally required procedure.

Homeowners Savings v. Felonia


G.R. No. 189477, February 26, 2014

Even if on the outset, a party is a mortgagee in good faith, if he subsequently purchases the property with notice of lis
pendens, he cannot claim to have a better right over the said property by interposing the argument that he is a
mortgagee in good faith.

FACTS:

Respondents Asuncion Felonia (Felonia) and Lydia de Guzman (De Guzman) were the registered owners of a
parcel of land covered by TCT No. T-402. Felonia and De Guzman mortgaged the property to Delgado to secure the
loan. However, instead of a real estate mortgage, the parties executed a Deed of Absolute Sale with an Option to
Repurchase. Felonia and De Guzman filed an action for Reformation of Contract (Reformation case) before the RTC.
The RTC rendered a judgment in their favor. Aggrieved, Delgado elevated the case to the CA. The CA affirmed the
RTC decision. Such CA decision eventually became final.
Inspite of the pendency of the Reformation case in which she was the defendant, Delgado filed a “Petition for
Consolidation of Ownership of Property Sold with an Option to Repurchase and Issuance of a New Certificate of
Title” (Consolidation case) in the RTC. After an ex parte hearing, the RTC ordered the issuance of a new title under
Delgado’s name.

By virtue of the RTC decision, Delgado transferred the title to her name. Hence, TCT No. T-402, registered in the
names of Felonia and De Guzman, was canceled and TCT No. 44848 in the name of Delgado, was issued. Aggrieved,
Felonia and De Guzman elevated the case to the CA through a Petition for Annulment of Judgment.

Meanwhile, Delgado mortgaged the subject property to Homeowners Savings and Loan Bank (HSLB) using her
newly registered title. Three (3) days later HSLB caused the annotation of the mortgage. Three months after,
Felonia and De Guzman caused the annotation of a Notice of Lis Pendens on Delgado’s title, TCT No. 44848. HSLB
foreclosed the subject property and later consolidated ownership in its favor, causing the issuance of a new title in
its name, TCT No. 64668.

The CA rendered a decision in favor of Felonia and De Guzman. The decision of the CA, declaring Felonia and De
Guzman as the absolute owners of the subject property and ordering the cancellation of Delgado’s title, became
final and executor.

Claiming to be the absolute owners of the subject property, instituted the instant complaint against Delgado, HSLB,
Register of Deeds of Las Piñas City and Rhandolfo B. Amansec before the RTC of Las Piñas City for Nullity of
Mortgage and Foreclosure Sale, Annulment of Titles of Delgado and HSLB, and finally, Reconveyance of Possession
and Ownership of the subject property in their favor. HSLB asserted that Felonia and De Guzman are barred from
laches as they had slept on their rights to timely annotate, by way of Notice of Lis Pendens, the pendency of the
Reformation case. HSLB also claimed that it should not be bound by the decisions of the CA in the Reformation and
Consolidation cases because it was not a party therein. Finally, HSLB asserted that it was a mortgagee in good faith
because the mortgage between Delgado and HSLB was annotated on the title on 5 June 1995, whereas the Notice of
Lis Pendens was annotated only on 14 September 1995.

The RTC ruled in favor of Felonia and De Guzman as the absolute owners of the subject property. The CA affirmed
the RTC decision with modification. Hence, this instant petition.

ISSUE:
Whether HLSB has a better right over the property.

RULING:

The rights of the parties to the present case are defined not by the determination of whether or not HSLB is a
mortgagee in good faith, but of whether or not HSLB is a purchaser in good faith. And, HSLB is not such a
purchaser.

A purchaser in good faith is defined as one who buys a property without notice that some other person has a right
to, or interest in, the property and pays full and fair price at the time of purchase or before he has notice of the
claim or interest of other persons in the property.

When a prospective buyer is faced with facts and circumstances as to arouse his suspicion, he must take
precautionary steps to qualify as a purchaser in good faith.

In the case at bar, HSLB utterly failed to take the necessary precautions. At the time the subject property was
mortgaged, there was yet no annotated Notice of Lis Pendens. However, at the time HSLB purchased the subject
property, the Notice of Lis Pendens was already annotated on the title.

Lis pendens is a Latin term which literally means, “a pending suit or a pending litigation” while a notice of lis
pendens is an announcement to the whole world that a real property is in litigation, serving as a warning that
anyone who acquires an interest over the property does so at his/her own risk, or that he/she gambles on the
result of the litigation over the property. It is a warning to prospective buyers to take precautions and investigate
the pending litigation.

The purpose of a notice of lis pendens is to protect the rights of the registrant while the case is pending resolution
or decision. With the notice of lis pendens duly recorded and remaining uncancelled, the registrant could rest
secure that he/she will not lose the property or any part thereof during litigation.

The doctrine of lis pendens is founded upon reason of public policy and necessity, the purpose of which is to keep
the subject matter of the litigation within the Court’s jurisdiction until the judgment or the decree have been
entered; otherwise, by successive alienations pending the litigation, its judgment or decree shall be rendered
abortive and impossible of execution.

Indeed, at the time HSLB bought the subject property, HSLB had actual knowledge of the annotated Notice of Lis
Pendens. Instead of heeding the same, HSLB continued with the purchase knowing the legal repercussions a notice
of lis pendens entails. HSLB took upon itself the risk that the Notice of Lis Pendens leads to. As correctly found by the
CA, “the notice of lis pendens was annotated on 14 September 1995, whereas the foreclosure sale, where the
appellant was declared as the highest bidder, took place sometime in 1997. There is no doubt that at the time
appellant purchased the subject property, it was aware of the pending litigation concerning the same property and
thus, the title issued in its favor was subject to the outcome of said litigation.”

The subject of the lis pendens on the title of HSLB’s vendor, Delgado, is the “Reformation case” filed against Delgado
by the herein respondents. The case was decided with finality by the CA in favor of herein respondents. The
contract of sale in favor of Delgado was ordered reformed into a contract of mortgage. By final decision of the CA,
HSLB’s vendor, Delgado, is not the property owner but only a mortgagee. As it turned out, Delgado could not have
constituted a valid mortgage on the property.

Insofar as the HSLB is concerned, there is no longer any public interest in upholding the indefeasibility of the
certificate of title of its mortgagor, Delgado. Such title has been nullified in a decision that had become final and
executory. Its own title, derived from the foreclosure of Delgado’s mortgage in its favor, has likewise been nullified
in the very same decision that restored the certificate of title in respondents’ name. There is absolutely no reason
that can support the prayer of HSLB to have its mortgage lien carried over and into the restored certificate of title
of respondents.
Rotairo v. Alcantara
G.R. No. 173632, Sept. 29, 2014

Where the vendor is not in possession of the property, the prospective vendees are obligated to investigate the rights of
one in possession. A purchaser cannot simply close his eyes to facts which should put a reasonable man on guard, and
thereafter claim that he acted in good faith under the belief that there was no defect in the title of the vendor.

FACTS:

Victor Alcantara and Alfredo Ignacio owned a parcel of land in Cainta, Rizal which they mortgaged to Pilipinas Bank
and Trust Company (Bank). They sold the same lot to Ambrosio Rotairo, who constructed his house on a portion of
the lot. Alcantara and Ignacio meanwhile defaulted on the loan, so the Bank foreclosed the mortgage. The Bank
then sold the entire property to Rovira Alcantara, who happens to be Victor’s daughter. Rovira filed an action for
recovery of possession and damages against Rotairo whose house was still on the subject lot. RTC dismissed the
case. CA reversed.

ISSUE:

Who, as between the Petitioners and Rovira, has better right to the property in dispute?

RULING:

The Petitioners. Rovira cannot claim a better right to the property because she is not a buyer in good faith. Two
factors work against Rovira as a buyer in good faith. One, she cannot be considered a third person for purposes of
applying the rule. Rovira does not deny that she is the daughter and an heir of Victor, one of the parties to the
contract to sell (and the contract of sale) executed in favor of Rotairo. Based on such privity, Rovira is charged with
constructive knowledge of prior dispositions or encumbrances affecting the subject property made by her father.
Further, more than the charge of constructive knowledge, the surrounding circumstances of this case show
Rovira’s actual knowledge of the disposition of the subject property and Rotairo’s possession thereof. Rovira, who
lives only 50 meters away from the subject property, in fact, knew that there were "structures built on the
property."

Under different circumstances, the prior registration of the mortgage between the Bank and Alcantara and Ignacio,
and Rovira's subsequent purchase of the subject property would have been valid and binding, and could have
defeated Rotairo's unregistered claim over it. But given Rovira's privity with her father Victor and the fact that she
had actual knowledge of the disposition of the property and Rotairo's possession thereof, her acquisition of the
property cannot be upheld.

c. Bad faith, NCC 549, 1132, 1137, 552

Tiu v. Jangas
G.R. No. 200285, March 20, 2017

When a piece of land is in the actual possession of persons other than the seller, the buyer must be wary and should
investigate the rights of those in possession. Without making such inquiry, one cannot claim that he is a buyer in good
faith.

FACTS:

Gregorio Pajulas during his lifetime owned a parcel of land designated as Lot No. 480-A. Upon his death, the same
property was transmitted by succession to his heirs, the Pajulas Sisters. Upon the death of Isabel, one of the Pajulas
Sisters, her share was inherited by her heirs which includes Norma Gadiane (Norma). Norma then sold a portion of
her share to Spouses Jangas.
Bruna, also one of the Pajulas Sisters, sold her 1/3 share to Spouses Gaudencio Delayco (Gaudencio) and Lucia
Amigo-Delayco (Spouses Delayco).

Subsequently, the heirs of Gaudencio represented by Bridiana Delayco (Bridiana) applied for and was granted a
free patent over the entire lot and an OCT was issued in their name. Thereafter, Bridiana was able to transfer title
over the lot to her name alone and sold the property to the petitioner. A TCT was then issued to the petitioner and
the latter mortgaged the subject property with Rural Bank of Amlan, Inc. (RBAI). This prompted Spouses Jangas to
file a complaint for reconveyance and damages against the petitioner and RBAI.

The RTC then ruled against the petitioner which was affirmed by the appellate court. On appeal before the SC, the
petitioner claimed that he acquired the property in good faith and for value.

ISSUE:

Whether the petitioner is a buyer in good faith.

RULING:

No, petitioner is not a buyer in good faith.

Based on the factual findings of the appellate court, the petitioner testified that when he visited the subject
property for surveying he already saw two structures that were built thereon, thus, he already knew that someone
else besides his seller has possession over the same.

The Court emphasized that when a piece of land is in the actual possession of persons other than the seller, the
buyer must be wary and should investigate the rights of those in possession. Without making such inquiry, one
cannot claim that he is a buyer in good faith. The failure of buyer to take the ordinary precautions which a prudent
man would have taken under the circumstances, especially in buying a piece of land in the actual, visible and public
possession of another person, other than the vendor, constitutes gross negligence amounting to bad faith.

The Court ruled that far from being prudent, it is clear that the petitioner chose to close his eyes to facts which
should have put a reasonable man on his guard. Consequently, he cannot now claim that he acted in good faith on
the belief that there was no defect in the title of his predecessor-in-interest.

d. Applicable presumptions, NCC 541, 527, 559, 1505, 1132, 1518, 528, 529, 554, 561, 542, 426

Luga v. Arciaga
G.R. No. 175343, July 27, 2011 36

FACTS:

The suit concerns a parcel of land situated in the District of Toril, Davao City, presently registered in the name of
respondent Elena Arciaga (Elena), married to respondent Rogelio Arciaga (Rogelio). The land used to form part of
the Y. Furukawa Daliao Plantation which, after being turned over to the Philippine government, was initially
administered by the National Abaca and Other Fibers Corporation (NAFCO) and, later, by the Board of Liquidators
(BOL). It appears that Loreto Luga (Loreto) became a tenant of the BOL and, in said capacity, occupied the subject
parcel since 1957, eventually building a house of light of materials thereon. On 28 July 1960, however, it appears
that an Occupants Affidavit was executed by one Honorio Romero (Honorio), a former employee of NAFCO, over a
2.5 hectare landholding of which the land in litigation formed part. Honorio executed a Deed of Transfer of Right
over a 600 square meter portion of said landholding in favor of Rogelio. In receipt of Elena's application for
patent/title over the subject parcel, the BOL issued and caused the posting of the Notice directing person/s
affected thereby to make known their adverse claim/s, if any. Elena was apprised of the fact that her application
had been approved. Loreto commenced the instant suit with the filing of his complaint for reconveyance of title and
damages against Elena and Rogelio. RTC ruled in favor of the petitioner rendering decision that Elena obtained title
through fraud and misrepresentation and falsification of document.
ISSUE: Whether or not Loreto is the rightful owner of the parcel of land

RULING: NO.

Perusal of the record shows that, as the party asserting a right superior to that claimed by the Spouses Arciaga,
Loreto failed to prove that he was a bona fide occupant of the land in litigation. Despite his testimony and that
elicited from his witnesses to the effect that he occupied the subject parcel in 1957, Loretos documentary evidence
consisting of receipts issued by the NAFCO and BOL simply showed that he was a tenant on the plantation from
1955 to 1957, remitting a portion of the produce harvested therefrom to said government agencies. Even if Loreto
were, moreover, to be considered the bona fide occupant of the land in dispute, it cannot be gainsaid that he
effectively forfeited the priority accorded him under Section 3 of Republic Act No. 477, as amended, when he failed
to register his claim in writing after the notice relative to Elenas application was posted at the Barangay Hall of
Toril, the Davao City Hall and BOL Davao Branch. Called as witness by Loreto during the presentation of the
evidence in chief, then Toril Barangay Chairman Consing Te categorically admitted on cross-examination that the
following notice was duly posted for thirty days at the Barangay Hall, 500 meters away from the land in litigation.

As prima facie proofs of ownership or possession of the property for which such taxes have been paid, tax receipts
and tax declaration may, moreover, become a basis of a claim of ownership when coupled with proof of actual
possession. More than the Occupants Affidavit of Application executed by Elena, the joint affidavit executed by her
witnesses and the certification issued by the Toril Barangay Chairman, the Spouses Arciagas actual possession of
the lot was, additionally proved by the Inspection and Investigation Report filed under oath by BOL
Inspector/Investigator. In the absence of proof adduced to rebut the presumption of regularity in the performance
of official duty, the same report deserves credence over Loretos naked assertion of possession of the subject parcel.

3. Acquisition and loss of possession, NCC 531, 532, 555, 1125, 1189, 1120-1124

Republic v. Aldana
G.R. No. 175578, August 11, 2010

The voluntary declaration of a piece of property for taxation purposes manifests not only one’s sincere and honest
desire to obtain title to the property. It also announces his adverse claim against the state and all other parties who
may be in conflict with his interest. More importantly, it signifies an unfeigned intention to contribute to government
revenues—an act that strengthens one’s bona fide claim of acquisition of ownership.

FACTS:

Respondents filed an application for registration of title over 2 pieces of land, professing themselves to be co-
owners of these lots having acquired them by succession from their predecessors. That until the time of the
application, they and their predecessors-in-interest have been in actual, open, peaceful, adverse, exclusive and
continuous possession of these lots in the concept of an owner and that they had consistently declared the
property in their name for purposes of real estate taxation. In support of their application, respondents submitted
to the court the pertinent tax declarations, together with the receipts of payment thereof. Petitioner opposed the
application for the reason that the tax declaration submitted to the court did not constitute competent and
sufficient evidence of bona fide acquisition in good faith or of prior possession in the concept of an owner.

ISSUE:

Whether or not respondents have occupied and possessed the property openly, continuously, exclusively and
notoriously under a bona fide claim of ownership.

RULING: YES.

Respondents’ possession through their predecessors-in-interest dates back to as early as 1937 when the property
had already been declared for taxation by respondent’s father. Respondents could have produced more proof of
this kind had it not been for the fact that, the relevant portions of the tax records on file with the Provincial
Assessor had been burned when its office was razed by fire in 1997. With the tax assessments there came next tax
payments. Respondents’ receipts for tax expenditures were likewise in the records and in these documents the
predecessors of respondents were the named owners of the property. Tax declarations and realty tax payment are
not conclusive evidence of ownership, nevertheless, they are a good indication of possession in the concept of an
owner. No one in his right mind would be paying taxes for a property that is not in his actual or at least
constructive possession. Indeed, respondents herein have been in possession of the land in the concept of an
owner, open, continuous, peaceful and without interference and opposition from the government or from any
private individual. Itself makes their right thereto unquestionably settled and hence, deserving of protection under
the law.

The voluntary declaration of a piece of property for taxation purposes manifests not only one’s sincere and honest
desire to obtain title to the property. It also announces his adverse claim against the state and all other parties who
may be in conflict with his interest. More importantly, it signifies an unfeigned intention to contribute to
government revenues—an act that strengthens one’s bona fide claim of acquisition of ownership.

VSD Realty v. Uniwide


GR No. 170677, Oct. 24, 2012

Introduction of valuable improvements on the leased premises does not give the lessee the right of retention and
reimbursement which rightfully belongs to a builder in good faith— the doctrine is that a lessee is neither builder in
good faith nor in bad faith that would call for the application.

FACTS:

Petitioner alleged that it is the registered owner of a parcel of land in Caloocan City. Petitioner purchased the said
property from Felisa D. Bonifacio. Petitioner alleged that respondent Baello is the holder and registered owner of a
parcel of land covered by TCT No. (35788) 12754 in the Register of Deeds for the Province of Rizal. Petitioner
alleged that its title, is the correct, valid and legal document that covers the subject property, since it is the result of
land registration proceedings in accordance with law. Respondent Baello filed an Answer, alleging that the subject
property was bequeathed to her through a will by her adoptive mother, Jacoba Galauran. Baello alleged that during
her open and public possession of the subject property spanning over 40 years, nobody came forward to contest
her title thereto. It was only in September 1994, when Baello was absent from the Philippines that petitioner
demanded
rentals from Uniwide, asserting ownership over the land. respondent Uniwide alleged that, it entered into a
Contract of Lease with respondent Baello involving a parcel of land with, located in Caloocan City, which property
is in the name of Baello. As a consequence of the lease agreement, it constructed a building worth at least
P200,000,000.00 on the said property and that the lease contract is valid and enforceable.

ISSUE:

W/N the respondents are lessors in good faith.

RULING:

It is noted that when the contract of lease was executed, Uniwide was unaware that the property leased by it was
owned by another person other than Dolores Baello. Nevertheless, Uniwide cannot avail of the rights of a builder in
good faith under Article 448 of the Civil Code, in relation to Article 546 of the same Code, which provides for full
reimbursement of useful improvements and retention of the premises until reimbursement is made, as the said
provisions apply only to a possessor in good faith who builds on land with the belief that he is the owner thereof. It
does not apply where one’s only interest is that of a lessee under a rental contract. Introduction of valuable
improvements on the leased premises does not give the lessee the right of retention and reimbursement which
rightfully belongs to a builder in good faith— the doctrine is that a lessee is neither a builder in good faith nor in
bad faith that would call for the application.
Aldover v. CA
G.R. No. 167174, September 23, 2013

FACTS:

Siblings Tomas Reyes and Sidra Reyes, and their father Alfredo Reyes (the Reyeses) failed to pay the loan causing
their mortgaged lot to be extrajudicially foreclosed. Antonia Aldover (Aldover) emerged as the winning bidder and
a TCT was issued in her name. A Writ of Possession was issued in favor of Aldover by the RTC. Thereafter, the
Sheriff issued a Notice to Vacate but there are other persons claiming to be owners occupying the subject lot. The
respondents then filed before the RTC a Complaint alleging that that they have been residing in the same lot since
the 1960’s by virtue of lease contracts wherein they were allowed by the Reyeses to build their houses.
Subsequently, their occupation became in the concept of owners after the Reyeses sold to them portions of the lot
they respectively occupy. Respondents insisted that petitioners were aware of the lease and subsequent sale.
Respondents also claimed that the REM is a fictitious transaction because at the time of its execution the Reyeses
were no longer the owners of the entire property subject thereof. Hence, the mortgage as well as the subsequent
foreclosure sale is null and void.

However, it such complaint was denied by the court on the ground that it cannot interfere with the order of a
coordinate court. Meanwhile, Aldover filed a Motion for Special Order of Demolition which the court granted.
Respondents then filed before the CA a Petition for Certiorari, Prohibition, and Injunction with prayer for the
issuance of a TRO and/or Writ of Preliminary Injunction against Aldover and the Reyeses, which the CA granted.

ISSUE:

Whether the issuance of a writ of preliminary injunction proper?

RULING: YES.

Respondents are in actual possession of the disputed portions of subject property which raises a disputable
presumption that they are the owners thereof. Thus, petitioners cannot resort to procedural shortcut in ousting
them by the simple expedient of filing a Motion for Special Order of Demolition for under Article 433, petitioners
have to file the appropriate judicial process to recover the property from the respondents. This "judicial process
could mean no less than an ejectment suit or a reinvindicatory action, in which the ownership claims of the
contending parties may be properly heard and adjudicated."

Moreover, they were not impleaded and did not take part would be tantamount to taking of real property without
due process of law. In fine, the CA cannot be said to have acted capriciously, whimsically, arbitrarily or despotically
in issuing its Writ of Preliminary Injunction to prevent a threatened or continuous irremediable injury. Indeed, the
precipitate demolition of their houses would constitute material and substantial invasion of their right which
cannot be remedied under any standard compensation. Hence, the need for a Writ of Preliminary Injunction.

Nenita Quality Foods Corp. v. Galabo, et al.


G.R. No. 174191, January 30, 2013

FACTS:

The respondents are the heirs of Donato Galabo. In 1948, Donato obtained Lot No. 722, a portion of the Arakaki
Plantation in Davao City owned by National Abaca and Other Fibers Corporation. Donato and the respondents
assumed that Lot No. 722 included Lot No. 102, per the original survey of 1916 to 1920. When the Board of
Liquidators (BOL) took over the administration of the plantation in the 1950s, it had Lot No. 722 resurveyed.
Allegedly, the resurvey did not include Lot No. 102; thus, when Donato acquired TCT No. T-21496 for Lot No. 722,
Lot 102 was not included. The respondents, however, continue to posses, occupy and cultivate Lot 102.

When petitioner opened its business in Davao City, it allegedly offered to buy Lot No. 102. Galabo declined. In the
1970, the heirs fenced off the entire perimeter of Lot No. 102 and built his house on it. But in 1994, the
respondents received a letter from Santos demanding that they vacate Lot No. 102 as he claimed ownership of this
lot per the Deed of Transfer which the respondents and their mother allegedly executed in Santos’ favor. The
respondents denied and instead applied for free patent over Lot No. 102 in 2000.

Respondents filed a complaint for forcible entry with damages before the MTCC against NQFC, alleging that: (1)
they had been in prior physical possession; and (2) NQFC deprived them of possession through force, intimidation,
strategy, threats and stealth.

ISSUE

W/N NQFC had been in prior physical possession of Lot No. 102.

RULING: NO.

In a forcible entry case, a party who can prove prior possession can recover such possession even against the
owner himself. Whatever may be the character of his possession, if he has in his favor prior possession in time, he
has the security that entitles him to remain on the property until a person with a better right lawfully ejects him. SC
agreed with the respondents that instead of squarely addressing the issue of possession and presenting evidence
showing that NQFC or Santos had been in actual possession of Lot No. 102, the former merely narrated how it
acquired ownership of Lot No. 102 and presented documents to this effect. Its allegation that Santos occupied Lot
No. 102 in 1972 is uncorroborated. Even the tax declarations under Santos name are hardly of weight. This
Decision deals only with de facto possession and is without prejudice to an appropriate action for recovery of
possession based on ownership.

Republic v. Remman
G.R. No. 199310. February 19, 2014

FACTS:

On December 3, 2001, Remman Enterprises filed an application with the RTC for judicial confirmation of title over
two parcels of land situated in Taguig, Metro Manila, identified as Lot Nos. 3068 and 3077, Mcadm-590-D, Taguig
Cadastre, with an area of 29,945 square meters and 20,357 sq m, respectively.

The RTC found the application for registration sufficient in form and substance and set it for initial hearing on May
30, 2002. The Notice of Initial Hearing was published in the Official Gazette and was likewise posted in a
conspicuous places.

On the day of the hearing, only the Laguna Lake Development Authority (LLDA) appeared as oppositor. Hence, the
RTC issued an order of general default except LLDA, which was given 15 days to submit its comment/opposition to
the respondent’s application for registration. Sometime after, the Republic of the Philippines (petitioner) likewise
filed its Opposition, alleging that the respondent failed to prove that it and its predecessors-in-interest have been
in open, continuous, exclusive, and notorious possession of the subject parcels of land since June 12, 1945 or
earlier.

During the trial, the testimonies of the respondent’s witnesses showed that the respondent and its predecessors-
in-interest have been in open, continuous, exclusive, and notorious possession of the said parcels of land long
before June 12, 1945. The respondent purchased Lot Nos. 3068 and 3077 from Conrado Salvador and Bella Mijares,
respectively, in 1989. The subject properties were originally owned and possessed by Veronica Jaime, who
cultivated and planted different kinds of crops in the said lots, through her caretaker and hired farmers, since
1943. Sometime in 1975, Jaime sold the said parcels of land to Salvador and Mijares, who continued to cultivate the
lots until the same were purchased by the respondent in 1989. The respondent likewise alleged that the subject
properties are within the alienable and disposable lands of the public domain, as evidenced by the certifications
issued by the Department of Environment and Natural Resources (DENR).

In support of its application, the respondent, inter alia, presented the following documents: (1) Deed of Absolute
Sale dated August 28, 1989 executed by Salvador and Mijares in favor of the respondent; (2) survey plans of the
subject properties; (3) technical descriptions of the subject properties; (4) Geodetic Engineer’s Certificate; (5) tax
declarations of Lot Nos. 3068 and 3077 for 2002; and (6) certifications dated December 17, 2002, issued by
Corazon D. Calamno, Senior Forest Management Specialist of the DENR, attesting that Lot Nos. 3068 and 3077 form
part of the alienable and disposable lands of the public domain.

For its part, the LLDA alleged that the respondent’s application for registration should be denied since the subject
parcels of land are not part of the alienable and disposable lands of the public domain; it pointed out that pursuant
to Section 41(11) of R.A. No. 4850, lands, surrounding the Laguna de Bay, located at and below the reglementary
elevation of 12.50 meters are public lands which form part of the bed of the said lake. Engr. Magalonga, testifying
for the oppositor LLDA, claimed that, upon preliminary evaluation of the subject properties, based on the
topographic map of Taguig, which was prepared using an aerial survey conducted by the then Department of
National Defense-Bureau of Coast in April 1966, he found out that the elevations of Lot Nos. 3068 and 3077 are
below 12.50 m. That upon actual area verification of the subject properties on September 25, 2002, Engr.
Magalonga confirmed that the elevations of the subject properties range from 11.33 m to 11.77 m.

On rebuttal, the respondent presented Engr. Flotildes, who claimed that, based on the actual topographic survey of
the subject properties he conducted upon the request of the respondent, the elevations of the subject properties,
contrary to LLDA’s claim, are above 12.50 m. Particularly, Engr. Flotildes claimed that Lot No. 3068 has an
elevation ranging from 12.60 m to 15 m while the elevation of Lot No. 3077 ranges from 12.60 m to 14.80 m.

The RTC ruled in favor of respondent. The RTC pointed out that LLDA’s claim that the elevation of the subject
properties is below 12.50 m is hearsay since the same was merely based on the topographic map that was
prepared using an aerial survey on March 2, 1966; that nobody was presented to prove that an aerial survey was
indeed conducted on March 2, 1966 for purposes of gathering data for the preparation of the topographic map.

Further, the RTC posited that the elevation of a parcel of land does not always remain the same; that the elevations
of the subject properties may have already changed since 1966 when the supposed aerial survey, from which the
topographic map used by LLDA was based, was conducted. The RTC likewise faulted the method used by Engr.
Magalonga in measuring the elevations of the subject properties.

Even supposing that the elevations of the subject properties are indeed below 12.50 m, the RTC opined that the
same could not be considered part of the bed of Laguna Lake. The RTC held that, under Section 41(11) of R.A. No.
4850, Laguna Lake extends only to those areas that can be covered by the lake water when it is at the average
annual maximum lake level of 12.50 m. Hence, the RTC averred, only those parcels of land that are adjacent to and
near the shoreline of Laguna Lake form part of its bed and not those that are already far from it, which could not be
reached by the lake water. The RTC pointed out that the subject properties are more than a kilometer away from
the shoreline of Laguna Lake; that they are dry and waterless even when the waters of Laguna Lake is at its
maximum level. The RTC likewise found that the respondent was able to prove that it and its predecessors-in-
interest have been in open, continuous, exclusive, and notorious possession of the subject properties as early as
1943.

The CA affirmed the RTC Decision.

ISSUE:

Is respondent entitled to the registration of title to the subject properties?

HELD: NO

That the elevations of the subject properties are above the reglementary level of 12.50 m is a finding of fact by the
lower courts, which this Court, generally may not disregard. This Court is not a trier of facts and will not disturb
the factual findings of the lower courts unless there are substantial reasons for doing so. That the subject
properties are not part of the bed of Laguna Lake, however, does not necessarily mean that they already form part
of the alienable and disposable lands of the public domain. It is still incumbent upon the respondent to prove, with
well-nigh incontrovertible evidence, that the subject properties are indeed part of the alienable and disposable
lands of the public domain.

While deference is due to the lower courts’ finding that the elevations of the subject properties are above the
reglementary level of 12.50 m and, hence, no longer part of the bed of Laguna Lake, the Court nevertheless finds
that the respondent failed to substantiate its entitlement to registration of title to the subject properties.

"Under the Regalian Doctrine, xxxx all lands of the public domain belong to the State, which is the source of any
asserted right to any ownership of land. All lands not appearing to be clearly within private ownership are
presumed to belong to the State. Accordingly, public lands not shown to have been reclassified or released as
alienable agricultural land, or alienated to a private person by the State, remain part of the inalienable public
domain. The burden of proof in overcoming the presumption of State ownership of the lands of the public domain
is on the person applying for registration, who must prove that the land subject of the application is alienable or
disposable. To overcome this presumption, incontrovertible evidence must be presented to establish that the land
subject of the application is alienable or disposable."

The respondent filed its application for registration of title to the subject properties under Section 14(1) of
Presidential Decree (P.D.) No. 1529. Under said Section, applicants for registration of title must sufficiently
establish:

1) that the subject land forms part of the disposable and alienable lands of the public domain;
2) that the applicant and his predecessors-in-interest have been in open, continuous, exclusive, and notorious
possession and occupation of the same; and
3) that it is under a bona fide claim of ownership since June 12, 1945, or earlier.

The first requirement was not satisfied in this case. To prove that the subject property forms part of the alienable
and disposable lands of the public domain, the respondent presented two certifications issued by Calamno,
attesting that Lot Nos. 3068 and 3077 form part of the alienable and disposable lands of the public domain "under
Project No. 27-B of Taguig, Metro Manila as per LC Map 2623, approved on January 3, 1968."

However, the said certifications presented by the respondent are insufficient to prove that the subject properties
are alienable and disposable. In Republic of the Philippines v. T.A.N. Properties, Inc., the Court clarified that, in
addition to the 1) certification issued by the proper government agency that a parcel of land is alienable and
disposable, applicants for land registration must 2) prove that the DENR Secretary had approved the land
classification and released the land of public domain as alienable and disposable. They must 3) present a copy of
the original classification approved by the DENR Secretary and 4) certified as true copy by the legal custodian of
the records.

The DENR certifications that were presented by the respondent in support of its application for registration are
thus not sufficient to prove that the subject properties are indeed classified by the DENR Secretary as alienable and
disposable. It is still imperative for the respondent to present a copy of the original classification approved by the
DENR Secretary, which must be certified by the legal custodian thereof as a true copy. Accordingly, the lower
courts erred in granting the application for registration in spite of the failure of the respondent to prove by well-
nigh incontrovertible evidence that the subject properties are alienable and disposable.

Nevertheless, the respondent claims that the Court’s ruling in T.A.N. Properties, which was promulgated on June
26, 2008, must be applied prospectively, asserting that decisions of this Court form part of the law of the land and,
pursuant to Article 4 of the Civil Code, laws shall have no retroactive effect. The respondent points out that its
application for registration of title was filed and was granted by the RTC prior to the Court’s promulgation of its
ruling in T.A.N. Properties.

The Court does not agree.

Notwithstanding that the respondent’s application for registration was filed and granted by RTC prior to the
Court’s ruling in T.A.N. Properties, the pronouncements in that case may be applied to the present case; it is not
antithetical to the rule of non-retroactivity of laws pursuant to Article 4 of the Civil Code. It is elementary that the
interpretation of a law by this Court constitutes part of that law from the date it was originally passed, since this
Court’s construction merely establishes the contemporaneous legislative intent that the interpreted law carried
into effect. "Such judicial doctrine does not amount to the passage of a new law, but consists merely of a
construction or interpretation of a pre-existing one."

Anent the second and third requirements, the Court finds that the respondent failed to present sufficient evidence
to prove that it and its predecessors-in-interest have been in open, continuous, exclusive, and notorious possession
and occupation of the subject properties since June 12, 1945, or earlier.

To prove that it and its predecessors-in-interest have been in possession and occupation of the subject properties
since 1943, the respondent presented the testimony of Cerquena which are but unsubstantiated and self-serving
assertions of the possession and occupation of the subject properties by the respondent and its predecessors-in-
interest; they do not constitute the well-nigh incontrovertible evidence of possession and occupation of the subject
properties required by Section 14(1) of P.D. No. 1529. Indeed, other than the testimony of Cerquena, the
respondent failed to present any other evidence to prove the character of the possession and occupation by it and
its predecessors-in-interest of the subject properties.

For purposes of land registration under Section 14(1) of P.D. No. 1529, proof of specific acts of ownership must be
presented to substantiate the claim of open, continuous, exclusive, and notorious possession and occupation of the
land subject of the application. Applicants for land registration cannot just offer general statements which are mere
conclusions of law rather than factual evidence of possession. Actual possession consists in the manifestation of
acts of dominion over it of such a nature as a party would actually exercise over his own property.

Further, assuming ex gratia argumenti that the respondent and its predecessors-in-interest have indeed planted
crops on the subject properties, it does not necessarily follow that the subject properties have been possessed and
occupied by them in the manner contemplated by law. The supposed planting of crops in the subject properties
may only have amounted to mere casual cultivation, which is not the possession and occupation required by law.

"A mere casual cultivation of portions of the land by the claimant does not constitute possession under claim of
ownership. For him, possession is not exclusive and notorious so as to give rise to a presumptive grant from the
state. The possession of public land, however long the period thereof may have extended, never confers title
thereto upon the possessor because the statute of limitations with regard to public land does not operate against
the state, unless the occupant can prove possession and occupation of the same under claim of ownership for the
required number of years."

Further, the Court notes that the tax declarations over the subject properties presented by the respondent were
only for 2002. The respondent failed to explain why, despite its claim that it acquired the subject properties as
early as 1989, and that its predecessors-in-interest have been in possession of the subject property since 1943, it
was only in 2002 that it started to declare the same for purposes of taxation. "While tax declarations are not
conclusive evidence of ownership, they constitute proof of claim of ownership." That the subject properties were
declared for taxation purposes only in 2002 gives rise to the presumption that the respondent claimed ownership
or possession of the subject properties starting that year.

Suarez v. Emboy, Jr.


G.R. No. 187944, March 12, 2014

The jurisdictional facts must appear on the face of the complaint. When the complaint fails to aver facts constitutive of
forcible entry or unlawful detainer, as where it does not state how entry was effected or how and when dispossession
started, the remedy should either be an accion publiciana or accion reivindicatoria.
There are circumstances where the unlawful detainer action must be abated due to pending action involving the
ownership of the same property, such as when the unlawful detainer would result to demolition of the house built
thereon. The litigants as well as the courts will be wasting much time and effort by proceeding at a stage wherein the
outcome is at best temporary, but the result of enforcement is permanent, unjust and probably irreparable.
FACTS:

The respondents (heirs of Claudia) have been living in the subject lot for decades, where they built their house. The
subject lot was a part of the lot inherited by, among others, Claudia and Vicente, from their parents. One time, the
petitioner Suarez claims ownership of the house. Petitioner’s claim rests on the fact that she bought the property
from the respondents’ cousins (heirs Vicente, Vicente is Claudia’s brother). Apparently, petitioner and the heirs of
Vicente show a deed of partition where the subject lot was given to Vicente instead of Claudia. The respondents
filed an action for the nullification of the deed of partition. But on the petitioner’s part, she filed a case of unlawful
detainer against the respondents. However, petitioner failed to allege how and when the respondents possessed
the lot in question, nor how the heirs of Vicente were merely tolerating the possession of the respondents.

ISSUES:

(1) Whether or not Suarez has cause of action for unlawful detainer.
(2) Whether or not the action for unlawful detainer must continue despite an action for the ownership of the
same property is being litigated in another court.

RULING:

(1) No. When the complaint fails to aver facts constitutive of forcible entry or unlawful detainer, as where it
does not state how entry was effected or how and when dispossession started, the remedy should either be an
accion publiciana or accion reivindicatoria. The jurisdictional requirement of possession by mere tolerance of the
owners had not been amply alleged and proven.

(2) No. The rule is that an action regarding the ownership of the land does not suspend the ejectment case
involving the same property. However, the respondents claim the defense of ownership, and point to the existence
of the civil case for nullification of the deed of partition involving the same lot, where ownership over the lot was
presented as an issue. If Suarez’ complaint will be granted, the respondents’ house, which has been standing in the
subject lot for decades, would be subject to demolition. The foregoing circumstances, thus, justify the exclusion of
the instant petition from the purview of the general rule. Suarez can ventilate the claim of ownership in the civil
case for nullification of the deed of partition.

Ocampo v. Dionisio
G.R. No. 191101, October 01, 2014

A judgment rendered in a forcible entry case will not bar an action between the same parties respecting title or
ownership because between a case for forcible entry and an accion reinvindicatoria, there is no identity of causes of
action.

FACTS:

Dionisio filed a complaint for forcible entry against Mario Ocampo and Felix Ocampo. Dionisio alleged that Mario
and Felix built a piggery thereon without his consent. Mario claimed that the disputed parcel of land is owned by
his wife, Carmelita Ocampo, who inherited the same from her father. Mario further claimed that they have been in
possession of the said parcel of land since 1969.

MTC ruled that Dionisio failed to establish his prior possession of the disputed parcel of land. Dionisio died on
September 27, 1997. Consequently, the heirs of Dionisio filed a complaint for recovery of possession with the MTC
against the spouses Mario and Carmelita.

Petitioners claimed that the respondents’ complaint for recovery of possession is barred by res judicata in the light
of the finality of the decision in the forcible entry case.

ISSUE:
Whether the finality of the decision in the forcible entry case constitutes res judicata?

RULING:

No. In an action for forcible entry and detainer, the only issue is possession in fact, or physical possession of real
property, independently of any claim of ownership that either party may put forth in his pleading. If plaintiff can
prove prior physical possession in himself, he may recover such possession even from the owner, but, on the other
hand, if he cannot prove such prior physical possession, he has no right of action for forcible entry and detainer
even if he should be the owner of the property.

A judgment rendered in a forcible entry case will not bar an action between the same parties respecting title or
ownership because between a case for forcible entry and an accion reinvindicatoria, there is no identity of causes of
action. Such determination does not bind the title or affect the ownership of the land; neither is it conclusive of the
facts therein found in a case between the same parties upon a different cause of action involving possession.

The decision in the forcible entry case is conclusive only as to the MTC’s determination that the petitioners are not
liable for forcible entry since the respondents failed to prove their prior physical possession; it is not conclusive as
to the ownership of the subject property. Besides, Section 18, Rule 70 of the Rules of Court expressly provides that
a "judgment rendered in an action for forcible entry or detainer shall be conclusive with respect to the possession
only and shall in no wise bind the title or affect the ownership of the land."

Novecio v. Hon. Lim


G.R. 193809 March 23, 2015

FACTS:

Respondents filed complaints for forcible entry with damages against petitioners.
The respondents alleged that on February 15, 2004, the petitioners, by force, intimidation, threat, strategy and
stealth, unlawfully squatted and took possession of several portions of land. The petitioners allegedly planted
crops, erected makeshift shelters, and continue to plant and /or improve the shelters as of the filing of the
complaints for forcible entry, all without the consent and/or against the will of the respondents.

The petitioners, on the other hand, contended that they have already been in possession of the land and planted on
the same for more than two years when the complaints were filed. They also alleged that they were harassed by
some men armed with shotguns and pistols on February 12, 2004.
The petitioners further maintained that Manuel V. Nieto, father of Maria Carmen J. Tuazon, had previous
landholding in the area but the same was covered by the Comprehensive Agrarian Reform Program (CARP) and so
it was subdivided in favor of the tenants.

MTC ruled in favor of the Petitioners, ruling that the two years have already passed since the respondents have
allegedly been dispossessed of their property, such period beyond the ambit of an action for Forcible entry and
unlawful Detainer. RTC, reversed the decision and ruled in favor of respondents. As the respondents sought the
execution of the RTC judgment, the petitioners filed on May 14, 2010 an Extremely Urgent Application for Writ of
Preliminary Injunction and Immediate Issuance of Temporary Restraining Order before the Court of Appeals. CA
denied the application.

ISSUE:

Whether or not the alleged dispossession done by the petitioners will give ground for the filing of a case for
forcible entry.

HELD: NO.

In a prayer for preliminary injunction, the plaintiff is not required to submit conclusive and complete evidence. He
is only required to show that he has an ostensible right to the final relief prayed for in his complaint. The Court
approved the application for writ of injunction of the petitioners on the basis of the findings of the MTC which
stated that:

xxx Force, intimidation[,] and threat usually connote actual knowledge of dispossession. One cannot force,
intimidate or threaten another who is not around. In stealth and strategy[,] the actual entry is usually done without
the knowledge of the plaintiff. If they are not sure how [the] defendants entered the land[,] the likelihood is that
they also do not know when [the] defendants] entered the land. The court is apt to believe that [the] defendants
have been in possession of the land for more than 2 years. And under Rule 70[,] the action of forcible entry must be
filed within one year from dispossession. The filing of these cases was beyond the one-year period.

The RTC, on the other hand, relied on a mere request for authority to conduct a land survey, allegedly showing that
respondent Manuel V. Nieto was the occupant and tiller of the land.

However, this document does not prove prior possession of the subject land. It only points to the fact that there
was an application for a land title in the name of one of the respondents, which application was not even shown to
have been granted. This document merely authorized the survey of the land; the declaration regarding possession
was just incidental to the application for land survey.

Between the clear findings of the MTC, which conducted the trial of the forcible entry cases, and the RTC acting as
an appellate court, which relied on documentary evidence but without sufficiently explaining how such evidence
would prove prior possession, we are inclined to give weight to the MTC's ruling.

Fullido v. Grilli
G.R. No. 215014, February 29, 2016

In Llantino v. Co Liong Chong, however, the Court clarified that a lease contract in favor of aliens for a reasonable
period was valid as long as it did not have any scheme to circumvent the constitutional prohibition, such as depriving
the lessors of their right to dispose of the land.

FACTS:

Grilli, an Italian national, met Fullido in Bohol and courted her. Grilli decided to build a residential house. He
financially assisted Fullido in procuring a lot from her parents. They constructed a house funded by Grilli and upon
completion, they maintained a common law relationship and live there whenever Grilli was on vacation in the
Philippines.

Grilli and Fullido executed a contract of lease, memorandum of agreement, and a special power of attorney to
define their rights over the house and lot. It stipulated among others that Grilli, as lessee, would rent the lot
registered in Fullido’s name for 50 years. Fullido, as lessor, was prohibited from selling, donating or encumbering
the lot with written consent of Grilli. The SPA allowed Fullido to administer, manage and transfer the property.

Their relationship turned sour after 16 years. They could not agree on who should leave the property. Grilli sent
formal demand letters to Fullido, demanding her to vacate the property. Grilli then filed a complaint for unlawful
detainer with a [rayer for the issuance of a writ of preliminary injunction. When he discovered that Fullido was
pregnant, he allowed Fullido to stay in the house without rent even if the child wasn’t his. She let he 2 children,
siblings and parents stay. He lost his personal belongings in the house and Fullido and her family was hostile
towards him. When he asked her to leave, she refused.

The MCTC dismissed the complaint as Fullido was a co-owner who cannot be ejected. RTC reversed the ruling,
holding that Grilli had the exclusive right to use and possess the property by virtue of the contract of lease. CA
upheld RTC’s decision.

ISSUE:
Whether or not Grilli can compel Fullido to vacate the property.

RULING:

The prohibition on the transfer of lands to aliens was adopted in the present 1987 Constitution, under Sections 2, 3
and 7 of Article XII thereof. Agricultural lands, whether public or private, include residential, commercial and
industrial lands. The purpose of prohibiting the transfer of lands to foreigners is to uphold the conservation of our
national patrimony and ensure that agricultural resources remain in the hands of Filipino citizens.

The prohibition, however, is not limited to the sale of lands to foreigners. It also covers leases of lands amounting
to the transfer of all or substantially all the rights of dominion. if an alien is given not only a lease of, but also an
option to buy, a piece of land by virtue of which the Filipino owner cannot sell or otherwise dispose of his property,
this to last for 50 years, then it becomes clear that the arrangement is a virtual transfer of ownership whereby the
owner divests himself in stages not only of the right to enjoy the land but also of the right to dispose of it — rights
which constitute ownership. If this can be done, then the Constitutional ban against alien landholding in the
Philippines, is indeed in grave peril.

The Court finds that the lease contract and the MOA in the present case are null and void for virtually transferring
the reigns of the land to a foreigner.

As can be gleaned from the contract, the lease in favor of Grilli was for a period of fifty (50) years, automatically
extended for another fifty (50) years upon the expiration of the original period. Moreover, it strictly prohibited
Fullido from selling, donating, or encumbering her land to anyone without the written consent of Grilli. For a
measly consideration of P10,000.00, Grilli would be able to absolutely occupy the land of Fullido for 100 years, and
she is powerless to dispose the same. The terms of lease practically deprived Fullido of her property rights and
effectively transferred the same to Grilli.

The title of Fullido over the land became an empty and useless vessel, visible only in paper, and was only meant as
a dummy to fulfill a foreigner’s desire to own land within our soils. It is disturbing how these documents were
methodically formulated to circumvent the constitutional prohibition against land ownership by foreigners. The
said contracts attempted to guise themselves as a lease, but a closer scrutiny of the same revealed that they were
intended to transfer the dominion of a land to a foreigner in violation of Section 7, Article XII of the 1987
Constitution. Even if Fullido voluntary executed the same, no amount of consent from the parties could legalize an
unconstitutional agreement. The lease contract and the MOA do not deserve an iota of validity and must be
rightfully struck down as null and void for being repugnant to the fundamental law. These void documents cannot
be the source of rights and must be treated as mere scraps of paper.

F. Usufruct
1. Objects of Usufruct

2. Elements in a Usufruct

3. Classes of Usufruct - NCC 563, 563, FC 226, NCC 564, 756, 863, 869, 564, 574, 573, 564, 598-599, 758-759

4. Rights of Usufructuary – NCC 566-569, 438, 570, 571-572, 590, 578-579, 594, 580

5. As to the Legal Right of Usufruct Itself – NCC 580, 572

6. Obligations of the Usufructuary – NCC 583, 586, 599, 585, 584, 587, FC 226, NCC 584

7. During the Usufruct – NCC 589, 610, 590, 591, 592, 593,594, 595, 596-597, 610, 602

8. At the Time of the Termination of the Usufruct – NCC 612

9. Special Cases of Usufruct – NCC 570, 582, 591, 575-576, 578, 600, 598, 573, 574
37

Fabie v. Sps. David


G.R. No. L-123, December 12, 1945

All the acts of administration; to collect the rents for herself, and to conserve the property by making all necessary
repairs and paying all the taxes, special assessments, and insurance premiums thereon belongs to the usufructuary.

FACTS:

Josefa Fabie is the usufructuary of several houses located in Binondo, Sta. Cruz Manila under the ninth clause of the
will of the deceased Rosario Fabie.
The owner of the said property in Binondo is Juan Grey.
In 1945 Fabie sued an individual named Ngo Soo for unlawful detainer on the said property for non-payment of
rent.
Ngo Soo defended that he had been paying the said rent to the owner Juan Grey.
Grey intervened that he was the absolute owner and therefore had the right and power of administration over the
said property.
The Municipal Trial Court ruled in favor of Fabie, ordering Soo to vacate the premises.
The CFI in turn dismissed the case stating that the controversy is not an issue of ownership but of possessory
rights.
Fabie challenged the said dismissal stating that her action was a possessory one.

ISSUES:

Between the usufructuary, and the owner who has the power of administration over the property, and the right to
collect rents from the same.
Who has jurisdiction over the said issue?

RULING:

Based on the 9th clause of the deceased Rosario fabie, the court found he said usufructuary has the right to
administer the property in question. All the acts of administration; to collect the rents for herself, and to conserve
the property by making all necessary repairs and paying all the taxes, special assessments, and insurance
premiums thereon

Grey admitted that before said judgment he had been collecting the rents as agent of the usufructuary under an
agreement with the latter. As long as the property is properly conserved and insured he can have no cause for
complaint, and his right in that regard is fully protected by the terms of the stipulation and the judgment of the
court above mentioned. To permit him to arrogate to himself the privilege to choose the tenant, to dictate the
conditions of the lease, and to sue when the lessee fails to comply therewith, would be to place the usufructuary
entirely at his mercy. It would place her in the absurd situation of having a certain indisputable right without the
power to protect, enforce, and fully enjoy it.

In terms of jurisdiction, the CFI was evidently confused and led to misconstrue the real issue by the complaint in
intervention of Juan Grey, who, allying himself with the defendant Ngo Soo, claimed that he is the administrator of
the property.

The action is purely possessory and not one in any way involving the title to the property.

In determining whether an action of this kind is within the original jurisdiction of the municipal court or of the
Court of First Instance, the averments of the complaint and the character of the relief sought are primarily to be
consulted; that the defendant in such an action cannot defeat the jurisdiction of the justice of the peace or
municipal court by setting up title in himself.
Baluran v. Navarro
79 SCRA 309 (1977)

The manner of terminating the right of usufruct may be stipulated by the parties such as in this case, the happening of
a resolutory condition.

FACTS:

Baluran and Paraiso (ancestor of Obedencio) entered into a contract, which they called barter. However, it was in
fact stipulated that they would only transfer the material possession of their respective properties to each other.
Thus, Baluran will be allowed to construct a residential house on the land of Paraiso while Paraiso is entitled to
reap the fruits of the riceland of Baluran.

The contract prohibited them from alienating the properties of the other and contained a stipulation that should
the heirs of Paraiso desire to re-possess the residential lot, Baluran is obliged to return the lot.

Indeed, years after, Obedencio (grandchild of Paraiso) acquired the ownership of the residential lot from his
mother and demanded that Baluran, who was in possession, vacate. Baluran now counters that the barter already
transferred ownership.

ISSUE:

Whether or not the contract was a barter or usufruct

RULING:

USUFRUCT.

First, the contract is what the law defines it to be and not what the parties call it. It is very clear that what
the parties exchanged was not ownership, but merely material possession or the right to enjoy the thing.

Now, because it is usufruct, the law allows the parties to stipulate the conditions including the manner of its
extinguishment. In this case, it was subject to a resolutory condition, which is in case the heir of Paraiso (a third
party) desires to repossess the property. Upon the happening of the condition, the contract is extinguished.

Therefore, Baluran must return the land to Obedencia. But, since Art. 579 allows the usufructuary to remove
improvements he made, Baluran may remove the house he constructed. At the time of this case, the Obedencias
were also in possession of the riceland of Baluran. Although it was not proper to decide the issue of possession in
this case, the Court nevertheless decided on the matter and order the Obedencias to vacate the property inasmuch
as there was an extinguishment of reciprocal obligations and rights.

Gaboya v Cui, 38 SCRA 85

10. Rights of the Naked Owner – NCC 574, 581

11. Extinguishment of the usufruct, NCC 603, 611, 606, 605, 607, 608, 1381, 709

Calingasan v. Rivera
G.R. No. 171555, April 17, 2013

Whatever may be the character of his possession, if he has in his favor prior possession in time, he has the security that
entitles him to remain on the property until a person with a better right lawfully ejects him." "The party in peaceable,
quiet possession shall not be thrown out by a strong hand, violence, or terror."

FACTS:
Wilfredo Rivera and his 2 daughters named Evangeline and Brigida Liza heirs of Loreto, executed an extrajudicial
settlement of the one half share of the latter. Wilfredo waived his rights to the properties with a reservation of his
usufructuary rights during his lifetime. Registered deed of sale was subsequently executed in favor of Evangeline
with an annotation of Wilfredo’s usufructuary rights.

A decade later, Wilfredo filed with the MTCC of Lipa City a complaint for forcible entry against petitioners and Star
Honda, inc. Wilfredo claimed that he is lawfully possessed and occupied the 2 parcels of land with a building used
for his furniture business.

Taking advantage of his absence due to his hospital confinement in September 2002, the petitioners and Star
Honda, Inc. took possession and caused the renovation of the building on the property.

Both the petitioners and Star Honda, Inc. countered that Wilfredo voluntarily renounced his usufructuary rights in
a petition for cancellation of usufructuary rights.

MTCC dismissed the complaint. On appeal to the RTC, the court affirmed the MTCC decision It held that Wilfredo
lacked a cause of action to evict the petitioners and Star Honda, Inc. since Evangeline is the registered owner of the
property and Wilfredo had voluntarily renounced his usufructuary rights.

Wilfredo sought reconsideration and the RTC ruled in favour of Wilfredo taking consideration of Evangeline’s
judicial admission of "J. Belen Street, Rosario, Batangas" as her residence since May 2002.
CA affirmed. Hence this petition.

ISSUE:

Who between petitioners and Wilfredo had a prior physical possession over the property?

HELD:

In this case, we are convinced that Wilfredo had been in prior possession of the property and that the petitioners
deprived him of such possession by means of force, strategy and stealth.

Ejectment cases involve only physical possession or possession de facto.

"Ejectment cases - forcible entry and unlawful detainer - are summary proceedings designed to provide
expeditious means to protect actual possession or the right to possession of the property involved. The only
question that the courts resolve in ejectment proceedings is: who is entitled to the physical possession of the
premises, that is, to the possession de facto and not to the possession de jure. It does not even matter if a party's
title to the property is questionable." Thus, "an ejectment case will not necessarily be decided in favor of one who
has presented proof of ownership of the subject property."
Indeed, possession in ejectment cases "means nothing more than actual physical possession, not legal possession in
the sense contemplated in civil law." In a forcible entry case, "prior physical possession is the primary
consideration." "A party who can prove prior possession can recover such possession even against the owner
himself.

Whatever may be the character of his possession, if he has in his favor prior possession in time, he has the security
that entitles him to remain on the property until a person with a better right lawfully ejects him." "The party in
peaceable, quiet possession shall not be thrown out by a strong hand, violence, or terror."
The respondents have proven prior physical possession of the property.

12. Conditions Not Affecting Usufruct – NCC 609, 610, 607, 608

Albar v. Carandang
106 Phil. 855 (1959)
Since only the building was destroyed and the usufruct is constituted not only on the building but on the land as well,
then the usufruct is not deemed extinguished by the destruction of the building for under the law usufruct is
extinguished only by the total loss of the thing subject of the encumbrance.

Since the usufruct has not been extinguished by the destruction of the building and the usufruct is for life as in this
case, it is but fair that the usufructuary continue to enjoy the use of the land and the materials that they may be
constructed on the land.

FACTS:

Doña Rosario Fabie y Grey was the owner of the lot in the City of Manila with a building and improvements thereon
erected at Ongpin. The former left a will upon her death which was duly probated and she devised the naked
ownership of the whole property to Rosario Grey Vda. de Albar, et al. but its usufruct to Josefa Fabie for life. Said
property was then registered in the name of Rosario Grey Vda. De Albar, et al. as naked owners of the right of
Josefa Fabie as life usufructuary was expressly noted on the new title. Pursuant to the 9th clause of the will, an
encumbrance was likewise noted on the title prohibiting the usufructuary from selling, mortgaging or transferring
her right to usufruct during her minority. During liberation, as a consequence of the fire that gutted the building in
many portions of Manila, the building on the Ongpin lot was burned, leaving only the walls and other
improvements that were not destroyed by the fire.

One Au Pit, a Chinaman, offered to lease the property for a period of five years a monthly rental of P500.00, at the
same time agreeing to construct on the lot a new building worth P30,000 provided the naked owners as well as the
usufructuary sign the agreement of the lease. As the usufructuary maintains that she has the exclusive right to cede
the property by lease and to receive the full rental value by virtue of her right to usufruct, on the other hand, the
naked owners maintain that the right of usufruct was extinguished when the building was destroyed, the right of
the usufructory being limited to the legal interest on the value of the lot and the materials. In order that the
agreement of lease may be affected, the parties agreed on a temporary compromise whereby the naked owners
would receive P100.00, or 20% of the monthly rental of P500.00 and the usufructuary the balance of 80% or
P400.00 of said monthly rental.

By reason of the destruction of the building on the Ongpin property, the United States War Damage Commission
approved the claim that was presented for the damage caused to the property the amount in the amount of
P8,574.00 which was paid to and received by the naked owners. In the meantime, the usufructuary paid the real
estate taxes due on the property at Ongpin for the years 1945 to 1952 in the total amount of P1,989.27, as well as
the real estate taxes for the years 1953 and 1954 in the annual sum of P295.80.

Petitioner contended that the provision in the will should be interpreted as constituting only a life usufruct on the
rentals of the buildings erected on the lands and that once these buildings are destroyed the usufruct is
extinguished. Respondent, on the other hand, contended that the provision should be interpreted as constituting a
life usufruct both on the buildings and the lands because the former cannot be separated from the latter. The CA
affirmed decision appealed from in so far it holds that appellee's right of life usufruct subsists and is in full force
and effect upon the Ongpin lot. Hence, this petition.

ISSUE:

Whether or not the usufruct is deemed extinguished by the destruction of the building.

RULING:

No. It is clear that when the deceased constituted the life usufruct on the rentals "fincas situadas" in Ongpin and
Sto. Cristo streets, she meant to impose the encumbrance both the building and the land on which it is erected, for
indeed the building cannot exist without the land. Since only the building was destroyed and the usufruct is
constituted not only on the building but on the land as well, then the usufruct is not deemed extinguished by the
destruction of the building for under the law usufruct is extinguished only by the total loss of the thing subject of
the encumbrance (Article 603, old Civil Code).

In our opinion, this case comes under Article 517 of the same Code which provides: "if the usufruct is constituted
on immovable property of which a building forms part, and the latter should be destroyed in any manner
whatsoever, the usufructuary shall have a right to make use of the land and materials." This is a temporary
measure calculated to maintain the usufruct alive until the very thing that has been destroyed be reconstructed or
replaced. The reason is obvious: since the usufruct has not been extinguished by the destruction of the building and
the usufruct is for life as in this case, it is but fair that the usufructuary continue to enjoy the use of the land and the
materials that they may be constructed on the land. To hold otherwise would be to affirm that the usufruct has
been extinguished.

We therefore hold that the Court of Appeals did not err in finding that appellee's right of usufruct subsists and is in
full force and effect upon the Ongpin lot and the building existing thereon, affirming the decision of the trial court.

G. Easements
1. Characteristics, NCC 613, 617, 618

Privatization v. Legaspi Towers


G.R. No. 174957, July 22, 2009

When the owner of two properties alienates one an apparent sign of easement exists between the two estates,
entitlement to it continues, unless there is a contrary agreement, or the indication that the easement exists is removed
before the execution of the deed.

FACTS:

Caruff obtained a loan from PNB to finance the construction of a condominium (Legaspi Towers), secured by a REM
over the land where such would be erected. When the building and its appurtenances was constructed, Caruff also
built a powerhouse (generating set) and two sump pumps in the adjacent lot. Caruff failed to pay its loan with PNB,
and the latter foreclosed the mortgage. Thereafter, the Asset Privatization Trust (APT) was created. The
Government became the assignee and transferee of all the rights, titles and interests of PNB regarding Caruff’s
properties, including the one foreclosed. Caruff sought the nullification of PNB’s foreclosure of its properties. A
Compromise Agreement was later entered into by Caruff, PNB, and the Government thru APT where Caruff would
transfer and convey in favor of the Government, the lot where the generating set and sump pumps were built. The
respondent then filed a case for Declaration of the existence of an easement. It alleged that the act of Caruff of
constructing the powerhouse and sump pumps on its property constituted a voluntary easement in its favor. APT
alleged that it acquired absolute ownership thereof by virtue of the Compromise Agreement free from any liens
and/or encumbrances. It was not a privy to any transaction or agreement entered into by and between Caruff,
respondent, and the bank.

ISSUE:

Whether or not there was any easement constituted.

RULING:

There was none. An easement or servitude is a real right constituted on another’s property, corporeal and
immovable, by virtue of which the owner of the same has to abstain from doing or to allow somebody else to do
something on his property for the benefit of another thing or person. This does not apply because both properties
were owned by Caruff. There are two sources of easements: by law or by the will of the owners. Art 619 of the
states: “Easements are established either by law or by the will of the owners. The former are called legal and the
latter voluntary easements.” In the present case, neither type of easement was constituted over the subject
property. Also, Art 624 is controlling, as it contemplates a situation where there exists an apparent sign of
easement between two estates established or maintained by the owner of both. The law provides: “The existence of
an apparent sign of easement between two estates, established or maintained by the owner of both, shall be
considered, should either of them be alienated, as a title in order that the easement may continue actively and
passively, unless, at the time the ownership of the two estates is divided, the contrary should be provided in the
title of conveyance of either of them, or the sign aforesaid should be removed before the execution of the deed…”. It
can be inferred that when the owner of two properties alienates one of them and an apparent sign of easement
exists between the two estates, entitlement to it continues, unless there is a contrary agreement, or the indication
that the easement exists is removed before the execution of the deed. Thus, when the subject property was
assigned to the Government, no easement arose or was voluntarily created from the transfer, since Caruff pledged
that it was assigning, transferring, and conveying the subject property in favor of the Government thru the APT
free from any and all liens and encumbrances.

Velasco v. Cui
105 SCRA 616

Easement of public highway; Public highway deemed a legal encumbrance to the lot originally registered despite the
absence of annotation on the title, as the highway was already subsisting when the title was issued.

FACTS:

Petitioner filed in the Court of First Instance of Davao an action against Davao City to quiet title to her lot known as
Lot 77-B-2, a portion of which she claims to having been occupied illegally as part of Bolton Street, Davao City. The
allegations in the complaint that the Bolton Street encroached on the lot of the plaintiff and that the defendant had
continuously occupied the portion so encroached upon do not, contrary to the conclusion of the plaintiff found in
the complaint, cast a cloud of doubt on the title of the plaintiff over said portion which would justify this action.

ISSUE:

Whether or not BOLTON STREET AS AN EASEMENT MUST REMAIN A BURDEN ON LOT 77-B-2 (LOT IN
QUESTION) PURSUANT TO SECTION 39 OF ACT 496 ON THE GROUND THAT IT IS SUBJECT TO EASEMENT OF
PUBLIC HIGHWAY.

RULING:

Yes. Bolton Street which is a public highway, already subsisting when O.C.T. No. 638 was issued, as this fact is
apparent too from the face of the complaint itself, is deemed to have attached as a legal encumbrance to the lot
originally registered Lot No. 77, notwithstanding the lack of an annotation thereof in O.C.T. No. 638. Petitioner,
therefore, cannot rely, as she almost entirely does for the relief she seeks, on the aforequoted provision, which she
had repeatedly cited but without making mention, perhaps conveniently, of the exception as expressly provided in
the later part of the legal provision invoked

Valisno v. Adriano
161 SCRA 398 91988)

Water rights, such as the right to use a drainage ditch for irrigation purposes, which are appurtenant to a parcel of
land, pass with the conveyance of the land, although not specifically mentioned in the conveyance. The purchaser's
easement of necessity in a water ditch running across the grantor's land cannot be defeated even if the water is
supplied by a third person. The fact that an easement by grant may also have qualified as an easement of necessity
does detract from its permanency as property right, which survives the determination of the necessity.

FACTS:

Valisno filed against the Adriano an action for damages. The complaint alleged that Valisno is the absolute owner
and actual possessor of a parcel of land in La Fuente, Nueva Ecija. That Valisno bought the land from Adriano’s
sister, Honorata. The land adjoins that of Adriano on the bank of the Pampanga River. At the time of the sale of the
land to Valisno, the land was irrigated by water from the Pampanga River through a canal about 70 meters long,
traversing Adriano’s land. Adriano leveled a portion of the irrigation canal so that Valisno be deprived of the
irrigation water and prevented from cultivating his 57-hectare land. Valisno filed in the Bureau of Public Works a
complaint for deprivation of water rights. A decision was rendered ordering Adriano to reconstruct the irrigation
canal, "otherwise judicial action shall be taken against him under the provisions of Section 47 of Act 2152 (the
Irrigation Act), as amended. The Secretary of Public Works and Communications reversed the Bureau's decision by
issuing a final resolution dismissing Valisno's complaint. Trial court held that controversies between persons
claiming a right to water from a stream are within the jurisdiction of the Secretary of Public Works and his decision
on the matter is final, unless an appeal is taken to the proper court within 30 days. Valisno argues that while the
trial court correctly held that the Secretary of Public Works may legally decide who between the parties is entitled
to apply for water rights under the Irrigation Act, it erred in ruling that the Secretary has authority to hear and
decide the plaintiff 's claim for damages for the defendant's violation of his right to continue to enjoy the easement
of aqueduct or water through the defendant's land under Articles 642, 643, and 646 of the Civil Code

ISSUE:

Whether the provisions of the Irrigation Act (Act No. 2152) or those of the Civil Code should apply to the case.

RULING:

The principal issue involved in this case falls under the subject of servitude of waters which are governed by
Article 648 of the new Civil Code and the suppletory laws which are the irrigation law and the Spanish Law of
Waters specifically Article 122 thereof.

Article 122. Whenever a tract of irrigated land which previously received its waters from a single point is
divided through inheritance, sale or by virtue of some other title, between two or more owners, the owners of
the higher estates are under obligation to give free passage to the water as an easement of conduit for the
irrigation of the lower estates, and without right to any compensation therefore unless otherwise stipulated in
the deed of conveyance. (Art. 122, Spanish Law of Waters of August 3, 1866.)

The deed of sale in favor of Valisno included the "conveyance and transfer of the water rights and improvements"
appurtenant to Honorata Adriano's property. By the terms of the Deed of Absolute Sale, the vendor Honorata
Adriano Francisco sold, ceded, conveyed and transferred to Valisno all "rights, title, interest and participations over
the parcel of land.

2. Kinds of easements, NCC 613, 615, 616, 619

La Vista v. CA
278 SCRA 498 (1997)

FACTS:

Dispute about whether there is a right of way over 15-m wide roadway aka Mangyan Road, located on the
boundary line of La Vista and Ateneo property.

Land was originally owned by Tuasons. 1⁄2 was sold to Ateneo (Deed of Sale w/ Assumption of Mortgage) while
the other half was retained by Tuasons and developed into a subdivision (La Vista). Part of their agreement was for
each to develop their own share in the Mangyad Road (7.5m each). La Vista developed it, but Ateneo deferred
improvement on its share and erected an adobe wall instead. Ateneo informed La Vista about its intention to
develop some 16ha of its property along Mangyan into a subdivision.

In a letter, La Vista clarified that the “the easement of a right-of-way on (Ateneo’s) 7.5m portion was created in (La
Vista’s) favor, and likewise, a 7.5m right of way was created in Ateneo’s favor. Subsequently, La Vista offered to buy
Ateneo’s property which it was intending to develop, w/ the condition that the right-of-way over Mangyan will be
extinguished. Ateneo did not accept, instead sold it to the public. It was Solid Homes which was the highest bidder.
Part of what was transferered was the privilege of the right-of-way.
Solid Homes developed a subdivision now known as Loyola Grand Villas, and now claim that they have an
easement of right-of-way along Mangyan Road which they could have accessed to Katipunan. Issue arose when La
Vista prevented them from using the road by stationing guards and building concrete posts along the road. La Vista
argues that it couldn’t recognize the right-of-way because Ateneo never complied with their obligation of providing
them w/ a 7.5m right of way on their portion of the road, and because it was intended exclusively for Ateneo. Solid
Homes filed a case for recognition of right of way.

ISSUE:

Whether there exists a right of way over Mangyan Road in favor of Solid Homes?

RULING:

Yes. From the facts of the instant case it is very apparent that the parties and their respective predecessors‐in-
interest intended to establish an easement of right-of way over Mangyan Road for their mutual benefit, both as
dominant and servient estates.

It is evident from the stipulations in the Deed of Sale and the clarificatory letter sent by La Vista. Like any other
contract, the same could be extinguished only by mutual agreement or by renunciation of the owner of the
dominant estate. The free ingress and egress along Mangyan Road created by the voluntary agreement between
Ateneo and Solid Homes, Inc., is thus legally demandable (Articles 619 and 625, New Civil Code) with the
corresponding duty on the servient estate not to obstruct the same so much so that when the owner of the servient
tenement performs acts or constructs works impairing the use of the servitude, the owner of the dominant
tenement may ask for the destruction of such works and the restoration of the things to their condition before the
impairment was committed, with indemnity for damages suffered. That it was established exclusively for Ateneo
holds no water because the parties in the contract made provisions for its observance by all who in the future
might succeed them in dominion.

Liwag v. Happy Glen


G.R. No. 189755, July 4, 2012

FACTS:

In 1978, F. G. R. Sales, the original developer of Happy Glen Loop, loaned from Ernesto Marcelo, owner of T. P.
Marcelo Realty Corporation. The former failed to settle its debts with the latter, so, he assigned all his rights to
Marcelo over several parcels of land in the Subdivision including the receivables from the lots already sold. As the
successor-in-interest, Marcelo represented to lot buyers, the National Housing Authority (NHA) and the Human
Settlement Regulatory Commission (HSRC) that a water facility is available in the subdivision. The said water
facility has been the only source of water of the residents for thirty (30) years.

In September 1995, Marcelo sold Lot 11, Block 5 to Hermogenes Liwag. In 2003, Hermogenes died. Petitioner, wife
of Hermogenes, subsequently wrote to the respondent Association demanding the removal of the overhead water
tank over the parcel of land. The latter refused and filed a case before the HLURB against T. P. Marcelo Realty
Corporation, petitioner and the surviving heirs of Hermogenes. The HLURB ruling was in favor of the respondent
Association. One of the things it affirmed was the existence of an easement for water system/facility or open space
on Lot 11, Block 5 of TCT No. C-350099 wherein the deep well and overhead tank are situated.

ISSUE:

WON An easement for water facility exists on Lot 11, Block 5 of Happy Glen Loop Subdivision

RULING:
Yes, Easements or servitudes are encumbrances imposed upon an immovable for the benefit of another immovable
belonging to a different owner, for the benefit of a community, or for the benefit of one or more persons to whom
the encumbered estate does not belong. It may be discontinuous or continuous and apparent or non-apparent.

In this case, the water facility is an encumbrance on Lot 11, Block 5 of the Subdivision for the benefit of the
community. It is continuous and apparent, because it is used incessantly without human intervention, and because
it is continually kept in view by the overhead water tank, which reveals its use to the public.

Contrary to petitioner’s contention that the existence of the water tank is merely tolerated, we find that the
easement of water facility has been voluntarily established either by Marcelo, the Subdivision owner and
developer; or by F.G.R. Sales, his predecessor-in-interest and the original developer of the Subdivision. For more
than 30 years, the facility was continuously used as the residents’ sole source of water. The Civil Code provides that
continuous and apparent easements are acquired either by virtue of a title or by prescription of 10 years. It is
therefore clear that an easement of water facility has already been acquired through prescription.

North Greenhills Assoc., Inc. v. Atty. Morales


G.R. No. 222821, August 09, 2017

A property owner has the right to enclose or fence his land or tenements by means of walls, ditches, live or dead
hedges, or by any other means without detriment to servitudes constituted thereon. He also has a right to exclude
others from access to, and enjoyment of his property.

FACTS:

Atty. Morales is a resident of North Greenhills Subdivision. His house is adjacent to McKinley Park, an open
space/playground area owned and operated by NGA. He also has a personal access door, which opens directly into
the park.

NGA started constructing a pavilion or kiosk occupying the side of the park adjacent to the residence of Atty.
Morales. Part of the design was a public restroom. Said restroom was constructed alongside the concrete wall
separating the house of Atty. Morales from the park. Objecting to the construction of the restroom, Atty. Morales
filed a complaint before the HLURB against NGA on the ground among others that the construction of the pavilion
violated his right to immediate access to the park.

NGA, on the other hand, contended that as the absolute owner of the park, it had the absolute right to fence the
property and impose reasonable conditions for the use thereof by both its members and third parties; and that
Atty. Morales' use of a side entrance to the park for 33 years could not have ripened into any right because
easement of right of way could not be acquired by prescription.

ISSUE:

Whether NGA has the right to block Atty. Morales' access to the park.

RULING:

YES. Under the Civil Code, NGA, as owner of the park, has the right to enclose or fence his land or tenements by
means of walls, ditches, live or dead hedges, or by any other means without detriment to servitudes constituted
thereon. It also has a right to exclude others from access to, and enjoyment of its property.

Atty. Morales never introduced any evidence that he had acquired any right by prescription or by agreement or
legal easement to access the park through his side door. Moreover, he never claimed that his side door was his only
access to the park. He has other means and, being adjacent to the park, going through other means is not
cumbersome.

3. Modes of acquiring easements, NCC 623, 624


Sps. Andres v. Sta. Lucia
G.R. No. 201405, August 24, 2015

Not all may demand for an easement of right-of-way. Under the law, an easement of right-of-way may only be
demanded by the owner of an immovable property or by any person who by virtue of a real right may cultivate or use
the same.

FACTS:

Petitioners and Liza Andres filed a Complaint for Easement of Right-of-Way against respondent before the RTC.
They alleged that they are co-owners and possessors for more than 50 years of three parcels of unregistered
agricultural land. Respondent acquired the lands surrounding the subject property, built a concrete perimeter
fence around it such that petitioners and Liza were denied access from subject property to the nearest public road.
Petitioners presented the application for tax declaration of the unregistered land which was denied by Provincial
Assessor on the ground that the subject property was already declared for taxation purposes under the name of
the Blancos. Sps. Andres testified that they are surviving heirs of the late Carlos who acquired ownership over the
same after he had been in continuous, public and peaceful possession thereof for 50 years. RTC rendered judgment
in favor of petitioners; CA held that the evidence adduced by petitioners and Liza failed to sufficiently establish
their asserted ownership and possession of the subject property, hence, no right to demand right-of-way from
respondent.

ISSUE:

Whether or not petitioners are entitled to demand an easement of right-of-way.

RULING:

NO. The records show that the subject property is an unregistered public agricultural land. Thus, being a land of the
public domain, petitioners, in order to validly claim acquisition through prescription, must first be able to show
that the State has expressly declared through either a law enacted by Congress or a proclamation issued by the
President that the subject property is no longer retained for public service or the development of the national
wealth or that the property has been converted into patrimonial. Consequently, without an express declaration by
the State, the land remains to be a property of public dominion and hence, not susceptible to acquisition by virtue
of prescription. In the absence of such proof of declaration in this case, petitioners' claim of ownership over the
subject property based on prescription necessarily crumbles. Conversely, they cannot demand an easement of
right-of-way from respondent for lack of personality.

4. Effects of Easement, NCC 625, 641, 626, 627, 628

Valderrama v. North Negros


G.R. No. L-23810, December 18, 1925

What is prohibited by Art. 543 is that in extending the line or repairing or using the same, a larger area of land is
occupied or excavations or materials deposited are outside the area occupied not by causing wagons to pass just
because of a change of ownership of the objects being transported.

FACTS:

Several hacienda owners in Manapla, Occidental Negros, entered into a milling contract with Miguel Osorio
wherein the latter would build a sugar central of a minimum capacity of 300 tons for the milling and grinding of all
the sugar cane to be grown by the hacienda owners who in turn would furnish the central with all the cane they
might produce in their estates for 30 years from the execution of the contract. Later on, Osorio’s rights and
interests were acquired by the North Negros Sugar Co., Inc.
Two years after, the current petitioners, Catalino Valderrama, Emilio Rodriguez, Santos Urra et. al, made other
milling contracts identical to the first one with the North Negros Sugar, Co., Inc. The hacienda owners, however,
could not furnish the central sufficient cane for milling as required by its capacity, so the North Negros made other
milling contracts with the various hacienda owners of Cadiz, Occidental Negros. This prompted Valderrama et. al to
each file a complaint against North Negros.

The CFI entered a single judgment for all of them, ruling in Valderrama et. al’s favor finding that North Negros had
no right to pass through the lands of the hacienda owners for the transportation of sugar cane not grown from
their lands. Thus the appeal to the SC.

ISSUE:

Whether or not the easement of way established was restricted to transporting only sugar cane from the hacienda
owners’ lands.

RULING: NO.

The contract entered into by each of the hacienda owners contained a clause that granted the North Negros an
easement of way 7 meters wide for the period of 50 years upon their properties for the construction of a railroad.
The owners allege ambiguity since it could permit the transportation of sugar cane which they did not produce
which is contrary to their intent but the SC held that it is clear that the easement was established for the benefit of
all producers and of the corporation as it is the intent of the milling contract.

Since the easement is a voluntary, apparent, continuous easement of way in favor of the corporation, it is contrary
to the nature of the contract that it is only limited to canes produced by the servient estates since it is a well settled
rule that things serve their owner by reason of ownership and not by reason of easement. The owners also cannot
limit its use for there is nothing in the contract prohibiting the central from obtaining other sources.

Transporting cane from Cadiz also does not make it more burdensome since what is prohibited in Art. 543 of the
CC is that in extending the road or in repairing it, it should occupy a greater area or deposit excavations outside the
granted 7 meters. This does not happen in this case when the North Negros transports sugar cane from Cadiz,
crossing the servient estates, since it continues to occupy the same area and the encumbrance is still the same
regardless of the number of times it passes through the estates.

Also the period of the easement is longer than the period of the milling contracts, so even if the owners no longer
desire to furnish the central canes for milling, the North Negros still has the right to the easement for the remaining
period so the contention that it should be limited to the canes produced by the owners has no basis.

Goldcrest Realty Corp. v. Cypress Gardens Condominium Corp.


G.R. No. 171072, April 7, 2009

The owner of the dominant estate cannot violate any of the following prescribed restrictions on its rights on the servient
estate, to wit: (1) it can only exercise rights necessary for the use of the easement; (2) it cannot use the easement except
for the benefit of the immovable originally contemplated; (3) it cannot exercise the easement in any other manner than
that previously established; (4) it cannot construct anything on it which is not necessary for the use and preservation of
the easement; (5) it cannot alter or make the easement more burdensome; (6) it must notify the servient estate owner of
its intention to make necessary works on the servient estate; and (7) it should choose the most convenient time and
manner to build said works so as to cause the least convenience to the owner of the servient estate. Any violation of the
above constitutes impairment of the easement.

FACTS:

Petitioner is the developer of Cypress Gardens, located at Makati City. Goldcrest executed a Master Deed and
Declaration of Restrictions which constituted Cypress Gardens into a condominium project and incorporated
respondent Cypress Gardens Condominium Corporation (Cypress) to manage the condominium project and to hold
title to all the common areas. Title to the land on which the condominium stands was transferred to Cypress under
a Transfer Certificate of Title. But Goldcrest retained ownership of the two-level penthouse unit on the ninth and
tenth floors of the condominium registered under Condominium Certificate of Title at the Register of Deeds
of Makati City.

Following the turnover of the administration and management of the Condominium to the board of directors
of Cypress in 1995, it was discovered that certain common areas pertaining to Cypress were being occupied and
encroached upon by Goldcrest. Thus, Cypress filed a complaint with damages against Goldcrest before the HLURB,
seeking to compel the latter to vacate the common areas it allegedly encroached on and to remove the structures it
built thereon. Cypress sought to remove the door erected by Goldcrest along the stairway between the 8th and
9th floors, as well as the door built in front of the 9th floor elevator lobby, and the removal of the cyclone wire fence on
the roof deck.

Goldcrest averred that it was granted the exclusive use of the roof decks limited common area by Section 4(c) of
the condominiums Master Deed. It likewise argued that it constructed the contested doors for privacy and security
purposes, and that, nonetheless, the common areas occupied by it are unusable and inaccessible to other
condominium unit owners.

HLURB Arbiter Decision: instructed Goldcrest, to: (1) remove the questioned structures, including all other
structures which inhibit the free ingress to and egress from the condominiums limited and unlimited common
areas; (2) vacate the roof decks common areas and to pay actual damages for occupying the same; and (3) pay an
administrative fine for constructing a second penthouse and for making an unauthorized alteration of the
condominium plan.

HLURB Special Division Decision: modified the decision of the Arbiter. Found that the encroached areas were not
actually measured and that there was no evidentiary basis for the rate of compensation. It likewise held
that Cypress has no cause of action regarding the use of the roof decks limited common area because only
Goldcrest has the right to use the same.

Office of the President Decision: dismissed the appeal. Contrary to the submissions of Cypress, the assailed decision
did not favor the building of structures on either the condominiums limited or unlimited common areas. It stressed
that the decision did not only order Goldcrest to remove the structures impeding the use of the unlimited common
areas, but also fined it for making unauthorized alteration and construction of structures on the condominiums
roof deck.

Court of Appeals Decision: Partly granted its appeal. The appellate court noted that the right of Goldcrest under
Section 4(c) of the Master Deed for the exclusive use of the easement covering the portion of the roof deck
appurtenant to the penthouse did not include the unrestricted right to build structures thereon or to lease such
area to third persons. It ordered the removal of the permanent structures constructed on the limited common area
of the roof deck.

ISSUE

WON PETITIONER IMPAIRED THE EASEMENT ON THE PORTION OF THE ROOF DECK DESIGNATED AS A LIMITED
COMMON AREA.

RULING:

We find no cogent reason to overturn the similar finding of the HLURB, the Office of the President and the Court of
Appeals that Goldcrest has no right to erect an office structure on the limited common area despite its exclusive right
to use the same. We note that not only did Goldcrests act impair the easement, it also illegally altered the
condominium plan, in violation of Section 22 of Presidential Decree No. 957.
Goldcrests acts shows that it breached a number of the aforementioned restrictions. First, it is obvious that the
construction and the lease of the office structure were neither necessary for the use or preservation of the roof
decks limited area. Second, the weight of the office structure increased the strain on the condominiums foundation
and on the roof decks common limited area, making the easement more burdensome and adding unnecessary
safety risk to all the condominium unit owners. Lastly, the construction of the said office structure clearly went
beyond the intendment of the easement since it illegally altered the approved condominium project plan and
violated Section 4 of the condominiums Declaration of Restrictions.

NPC v. Heirs of Makabingit


G.R. No. 165828, August 24, 2011

FACTS:

National Power Corporation (NPC) undertook the Agus River Hydroelectric Power Plant Project to generate
electricity for Mindanao. It included the construction of several underground tunnels to be used in diverting the
water flow from the Agus River to the hydroelectric plants.

On 1997, Respondents sued NPC for recovery of damages of the property and a prayer for just compensation. They
alleged that the tunnel deprived them of the agricultural, commercial, industrial and residential value of their land;
and that their land had also become an unsafe place for habitation, forcing them and their workers to relocate to
safer grounds.

ISSUE:

Whether the Heirs of Sangkay have the right to just compensation.

RULING:

Just compensation is the full and fair equivalent of the property taken from its owner by the expropriator. It has the
objective to recover the value of property taken in fact by the governmental defendant, even though no formal
exercise of the power of eminent domain has been attempted by the taking agency.

The underground tunnels impose limitations on respondents’ use of the property for an indefinite period and
deprive them of its ordinary use. Hence, respondents are clearly entitled to the payment of just compensation.

Notwithstanding the fact that petitioner only occupies the sub-terrain portion, it is liable to pay not merely an
easement fee but rather the full compensation for land. It is settled that the taking of private property for public
use, to be compensable, need not be an actual physical taking or appropriation. This is so because in this case, the
nature of the easement practically deprives the owners of its normal beneficial use. Compensable taking includes
destruction, restriction, diminution, or interruption of the rights of ownership or of the common and necessary use
and enjoyment of the property in a lawful manner, lessening or destroying its value.

Cabahug v. NAPOCOR
G.R. No. 186069; January 30, 2013

FACTS:

Spouses Cabahug, being owners of two parcels of land which were subjected to expropriation proceedings by the
National Power Corporation (NPC). NPC electrical cables would be installed in the portions of the province and
would traverse the land owned by the petitioners. Cabahug, in consideration of the easement fees, granted NPC a
continuous easement right of way. Two years thereafter, Cabahug filed a complaint before RTC for payment of just
compensation after having learned that the compensation given by NPC was very low compared to the appraisal
made by the province of Leyte. RTC rendered decision in favor of Cabahug. However, at the Court of Appeals, it was
ruled that vested right has already accrued in favour of NPC, and to allow spouses Cabahug to pursue the case
would be a violation of the contract and an unjust enrichment in favour of Cabahug.

ISSUE:
Whether or not NPC may still be held liable to pay for the full market value of the affected property despite the fact
transfer of title thereto was not required by the easement.

HELD:

Yes. The power of Eminent Domain may be exercised although title is not transferred to the expropriator in
easement of right of way. Just compensation which should be neither more nor less than the money equivalent of
the property is, moreover, due where the nature and effect of the easement is to impose limitations against the use
of the land for an indefinite period and deprive the landowner if ordinary use.

5. Legal Easements - Water Code, Arts. 31-52, NCC 640-641, 642-646, 647, 649-657, 658-666, 667-673

Valdez v. Tabisula
G.R. No. 175510, July 28, 2008

An easement or servitude is a real right constituted on anothers property, corporeal and immovable, by virtue of
which the owner of the same has to abstain from doing or to allow somebody else to do something on his property for
the benefit of another thing or person.

FACTS:

Petitioner-spouses Valdez purchased via a Deed of Absolute Sale from respondent-spouses Tabisula a 200 sqm of a
380 sqm parcel of land.

Respondents subsequently built a concrete wall on the western side of the subject property. Believing that that
side is the intended road right of way mentioned in the deed, petitioners, reported the matter to the barangay.
Petitioners filed, more than six years after the execution of the deed, a Complaint for Specific Performance with
Damages against respondents before the RTC of San Fernando City.

Petitioners alleged that they purchased the subject property on the strength of respondent’s assurance of
providing them a road right of way. They thus prayed that respondents be ordered to provide the subject property
with an easement and to remove the concrete wall blocking the same.

Respondents, in their Answer with Compulsory Counterclaim averred that the easement should be taken from the
western portion of the subject property and not from theirs Respondents further averred that they did not agree to
providing petitioners an easement on the western side of their lot.

The RTC dismissed petitioner’s complaint and granted respondents Counterclaim. On appeal by petitioners, the
CA affirmed that of the trial court.

ISSUE:

WON the Petitioners are entitled to an easement.

RULING:

No. Petitioners are neither entitled to a legal or compulsory easement of right of way. For to be entitled to such
kind of easement, the preconditions under Articles 649 and 650 of the Civil Code must be established.

To be conferred a legal easement of right of way under Article 649, the following requisites must be complied
with: (1) the property is surrounded by other immovables and has no adequate outlet to a public highway; (2)
proper indemnity must be paid; (3) the isolation is not the result of the owner of the dominant estates own
acts; (4) the right of way claimed is at the point least prejudicial to the servient estate; and (5) to the extent
consistent with the foregoing rule, the distance from the dominant estate to a public highway may be the
shortest. The onus of proving the existence of these prerequisites lies on the owner of the dominant estate, herein
petitioners.

As found, however, by the trial court, petitioners and their family are also the owners of two
properties adjoining the subject property which have access to two public roads. Since petitioners then have more
than adequate passage to two public roads, they have no right to demand the grant by respondents of an easement
on the western side of respondents lot.

St. Michael School of Cavite v. Masaito Dev’t. Corp.


G.R. No. 166301, Feb. 29, 2008

For a complaint to state a cause of action in an easement case, more specifically, Art. 649 of the Civil Code has laid
down the following requirements: (1) the dominant estate is surrounded by other immovables and has no adequate
outlet to a public highway; (2) there is payment of proper indemnity; and (3) the isolation is not due to the acts of the
proprietor of the dominant estate.

FACTS:

Petitioner St. Michael is a non-stock corporation owned by spouses Claveria. Respondents Masaito Development
and Rexlon Realty are domestic corporations that own, operate and manage Citihomes. St. Michael is located
outside the northern perimeter fence of Citihomes. Its passageway occupies a portion of the 61-square meter lot of
Citihomes. The gate to the school is located at the subdivision’s northern perimeter fence and is the only entrance
and exit for the entire school population.

Rexlon informed petitioners that the value of the Citihomes lots when fully developed was P3,872 per sq. m. as
appraised. Masaito advised petitioners to purchase the lot fronting the school at P3,579,000. Masaito sent another
offer to sell it with the right of way through the private roads/drainage facilities of Citihomes at P2,000,000.
Petitioners refused both offers because it did not need the entire area mentioned and that it was overpriced.

Petitioners with 4 other homeowners filed a complaint against respondents for easement of right of way with
damages under Article 649 of the Civil Code and preliminary injunction and/or TRO. RTC issued a TRO for 72
hours which was extended, enjoining respondents from blocking the passageway and school gate. Respondents file
a Motion to Dismiss.

RTC dismissed the action as to the 4 homeowners and denied petitioners’ application for issuance of writ of
preliminary injunction. RTC granted respondents’ partial motion for reconsideration and dismissed petitioners’
complaint. Petitioners’ omnibus motion was also denied. CA likewise dismissed petitioners’ petition for certiorari.

ISSUE:

WON there is merit in petitioners’ complaint for easement.

RULING:

Yes. Three elements must be present for a complaint to state a cause of action: (1) the legal right of the plaintiff, (2)
the correlative obligation of the defendant, and (3) the act or omission of the defendant violating said legal right.
For a complaint to state a cause of action in an easement case, more specifically, Art. 649 of the Civil Code has laid
down the following requirements: (1) the dominant estate is surrounded by other immovables and has no
adequate outlet to a public highway; (2) there is payment of proper indemnity; and (3) the isolation is not due to
the acts of the proprietor of the dominant estate. We rule that the Complaint satisfies these three elements and
thus sufficiently alleges a cause of action. The Complaint, first, asserts that petitioners have a right to an easement
of right-of-way that cuts across respondents’ property; second, it refers to respondents’ correlative obligation not
to fence off and close the single gate which is used as the only entry and exit points of the school population; and
third, it refers to respondents’ expansion and excessive terms and conditions, constituting the acts violating
petitioners’ right. We thus hold that the Complaint’s material allegations are enough to entitle petitioners to a
favorable judgment if these are assumed to be true.

The trial court erred when it ruled that the school, not being the registered owner of the subject lot, is not a real
party-in-interest. It will suffice under Art. 649 of the Civil Code that “any person who by virtue of a real right may
cultivate or use any immovable which is surrounded by other immovables pertaining to other persons and without
adequate outlet to a public highway, is entitled to demand a right of way.” Clearly, the school is a real party-in-
interest since it has established a right to use the passageway for the benefit of its students. More importantly, the
records reveal that petitioners-spouses are the owners of the lot where the school is located and they are the
incorporators, trustees, and officers of St. Michael. They are also authorized to represent the corporation in the
complaint and subsequent actions. Thus, petitioners are real parties-in-interest and we rule that the dismissal of
the complaint is patently erroneous and bereft of any legal basis. Petitioners must be allowed to pursue their case
before the trial court.

Quintanilla v. Abangan
544 SCRA 494

As between a right of way that would demolish a fence of strong materials to provide ingress and egress to a public
highway and another right of way which although longer will only require a van or vehicle to make a turn, the second
alternative should be preferred. Mere convenience for the dominant estate is not what is required by law as the basis
for setting up a compulsory easement.

FACTS:

Perfecta Quintanilla bought a subdivided lot (the dominant estate). She would later donate the said lot to her son,
Apolinardito. They used the said property for their business, and constructed a warehouse on it. They asked
Abangan for a right of way over his property (the servient estate), however, it would appear that Abangan had
already sold the property to Daryl’s Collection, Inc., which fenced off the said property.

The Quintanillas filed an action with the RTC for the imposition of an easement of right of way. Both RTC and CA
dismissed the case.

ISSUE:

Whether or not compliance with the preconditions set forth in Articles 649 and 650 of the New Civil Code is
superior to the “mere convenience rule against the owner of the dominant estate.”

RULING: NO.

We hold that Apolinardito as owner of the dominant estate together with Perfecta failed to discharge the burden of
proving the existence and concurrence of all the requisites in order to validly claim a compulsory right of way
against respondents. It should be remembered that to be entitled to a legal easement of right of way, the following
requisites must be satisfied: (1) the dominant estate is surrounded by other immovables and has no adequate
outlet to a public highway; (2) proper indemnity has been paid; (3) the isolation was not due to acts of the
proprietor of the dominant estate; and (4) the right of way claimed is at the point least prejudicial to the servient
estate. The fourth requisite is absent

As between a right of way that would demolish a fence of strong materials to provide ingress and egress to a public
highway and another right of way which although longer will only require a van or vehicle to make a turn, the
second alternative should be preferred. Mere convenience for the dominant estate is not what is required by law as
the basis for setting up a compulsory easement. Even in the face of necessity, if it can be satisfied without imposing
the easement, the same should not be imposed. Finally, worthy of note, is the undisputed fact that there is already a
newly opened public road barely fifty (50) meters away from the property of appellants, which only shows that
another requirement of the law, that is, there is no adequate outlet, has not been met to establish a compulsory
right of way.
Obra v. Sps Badua
G.R. No. 149125, Aug. 9, 2007

A right-of-way is an interest in the land, any agreement creating it should be drawn and executed with the same
formalities as a deed to a real estate, and ordinarily must be in writing.

FACTS:

Respondents Sps. Badua et al alleged that their residential houses, erected on a lot commonly owned by them
situated in La Union, were located west of the properties of the Obras, Bucasases, and Baduas. Their only access to
the national highway was a pathway traversing the northern portion of petitioners property and the southern
portion of the properties of the Bucasases and Baduas. The pathway was more than one meter wide and sixteen
meters long. They claimed that this pathway had been established as early as 1955. In 1995, however, petitioner
Obra constructed a fence on the northern boundary of their property; thus, blocking respondents access to the
national highway. Respondents demanded the demolition of the fence, but petitioner refused.

In her Answer, petitioner averred that respondents had not established any easement of right-of-way either by law
or agreement. She claimed that respondents failed to satisfy the requisites provided in Articles 649 and 650 of the
Civil Code in order to establish an easement of right-of-way on the northern portion of her property. Moreover, she
alleged that respondents had another access as ingress and egress to the public road other than the one traversing
her property.

The spouses Badua and Bucasas failed to file an answer; consequently, they were declared in default.

On July 7, 2000, after trial, the RTC rendered a Decision dismissing the complaint. It held that respondents were
not able to satisfy all the requisites needed for their claim of an easement of right of way. It observed that when
petitioner fenced the northern portion of her property, respondents were able to use another pathway as ingress
and egress to the highway. It stated further that the new pathway is more than adequate for respondents use. Thus,
the applied easement of right-of-way on the northern portion of petitioners property was not allowed. The said
Decision became final and executory.

Sometime in 2001, petitioner constructed a fence on this portion of her lot, which again restricted the use of
respondents new pathway.

Respondents filed on March 6, 2001 a Motion to Enforce the July 7, 2000 Decision of the RTC. They alleged that the
Decision of the RTC dismissing the case was based on the existence of a new pathway which they had been using
since 1995. Thus, they asserted that petitioner was prohibited from closing said passage.

On March 20, 2001, the RTC granted the said motion. Clarifying its July 7, 2000 Decision, the trial court, in its March
20, 2001 Order, held that the dismissal of the complaint depended on petitioners representation that she was
allowing respondents to use the southern portion of her property as an alternative pathway. Since the southern
portion was an agreed pathway, petitioner could not reduce its width; thus, the trial court ordered petitioner to
remove the fence blocking the passage.

ISSUE:

Whether or not the trial courts can issue an order clarifying its final and executory decision and effectively
establishing an easement on petitioners property without proper adjudication.

RULING: No.

It is a settled doctrine that a decision, after it becomes final, becomes immutable and unalterable. Thus, the court
loses jurisdiction to amend, modify, or alter a final judgment and is left only with the jurisdiction to execute and
enforce it. To recapitulate, the dismissal of Civil Case No. 5033 meant that no easement was ever established on
petitioners property. However, the trial court, by issuing its March 20, 2001 Order directing petitioner to remove
the fence that limited respondents passage, effectively created a right-of-way on petitioners property in favor of
respondents allegedly on the basis of a voluntary agreement between the parties. This directive was in
contravention of its July 7, 2000 Decision; thus, it was null and void for having been issued outside of the courts
jurisdiction.

Granting for the sake of argument that the issue of voluntary easement of right-of-way, subject of the assailed
March 20, 2001 Order, was proper, relevant, and material to the issue of right-of-way as averred in the complaint
in Civil Case No. 5033, still, the conclusion that there was an agreed or voluntary easement of right-of-way
had no basis. The records of Civil Case No. 5033 do not reveal any agreement executed by the parties on the
claimed right-of-way. Glaring is the fact that the terms of the arrangement were not agreed upon by the parties,
more particularly, the payment of the proper indemnity. The evidence is not ample enough to support the
conclusion that there was a verbal agreement on the right-of-way over the southern portion.

More so, since a right-of-way is an interest in the land, any agreement creating it should be drawn and executed
with the same formalities as a deed to a real estate, and ordinarily must be in writing. No written instrument on
this agreement was adduced by respondents.

Dichoso v. Marcos
G.R. No. 180282, April 11, 2011

Doctrine: In order to justify the imposition of an easement of right of way, there must be real, not fictitious or artificial,
necessity for it. Mere convenience for the dominant estate is not what is required by law as the basis of setting up a
compulsory easement. Even in the face of necessity, if it can be satisfied without imposing the easement, the same
should not be imposed.

FACTS:

Crispin Dichoso et al. filed a complaint for easement of right of way against Patrocinio Marcos, claiming that they
previously used a portion of the latter’s lot as ingress and egress but Marcos subsequently blocked the same with
piles of sand. In his answer, Marcos contended that the claim of Dichoso is not due to necessity but mere
expediency and that there is an existing easement of right of way granted to them by the Spouses Arce which
makes it unnecessary to establish another easement over his property.

ISSUE:

Whether Dichoso et al. are entitled to an easement of right of way in Marcos’ property.

RULING:

No, Dichoso et al. are not entitled to a right of way in Marcos’ property. It was undisputed that Dichoso et al. had
been granted a right of way in the property of Spouses Arce, the same is also the passageway used by others lot
owners. Petitioners’ contention that the passageway granted by the Spouses Arce is circuitous and longer is not
adequate to grant an easement of right of way in their favor over the property of Marcos. The convenience of the
dominant estate has never been the gauge for the grant of compulsory right of way. The true standard for the grant
of the legal right is adequacy. Hence, when there is already an existing adequate outlet from the dominant estate to
a public highway, as in this case, even when the said outlet, for one reason or another, be inconvenient, the need to
open up another servitude is entirely unjustified.

Regala v. Carin
G.R. No. 188715, April 6, 2011

FACTS:
Petitioner and respondent are adjacent neighbors. When petitioner decided to renovate his one storey residence
by constructing a second floor, he under the guise of merely building an extension to his residence, approached
respondent for permission to bore a hole through a perimeter wall shared by both their respective properties, to
which respondent verbally consented on condition that petitioner would clean the area affected by the work.

Petitioner’s real intention was to build a second floor, in fact with a terrace atop the dividing wall. In the course of
the construction of the second floor, respondent and his wife Marietta suffered from the dust and dirt which fell on
their property. As petitioner failed to address the problem to respondent’s satisfaction, respondent filed a letter-
complaint.

Petitioner having continued the construction work despite issuance of several stop-work notices from the City
Engineer’s Office for lack of building permit, respondent filed a complaint for damages against petitioner. In his
complaint, respondent alleged in the main that, instead of boring just one hole as agreed upon, petitioner
demolished the whole length of the wall from top to bottom into five parts for the purpose of constructing a second
floor with terrace. Respondent thus prayed for the award of moral and exemplary damages. Petitioner, denying
respondent’s allegations, claimed that he was the sole and exclusive owner of the wall referred to as a perimeter
wall, the same having been built within the confines of his property and being part and parcel of the house and lot
package he purchased from the developer. Engineer Haduca found an encroachment by petitioner of six
centimeters. Hence, RTC rendered judgment in favor of respondent.

ISSUE:

Whether or not the perimeter wall is owned by petitioner.

RULING:

Yes. The testimony of petitioner and his witnesses, specifically Architect Punzalan, demonstrates that they had
actually taken measures to prevent, or at the very least, minimize the damage to respondent’s property occasioned
by the construction work. Punzalan surveyed petitioner’s property based on the TCT and Tax Declarations and
found that the perimeter wall was within the confines of petitioner’s property.

It bears noting that petitioner was engaged in the lawful exercise of his property rights to introduce renovations to
his abode. While he initially did not have a building permit and may have misrepresented his real intent when he
initially sought respondent’s consent, the lack of the permit was inconsequential since it only rendered petitioner
liable to administrative sanctions or penalties.

Respondent failed to establish by clear and convincing evidence that the injuries he sustained were the proximate
effect of petitioner’s act or omission. It thus becomes necessary to instead look into the manner by which petitioner
carried out his renovations to determine whether this was directly responsible for any distress respondent may
have suffered since the law requires that a wrongful or illegal act or omission must have preceded the damages
sustained by the claimant. Respondent is not entitled to damages.

Fabie v. Lichauco
G.R. No. L-3598, July 24, 1908

FACTS:

Petitioner Miguel Fabie applied for the registration of his property in Manila free from any encumbrances except
the easement of right of way in favor of respondents Julita Lichauco and Hijos de Roxas. In addition to the said right
of way, respondents also claim that of light and view and drainage. However, the claim was later reduce only to
that of the light and view. Lichauco cliamed that when Juan Bautista Coloma, the original owner of both estates,
established not only an easement of right of way but also that of light and view and that when both the properties
were alienated, the apparent signs were not removed. The apparent sign allegedly consists of a gallery with
windows through which light is admitted. It was supported on columns erected on the ground belonging to the
petitioner and the balcony on Lichauco’s property is supported by uprights erected on the land by petitioner. The
parties admitted the existence of such gallery. The house was now a frontage of 18 meters and 60 centimeters, of
which 16 meters and 60 centimeters correspond to the main part of the same, and 1 meter and 90 centimeters to
the gallery in question. It results, therefore, that at the present day, the house has nearly 2 meters more frontage
than when it was alienated by Coloma. Therefore, at the present day the house is erected partly on the land
belonging to the owner and partly, the gallery, over a lot belonging to another; that is, over that of the petitioner.
When it was sold in October, 1848, no portion of the house occupied the lot last mentioned, but the entire building
was erected over a lot belonging to the owner as set forth in the instrument of sale. The lower court held that the
right of way and drainage exist in favor of the respondents’ respective properties. The claim as to the easement of
light and view was dismissed by the court.

ISSUE:

Whether or not Respondents are entitled to the easement of light and view.

HELD:

No. The burden is not on the petitioner to prove on what time the gallery in controversy was constructed inasmuch
as he limits himself to sustaining nad defending the freedom of his property, denying the easement o flight and
view of the respondent pretends to impose over it. A property is assumed to be from all encumbrance unless the
contrary is proved. Respondent who claims the said easement is obliged to prove the aforementioned gallery, in
which the apparent sign of the easement is made to consist in the present case, existed at the time of ownership of
her property and that of the petitioner were separated. And inasmuch as this issue has not been proved, the claim
of the respondents as to the easements of the light and view which the petitioner does not admit, must of necessity
be dismissed. Therefore, it does not appear from the agreement of the parties that the respondents has
balconies over the land of the petitioner; and as it is, since it has been positively shown that the said balconies
exceed the limit of the lot owned by the former, nor less that they invade the atmospheric area of the lot belonging
to the latter, it follows that, even in accordance with the theory maintained by the respondents with which on
account of its lack of basis, we consider it unnecessary to deal herein as to its other aspect, the easement of view,
which might result in such case from the existence of the balconies alluded to, would be negative and not a positive
one, because the erection of the same would not constitute, according to their own statement, an invasion of the
right of another, but the lawful exercise of the right inherent to the dominion of the respondents to construct
within their own lot. And as said easement is negative, it cannot have prescribed in favor of the property of the
respondents in the absence of any act of opposition, according to the agreement, by which they or their principals
would have prohibited the petitioner or his principals to do any work which obstruct the balconies in question,
inasmuch as said act of opposition is what constitutes the necessary and indispensable point of departure for
computing the time required by law for the prescription of negative easements. Thus, the judgment appealed from
was affirmed in toto by the Court.

Williams v. Zerda
G.R. No. 207146, March 15, 2017

An entitlement to the easement of right of way requires that the following requisites must be met.1. The dominant
estate is surrounded by other immovables and has no adequate outlet to a public highway (Art. 649, par. 1); 2. There is
payment of proper indemnity (Art. 649, par. 1); 3. The isolation is not due to the acts of the proprietor of the dominant
estate (Art. 649, last par.); and 4. The right of way claimed is at the point least prejudicial to the servient estate; and
insofar as consistent with this rule, where the distance from the dominant estate to a public highway may be the
shortest (Art. 650).

FACTS:

Zerda was the owner of a parcel of land (dominant estate) in Surigao City, Immediately behind the dominant
estate was a swampy mangrove area owned by the Republic of the Philippines. On both sides were lots registered
under the name of Woodridge Properties, Inc. and a lot, in the name of Luis G. Dilag. In front was Lot No. 1201-A
owned by Spouses Williams, where the national highway ran along.
Zerda filed a complaint against Spouses Williams for easement of right of way. The complaint alleged that Zerda's
lot was without adequate outlet to a public highway, that it could not be accessed except by passing through
Spouses Williams' property; that the isolation of Zerda's property was not due to his own acts, as it was the natural
consequence of its location; that they wrote to Spouses Williams formally asking them to provide him with right of
way, for which he was willing to pay its reasonable value or to swap a portion of his property, but Spouses
Williams refused.

Spouses Williams countered that Zerda failed to establish the requisites for the existence of right of way. They
claimed that they were in negotiation with Sierra, the former owner of the dominant estate, for its sale to them but
the sale did not materialize due to the intervention of Zerda. Spouses Williams further averred that they undertook
visible development projects on their property; that the isolation of the dominant estate was Zerda's fault; and that
his requested right of way would cause great damage and prejudice to them.

ISSUE:

Whether respondent Zerda is entitled to an easement of right of way.

RULING: YES.

All the requisites for entitlement to an easement of right of way are present in this case.

An entitlement to the easement of right of way requires that the following requisites must be met:
1. The dominant estate is surrounded by other immovables and has no adequate outlet to a public highway (Art.
649, par. 1);
2. There is payment of proper indemnity (Art. 649, par. 1);
3. The isolation is not due to the acts of the proprietor of the dominant estate (Art. 649, last par.); and
4. The right of way claimed is at the point least prejudicial to the servient estate; and insofar as consistent with this
rule, where the distance from the dominant estate to a public highway may be the shortest (Art. 650).

As regards the first requisite, there is no dispute that the respondent's property was surrounded by other
immovables owned by different individuals, including Spouses Williams. The isolation was further shown in the
Sketch Plan prepared a geodetic engineer.

The second requisite of payment of indemnity was also complied with by the respondent when he wrote Spouses
Williams formally asking them to provide him with a right of way, for which he was willing to pay a reasonable
value or to swap a portion of his property.

Anent the third requisite, when the respondent bought the dominant estate there could have been no existing
contract of sale yet considering that Spouses Williams and Sierra were still in negotiation. Hence, consent, one of
the essential requisites for a valid contract, was lacking.

As to the fourth requisite, the Court finds that the right of way sought by the respondent is at the point least
prejudicial to the servient estate and it is the shortest distance to the national highway.

It must be pointed out that where the easement may be established on any of several tenements surrounding the
dominant estate, the one where the way is shortest and will cause the least damage should be chosen. If having
these two (2) circumstances do not concur in a single tenement, the way which will cause the least damage should
be used, even if it will not be the shortest."

6. Other easements – NCC 674-676, 677-681, 682-683, 667-673

Calma v. CA
G.R. No. 78447 August 17, 1989

FACTS:
In 1975, the spouses Restituto and Pilar Calma purchased a lot, built a house and established residence in
respondent Pleasantville. Fabian and Nenita Ong also purchased from PLEASANTVILLE a lot fronting that of the
Calma spouses and constructed their own buildings where they resided and conducted their business. Calma
complaint about the utilization of Ongs residence as a lumber yard and that a “loathsome noise and nervous
developing sound” emanating therefrom disturbed him and his family and caused them and their son to suffer
nervous tension and illness. The Calma spouses filed a complaint for damages against the Ong spouses and
PLEASANTVILLE before the Court of First Instance of Negros Occidental.

Petitioner also filed with the National Housing Authority (NHA), a complaint for “Violation of the Provisions, Rules
and Regulations of the Subdivision and Condominium Buyers Protective Decree under Presidential Decree No. 957.
Petitioner prayed that PLEASANTVILLE be ordered to abate the alleged nuisance and recover damages for their
medical problems purportedly caused by the nuisance.

The COMMISSION (which had in the meantime taken over the powers of the NHA,) *rendered a decision dismissing
the complaint of the petitioner for lack of merit, but included a portion holding PLEASANTVILLE responsible for
the abatement of the alleged nuisance on the Ground that it was part of its implied warranty that its subdivision
lots would be used solely and primarily for residential purpose.

Aggrieved, PLEASANTVILLE filed a petition for prohibition with preliminary injunction with this Court assailing
the portion of the COMMISSION’s decision. PLEASANTVILLE asserted that since the COMMISSION had found that it
did not violate any provision of P.D. No. 957, the COMMISSION exceeded its jurisdiction when it ordered
PLEASANTVILLE to prevent / abate the alleged nuisance complained of.

The Court of Appeals rendered judgment holding that the COMMISSION “acted capriciously and in excess of its
jurisdiction in imposing an obligation upon the petitioner after absolving it of the complaint filed against it”.

Petitioner moved for reconsideration but the Court of Appeals denied his motion. Hence, the instant petition.

ISSUE:

Whether or not the Commission gravely abuse its discretion in ruling that Ongs property constituted a nuisance.

RULING:

Yes. The COMMISSION’s conclusion that the activities being conducted and the structures in the property of the
Ongs constituted a nuisance was not supported by any evidence. The Solicitor General himself, in his comment filed
in the Court of Appeals, admits that the decision of the COMMISSION did not make any finding of a nuisance.
Apparently, on the basis of position papers, the COMMISSION assumed the existence of the nuisance, without
receiving evidence on the matter, to support its order for the prevention or abatement of the alleged nuisance.

Moreover, the spouses Ong, were not even party to the proceedings before the COMMISSION who would be directly
affected by a decision favorable to petitioner. To declare their property or the activities being conducted therein a
nuisance, and to order prevention and abatement, without giving them an opportunity to be heard would be in
violation of their basic right to due process.

Hence, no reversible error was committed by the Court of Appeals when it nullified the assailed portion of the
COMMISSION’s decision, the order granting the writ of execution, and any writ of execution issued pursuant
thereto.

But all is not lost for petitioner and his family. There is still a pending civil case instituted by petitioner. In said
proceeding the factual issues can be fully threshed out and the Ong spouses, the parties who shall be directly
affected by any adverse judgment, shall be afforded the opportunity to be heard as they had been impleaded as
defendants therein together with PLEASANTVILLE.
Gancayco v. City Government of Quezon City and MMDA
Novemeber 11, 2011, G.R. No. 177807

Article 694 of the Civil Code defines nuisance as any act, omission, establishment, business, condition or property, or
anything else that (1) injures or endangers the health or safety of others; (2) annoys or offends the senses; (3) shocks,
defies or disregards decency or morality; (4) obstructs or interferes with the free passage of any public highway or
street, or any body of water; or, (5) hinders or impairs the use of property. A nuisance may be per se or per accidens. A
nuisance per se is that which affects the immediate safety of persons and property and may summarily be abated
under the undefined law of necessity.

FACTS:

The consolidated petitions of Retired Justice Emilio Gancayco, City Government of Quezon City and the Metro
Manila Development Authority stemmed from a local ordinance pertaining to Construction of Arcades, and the
clearing of Public Obstructions.

Gaycanco owns a property, of which he was able to obtain a building permit for a two-storey commercial building,
which was situated along EDSA, in an area which was designated as part of a Business/Commercial Zone by the
Quezon City Council. The Quezon City Council also issued Ordinance No. 2904, which orders the construction of
Arcades for Commercial Buildings. The ordinance was amended to not require the properties located at the Quezon
City - San Juan boundary, and commercial buildings from Balete - Seattle Street to construct the arcades, moreover,
Gancayco had been successful in his petition to have his property, already covered by the amended ordinance,
exempted from the ordinance. MMDA on April 28, 2003, sent a notice to Gancayco, under Ordinance no. 2904, part
of his property had to be demolished, if he did not clear that part within 15 days, which Gancayco did not comply
with, and so the MMDA had to demolish the party wall, or “wing walls.”

Gancayco then filed a temporary restraining order and/or writ of preliminary injunction before the RTC of Quezon
City, seeking to prohibit the demolition of his property, without due process and just compensation, claiming that
Ordinance no. 2904 was discriminatory and selective. He sought the declaration of nullity of the ordinance and
payment for damages. MMDA contended that Gancayco cannot seek nullification of an ordinance that he already
violated, and that the ordinance had the presumption of constitutionality, and it was approved by the Quezon City
Council, taking to note that the Mayor signed the ordinance.

The RTC, however, declared that the Ordinance was unconstitutional, invalid and void ab initio. MMDA appealed to
the Court of Appeals, and the CA partly granted the appeal, with the contention that the ordinance was to be
modified; it was constitutional because the intention of the ordinance was to uplift the standard of living, and
business in the commercial area, as well as to protect the welfare of the general public passing by the area,
however the injunction against the enforcement and implementation of the ordinance is lifted. With that decision,
the MMDA and Gancayco filed Motions for Reconsideration, which the CA denied, as both parties have no new
issues raised. Therefore they petitioned to the Court.

ISSUE:

Whether or not the wing wall of Gancayco’s property can be constituted as a public nuisance. Whether or not
MMDA was in their authority to demolish Gancayco’s property.

HELD:

The court affirmed the decision of the Court of Appeals. The court decided that the wing wall of Gancayco’s
building was not a nuisance per se, as under Art. 694 of the Civil Code of the Philippines, nuisance is defined as any
act, omission, establishment, business, condition or property, or anything else that (1) injures of endangers the
health or safety of the others; (2) annoys or offends the senses; (3) shocks, defies or disregards decency or
morality; (4) obstruct or interferes with the free passage of any public highway or street, or any body of water; or
(5) hinders or impairs the use of property. A nuisance may be a nuisance per se or a nuisance per accidens. A
nuisance per se are those which affect the immediate safety of persons and property and may summarily be abated
under the undefined law of necessity. As Gaycanco was able to procure a building permit to construct the building,
it was implied that the city engineer did not consider the building as such of a public nuisance, or a threat to the
safety of persons and property. The MMDA was only to enforce Authoritative power on development of Metro
Manila, and was not supposed to act with Police Power as they were not given the authority to do such by the
constitution, nor was it expressed by the DPWH when the ordinance was enacted. Therefore, MMDA acted on its
own when it illegally demolished Gancayco’s property, and was solely liable for the damage.

Unisource Commercial v. Chung


July 17, 2009, G.R. No. 73252

As defined, an easement is a real right on anothers property, corporeal and immovable, whereby the owner of the
latter must refrain from doing or allowing somebody else to do or something to be done on his property, for the benefit
of another person or tenement. Easements are established either by law or by the will of the owner. The former are
called legal, and the latter, voluntary easements.

FACTS:

Petitioner Unisource Commercial and Development Corporation is the registered owner of a parcel of land covered
by Transfer Certificate of Title (TCT) No. 176253 of the Register of Deeds of Manila. The title contains a
memorandum of encumbrance of a voluntary easement which has been carried over from the Original Certificate
of Title of Encarnacion S. Sandico. As Sandico's property was transferred to several owners, the memorandum of
encumbrance of avoluntary easement in favor of Francisco M. Hidalgo was consistently annotated at the back of
every title covering Sandico's property until TCT No. 176253 was issued in petitioner's favor. On the other hand,
Hidalgo's property was eventually transferred to respondents Joseph Chung, Kiat Chung and Cleto Chung under
TCT No. 121488. On May 26, 2000, petitioner filed a Petition to Ca nc e l t he E nc um br anc e of V o lun ta r y
Easement of Right of Way on the ground that the dominant estate has an adequate access toa public road which is
Matienza Street. The trial court dismissed the petition on the ground that it is a land registration case.
Petitioner moved for reconsideration. Thereafter, the trial court conducted an ocular inspection of the property.
On August 19, 2002, the trial court ordered the cancellation of the encumbrance of voluntary easement of right of
way in favor of the dominant estate owned by respondents. It found that the dominant estate has no more
use for the easement since it has another adequate outlet to a public road which is Matienza Street.

ISSUE

Whether or not to cancel the encumbrance of voluntary easement of right of way.

HELD:

No, petitioner itself admitted that a voluntary easement of right of way exists in favor of respondents. In
its petition to cancel the encumbrance of voluntary easement of right of way, petitioner alleged that the
easement is personal. It was voluntarily constituted in favor of a certain Francisco Hidalgo y Magnifico, the owner
of described as Lot No. 2, Block 2650. It further stated that the voluntary easement of the right of way in favor of
Francisco Hidalgo y Magnifico was constituted simply by will or agreement of the parties. It was not a
statutory easement and definitely not an easement created by such court order because the Court merely
declares the existence of an easement created by the parties." In its Memorandum dated September 27, 2001,
before the trial court, petitioner reiterated that "[t]he annotation found at the back of the TCT of Unisource is a
voluntary easement."

Vergara v. Sonkin
G.R. No. 193659, June 15, 2015

Lower estates are obliged to receive the waters which naturally and without the intervention of man descend from the
higher estates, as well as the stones or earth which they carry with them. The owner of the lower estate cannot
construct works which will impede this easement; neither can the owner of the higher estate make works which will
increase the burden.
FACTS:

Petitioners-spouses Fernando Vergara and Herminia Vergara (Sps. Vergara) and Spouses Ronald Mark Sonkin and
Erlinda Torrecampo Sonkin (Sps. Sonkin) are adjoining landowners in Poblacion, Norzagaray, Bulacan. In view of
the geographical configuration of the adjoining properties the property owned by Sps. Sonkin (Sonkin Property) is
slightly lower in elevation than that owned by Sps. Vergara (Vergara Property).

Sometime in 2001, Sps. Vergara levelled the uneven portion of the Vergara Property by filling it with gravel, earth,
and soil. As a result, the level of the Vergara Property became even higher than that of the Sonkin Property by a
third of a meter. Eventually, Sps. Sonkin began... to complain that water coming from the Vergara Property was
leaking into their bedroom through the partition wall, causing cracks, as well as damage, to the paint and the
wooden parquet floor. Sps. Sonkin repeatedly demanded that Sps. Vergara build a retaining wall on their...
property in order to contain the landfill that they had dumped thereon, but the same went unheeded. Hence, Sps.
Sonkin filed the instant complaint for damages and injunction with prayer for preliminary mandatory injunction
and issuance of a temporary... restraining order against Sps. Vergara

ISSUE:

Whether or not it should have ordered the demolition of the portion of the Sps. Sonkin's house that adjoins the
partition wall.

RULING:

In the case at bar, it is undisputed that the Sonkin property is lower in elevation than the Vergara property, and
thus, it is legally obliged to receive the waters that flow from the latter, pursuant to Article 637 of the Civil Code.
This provision refers to the legal easement pertaining to the natural drainage of lands, which obliges lower estates
to receive from the higher estates water which naturally and without the intervention of man descends from the
latter, i.e., not those collected artificially in reservoirs, etc., and the stones and earth carried by the waters.

Hence, the CA correctly held that while the proximate cause of the damage sustained by the house of Sps. Sonkin
was the act of Sps. Vergara in dumping gravel and soil onto their property, thus, pushing the perimeter wall back
and causing cracks thereon, as well as water seepage,... the former is nevertheless guilty of contributory negligence
for not only failing to observe the two (2)-meter setback rule under the National Building Code, but also for
disregarding the legal easement constituted over their property. As such, Sps. Sonkin must necessarily and equally
bear their own loss.

7. Extinguishment of Easements – NCC 631, 655

DIFFERENT MODES OF ACQUIRING OWNERSHIP


A. In general, NCC 712

Acap v. CA
251 SCRA 30 (1995)

Ownership and real rights are acquired only pursuant to a legal mode or process. While title is the juridical
justification, mode is the actual process of acquisition or transfer of ownership over a thing in question.

FACTS:

The title to Lot No. 1130 of the Cadastral Survey of Hinigaran, Negros Occidental was issued and is registered in the
name of spouses Santiago Vasquez and Lorenza Oruma. After both spouses died, their only son Felixberto inherited
the lot. In 1975, Felixberto executed a duly notarized document entitled “Declaration of Heirship and Deed of
Absolute Sale” in favor of Cosme Pido.
The evidence before the court a quo established that since 1960, petitioner Teodoro Acap had been the tenant of a
portion of the said land. When ownership was transferred in 1975 by Felixberto to Cosme Pido, Acap continued to
be the registered tenant thereof and religiously paid his leasehold rentals to Pido and thereafter, upon Pido’s death,
to his widow Laurenciana.

The controversy began when Pido died intestate and on 27 November 1981, his surviving heirs executed a
notarized document denominated as “Declaration of Heirship and Waiver of Rights of Lot No. 1130 Hinigaran
Cadastre.” The document was signed by all of Pido’s heirs. Private respondent Edy de los Reyes did not sign said
document.

It will be noted that at the time of Cosme Pido’s death, title to the property continued to be registered in the name
of the Vasquez spouses. Upon obtaining the Declaration of Heirship with Waiver of Rights in his favor, private
respondent Edy de los Reyes filed the same with the Registry of Deeds as part of a notice of an adverse claim
against the original certificate of title.

Thereafter, private respondent sought for petitioner (Acap) to personally inform him that he (Edy) had become the
new owner of the land and that the lease rentals thereon should be paid to him. In 1982, petitioner allegedly
complied with said obligation. In 1983, however, petitioner refused to pay any further lease rentals on the land,
prompting private respondent to seek the assistance of the then Ministry of Agrarian Reform. An officer of the
Ministry informed Acap’s wife about private respondent’s ownership of the said land but she stated that she and
her husband (Teodoro) did not recognize private respondent’s claim of ownership over the land.

ISSUE:

Whether or not the subject declaration of heirship and waiver of rights is a recognized mode of acquiring
ownership by private respondent over the lot in question.

RULING:

In the first place, an asserted right or claim to ownership or a real right over a thing arising from a juridical act,
however justified, is not per se sufficient to give rise to ownership over the res. That right or title must be
completed by fulfilling certain conditions imposed by law. Hence, ownership and real rights are acquired only
pursuant to a legal mode or process. While title is the juridical justification, mode is the actual process of
acquisition or transfer of ownership over a thing in question.

Under Article 712 of the Civil Code, the modes of acquiring ownership are generally classified into two (2) classes,
namely, the original mode (i.e., through occupation, acquisitive prescription, law or intellectual creation) and the
derivative mode (i.e., through succession mortis causa or tradition as a result of certain contracts, such as sale,
barter, donation, assignment or mutuum).

In the case at bench, the trial court was obviously confused as to the nature and effect of the Declaration of
Heirship and Waiver of Rights, equating the same with a contract (deed) of sale. They are not the same. In a
Contract of Sale, one of the contracting parties obligates himself to transfer the ownership of and to deliver a
determinate thing, and the other party to pay a price certain in money or its equivalent. Upon the other hand, a
declaration of heirship and waiver of rights operates as a public instrument when filed with the Registry of Deeds
whereby the intestate heirs adjudicate and divide the estate left by the decedent among themselves as they see fit.
It is in effect an extrajudicial settlement between the heirs under Rule 74 of the Rules of Court.

Hence, there is a marked difference between a sale of hereditary rights and a waiver of hereditary rights. The first
presumes the existence of a contract or deed of sale between the parties. The second is, technically speaking, a
mode of extinction of ownership where there is an abdication or intentional relinquishment of a known right with
knowledge of its existence and intention to relinquish it, in favor of other persons who are co-heirs in the
succession. Private respondent, being then a stranger to the succession of Cosme Pido, cannot conclusively claim
ownership over the subject lot on the sole basis of the waiver document which neither recites the elements of
either a sale, or a donation, or any other derivative mode of acquiring ownership.
B. Modes of Acquiring Ownership
1. Intellectual Creation
* Now governed by the Intellectual Property Code and the TRIPS Agreement.
2. Occupation, NCC 713-720 39

Palero-Tan v. Urdaneta
A.M. P-07-2399, June 18, 2008

When a person who finds a thing that has been lost or mislaid by the owner takes the thing into his hands, he acquires
physical custody only and does not become vested with legal possession. In assuming such custody, the finder is
charged with the obligation of restoring the thing to its owner. It is thus respondent’s duty to report to his superior or
his officemates that he found something.

FACTS:

Edna Palero-Tan charged Ciriaco I. Urdaneta (court employee) for stealing her ring and bracelet. Complainant
claimed that she kept her jewelry in the locked drawer of her table because she fears that they might be lost at the
boarding house she is renting. However, she discovered that her ring and bracelet worth P15,000.00 were
missing. She maintained that the only person who was present and saw her take out the jewelry from her table
drawer was respondent (Ciriaco), whose table is adjacent to hers.

An officemate, Altone, confided to her that he heard from his landlady, Nable, that respondent and his wife,
Milagros, had a quarrel because the latter discovered a ring and a bracelet in respondents coin purse. Milagros
suspected that respondent bought the jewelry for his mistress.

Respondent denied that he stole complainants jewelry. He claimed that he found a small plastic sachet containing a
ring and a bracelet under his table. When nobody claimed the jewelry, he placed them inside his coin purse and
took them home. However, he threw the pieces of jewelry when his wife starts nagging.

ISSUE:

Whether or not respondent is guilty.

RULING: YES.

The Civil Code, in Art. 719, explicitly requires the finder of a lost property to report it to the proper authorities.

Article 719. Whoever finds a movable, which is not treasure, must return it to its previous possessor. If the
latter is unknown, the finder shall immediately deposit it with the mayor of the city or municipality where
the finding has taken place. x x x

Contrary to respondents claim, this Court is convinced that respondent had the intention to appropriate the
jewelry to himself had these not been discovered by his wife. His claim that the ring and bracelet were worthless
fancy jewelry is immaterial because the basis for his liability is his act of taking something which does not belong
to him.

By admittedly finding complainant’s ring and bracelet without returning them to the rightful owner, respondent
blatantly degraded the judiciary and diminished the respect and regard of the people for the court and its
personnel. Every employee of the judiciary should be an example of integrity, morality and honesty. Thus, he is
guilty of Grave Misconduct.

3. Law, NCC 681, 1434, 1456


4. Tradition, NCC 1496-1501
5. Donation, NCC 725 – 769
a. Features

Seventh Day Adventist Conference Church of Southern Philippines v. Northeastern Mindanao Mission of
Seventh Day Adventist, Inc.
496 SCRA 215

Donation is an act of liberality whereby a person disposes gratuitously of a thing or right in favor of another person
who accepts it.

FACTS:

Spouses Felix Cosio and Felisa Cuysona donate a parcel of land to South Philippine [Union] Mission of Seventh Day
Adventist Church, and was received by Liberato Rayos, an elder of the Seventh Day Adventist Church, on behalf of
the donee.

However, twenty years later, the spouses sold the same land to the Seventh Day Adventist Church of Northeastern
Mindanao Mission.

Claiming to be the alleged donee’s successors-in-interest, petitioners asserted ownership over the property. This
was opposed by respondents who argued that at the time of the donation, SPUM-SDA Bayugan could not legally be
a donee because, not having been incorporated yet, it had no juridical personality. Neither were petitioners
members of the local church then, hence, the donation could not have been made particularly to them.

ISSUE:

Whether or not the Seventh Day Adventist Church of Northeastern Mindanao Mission's ownership of the lot should
be upheld

HELD:

Yes.Donation is undeniably one of the modes of acquiring ownership of real property. Likewise, ownership of a
property may be transferred by tradition as a consequence of a sale.

Donation is an act of liberality whereby a person disposes gratuitously of a thing or right in favor of another person
who accepts it. The donation could not have been made in favor of an entity yet inexistent at the time it was made.
Nor could it have been accepted as there was yet no one to accept it.

Talayan Holdings v. Homeowners Assoc. of Talayan


G.R. No. 203883, Nov. 10, 2015

The purpose of the formal requirement for acceptance of a donation is to ensure that such acceptance is duly
communicated to the donor. Since the donation is considered perfected only upon the moment the donor is apprised of
such acceptance, it has been ruled that lack of such acceptance, as expressly provided under the law, renders the
donation null and void.

FACTS:

The Quezon City Council passed Ordinance No. 5095, series of 1962, directing all subdivision owners to
turn over to the city government the open spaces in city subdivisions which were required to be equivalent to 6%
of the total land area being developed. Ostensibly in compliance with said ordinance, J.M. Tuason Co., Inc., through
Gregorio Araneta, Inc., executed in favor of the city government a Deed of Donation and Acceptance over its
subdivisions' open spaces which included Block 494. No record or document exists to show that the donation was
accepted. Block 494 became the site of the Talayan Village Barangay Hall, a multi-purpose hall, basketball, tennis
and football courts and a children's playground which were developed at the expense of Homeowners Association
of Talayan Village, Inc. (HATVI) and the Quezon City government. J.M. Tuason subsequently sold Block 494 to
Talayan Holdings, Inc. (THI). THI eventually obtained a loan from Equitable Bank, secured by real estate mortgages
over the four lots into which Block 494 had been subdivided. J.M. Tuason and THI maintained that no donation was
perfected and that, if at all, the right under said contract pertained to the Quezon City government.

ISSUE:

Whether the donation is void.

RULING:

Yes. That J.M., Tuason donated Block 494 to the Quezon City government in compliance with the latter's
ordinances did not operate to divest the property of its private character. In addition to the fact that the donation
was not embodied in a public document as provided under Article 749 of the Civil Code, the record is entirely
bereft of showing that said donation was duly accepted in accordance with Article 745. The purpose of the formal
requirement for acceptance of a donation is to ensure that such acceptance is duly communicated to the donor.
Since the donation is considered perfected only upon the moment the donor is apprised of such acceptance, it has
been ruled that lack of such acceptance, as expressly provided under the law, renders the donation null and void.

b. Classifications

Del Rosario v. Ferrer


630 SCRA 683 (2010)

A Donation inter vivos is immediately operative and final. The reason is that such kind of donation is deemed
perfected from the moment the donor learned of the donee's acceptance of the donation. The acceptance makes the
donee the absolute owner of the property donated. In case of doubt, the conveyance should be deemed a donation inter
vivos rather than mortis causa, in order to avoid uncertainty as to the ownership of the property subject of the deed.

FACTS:

Spouses Leopoldo and Guadalupe Gonzales during their lifetime executed a document entitled “Donation Mortis
Causa” in favor of their 2 children, Asuncion and Emiliano, and their granddaughter, Jarabini (daughter of their
predeceased son, Zoilo) covering a house and lot. The document stated that it shall be irrevocable and shall be
respected by the surviving spouse. Guadalupe, the donor wife, died and few months later, Leopoldo, the donor
husband, executed a deed of assignment of his rights and interests in subject property to their daughter Asuncion.
Leopoldo died in 1972.

Jarabini then filed a petition for the probate of the deed of donation mortis causa before the RTC of Manila which
was opposed by Asuncion invoking his father Leopoldo's assignment of his rights and interests in the property to
her. The RTC ruled that the donation was in fact one made inter vivos, the donors' intention being to transfer title
over the property to the donees during the donors' lifetime, given its irrevocability. Consequently, Leopoldo's
subsequent assignment of his rights and interest in the property was void since he had nothing to assign.

On appeal, the CA reversed the RTC’s decision finding the deed as donation mortis causa, and since it did not
comply with the requirements of a notarial will, the same is void.

ISSUE:

Whether the deed is a donation inter vivos or a donation mortis causa.

RULING:

It is a donation inter vivos.


The Court ruled that the caption of the document is not controlling. The Court, in addition, citing Austria-Magat vs.
CA, held that the express "irrevocability" of the donation is the "distinctive standard that identifies the document as
a donation inter vivos." In this case, the donors plainly said that it is "our will that this Donation Mortis Causa shall
be irrevocable and shall be respected by the surviving spouse." The intent to make the donation irrevocable
becomes even clearer by the proviso that a surviving donor shall respect the irrevocability of the
donation. Consequently, the donation was in reality a donation inter vivos.

Since the donation in this case was one made inter vivos, it was immediately operative and final. The reason is that
such kind of donation is deemed perfected from the moment the donor learned of the donee's acceptance of the
donation. The acceptance makes the donee the absolute owner of the property donated.

Given that the donation in this case was irrevocable or one given inter vivos, Leopoldo's subsequent assignment of
his rights and interests in the property to Asuncion should be regarded as void for, by then, he had no more rights
to assign. He could not give what he no longer had. Nemo dat quod non habet.

Meralco v. Heirs of Sps. Martonito


GR No. 192893, June 5, 2013

FACTS:

Respondents are the owners, by way of succession, of a parcel of land located in Trece Martires City (Trece
Martires property). In 1965, Dionisio, respondents’ predecessor-in-interest, donated a 680-square meter portion
(subject land) of the 8,550 square meter property to the Communications and Electricity Development Authority
(CEDA) for the latter to provide cheap and affordable electric supply to the province of Cavite. A deed of
donation was executed to reflect and formalize the transfer. Sometime in 1985, CEDA offered for sale to MERALCO,
its electric distribution system necessary for providing electrical service in Cavite. After the approval of the MOA,
CEDA and MERALCO executed the Deed of Absolute Sale. Thereafter, MERALCO occupied the subject land.
MERALCO requested Dionisio’s permission for the continued use of the subject land as a substation site. The
parties were not able to reach any agreement. Meanwhile, respondents claimed that they had no immediate use for
the subject land and that they were preoccupied with the judicial proceedings to rectify errors involving the
reconstituted title of the Trece Martires property, which included the subject land. Respondents demanded that
MERALCO vacate the subject land. Despite the written demand, MERALCO did not move out of the subject land.
Thus, respondents were constrained to file the complaint for unlawful detainer.

ISSUE:

Whether or not title to the property donated to CEDA was validly transferred to the petitioner.

RULING: NO.

Evidently in a letter sent by MERALCO to Dionision seeking permission for the continued use of the subject land,
MERALCO acknowledged that the owners of the subject land were the Deloys. The first letter was written barely
four (4) months after the deed of sale was accomplished. MERALCO never disputed the declarations contained in
these letters. Guided by the foregoing rules and jurisprudence, the Court holds that the letter and the internal
memorandum presented, offered and properly admitted as part of the evidence on record by MERALCO itself,
constitute an admission against its own interest. Hence, MERALCO should appropriately be bound by the contents
of the documents.

The Court has read the MOA and the Deed of Absolute Sale but found nothing that clearly stated that the subject
land was included therein. What were sold, transferred and conveyed were "its electric distribution facilities,
service drops, and customers' electric meters except those owned by the VENDOR'S customers, x x x, and all the
rights and privileges necessary for the operation of the electric service x x x." No mention was made of any land.
Rights and privileges could only refer to franchises, permits and authorizations necessary for the operation of the
electric service. The land on which the substation was erected was not included, otherwise, it would have been so
stated in the two documents. Otherwise, also, MERALCO would not have written Dionisio to ask permission for the
continued use of the subject land.

c. Form

Pajarillo v. IAC
176 SCRA 340 (1989)

The purpose of the formal requirement is to insure that the acceptance of the donation is duly communicated to the
donor. The donation cannot be declared ineffective just because there is no notation in the Extra judicial settlement of
donees acceptance that would be placing too much stress on mere form over the substance.

FACTS:

Perfecta Balane de Cordero died intestate in 1945 and leaving a tract of 28 hectares of land with buildings and
improvements in the Quezon Province. Perfecta’s siblings Juana and Felipe executed a public instrument entitled “
Extra-judicial settlement of the estate of the decease Perfecta Balane de Cordero.” In it they disposed that in
according to Perfecta’s wishes and in consideration of love and affection, the said property be donated to private
respondent Salud Suterio de Matias, Perfecta’s niece, who will assume the encumbrance/obligation to the
Philippine National Bank in the amount of 1,000. In the same document, the donee accepted the donation in a
public instrument. The instrument was never registered nor the title transferred to Salud’s name although she
immediately took possession of the land. Sometime in 1951, Salud transferred the possession of the land to her
mother Juana, who was then staying with her brother Claudio and his family. During the period they were
occupying the land, Claudio paid realty taxes thereon. On May 25, 1956, Juana executed a deed of absolute sale
conveying the land to Claudio. Two years later, Claudio had the land registered in his name. Claudio died in 1961
and his mother in 1963. On June 30, 1965, the private respondents Salud and Pedro Matias filed a complaint for the
reconveyance of the property on the ground that the deed of sale in favour of Claudio was fictitious and the
registration in his name was null and void. Salud claimed that no compensation was paid by Claudio and that the
transaction was deliberately concealed from her by her brother and the defendants.

ISSUE:

Whether or not the donation is valid.

RULING: YES.

It is true that there is nothing in either of the two instruments showing that Salud made authentic notice of the
acceptance to Felipe. While the first instrument contains the statement that “the donee does hereby accept this
donation and does hereby express her gratitude for the kindness and liberality of the donor,” Felipe and Juana
were the only signatories thereof. That was in fact the reason for separate instrument of acceptance signed by
Salud a month later.

A strict interpretation of Art. 633 of the old civil code, can lead to no other conclusion that on the annulment of the
donation for being defective in form. This would be in keeping with the unmistakable language of Art. 633. A literal
adherence to the requirement of the law might result not in justice to the parties but conversely a distortion of
their intentions it is also a policy of the court to avoid such an interpretation. The purpose of the formal
requirement is to insure that the acceptance of the donation is duly communicated to the donor. Here, it is not even
suggested that Juana was unaware of the acceptance for she in fact confirmed it later and requested that the
donated land be not registered during her lifetime by Salud. The donation cannot be declared ineffective just
because there is no notation in the Extra judicial settlement of donees acceptance that would be placing too much
stress on mere form over the substance. It would also be disregard the clear reality of the acceptance of the
donation as manifested in these separate instrument and as later acknowledged by Juana.

Echavez v. Dozen
G.R. No. 192916, October 11, 2010
That the requirements of attestation and acknowledgment are embodied in two separate provisions of the Civil Code
(Articles 805 and 806, respectively) indicates that the law contemplates two distinct acts that serve different purposes.

FACTS:

Vicente Echavez (Vicente) was the absolute owner of several lots in Cebu City. Vicente donated the subject lots to
petitioner Manuel Echavez (Manuel) through a Deed of Donation Mortis Causa. Manuel accepted the donation.

Vicente executed a Contract to Sell over the same lots in favor of Dozen Construction and Development
Corporation (Dozen Corporation). They executed two Deeds of Absolute Sale over the same properties covered by
the previous Contract to Sell.

Vicente died. Emiliano Cabanig, Vicente’s nephew, filed a petition for the settlement of Vicente’s intestate estate. On
the other hand, Manuel filed a petition to approve Vicente’s donation mortis causa in his favor and an action to
annul the contracts of sale Vicente executed in favor of Dozen Corporation.

The RTC granted her petition and declared Diego for all legal intents and purposes presumptively dead in
accordance with Art. 41 of the Family Code. The OSG contested the grant of the petition. The appellate court denied
the petition and affirmed the decision of the RTC.

ISSUE:

W/N the donation mortis cause is valid

RULING:

A donation mortis causa must comply with the formalities prescribed by law for the validity of wills, “otherwise the
donation is void and would produce no effect.”—The CA correctly declared that a donation mortis causa must
comply with the formalities prescribed by law for the validity of wills, “otherwise, the donation is void and would
produce no effect.” Articles 805 and 806 of the Civil Code should have been applied.

That the requirements of attestation and acknowledgment are embodied in two separate provisions of the Civil
Code (Articles 805 and 806, respectively) indicates that the law contemplates two distinct acts that serve different
purposes. An acknowledgment is made by one executing a deed, declaring before a competent officer or court that
the deed or act is his own. On the other hand, the attestation of a will refers to the act of the instrumental witnesses
themselves who certify to the execution of the instrument before them and to the manner of its execution.

d. Distinctions between inter vivos and mortis causa

Del Rosario v. Ferrer


630 SCRA 683 (2010)

A Donation inter vivos is immediately operative and final. The reason is that such kind of donation is deemed
perfected from the moment the donor learned of the donee's acceptance of the donation. The acceptance makes the
donee the absolute owner of the property donated. In case of doubt, the conveyance should be deemed a donation inter
vivos rather than mortis causa, in order to avoid uncertainty as to the ownership of the property subject of the deed.

FACTS:

Spouses Leopoldo and Guadalupe Gonzales during their lifetime executed a document entitled “Donation Mortis
Causa” in favor of their 2 children, Asuncion and Emiliano, and their granddaughter, Jarabini (daughter of their
predeceased son, Zoilo) covering a house and lot. The document stated that it shall be irrevocable and shall be
respected by the surviving spouse. Guadalupe, the donor wife, died and few months later, Leopoldo, the donor
husband, executed a deed of assignment of his rights and interests in subject property to their daughter Asuncion.
Leopoldo died in 1972.

Jarabini then filed a petition for the probate of the deed of donation mortis causa before the RTC of Manila which
was opposed by Asuncion invoking his father Leopoldo's assignment of his rights and interests in the property to
her. The RTC ruled that the donation was in fact one made inter vivos, the donors' intention being to transfer title
over the property to the donees during the donors' lifetime, given its irrevocability. Consequently, Leopoldo's
subsequent assignment of his rights and interest in the property was void since he had nothing to assign.

On appeal, the CA reversed the RTC’s decision finding the deed as donation mortis causa, and since it did not
comply with the requirements of a notarial will, the same is void.

ISSUE:

Whether the deed is a donation inter vivos or a donation mortis causa.

RULING:

It is a donation inter vivos.

The Court ruled that the caption of the document is not controlling. The Court, in addition, citing Austria-Magat vs.
CA, held that the express "irrevocability" of the donation is the "distinctive standard that identifies the document as
a donation inter vivos." In this case, the donors plainly said that it is "our will that this Donation Mortis Causa shall
be irrevocable and shall be respected by the surviving spouse." The intent to make the donation irrevocable
becomes even clearer by the proviso that a surviving donor shall respect the irrevocability of the
donation. Consequently, the donation was in reality a donation inter vivos.

Since the donation in this case was one made inter vivos, it was immediately operative and final. The reason is that
such kind of donation is deemed perfected from the moment the donor learned of the donee's acceptance of the
donation. The acceptance makes the donee the absolute owner of the property donated.

Given that the donation in this case was irrevocable or one given inter vivos, Leopoldo's subsequent assignment of
his rights and interests in the property to Asuncion should be regarded as void for, by then, he had no more rights
to assign. He could not give what he no longer had. Nemo dat quod non habet.

e. Limitations
a. Who may donate, NCC 735, 41, 1390 (1)
b. Who may be a donee, NCC 738
c. Void donations, 739 (1), 736, 1409, FC 87

Insular Life v. Ebrado


G.R. No. L-44059, Oct. 28, 1977

FACTS:

Buenaventura Cristor Ebrado was married to Pascuala Ebrado. During his lifetime, he was living with his
common-law wife, Carponia Ebrado, although he was not legally separated from his legal wife. Buenaventura was
issued by the insurer a policy on a whole-life plan with a rider for Accidental Death Benefits. Buenaventura
designated Carponia Ebrado as the revocable beneficiary in his policy. Buenaventura died as a result of an accident
when he was hit by a falling branch of a tree.

As the insurance policy was still in force, the insurer stands liable to pay the coverage. Carponia Ebrado
then filed with the insurer a claim for the proceeds of the policy as the designated beneficiary therein. Pascuala
Vda. de Ebrado also filed her claim as the widow of the deceased insured. She asserts that she is the one entitled to
the insurance proceeds, not the common-law wife. In doubt as to whom the insurance proceeds shall be paid, the
insurer commenced an action for Interpleader before the CFI.

ISSUE:

Whether a common-law wife of a man who was not legally separated from his legal wife be a beneficiary of his life
insurance plan?

RULING: NO.

A life insurance policy is no different from a civil donation insofar as the beneficiary is concerned. Both are
founded upon the same consideration: liberality. A beneficiary is like a donee, because from the premiums of the
policy which the insured pays out of liberality, the beneficiary will receive the proceeds or profits of said in-
surance. As a consequence, the proscription in Article 739 of the new Civil Code should equally operate in life
insurance contracts. The mandate of Article 2012 cannot be laid aside: any person who cannot receive a donation
cannot be named as beneficiary in the life insurance policy of the person who cannot make the donation.

Policy considerations and dictates of morality rightly justify the institution of a barrier between common-
law spouses in regard to property relations since such relationship ultimately encroaches upon the nuptial and
filial rights of the legitimate family. There is every reason to hold that the bar in donations between legitimate
spouses and those between illegitimate ones should be enforced in life insurance policies since the same are based
on similar consideration. As above pointed out, a beneficiary in a life insurance policy is no different from a
donee. Both are recipients of pure beneficence. So long as marriage remains the threshold of family laws, reason
and morality dictate that the impediments imposed upon married couple should likewise be imposed upon extra-
marital relationship. If legitimate relationship is circumscribed by these legal disabilities, with more reason should
an illicit relationship be restricted by these disabilities.

Republic v AFP, Jan. 16, 2013 40

d. Others, NCC 740, 1027, 1028, 1027 (ex par. 4), 741 – 744, 1544, 745, 746

Lavarez v. Guevarra
G.R. No. 206103, March 29, 2017

A donation is an act of liberality whereby a person disposes a thing or right gratuitously in favour of another, who in
turn, accepts it.

FACTS:

Rebecca Zaballero died intestate without any issue, leaving several properties to be settled among her nearest kins,
the sons and daughters of her siblings, who later became the parties in this case.

Lavarez et al filked an action for reconveyance, partition, accounting, and nullification of documents, against
respondents Guevarra et al.

Guevarra et al alleged that there was nothing to partition since they were not aware of any real or personal
properties which their aunt has left behind. Subject properties which were included in the complaint had already
been donated to them by Rebecca, resulting to new TCT titles issued in their names.

RTC granted the complaint. CA granted the respondents’ appeal and sustained the validity of the Deeds of
Donation.

ISSUE:
W/N Rebecca possessed sufficient mentality to make the deeds of donation which would meet the legal test
regarding the required capacity to dispose.

RULING:

No. Rebecca’s doctor during her lifetime, Dr. Conde, testified that she had been suffering from dementia, which was
more or less permanent, and had been taking medications for years. The records show that Rebecca lived with the
respondents, and the old lady was dependent on their care. During the execution of the deeds in question, Rebecca
was already 75 years old, and was confined at a hospital.

A donation is an act of liberality whereby a person disposes a thing or right gratuitously in favour of another, who,
in turn, accepts it. The parties’ intention must be clear and the attendance of vice of consent, renders the donation
voidable. Certainly, there lies no doubt in the fact that insanity impinges on consent freely given. To constitute
complete senile dementia, there must be such failure of the mind as to deprive the donor of intelligent action. In
this case, the petitioners succeeded in discharging proof that Rebecca did not possess sufficient mentality to
perform the donation.

6. Reduction and revocation, NCC 760-773

Central Philippines University v. CA


246 SCRA 511

Non-fulfillment of the condition imposed upon the donee gives right to the revocation of the donation. Even if no
period was indicated, 50 years is long enough to be deemed non-fulfillment.

FACTS:

In 1939, Don Ramon Lopez, Sr. donated a parcel of land in favor of Central Philippine College (later on, University).
The deed of donation stated that the land described shall be utilized by the CPU exclusively for the establishment of
a medical college with all its buildings as part of the curriculum; that the said college shall not sell, transfer or
convey to any third party nor in any way encumber said land; that the said land shall be called “RAMON LOPEZ
CAMPUS”, and the college shall be under obligation to erect a cornerstone bearing that name. Any net income from
the land or any of its parks shall be put in a fund to be known as the “RAMON LOPEZ CAMPUS FUND” to be used for
improvements of said campus and erection of a building thereon.

In 1989, the heirs of Ramon Lopez filed an action for annulment of donation, reconveyance and damages against
CPU alleging that for 50 years, the CPU never complied with the conditions, and even violated it for negotiating
with National Housing Authority to exchange the donated property with another land.

The CPU alleged that the right to file the action had prescribed, and also that no violation was made by the CPU.
The RTC held that CPU failed to comply with the conditions and declared the donation null and void. It was
appealed to the CA which ruled that the obligations imposed were resolutory conditions, which failure to comply
renders the donation revocable. But it ruled that since there’s no period for compliance with the condition, CPU
cannot be deemed to have failed to perform it yet, so the CA remanded the case for determination of the period.

ISSUE:

Whether or not the donation can be revoked.

RULING:

Yes. Where Don Ramon Lopez donated the subject parcel of land to petitioner but imposed an obligation upon the
latter to establish a medical college thereon, the donation must be for an onerous consideration. The condition was
not suspensive, but resolutory. If there was no fulfillment of the condition, such as what obtains in the instant case,
the donation may now be revoked and all rights which the donee may have acquired under it shall be deemed lost
and extinguished. The action to revoke has not expired yet since there’s no legal possibility of bringing the suit to
start the count of the prescriptive period, because there’s no period of compliance in the deed. However, this rule
cannot be applied on the part of the CPU to comply with the obligation, considering that 50 years had lapsed and it
did not do anything to fulfill the condition, to apply the rule of fixing the period by the court will just result to
multiplicity of suits and more delay. In the absence of any just cause for the court to determine the period of
compliance, there’s no obstacle for the court to grant the decree of rescission. Morever, being a gratuitous contract,
doubts shall be resolved in favor of least transmission of rights. The CPU is ordered to reconvey the property to the
private respondents (heirs of Lopez).

Lagazo v. CA and Cabanlit


287 SCRA 18 (1998)

Acceptance of the donation by the donee is, therefore, indispensable; its absence makes the donation null and void.

FACTS:

Catalina was the grantee of a portion of the Monserrat estate. The Monserrat Estate is a public land owned by the
City of Manila and distributed for sale to bona fide tenants under its land-for-the-landless program. Catalina Jacob
constructed a house on the lot.

She had to leave for Canada to become a permanent resident therein and she appointed Espanol to be her attorney-
in-fact to fix the requirements needed. Failing to accomplish what he ought to do, Catalina appointed Lagazo as her
new attorney-in-fact.

The grant was subsequently given and later, the land was donated to Lagazo. Lagazo then sought to remove
Cabanlit from the property. The latter claims ownership over the land by virtue of a deed of sale executed in favor
of him by Espanol.

ISSUE/S:

Where the acceptance of a donation was made in a separate instrument but not formally communicated to the
donor, may the donation be nonetheless considered complete, valid and subsisting?

RULING:

No. Like any other contract, an agreement of the parties is essential. The donation, following the theory of
cognition (Article 1319, Civil Code), is perfected only upon the moment the donor knows of the acceptance by the
donee. Furthermore, if the acceptance is made in a separate instrument, the donor shall be notified thereof in an
authentic form, and this step shall be noted in both instruments. Acceptance of the donation by the donee is,
therefore, indispensable; its absence makes the donation null and void.

The deed of donation does not show any indication that petitioner-donee accepted the gift. During the trial, he did
not present any instrument evidencing such acceptance despite the fact that private respondent already raised this
allegation in his supplemental pleading to which petitioner raised no objection. It was only after the CA had
rendered its decision, when petitioner came before this Court, that he submitted an affidavit dated August 28,
1990, manifesting that he "wholeheartedly accepted" the lot given to him by his grandmother, Catalina Reyes. This
is too late, because arguments, evidence, causes of action and matters not raised in the trial court may no longer be
raised on appeal.

True, the acceptance of a donation may be made at any time during the lifetime of the donor. And
granting arguendo that such acceptance may still be admitted in evidence on appeal, there is still need for proof
that a formal notice of such acceptance was received by the donor and noted in both the deed of donation and the
separate instrument embodying the acceptance. At the very least, this last legal requisite of annotation in both
instruments of donation and acceptance was not fulfilled by petitioner. For this reason, the subject lot cannot be
adjudicated to him.
Imperial v. CA
G.R. No. 112483, October 8, 1999

FACTS:

In 1951, Leoncio sold his 32, 837m² parcel of land to his acknowledged natural son Eloy Imperial. However, in
1953, Leoncio filed a copmplaint for annulment of the sale alleging that he was deceived by his son. They entered
into a compromise agreement that Leoncio will recognize the validity and legality of the sale but Eloyhas to sold
the 1000m² to Leoncio. In 1962, Leoncio died and Victor, his adopted child, substituted him in the execution of the
compromise agreement. However, in 1977, Victor died single and without issue. After 4 years, Ricardo, Victors’s
natural father, died too. Cesar and Trasa, children of Ricardo, filed a complaint for annulment of the donation. A
motion to dismiss was filed on the ground of res judicata. The trial court dismissed the case. On appeal, the CA
reversed the ruling of the trial court and remanded the case for further proceedings. Cesar and Teresa filedan
Amended Complaint alleging that the conveyance impaired the legitime of Victor. The RTC ruled that the donation
is inofficious imparing the legitime of Victor. CA affirmed in toto the decision of the RTC.

ISSUE:

Whether or not the action for inofficious donation has prescribed.

RULING:

Yes. The Supreme court applied Article 1144 of the Civil Code which atates that “actions upon an obligation created
by law must be brought within ten years from the time the right accrues. Here, the right accrues from
the moment Leoncio died, but it took the respondents 24 years to file the action.
Also, there is estoppel by laches on their part. First, Victor is a lawyer; he even substituted his father in the
execution of the compromise agreement regarding the contested conveyance of parcel of land. Second, Richardo is
the lessee of the contested land and it is expected that he was aware of the sale of the land. And, the respondents
only institud the complaint five years after the death of Ricardo. The petition is granted.

7. Prescription
NCC 1106-1155

Solis v. CA
G.R. No. L-46753-54, August 25, 1989

Even a void donation may be the basis of claim of ownership which may ripen into title by prescription based on
the old Civil Code

FACTS:

The Solis’ owned a piece of land in Calasiao Pangasinan inherited from their parents.
In 1939 they allowed Jose Solis, and his wife Florencia Dioquino to construct a house on the eastern portion of the
land with the condition that they would vacate the same as soon as their means permit them to do so.
The defendants claimed that the already acquired the said lot by way of donation proper nupcias based on their
wedding back in 1931. Plus they were in possession of said property openly, continuously and adversely, to the
exclusion of all others, and in the concept of owners and that since 1931 they have paid the taxes due on the
property.
The CFI ruled that while the defendants had possessed the eastern portion of the land in question for more than
30 years, such possession cannot be held adversely against the plaintiffs who had shown a better title thereof.
Hence, prescription does not lie.
Whereas the donation propter nupcias was executed by a certain Tomas Solis, the owner’s brother who had no
right to execute the same.
The CA reversed the said decision based on the fact that the defendants period of possession and payment of taxes.
ISSUE:

Did the Solis’ have the right to evict Jose Solis, and Dioquino from the said land?

RULING:

The Court says no. The donacion was made in 1931 and spouses Jose Solis and Florencia Dioquino took possession
of the land in 1933 by virtue of the donacion. It was the Code of Civil Procedure which was then in force. Under the
Code of Civil Procedure, ten years of adverse possession by a person claiming to be the owner, in whatever way
such occupancy may have commenced shall vest in every actual possessor of such land a full complete title.

'Under the Code of Civil Procedure formerly in force, good or bad faith was immaterial for purposes of acquisitive
prescription. Adverse possession in either character ripened into ownership after the lapse of ten years

Sec. 41. Title to land by prescription. — Ten years of adverse possession by any person claiming to be the owner
for that time of any land or interest in land, uninterruptedly, continuously for ten years by occupancy, descent,
grants or otherwise, in whatever way such occupancy may have commenced or continued, shall vest in every
actual possessor of such land a full complete title, saving to the person under disabilities the rights, secured by the
next Section.

Although petitioners' action for quieting of title was filed in May 30, 1967 when the New Civil Code was already in
effect, Article 1116 of the New Civil Code provides that "prescription already running before the effectivity of this
Code (August 30, 1950) shall be governed by laws previously in force; ..." which in this case is the aforequoted
Section 41 of the Old Civil Code.

Therefore, whatever claim petitioners had over the disputed property had prescribed in view of private
'respondents' open, actual, peaceful, continuous and adverse possession of the same property for more than thirty
years or at the least, for more than twenty (20) years.

Republic v. Rizalvo
G.R. No. 172011, March 7, 2011

FACTS:

On December 7, 2000, respondent Teodoro P. Rizalvo, Jr. filed before the MTC of Bauang, La Union an application
for the registration of a parcel of land referred to in Survey Plan Psu-200706,4 located in Bauang, La Union and
containing an area of 8,957 square meters. Respondent alleged that he is the owner in fee simple of the subject
parcel of land, that he obtained title over the land by virtue of a Deed of Transfer, alleging that he bought the
property from his mother and that he is currently in possession of the land. He thus presented a tax declaration
under his name and a Proof of Payment of real property taxes from 1952 up to the time of his filing of the
application. OSG filed an Opposition alleging that neither respondent nor his predecessors-in-interest had been in
open, continuous, exclusive and notorious possession and occupation of the subject property since June 12, 1945
or earlier and that the tax declarations and tax payment receipts did not constitute competent and sufficient
evidence of ownership, and that the subject property was a portion of public domain and hence not subject to
private acquisition.

ISSUE:

WON respondent and his predecessors-in-interest were in open, continuous, adverse, and public possession of the
land in question in the manner and length of time required by law as to entitle respondent to judicial confirmation
of imperfect title

RULING: NO.
Applicant failed to comply with PD 1529. An applicant for judicial confirmation of imperfect title must prove
compliance with Section 14 of PD No. 1529 or the Property Registration Decree. Among the requirements, one of
which was not satisfied because Rizalvo merely presented a certification and report from the DENR-CENRO dated
July 17, 2001 certifying that the land in question entirely falls within the alienable and disposable zone since
January 21, 1987; that it has not been earmarked for public use; and that it does not encroach any area devoted to
general public use.

Unfortunately, such certification and report is not enough in order to commence the thirty (30)-year prescriptive
period under Section 14 (2). There is no evidence in this case indicating any express declaration by the state that
the subject land is no longer intended for public service or the development of the national wealth. Thus, there
appears no basis for the application of the thirty (30)-year prescriptive period in this case.

However, even if the DENR-CENRO report was enough, Rizalvo would still not be entitled to the registration
of the land. Indeed, even assuming arguendo that the DENR-CENRO certification and report is enough to signify
that the land is no longer intended for public service or the development of the national wealth, respondent is still
not entitled to registration because the land was certified as alienable and disposable in 1987, while the
application for registration was filed on December 7, 2000, a mere thirteen (13) years after and far short of the
required thirty (30) years under existing laws on prescription.

(a) Distinctions between Acquisitive and Extinctive Prescription


Rule 9, Sec. 1 (1997)

Sunga v. De Guzman
June 19, 1979

Appellees were not aware that appellant's possession extended over the whole fishpond, including that which
pertained to them as their share, thus said possession cannot be said to be adverse and open as to give rise to title by
prescription.

FACTS:

A private contract of sale was signed by five (5) of the nine (9) legitimate heirs of the spouses Juan de Guzman and
Lucia Montemayor for a consideration of P700.00. The property in question was a fishpond located in Pampanga.

However, the referred deed of sale is not notarized nor registered in the Register of Deeds of Pampanga, hence the
said property was still registered in the name of the father of the heirs up to October 5, 1962. Three out of the four
who did not sign questioned the sale. Respondents claim they have acquired thru prescription.

ISSUE:

Whether or not petitioners have acquired ownership thru acquisitive and extinctive prescription

RULING:

NO.

Petitioners’ claim of prescription is made to rest on their alleged adverse possession of the whole fishpond, dating
back from 1948. Respondents were not definitely aware that appellant's possession extended over the whole
fishpond, including that which pertained to them as their share.

In that state of their knowledge as to the extent and nature of petitioners-appellant's possession, said possession
cannot be said to be adverse and open as to give rise to title by prescription in favor of petitioners-appellants.
A fishpond is not as physically or actually occupied or held in possession as a parcel of land, in that the signs of
possession in the latter are more visible, and the extent of its exercise or enjoyment, more manifest and easily
determined.

The tax declaration over the land has remained up to the present in the name of the original owners, the deceased
parents of respondents-appellees. The possession of petitioners-appellants, was, therefore, not completely adverse
or open, nor was it truly in the concept of an owner, which are indispensable elements for prescription to become
legally effective as a means of acquiring real property

Overseas Bank v Geraldez, December 28, 1979

Ledesma v. CA
224 SCRA 174

Without the mandatory personal confrontation, no complaint could be filed with the MTC.

FACTS:

Petitioner is the owner-lessor of the apartment building located in Malate, Manila. Two units of the said building
were leased to private respondents herein. Private respondent however failed to comply with the terms of their
contract, particularly for the payment of montlhy rentals. Proceedings before the barangay conciliation was
conducted despite the non-appearance of the petitioner due to her alleged recurring psychological ailments which
eventually issued a certification to file action. Demands of the pletitioner remained unheeded which constrained
the latter to file for an ejectment case before the MTC Manila. MTC ordered the private respondent to vacate the
premises. On appeal, private respondendts found favor before the CA.

Petitioner assails private respondent for raising the issue of non-compliance with Sections 6 and 9 of P.D. 1508
only in his petition for review with the appellate court and which mislead the court to erroneously dismiss her
complaint for ejectment. Petitioner submits that said issue, not having been raised by private respondent in the
court below, cannot be raised for the first time on appeal.

ISSUE:

Whether compliance with the Barangay Conciliation proceeding is a condition sine qua non prior to the filing of
ejectment case before the MTC?

HELD:

We do not agree with petitioner that the issue of non-compliance with Sections 6 and 9 of P.D. 1508 was raised
only for the first time in the Court of Appeals. When private respondent stated that he was never summoned or
subpoenaed by the Barangay Chairman, he, in effect, was stating that since he was never summoned, he could not
appear in person for the needed confrontation of the parties before the Lupon Chairman for conciliation and/or
amicable settlement. Without the mandatory personal confrontation, no complaint could be filed with the MTC.
Petitioner's non-compliance with Secs. 6 and 9 of P.D. 1508 legally barred her from pursuing the ejectment case in
the MTC of Manila.

Tanyag v. Gabriel
April 11, 2012

Acquisitive prescription is a mode of acquiring ownership by a possessor through the requisite lapse of time. In order
to ripen into ownership, possession must be in the concept of an owner, public, peaceful and uninterrupted. Possession
is open when it is patent, visible, apparent, notorious and not clandestine. It is continuous when uninterrupted,
unbroken and not intermittent or occasional; exclusive when the adverse possessor can show exclusive dominion over
the land and an appropriation of it to his own use and benefit; and notorious when it is so conspicuous that it is
generally known and talked of by the public or the people in the neighborhood. The party who asserts ownership by
adverse possession must prove the presence of the essential elements of acquisitive prescription.

FACTS:

Subject of controversy are two adjacent parcels of land located at Ruhale, Barangay Calzada, Municipality of Taguig
(now part of Pasig City, Metro Manila). The first parcel (Lot 1) was originally declared in the name of Jose Gabriel
for the years 1949 and 1966, while the second parcel (Lot 2) was originally declared in the name of Agueda
Dinguinbayan for the years 1966 and 1967. For several years, these lands lined with bamboo plants remained
undeveloped and uninhabited.

Petitioners claimed that Lot 1 was owned by Benita Gabriel, sister of Jose Gabriel, as part of her inheritance as
declared by her in a 1944 notarized instrument ("Affidavit of Sale") whereby she sold the said property to spouses
Gabriel Sulit and Cornelia Sanga. Lot 1 allegedly came into the possession of Benita Gabriel’s own daughter,
Florencia Gabriel Sulit, when her father-in-law Gabriel Sulit gave it to her as part of inheritance of his son, Eliseo
Sulit who was Florencia’s husband. Florencia Sulit sold the same lot to Bienvenido S. Tanyag, father of petitioners,
as evidenced by a notarized deed of sale dated October 14, 1964. As to Lot 2, petitioners averred that it was sold by
Agueda Dinguinbayan to Araceli Tanyag under Deed of Sale executed on October 22, 1968. Petitioners claimed to
have continuously, publicly, notoriously and adversely occupied both Lots 1 and 2 through their caretaker Juana
Quinones; they fenced the premises and introduced improvements on the land. Sometime in 1979, Jose Gabriel,
father of respondents, secured TD No. 120-014-01013 in his name over Lot 1 indicating therein an increased area
of 1,763 square meters. Consequently, the petitioners instituted a civil action against the respondents. The trial
court dismissed the complaint as well as the counterclaim, holding that petitioners failed to establish ownership of
the subject property and finding the respondents to be the declared owners and legal possessors. It likewise ruled
that petitioners were unable to prove by preponderance of evidence that respondents acquired title over the
property through fraud and deceit. Petitioners appealed to the CA which affirmed the trial court’s ruling.
Petitioners’ motion for reconsideration was likewise denied by the CA. Hence, this petition.

It is the petitioner’s contention that they have acquired ownership of the subject lots by virtue of acquisitive
prescription. They also assailed the CA decision in not finding that the respondents obtained OCT No. 1035 in their
names fraudulently and in bad faith.

ISSUE:

Whether petitioners acquired the property through acquisitive prescription.

RULING:

Yes. In this case, the CA was mistaken in concluding that petitioners have not acquired any right over the subject
property simply because they failed to establish Benita Gabriel’s title over said property. The appellate court
ignored petitioners’ evidence of possession that complies with the legal requirements of acquiring ownership by
prescription.

Acquisitive prescription is a mode of acquiring ownership by a possessor through the requisite lapse of time. In
order to ripen into ownership, possession must be in the concept of an owner, public, peaceful and uninterrupted.
Possession is open when it is patent, visible, apparent, notorious and not clandestine. It is continuous when
uninterrupted, unbroken and not intermittent or occasional; exclusive when the adverse possessor can show
exclusive dominion over the land and an appropriation of it to his own use and benefit; and notorious when it is so
conspicuous that it is generally known and talked of by the public or the people in the neighborhood. The party
who asserts ownership by adverse possession must prove the presence of the essential elements of acquisitive
prescription.

On the matter of prescription, the Civil Code provides:

Art. 1117. Acquisitive prescription of dominion and other real rights may be ordinary or extraordinary.
Ordinary acquisitive prescription requires possession of things in good faith and with just title for the time fixed by
law.

Art. 1134. Ownership and other real rights over immovable property are acquired by ordinary prescription
through possession of ten years.

Art. 1137. Ownership and other real rights over immovables also prescribe through uninterrupted adverse
possession thereof for thirty years, without need of title or of good faith. (Emphasis supplied.)

Petitioners’ adverse possession is reckoned from 1969 with the issuance of TD No. 1145 in the name of Araceli
Tanyag, which tax declaration cancelled TD No. 6425 in the name of Jose Gabriel. It is settled that tax receipts and
declarations are prima facie proofs of ownership or possession of the property for which such taxes have been
paid. Coupled with proof of actual possession of the property, they may become the basis of a claim for ownership.
Petitioners’ caretaker, Juana Quinones, has since lived in a nipa hut, planted vegetables and tended a piggery on the
land. Aside from paying taxes due on the property, petitioners also exercised other acts of ownership such as
selling the 468-square meter portion to Sta. Barbara who had constructed thereon a nine-door apartment building.

It was only in 1979 that respondents began to assert a claim over the property by securing a tax declaration in the
name of Jose Gabriel albeit over a bigger area than that originally declared. In 1998, they finally obtained an
original certificate of title covering the entire 1,763 square meters which included Lot 1.

From 1969 until the filing of this complaint by the petitioners in March 2000, the latter have been in continuous,
public and adverse possession of the subject land for 31 years. Having possessed the property for the period and in
the character required by law as sufficient for extraordinary acquisitive prescription, petitioners have indeed
acquired ownership over the subject property. Such right cannot be defeated by respondents’ acts of declaring
again the property for tax purposes in 1979 and obtaining a Torrens certificate of title in their name in 1998.

Mercado v. Espenocilla
February 1, 2012

Prescription, as a mode of acquiring ownership and other real rights over immovable property, is concerned with
lapse of time in the manner and under conditions laid down by law, namely, that the possession should be in the
concept of an owner, public, peaceful, uninterrupted, and adverse.

FACTS:

Doroteo Espinocilla owned a parcel of land and after he died, his five children, Salvacion, Aspren, Isabel, Macario,
and Dionisia divided it equally among themselves. Dionisia died and Macario took possession of her share. He
claimed that Dionisia had donated her share to him. Thereafter he sold 225 sq. m. to his son Roger. The latter sold
114 sq. m. to Atienza. Per actual survey, His wife Belen (respondent) occupies 109 sq. m., Atienza occupies 120 sq.
m., Yu occupies 209 sq. m., and petitioner, Salvacion's son, occupies 132 sq. m. Petitioner avers that he inherited
142.5 sq. m. from Salvacion and bought 28.5 sq. m. from his aunt. The area he occupies is only 132 sq. m., he claims
that respondents encroached on his share by 39 sq. m. Respondents claim that they rightfully possess the land
they occupy by virtue of acquisitive prescription and that there is no basis for petitioners claim of encroachment.
There being no public document to prove Dionisia’s donation, it was held that Macario’s affidavit is void and is an
invalid repudiation of the shares of his sisters. Macario cannot acquire said shares by prescription. Also the oral
partition by Doroteo’s heirs did not include Dionisia’s share and that partition should have been the main action.

ISSUE:

W/N the action to recover the subject portion is barred by prescription.

RULING:
Yes it is. The action for reconveyance based on an implied or constructive trust prescribes in 10 years from the
time the right of action accrues. This is called extinctive prescription, where rights and actions are lost by the lapse
of time. The action was only filed 55 years after Macario’s occupation. The court also said that the sisters wasted
their opportunity to question his acts. Aside from this, Even if Macario’s affidavit is fraudulent, it does not matter.
Extraordinary prescription is unconcerned with title or good faith.

Prescription, as a mode of acquiring ownership and other real rights over immovable property, is concerned with
lapse of time in the manner and under conditions laid down by law, namely, that the possession should be in the
concept of an owner, public, peaceful, uninterrupted, and adverse. Acquisitive prescription of real rights may be
ordinary or extraordinary. Ordinary acquisitive prescription requires possession in good faith and with just title
for 10 years. In extraordinary prescription, ownership and other real rights over immovable property are acquired
through uninterrupted adverse possession for 30 years without need of title or of good faith. The petitioner himself
admits the adverse nature of respondent’s possession with his assertion that Macario’s fraudulent acquisition of
Dionisia’s share created a constructive trust. In a constructive trust, there is neither a promise nor any fiduciary
relation to speak of and the so-called trustee (Macario) neither accepts any trust nor intends holding the property
for the beneficiary (sisters). The holding of a constructive trust is for the trustee himself and at all times adverse.
Prescription may supervene even if the trustee does not repudiate the relationship. Aside from this, strengthening
the position of adverse possesseion, Macario and Roger exercised acts of ownership over the land, specifically
alienating portions thereof.

Fudalan v. Ocial
G.R. No. 194516, June 17, 2015

Prescription, as a mode of acquiring ownership and other real rights over immovable property, is concerned with
lapse of time in the manner and under conditions laid down bylaw, namely, that the possession should be in the
concept of an owner, public, peaceful, uninterrupted, and adverse. Acquisitive prescription of real rights may be
ordinary or extraordinary. Ordinary acquisitive prescription requires possession in good faith and with just title for 10
years. When the Court speaks of possession in "good faith," it consists in the reasonable belief that the person from
whom the thing is received has been the owner thereof, and can transmit his ownership. There is "just title," on the
other hand, when the adverse claimant comes into possession of the property through one of the modes recognized by
law for the acquisition of ownership or other real rights, but the grantor is not the owner or cannot transmit any
right.

FACTS:

Baldomera filed against the Fuderanans for specific performance, quieting of title and nullification of the deed of
extra-judicial settlement with simultaneous sale in favor of Spouses Ocial. She alleged therein that, although still
declared in the name of the late Juana Fuderanan, the property was absolutely owned by her parents, the late
Spouses Eusebio Fucolan and Catalina Bolias, who acquired the property in 1935 and thereafter took actual
possession of the land. She averred that the possession was continuous, peaceful, open, public, adverse, and in the
concept of an owner which was never disturbed by any person until Spouses Ocial, informed the Fudalans and
Baldomera that they had already bought the land from the Fuderanans.

ISSUE:

Whether or not Baldomera acquired the subject land through prescription.

RULING:

No. In the present controversy, aside from Baldomera’s bare allegation that her family had been in possession of
the subject property since it was sold to her parents, no other evidence, documentary or otherwise, showing that
the title to the subject property was indeed transferred from Juana to her parents was presented. In fact, she never
denied that the tax declaration of the property was still in the name of Juana Fuderanan. As such, for lack of "just
title," she could not have acquired the disputed property by ordinary prescription through possession of ten (10)
years. Occupation or use alone, no matter how long, cannot confer title by prescription or adverse possession
unless coupled…

Can Baldomera acquire the property through extraordinary acquisitive prescription? Still no. In extraordinary
prescription, ownership and other real rights over immovable property are acquired through uninterrupted
adverse possession for 30 years even without need of title or of good faith. Taking cue from the foregoing,
Baldomera’s alleged possession could not have amounted to an ownership by way of extraordinary acquisitive
prescription. According to the factual findings of the trial court, it was only in 1994 that her husband, Flavio was
named administrator; that it was also then that they started paying taxes; and that it was also then that they
started occupying the subject property. This observation of the trial court was contrary to her assertion that they
had been paying taxes and had been in possession of the land even before the said period. On this note, the thirty–
year period would only be completed in the year 2024.

(b) Distinctions between Extinctive Prescription and Laches

Miguel v. Catalino
November 29, 1968

Laches is different from the statute of limitations. Prescription is concerned with the fact of delay, whereas laches is
concerned with the effect of delay. Prescription is a matter of time; laches is principally a question of inequity of
permitting a claim to be enforced, this inequity being founded on some change in the condition of the property or the
relation of the parties. Prescription is statutory; laches is not. Laches applies in equity, whereas prescription applies at
law. Prescription is based on fixed time laches is not.

FACTS:

Simeon, Emilia and Marcelina Miguel, and Ventura brought suit in the CFI of Baguio against Catalino for the
recovery of the parcel of land, the plaintiffs claiming to be the children and heirs of the original registered owner,
and averred that defendant, without their knowledge or consent, had unlawfully taken possession of the land,
gathered its produce and unlawfully excluded plaintiffs therefrom. Defendant answered pleading ownership and
adverse possession for 30 years. CFI dismissed the complaint, declared defendant to be the rightful owner, and
ordered the RD to issue a transfer certificate in lieu of the original. As found by the CFI, the land in dispute is
covered by Original Certificate of Title in the name of Bacaquio, a widower. No encumbrance or sale has ever been
annotated in the certificate of title. Ventura is the only child of Bacaquio by his first wife and the other plaintiffs are
his children by his third wife. He begot no issue with his second wife. The three successive wives have all died.
Bacaquio, acquired the land when his second wife died and sold it to Catalino Agyapao, father of the defendant. No
formal deed of sale was executed, but since the sale in 1928, or for more than 30 years, vendee Catalino Agyapao
and his son, defendant, had been in possession of the land, in the concept of owner, paying the taxes thereon and
introducing improvements.

ISSUE:

Whether or not plaintiffs are estopped from claiming ownership over the land

RULING:

YES. Though the appellants are correct in claiming that the sale of the land in 1928 by Bacaquio to Catalino
Agyapao, defendant's father, is null and void ab initio, for lack of executive approval under Section 145(b) of the
Administrative Code of Mindanao and Sulu, vendor Bacaquio suffered the latter (Catalino) to enter, possess and
enjoy the land in question without protest, from 1928 to 1943, when the seller died; and the appellants, in turn,
while succeeding the deceased, also remained inactive, without taking any step to reivindicate the lot from 1944 to
1962, when the present suit was commenced in court. Courts can not look with favor at parties who, by their
silence, delay and inaction, knowingly induce another to spend time, effort and expense in cultivating the land,
paying taxes and making improvements thereon for 30 long years, only to spring from ambush and claim title
when the possessor's efforts and the rise of land values offer an opportunity to make easy profit at his expense.
Lola v. CA
145 SCRA 439 (1986)

The long inaction and delay of the title holder in asserting his right over the disputed lot bars him from recovering the
same.

FACTS:

On June 29, 1936, the petitioners, Fr. Pablo Lola and his sister Maxima Lola, bought from respondent Dolores
Zabala a parcel of land, which the petitioners immediately occupied upon consummation of the sale.

32 years after the sale, Zabala filed a complaint for the recovery of the real property and damages. She alleged that
the parcel of land occupied by the petitioners consists of Lot 5516 and Lot 5517. She averred that the Sale
transaction in 1936 covered only Lot 5516 hence the petitioners shall be ordered to vacate Lot 5517 and demolish
the improvements introduced thereon.

In their answer, the petitioners alleged that at the time of sale, Zabala did not inform them of any division in the
property. Further, the petitioners invoke the doctrine of laches because of the unexplained delay, inaction, and
neglect on the part of the respondent to assert her claim over the disputed lot for over thirty (30) years.

ISSUE:

Whether laches already barred the respondent from recovering the lot in dispute

RULING:

Although the defense of prescription is unavailing to the petitioners because, admittedly, the title to Lot No. 5517 is
still registered in the name of the respondent, still the petitioners have acquired title to it by virtue of the equitable
principle of laches due to the respondent's failure to assert her claim and ownership for thirty two (32) years.

This defense is an equitable one and does not concern itself with the character of the defendant's title, but only
with whether or not by reason of the plaintiff's long inaction or inexcusable neglect he should be barred from
asserting this claim at all, because to allow him to do so would be inequitable and unjust to the defendant.

Borromeo v. Almazora
G.R. No. 200558, July 01, 2015

FACTS:

Wilhelm Jambrich (An Austrian) went to the Philippines for his work. There he met respondent Antonietta Opalla-
Descallar. They eventually fell in love and decided to live together in a rented house in Mandaue City. They bought
a house and lot and an Absolute Deed of Sale was issued in their names. However, when the Deed of Absolute Sale
was presented for registration, it was refused on the ground that Jambrich was an alien and could not acquire
alienable lands of the public domain. Consequently, his name was erased but his signature remained and the
property was issued on the name of the Respondent alone. However their relationship did not last long --
respondent found a new boyfriend; Jamrich lived with another woman in Danao.

Jambrich met the petitioner (Borromeo) who was engaged in real estate business. Jambrich bought an engine and
some accessories for his boat from the petitioner, for which he became indebted to. To pay his debt, he sold some
of his properties to the petitioner and a Deed of Absolute Sale/Assignment was issued in his favor. However, when
the Petitioner sought to register the deed of assignment,it found out that said land was registered in the name of
Respondent and that it has already been mortgaged. Petitioner filed a complaint against respondent for recovery of
real property.
ISSUES:

1. Whether or not Jambrich has no title to the properties in question and may not transfer and assign any rights
and interest in favor of the petitioner?

2. Whether or not the registration of the properties in the name of respondents make his the owner thereof.

RULINGS:

The evidence clearly shows that between respondent and Jambrich, Jambrich is the one who is financially capable
to possess the properties in dispute. At the time of the acquisition of the properties, Jamrich was the source of
funds used to purchase the three parcels of land, and to construct the house. Jambrich was the owner of the
properties in question, but his name was deleted in the Deed of Absolute Sale because of legal constraints.
Nevertheless, his signature remained in the deed of sale where he signed as a buyer. Thus, Jambrich has all
authority to transfer all his rights, interest and participation over the subject properties to petitioner by virtue of
Deed of Assignment. Furthermore, the fact that the disputed properties were acquired during the couples
cohabitation does not help the respondent.

The rule of co-ownership applies to a man and a woman living exclusively with each other as husband and wife
without the benefit of marriage, but otherwise capacitated to marry each other does not apply. At the case at bar,
respondent was still legally married to another when she and Jambrich lived together. In such an adulterous
relationship and no co-ownership exists between the parties. It is necessary for each of the partners to prove his or
her actual contribution to the acquisition of property in order to able to lay claim to any portion of it.

2. It is settled rule that registration is not a mode of acquiring ownership. It is only a means of confirming the
existence with notice to the world at large. The mere possession of a title does not make one the true owner of the
property. Thus, the mere fact that respondent has the titles of the disputed properties in her name does not
necessarily, conclusively and absolutely make her the owner.

(c ) Special Cases

Vda. De Espiritu v. CIF


G.R. No. L-30486, October 31, 1972

An action to compel execution of deed is subject to prescription.

FACTS:

Sometime in 1948, respondents verbally sold to petitioner two parcels of land. No deed of sale was executed at the
time because the titles were then in the name of their predecessor in interest and were still to be transferred to
their names. However, despite demands made by petitioner for the execution of such deed, respondents failed and
refused to comply with such demands. Hence, on October 20, 1964, petitioner filed an action to compel
respondents to execute the proper deed of conveyance of the two parcels of land to her.

In their answer, respondents denied that the transaction was a sale and alleged that it was merely a contract of
antichresis. By way of affirmative defense, they interposed prescription of petitioner’s action, the same having
allegedly accrued in 1948. Subsequently, respondents reiterated their said affirmative defense of prescription in a
motion to dismiss and as no opposition thereto was filed by petitioner, the CFI issued the impugned order of
dismissal.

It is petitioner’s pose that her action to compel respondents to execute a promised deed of sale in her favor is
imprescriptible under Section 38 of Art. 190, the Code of Civil Procedure.

ISSUE:
Whether the action has already prescribed.

RULING:

YES. The whole statute of limitations embodied in Chapter III of the Code of Civil Procedure must be deemed
supplanted are replaced by Chapter 3, Title V Book III of the Civil Code.

Differently from the Code of Civil Procedure, the Civil Code does not consider the action by the vendee of real
property to compel execution of a deed of conveyance as imprescriptible. In fact, under Article 1143, only the
following rights "are not extinguished by prescription: (1) to demand a right way, regulated in Article 649 and (2)
to bring an action to able a public or private nuisance", which are actions involving public policy. Nor is there any
other provision of the Civil Code or any unrepealed law or jurisprudential ruling of this Court, under which
petitioner’s claim of imprescriptibility can be sustained. The specific enumeration in the Civil Code of
imprescriptible actions excludes any other ones.

The nature of petitioner’s action may be said to be one founded on an oral contract, which, to be sure, cannot be
considered as among those rendered unenforceable by the statute of frauds, for the simple reason that it has
already been, from petitioner’s own point of view, almost fully consummated by the delivery of the lands and the
corresponding titles to her.

Consequently, petitioner’s action prescribes in six years according to Article 1145 of the Civil Code. Assuming
otherwise, the only other possibility is that petitioner’s case comes under Article 1149 and the action prescribes in
five years. In either case, since the cause of action of petitioner accrued in 1948 and the present suit was instituted
in 1964 or sixteen years later, and none of the interrupting circumstances enumerated in Article 1155 has been
shown to have intervened, it is unquestionable that petitioner’s action filed in the court below has already
prescribed.

Solidarios v. Alampay
January 28, 1975

The Civil Code provides that where, as alleged in the complaint, two parties agree upon the mortgage of real property
but the instrument states that the property is sold absolutely or with a right of repurchase, an action for reformation
of the instrument is proper. The prescriptive period must be determined on the basis of the allegations of the
complaint for reformation of instrument, which is ten years. Moreover, the existence of a bona fide mortgage in favor
of a third party clearly constitutes no impediment to petitioners' action for reformation and recovery of title.

FACTS:

Petitioner-spouses filed a complaint against private respondent in the CFI presided by respondent judge for
reformation of instrument and praying that the deed of absolute sale of the parcel of land executed by them in
favor of private respondent which did not embody their true agreement be reformed and declared a contract of
mortgage. Respondent raised the defense of prescription. Respondent court after pre-trial directed respondent to
raise the issue of prescription squarely in a motion to dismiss. Thereafter, with no evidence on the sole issue of
prescription received, respondent court granted the motion to dismiss on the ground that (1) the proper remedy is
annulment of the sale on the ground of “vitiated consent” which action has prescribed in 4 years, and (2) existence
of mortgage by respondent in favor of a third party who is a mortgagee in good faith.

ISSUE:

Whether or not the action for reformation of the instrument by petitioner has prescribed.

RULING:
NO. The complaint for reformation of instrument clearly alleged that the deed of sale did not express the true
agreement of the parties and should be reformed into the mortgage that it actually was and prayed that petitioners
be allowed to redeem the property by repaying the loan. Such allegations are binding for purposes of the dismissal
motion and therefore the applicable prescription period for such actions based upon a written contract and for
reformation thereof as provided by law is 10 years. In this case, petitioners' action for reformation and recovery of
title was brought less than 8 years after execution of the questioned deed and had therefore not prescribed. In
addition, the mortgagee's rights over the property are recognized but that would in no way defeat petitioners'
action for reformation and recovery of title to the property.

Jalandoni v. PNB
October 9, 1981

Moreover, article 478 of the Civil Code provides that "there may also be an action to quiet title or remove a cloud
therefrom when the contract, instrument or other obligation has been extinguished or has terminated, or has been
barred by extinctive prescription"

FACTS:

On March 31, 1959 the Court of First Instance of Manila rendered a judgment ordering Eduardo Jalandoni to pay
the Philippine National Bank the sum of P63,297.53, together with daily interest of P12.57 from March 6, 1959
until fully paid, and ten percent of the total amount due as attorney's fees, plus the costs (Civil Case No. 38393).

That judgment became final and executory. Within five years from the entry of judgment in that case, the sheriff of
Silay City, levied upon Lot No. 657-C of the Silay cadastre, with an area of seventeen hectares.
No effort was made by the bank up to this time to have that land sold at public auction to satisfy the judgment
against Jalandoni.

On April 22, 1974, or more than ten years after the levy was made, Jalandoni filed with the Court of First Instance
of Negros Occidental at Silay City in the land registration proceeding, LRC Cadastral Record No. 86 for Lot No. 657-
C, a petition for the cancellation of the levy on the ground of prescription. The petition was opposed by the bank.

On May 20, 1975, Jalandoni filed in the same court an action to quiet title or for the cancellation of the notice of
embargo on the ground that, although more than ten years had elapsed from the time the levy was made, no
execution sale had been held and, therefore, the levy had become inefficacious and was a cloud on his title (Civil
Case No. 685).

The trial court rendered a Decision dismissing the complaint. The heirs of Jalandoni (he died on January 20, 1977)
appealed to this Court under Republic Act No. 5440.

ISSUE:

Whether the CFI was correct in dismissing the case?

RULING: NO.

The employees of the bank were negligent. They did not require the sheriff to sell Jalandoni's land at public
auction. The bank is bound by its employees' negligence. This case should teach the responsible officers of the bank
to be more vigilant in exercising its rights and in supervising its employees. The law helps the diligent and vigilant,
not those who sleep on their rights.

For laches and neglect on the part of those, who, under the law are entitled to require of others the fulfillment of
their obligations, the statute of limitations has been enacted, which provides that such rights prescribe after a
certain period of time, in order that it may serve alike as a punishment for those who do not know how to look
after their own interests, and as a source of reassurance to those who may have rested in the belief that their
creditors had waived their rights, and also to insure economic stability and the certainty of rights. (Villareal, J., in
Lutero vs. Siuliong & Co., 54 Phil. 272, 280.)

We find that the "notice of embargo" annotated in 1964 on Jalandoni's title is no longer enforceable and has
become a cloud upon his title. Following the rule in the Ansaldo case, he and his heirs have a good cause of action
under article 476 of the Civil Code for the removal of that state encumbrance.

Moreover, Article 478 of the Civil Code provides that "there may also be an action to quiet title or remove a cloud
therefrom when the contract, instrument or other obligation has been extinguished or has terminated, or has been
barred by extinctive prescription". (See sec. 112 of Act No. 496.)

A court of equity will remove a cloud cast upon title to property by a lien, interest, or title which has become barred
by reason of laches or the running of the statute of limitations. ...

Liens which were acquired by virtue of judgments or levies of execution, and which have become barred by
limitations or by delay in enforcing them, and sales based on such hens, have been held to be removable as clouds.
(65 Am Jur 2nd 163-164).

Republic v. CFI
January 27, 1983

Ordinarily, the counting of the prescriptive period should be reckoned from the date the debt became due and
demandable. However, the moratorium decrees supervene suspending the enforcement of payments of all debts and
other monetary obligations contracted during the war, although in the case of Royal L. Rutter v. Placido J. Esteban, the
moratorium laws were declared unconstitutional. Nevertheless, said laws were in effect from the time of their
respective promulgations until May 18, 1953. As a consequence, before they were declared unconstitutional, they
suspended the running of the prescriptive period during their effectivity. Hence in the case at bar, the 10-year period
within which to institute the action against herein appellee began the day after the moratorium laws were declared
unconstitutional or, to be precise, on May 19, 1953.

Where the loans in question did not have any maturity dates and, therefore, payable on demand, prescription could
have accrued, if at all, only on September 27, 1954 when petitioners made the extra-judicial demand. Plaintiff’s cause
of action will therefore prescribe only on September 27, 1964 and, since the complaint in this case was filed on
September 15, 1961, which is within the 10-year period, the action has not yet prescribed.

FACTS:

In 1943 defendant Dolores Infante obtained loans from the Bank of Taiwan, Ltd., payable at its office in Bacolod
City in the total amount of P683.10 with interest at the rate of six percent per annum, compounded quarterly.

On September 15, 1961, plaintiff Republic of the Philippines filed a complaint to collect from the defendant the said
amount. The defendant moved to dismiss the complaint on the ground of prescription. The Justice of the Peace,
after hearing, dismissed the case on the ground that the action had prescribed. The plaintiff appealed to the CFI and
its decision, the case was dismissed on the ground that plaintiff’s action had already prescribed.

ISSUE:

WON the action has prescribed.

RULING:
In the case at bar, the loans which had no maturity dates were contracted in 1943, or during the period of the
Japanese occupation of the Philippines. Ordinarily, the counting of the prescriptive period should be reckoned from
the date the debt became due and demandable. However, the moratorium decrees supervene suspending the
enforcement of payments of all debts and other monetary obligations contracted during the war, although in the
case of Royal L. Rutter v. Placido J. Esteban, the moratorium laws were declared unconstitutional. Nevertheless,
said laws were in effect from the time of their respective promulgations until May 18, 1953. As a consequence,
before they were declared unconstitutional, they suspended the running of the prescriptive period during their
effectivity.

Thus, the 10-year period within which to institute the action against herein appellee began the day after the
moratorium laws were declared unconstitutional or, to be precise, on May 19, 1953. It was on September 27, 1954
when plaintiff (appellant) made extra-judicial written demand on defendant (appellee). As the loans in question
did not have any maturity dates and, therefore, payable on demand, prescription could have accrued, if at all, only
on September 27, 1954 when petitioner made the extra-judicial demand. Plaintiff’s cause of action will therefore
prescribe only on September 27, 1964. And, since the complaint in this case was filed on September 15, 1961,
which is within the 10-year period, the action has not yet prescribed.

ACCORDINGLY, the order of the lower court, dated October 28, 1963, dismissing the complaint is hereby SET
ASIDE and the case remanded to the court below for further proceedings. With costs against the appellee.

8. Succession
C. Modes of Extinguishing Ownership

S-ar putea să vă placă și